You are on page 1of 150

P

Engineering
Mathematics

A
Chapter 1: Linear Algebra2.3
Chapter 2: Calculus2.26
Chapter 3: Differential Equations 2.63
Chapter 4: complex variables2.98

R
Chapter 5: probability and statistics2.112
Chapter 6: numerical methods2.131

t
II
Chapter 01.indd 1 8/29/2015 9:00:35 AM
This page is intentionally left blank

FM.indd 6 1/4/2016 2:22:07 PM


Chapter 1
Linear Algebra
LEARNING OBJECTIVES

After reading this chapter, you will be able to understand:

Matrix Systems of Linear Equations


Type of Matrices Cramers Method
Determinants Inverse Method
Minor and Cofactor of a Matrix Gauss - Jordan Method
Singular and Non-singular Matrices Gauss Elimination Method
Adjoint of a Matrix The Characteristic Equation of a Matrix
Evaluating Inverse of a Square Matrix Caley - Hemilton Theorem
Elementary Operations (or) Elementary Transformations

A set of mn elements arranged in the form of rectangular array Diagonal matrix A square matrix is said to be a diagonal matrix,
having m rows and n columns is called a m n matrix (read if all its elements except those in the principal diagonal are zeros.
as m by n matrix) and is denoted by A = [aij] where 1 i m; i.e. if
1 j n or
1. m = n (A is a square matrix) and
a11 a12 a13 a1n
2. aij = 0 if i j (The non-diagonal elements are zeros)
a a22 a23 a2 n
A = 21 A diagonal matrix of order n with diagonal elements d1, d2, ... dn
: : : :
is donated by Diag [d1 d2 .. dn]
am 1 am 2 am 3 amn
Scalar matrix A diagonal matrix whose diagonal elements are all
The element aij lies in the i-th row and j-th column. equal is called a scalar matrix. i.e. if
1. m = n
Type of Matrices
2. aij = 0 if i j
Square matrix A matrix A = [aij]m n is said to be a square matrix, 3. aij = k if i = j for some constant k.
if m = n (i.e. No. of rows of A = No. of columns of A)
The elements a11, a22, a33 ........ ann are called DIAGONAL Unit or identity matrix A scalar matrix of order n in which each
ELEMENTS. diagonal elements is 1 (unity) is called a unit matrix or identity
The line containing the diagonal elements is the PRINCIPAL matrix of order n and is denoted by In. i.e.
DIAGONAL. 1. m = n
The sum of the diagonal elements of A is the TRACE of A. 2. aij = 0 if i j
Row matrix A matrix A = [aij]m n is said to be a row matrix, if 3. aij = 1 if i = j
m = 1 (i.e. the matrix has only one row) General form is A = [a1, a2 1 0 0
1 0
....... an] or [aij]1 n E.g. I1 = [1], I2 = , I =
3 0 1 0
0 1 0 0 1
Column matrix A matrix which has only one column.
a1 Null matrix or zero matrix A matrix is a null matrix or zero
matrix, if all its elements are zeros.
a
A = 2 or [aij]n 1
: Upper triangular matrix A square matrix is said to be an upper
triangular matrix, if each element below the principal diagonal is
a
n zero. i.e.

Chapter 01.indd 3 8/29/2015 9:00:36 AM


2.4|Engineering Mathematics

aij = 0 if i > j
1. m = n 2. Thus, if A = [aij]n n then trace of A denoted by tr A = a11
+ a22 + ... + ann
1 4 3 2 Properties of trace of a matrix Let A and B be any two
E.g. 0 1 6 1
square matrices and K any Scalar, then
0 0 3 2
tr(A + B) = trA + trB
1.
0 0 0 9 44 tr(KA) = KtrA
2.
tr(AB) = tr(BA)
3.
Lower triangular matrix A square matrix is said to be a Conjugate of a matrixA matrix obtained by replacing
lower triangular matrix, if each element above the principal each element of a matrix A by its complex conjugate is
diagonal is zero, i.e. if called the conjugate matrix of A and is denoted by A. If
1. m = n 2.
aij = 0 if i < j A = [aij]m n, then A = aij where aij is the conjugate of aij.

1 0 0 0 Properties of Conjugate of a Matrix


2 1 0 0
E.g.
0 7 8 0
C 1 (( A)) = A for any matrix A
5 4 2 1
C 2( A + B ) = A + B for any matrices A, B of same
order.
Horizontal matrix If the number of rows of a matrix is less
than the number of columns, i.e. m < n, then the matrix is C 3 ( KA) = K A for any matrix A and any Scalar K.
called a horizontal matrix.
C 4( A B ) = ( A ). B for any matrices A and B with the
Vertical matrix If the number of columns in a matrix is less condition that number of columns of A = number
than the number of rows, i.e. if m > n, then the matrix is of rows of B.
called a vertical matrix.
C 5 ( A ) n = ( A n ) for any square matrix A.
Comparable matrices Two matrices A = [aij]m n and B = [bij]p q
are said to be comparable, if they are of same order, i.e. Tranjugate or transposed conjugate of a matrix
m = p; n = q. Tranjugate of a matrix A is obtained by transposing the
( )
T
Equal matricesTwo comparable matrices are said be conjugate of A and is denoted by Aq. Thus Aq = A
equal, if the corresponding elements are equal, i.e.
A = [aij]mn and B = [bij]pq are equal if Properties of T
ranjugate of a Matrix
1. m = p; n = q (i.e. they are of the same order) TC 1(Aq)q = A for any matrix A
2. aij = bij i, j (i.e. the corresponding elements are
TC 2(A + B)q = Aq + Bq for any matrices A, B of the
equal) same order.
Transpose of a matrix The matrix obtained by interchanging TC 3 (KA)q = KAq for any matrix A and any scalar K
the rows and the columns of a given matrix A is called the TC 4(BA)q = BqAq for any matrix A, B with the con-
transpose of A and is denoted by AT or A. If A is an (m n) dition that number of columns of A = number
matrix, AT will be an (n m) matrix. Thus, if of rows of B
A = [aij]mn then AT = [uij]nm where uij = aji. TC 5 (An)q = (Aq)n for any square matrix A.
Symmetric matrix A matrix A is said to be Symmetric, if
Properties of Transpose AT = A (i.e. Transpose of A = A)
T 1: (A) = A for any matrix A
Note: A Symmetric matrix must be a square matrix.
T 2: (A + B) = A + B for any two matrices A, B of
same order Skew-symmetric matrix A matrix A is said to be Skew-
T 3: (KA) = KA for any matrix A Symmetric Matrix, if AT = (A) i.e. A = [aij]m n is Skew-
T 4: (AB) = BA for any matrices A, B such that Symmetric if
number of columns of A = number of rows of B
1. m = n 2.
aji = aij i, j
(REVERSAL LAW)
T 5: (An) = (A)n for any square matrix A Note: In a skew-symmetric matrix, all the elements of the
principal diagonal are zero.
Trace of a matrixLet A be a square matrix. The trace
of A is defined as the sum of elements of A lying in the Orthogonal matrix A square matrix A of order n n is
principal diagonal. said to be an Orthogonal Matrix, if AAT = ATA = In.

Chapter 01.indd 4 8/29/2015 9:00:41 AM


Chapter 1 Linear Algebra | 2.5

Involutory matrixA square matrix A is said to be an A 4:A + (A) = (A) + A = O (A is the ADDITIVE
involutory matrix, if A = I (where I is identity matrix). INVERSE)
A 5: A + B = B + A (COMMUTATIVITY)
Idempotent matrixA square matrix A is said to be an
idempotent matrix, if A = A. Note: The set of matrices of same order form an Abelian
Group under addition.
Nilpotent matrixA square matrix A is said to be a
nilpotent matrix, if there exists a natural number n such Multiplication of matrices Let A and B be two matrices.
that An = O. If n is the least natural number such that A and B are conformable for multiplication, only if the
An = O, then n is called the index of the nilpotent matrix number of columns of A is equal to the number of rows of B.
A. (Where O is the null matrix) Let A = [aij] be an m n matrix, B = [bjk] be an n p matrix.
Unitary matrix A square matrix A is said to be a unitary Then, the product AB is defined as the matrix C = [cik] of
matrix if, AAq = AqA = I (Where Aq is the transposed n
order m p where cik = ai1b1k + ai2b2k + ... + ainbnk = aij b jk
conjugate of A) j =1

Hermitian matrix A Matrix A is said to be a hermitian cij calculated for i = 1, 2,... m and k = 1, 2, ..... m will give
matrix, if Aq = A, i.e. A = [aij]m n is Hermitian if all the elements of the matrix C.
1.m = n2.aij = ij i, j
Properties of Multiplication
Note: The diagonal elements in a hermitian matrix are real
M 1:If A, B, C be m n, n p, p q matrices respec-
numbers. tively, then (AB)C = A(BC) (ASSOCIATIVITY)
Skew-Hermitian matrix A matrix A is said to be a skew- M 2:If A is a mn matrix, then AIn = A and
hermitian matrix, if Aq = A. Im A = A and if A is a square matrix i.e. m = n,
then AI = IA = A (I is the MULTIPLICATIVE
Operations on Matrices IDENTITY)
M 3:If A, B, C be m n, n p, p q matrices respec-
Scalar multiplication of matrices If A is a matrix of order
tively, then A(B + C) = AB + AC (DISTRIBUTIVE
m n and K be any scalar (a real or complex number),
LAW)
then KA is defined to be a m n matrix whose elements are
M 4:Matrix multiplication is NOT COMMUTATIVE
obtained by multiplying each element of A by K, i.e. if
in general.
A = [aij]m n, then KA = [Kaij]m n in particular if K = 1;
M 5:The INVERSE of a given matrix may not
then KA = A is called the negative of A and is such that
always exist.
A + (A) = [aij] + [aij] = [aij aij] = [0] = O (Zero Matrix)
Determinants Let A = [aij] be a Square Matrix of order n.
(A) + A = [aij] + [aij] = [aij + aij] = [0] = O
Then, the determinant of order n associated with A is
i. e. A + (A) = (A) + A = O
denoted by |A| or |aij| or Det(A) or .

Properties of Scalar Multiplication Note: 1. Determinant of a matrix exists, only if it is a


square matrix.
Let A, B be two matrices of same order and a, b be any
2. The value of a determinant is a single number.
scalars, then
S 1: a(A + B) = aA + aB Determinant of order 1 (or first order determinant)
S 2: (a + b)A = aA + bA If a be any number, then determinant of a is of order 1
S 3: a(bA) = (ab)A and is denoted by |a|. The value of |a| = a
S 4: 1.A = A Determinant of order 2 (or second order determinant)
Addition of matrices If A and B are two matrices of the a1 b1
If A is a square matrix of order 2 given by A = ,
same order, then they are conformable for addition and a2 b 2
their sum A + B is obtained by adding the corresponding a1 b1
elements of A and B i.e. if A = [aij]m n; B = [bij]m n, then A then Det (A) = is determinant of order 2 and its
a2 b 2
+ B = [aij + bij]m n
value is = a1b2 - a2b1
Properties of addition Let A, B and C be three matrices of
same order say m n, then Minor and Cofactor of a Matrix
A 1: A + B is also a m n matrix (CLOSURE) a1 b1 c1
A 2:(A + B) + C = A + (B + C) (ASSOCIATIVITY)
Let A = a2 b2 c2 be a 3 3 matrix
A 3:If O is the m n Zero (Null) matrix, then A + O
a b3 c3
= O + A = A (O is the ADDITIVE IDENTITY) 3

Chapter 01.indd 5 8/29/2015 9:00:42 AM


2.6|Engineering Mathematics

Then, the minor of an element aij of A is the determi- of elements of any row (or column) by their corresponding
nant of the 2 2 matrix obtained after deleting the i-th row cofactors. We can similarly define determinant of a square
and j-th column of A and is denoted by Mij. matrix of any order.
The cofactor of aij small size is denoted by Aij and is
defined as (-1)i + j Mij i.e., Aij = (1)i + j Mij Properties of Determinant
Determinant of order 3 (third order determinant) 1. If two rows (or columns) of a determinant are
a1 b1 c1 interchanged, the value of the determinant is
multiplied by (-1).
If A is a square matrix of order 3given by A = 2a b2
c2 .
a b c 2. If the rows and columns of a determinant are
3 3 3 interchanged, the value of the determinant remains
Then, the determinant of A given by = Det A unchanged, i.e. Det(A) = Det(AT).
3. If all the elements of a row (or column) of a
a1 b1 c1 determinant are multiplied by a scalar (say K), the
= a2 b2 c2 is a determinant of order 3 and the value is value of the new determinant is equal to K times the
a3 b3 c3 value of the original determinant.
4. If two rows (or columns) of a determinant are
obtained by taking the sum of the products of the elements identical, then the value of the determinant is zero.
of any row (or column) by their corresponding cofactors. 5. If the elements of a row (or a column) in a determinant
Thus for A, = a1A1 + b1B1 + c1C1 are proportional to the elements of any other row (or
column), then the determinant is 0.
b2 c2 a2 c2 a2 b2 6. If every element of any row (or column) is zero, then
= a1 b1 + c1 or also
b3 c3 a3 c3 a3 b3 determinant is 0
= a1A1 + a2A2 + a3A3 7. If each element in a row (or column) of a determinant
is the sum of two terms, then its determinant can be
b2 c2 b1 c1 b1 c1 expressed as the sum of two determinants of the same
= a1 a2 + a3
b3 c3 b3 c3 b2 c2 order.
8. (The theorem of false cofactor) The sum of products
(This is by expanding by C1) and so on. of elements of a row (or column) with the cofactors of
The sign to be used before a particular element can be any other row (or column) is zero.
judged by using the following rule:
a1 b1 c1
++
+ Thus, in A = a2 b2 c2
++ a b c
3 3 3
The value of the determinants of order 3 can also be eval-
uated by using Sarrus method given below a1A2 + b1B2 + c1C2 = 0
a2A1 + b2B1 + c2C1 = 0 and so on in general
a1 b1 c1 arAs + brBs + crCs = 0 if r s
Let = a2 b2 c2 9. If the elements of a determinant are polynomials in x
a3 b3 c3 and the determinant vanishes for x = a, then x - a is a
factor of the determinant.
Enter the first column and then the second column after
the third column and take the product of numbers as shown
by the arrows, taking care of signs indicated Singular and non-singular matrices A square matrix A
c1
is said to be singular, if Det(A) = 0 and is non-singular, if
a1 b1 a1 b1
Det(A) 0.
a2 b2 c2 a2 b2 Note:
a3 b3 c3 a3 b3 1. A unit matrix is non-singular (as its Det = 1)
2. If A and B are non-singular matrices of the same
Then, type, then AB is non-singular of the same type.
= a1b2c3 + b1c2a3 + c1a2b3 - a3b2c1 b3c2 a1 - c3a2b1
We can now define the cofactor of an element aij in a
Inverse of a matrix Let A be a square matrix. A matrix B
4 4 matrix as (1)i + j (Determinant of the 3 3 matrix
is said to be an inverse of A, if AB = BA = I
obtained by deleting the i-th row and j-th column) and
Determinant of a 4 4 matrix to be the sum of products Note: If B is the inverse of A, then A is the inverse of B.

Chapter 01.indd 6 8/29/2015 9:00:45 AM


Chapter 1 Linear Algebra | 2.7

Some Results of Inverse Rank and Nullity of a Matrix


1. Inverse of a square matrix, when it exists, is unique. 1. Rank of a matrix: The Matrix A is said to be of rank
2. The inverse of a square matrix exists, if and only if it r, if and only if it has at least one non-singular square
is non-singular. submatrix of order r and all square sub-matrices of
3. If A and B are square matrices of the same order, order (r + 1) and higher orders are singular. The rank
then AB is invertible (i.e. inverse of AB exists) if A of a matrix A is denoted by rank (A) or r(A).
and B are both invertible. 2. Nullity of a matrix: If A is a square matrix of order
4. If A and B are invertible matrices of the same n, then n r(A) i.e. n rank (A) is defined as nullity
order, then (AB)1 = B1A1 of matrix A and is denoted by N(A).
5. If A is invertible, then so is AT and (AT) 1 = (A1)T.
Remark 1: If there is a non-singular square submatrix
6. If A is invertible, then so is Aq and (Aq)1 = (A1)q.
of order K, then r(A) K.
Adjoint of a matrix Remark2: If there is no non-singular square
submatrix of order K, then r(A) < K.
The adjoint of a square matrix A is the Transpose of the
matrix obtained by replacing the elements of A by their Remark 3: If A1 is the transpose of A, then r(A) = r(A1)
corresponding cofactors. Remark 4: The rank of a null matrix is 0.
Remark 5: The rank of a non-singular square matrix
Note: The adjoint is defined only for square matrices and
of order n is n and its Nullity is 0.
the adjoint of a matrix A is denoted by Adj(A).
Remark 6: Elementary operations do not change the
a1 a2 .... an rank of a matrix.
b b2 .... bn Remark 7: If the product of two matrices A and B
If A = 1
.... .... .... .... is defined, then r(AB) r(A) and r(AB) r(B). i.e.
 2 ....  n the rank of product of two matrices cannot exceed the
1
rank of either of them.
T
A1 A 2 .... An
B B A B1 .... L1 Elementary Operations (or) Elementary
.... Bn 1
1 2 A B2 .... L2 Transformations
Adj A = .... .... .... .... = 2
.... .... .... .... .... .... 1. Elementary Row Operations
.... ....

L L
A Bn .... L n (a) Ri Rj: Interchanging of ith and jth rows
1 2
..... L
n
n
(b) Ri KRi: Multiplication of every element of ith
Results row with a non zero scalar K
1. If A is of order 3 3 and K is any number, then (c) Ri Ti + KRj: Addition of k times the elements
Adj(KA) = K(Adj A) of jth row to the corresponding elements
2. A(Adj A) = (Adj A)A = |A| I for any square matrix A of ith row
3. Adj I = I; Adj O = O where I is the identity matrix and 2. Elementary column operations
O is the null matrix. (a) Ci Cj: Interchanging of ith and jth columns
4. Adj(AB) = (Adj B) (Adj A) if A, B are non-singular Ci KCi: Multiplication of every element of ith
(b) 
and are of same type. column with a non zero scalar K
5. If A = An n, then Ci Ci + KCj: Addition of K times the elements
(c) 
det(Adj A) = (det A)n - 1. of jth column to the corresponding element of
Adj(Adj A) = (det A)n - 2(A). ith column
2 3 4 1
|Adj(Adj A)| = (det A)( n1) Consider the matrix A = 3 0 1 5
2

Evaluating Inverse of a Square Matrix 4 7 1 2


R2 2R2
1
If A is a square matrix, then A1 = (Adj A) 2 3 4 1
A
Note: 6 0 2 10
4 7 1 2
1. The inverse of an identity matrix is itself.
1 C2 C3
2. (Adj A)1 = A
A 2 4 3 1
3. If A is a non-singular square matrix (say of order 3) 3 1 0 5

1 1
and K is any non-zero number, then (KA)-1 = A 4 1 7 2
K

Chapter 01.indd 7 8/29/2015 9:00:48 AM


2.8|Engineering Mathematics

C1 C1 - 2C4 Row reduced echelon matrix


0 4 3 1 A matrix X is said to be row reduced echelon matrix if
7 1 0 5 1. X is row reduced.
2.There exists integer P (0 p m) such that first p
0 1 7 2
rows of X are non-zero and all the remaining rows are
Note: The rank of a matrix is invariant under elementary zero rows.
operations 3.For the ith non-zero row, if the first non-zero element
of the row (i.e.,1) occurs in the jth column then,
Row and Column Equivalence Matrices j1 < j2 < j3 < .. < jp
Row equivalence matrix 1 0 0 2
If B is a matrix obtained by applying a finite number of 0 1 2 0
0 1 0 3
elementary row operations successively on matrix A, then Example: P = ; Q = 0 0 0 1 echelon
0 0 1 4
matrix B is said to be row equivalent to A. (or a row equiva- 0 0 0 0

lent matrix of A) 0 0 0 0
matrices. The number of non-zero rows (i.e. value of P) are
Column equivalence matrix
3 and 2 respectively. The value of i and j are tabulated below
If B is obtained by applying a finite number of elementary
column operations successively on matrix A, then matrix B i 1 2 3
is said to be column equivalent to A. (or a column equivalent P:
j 1 2 3
matrix of A)
1 3 4 i 1 2
Q:
Example: A = 2 5 2
j 1 4
1 4 3
Normal form of a matrix
1 3 4
By means of Elementary transformations, every matrix A
R 2 2R1 0 1 10 = B (say )

of order m n and rank r(> 0) can be reduced to one of the
0 1 7 following forms.
B is a row equivalent matrix of A. I 0
(i) r (ii) [Ir/0](iii) [Ir]
1 3 2 0 0
Example: B = 3 4 4 C2 - 3C1, and these are called the normal forms. Ir is the unit matrix
of order r.
1 1 6
Note: If a m n matrix A has been reduced to the normal
1 0 1
C3 I 0
3 5 2 = C (say ) form say r then r is the rank of A.
2
1 2 6 0 0
Systems of Linear Equations
C is a column equivalent to B.
Let a11x1 + a12x2 + ..... a1nxn = b1
Row reduced matrix a12x1 + a22x2 + ..... a2nxn = b2

A matrix A of order m n is said to be row reduced if ................................
(1)
................................
1. The first non zero element of a non zero row is 1. an1x1 + an2x2 + ..... annxn = bn
2. Every other element in the column in which such 1s
be a system of n linear equations in n variables x1, x2,
occur is 0.
........ xn. The above system of equations can be written as
1 0 2
a11 a12 ........ a1n x1 b1
A = 0 1 3 is a row reduced matrix
0 0 0
a21 a22 .........a2 n x2 b2
.......................... ... = ... or AX = B
1 0 4
......................... ... ...
B = 0 5 0 is not a row reduced matrix.

0 0 0 an1 a2 n ..........an n xn bn

Chapter 01.indd 8 8/29/2015 9:00:52 AM


Chapter 1 Linear Algebra | 2.9

where ................................
a11 a12 ......... a1n x1 b1 am1x1 + am2x2 + ..... amnxn = bm

a21 a22 ......... a2 n x2 b2 This is a system of m equations in n variables

A = .......................... B = ... x1, x2, ... xn. The system of equations can be written as
, X = ... , AX = B where
......................... ... ...
a11 a12.......... a1n x1 b1
am1 am 2 ....... am n xn bm

a21 a22 ......... a2 n x2 b2
A is called the coefficient matrix.
A = .......................... , X = ... , B = ...

Any set of values of x1, x2, x3 ..... which simultaneously
......................... ... ...
satisfy these equations is called a solution of the system.
When the system of equations has one or more solutions, am1 am 2 am n x bm
n
the equations are said to be CONSISTENT and the system
of equations are said to be INCONSISTENT if it does not a11 a12 ..........a1n b1
admit any solution. The system of equation (1) is said to be
a21 a22 .........a2 n b2
HOMOGENEOUS, if B = 0 The matrix .................................. is called the aug-
NON-HOMOGENEOUS, if B 0 .................................

Let the system of equations be am1 am 2 .........amn bm

a11x1 + a12x2 + ..... a1nxn = b1
a12x1 + a22x2 + ..... a2nxn = b2 mented matrix of the system of equations and is denoted by
................................ [A : B].

Let AX = B represent m linear equations with n variables. Let Rank of A = r and Rank (A, B) = r1 [where (A, B) is an
augmented matrix] If r1 r, then the system of equations are inconsistent.
If r1 = r, the table follows:

m=n m>n m<n


r=n r<n r=n r<n r=m r<m
Homo Only trivial Infinite Only trivial Infinite Infinite Infinite
geneous solution solutions solution solutions solutions solutions
Non Homo Infinite Infinite Infinite Infinite
Unique solution Unique solution
geneous solutions solutions solutions solutions

Solving system of linear equations For solving the system of Linear Equation when the
The following methods of solving system of linear equa- coefficient matrix A is singular, i.e. when |A| = 0, we apply
tions (1) is applicable only when the coefficient matrix A the following method.
is non singular, i.e. |A| 0.
Gauss - jordan method
Cramers method Consider the augmented matrix [A : B] of the system of n
Let AX = B represent the system of equations (1) where A, X non-homogeneous equations (1) in n-variables
and B are as defined earlier. a11 a12 ..... a1n b1
Let be |A| and 1, 2, ......., n be the determinants
obtained by replacing the elements of 1st, 2nd ... nth column a21 a22 ..... a2 n b2
............................
of A by the elements of B. Then, if 0, we have
x1 = 1/ ; x2 = 2/ ; x3 = 3/ ..........., xn = n/ an1 an 2 ..... ann bn

Inverse method Reduce this augmented matrix to the standard form


Let the system of linear equations be AX = B, where A, X, B 1 0 ..... 0 d1
are as defined earlier.
0 1 ..... 0 d2
If |A| 0, then premultiplying with A1, we get ...........................
A1 (AX) = A1B.
0 0 ..... 1 dn
X = A1B which gives the values of the variables.

Chapter 01.indd 9 8/29/2015 9:00:55 AM


2.10|Engineering Mathematics

By applying the elementary operations, the solution of Step 1: Write A = LU, where L Lower triangular matrix
the equations is x1 = d1, x2 = d2 ... xn = dn. with principal diagonal elements being equal to 1 and U
Upper triangular matrix.
Gauss elimination method 1 0 0 u11 u12 u13
Let the system of linear equations given by
i.e. L = l21 1 0 and U = 0 u22 u23
a11 x1 + a12 x2 + ......... + a1n xn = c1 l 0 0 u33
31 l32 1

a21 x1 + a22 x2 + .......... + a2 n x n = c2 Step 2: Now, equation (2) becomes
LUX = B  (3)
a31 x1 + a32 x2 + .......... + a3n xn = c3
(1) y1
. . . .
. . . . Step 3: Let UX = Y (4) where Y = y2
. . . . y
3
am1 x1 + am 2 x2 + ........... + amn xn = cn Step 4: Combining (3) and (4), we get
LY = B (5)
Let a11 0 write the above equations in the matrix form On solving (5) we get y1, y2, y3
AX = B Step 5: Substituting Y in (4), we get UX = Y
Write the augmented matrix [A B] On solving, we get X i.e. x1, x2, x3.
Using elementary row operations eliminate the unknown
x1 from all the equations except the first. Eliminate the
unknown x2 from all the equations except from first and The Characteristic Equation of a
second rows, continuing in this way we finally get the fol- Matrix
lowing equivalent system of equations at the (n - 1)th step.
Characteristic matrix
a11x1 + a12x2 + a13x3. + a1nxn = c1 If A is any square matrix, the matrix A l I where l is a
a22x2 + . + a2nxn = c2 scalar, is called the characteristic matrix of A.
a33x3 + . + a3nxn = c3 Characteristic polynomial
amnn = Cn If A is any square matrix of order n, then the determinant
|A lI| yields a polynomial f(l) of degree n in l which
From the above system of equations we can find the val- is known as the characteristic polynomial of the matrix A.
ues of the unknowns.
Characteristic equation
Linear dependence If f(l) is the characteristic polynomial of a matrix A, then
A set of vectors of n dimensions is said to be linearly f(l) = 0, is called the characteristic equation of A.
dependent if one of these that vector can be expressed as a And the roots of this equation, say l1, l2, ... ln are called
linear combination of some other vectors in the set. the characteristic roots or latent roots or eigen values. If l is
If no vector can be expressed as a linear combination of the a characteristic root of order t, then t is called the algebraic
others, then the set of vectors is said to be linearly independent multiplicity of l.
Note: The number of linearly independent rows or columns
Characteristic vectors
of a matrix is called the rank of the matrix.
Corresponding to each characteristic root l, there is a
LU Decomposition Method of Factorisation non-zero vector which satisfies the characteristic equation
|A lI| = 0. These non-zero vectors are called the character-
(or) Method of T
riangularisation istic vectors or eigen vectors or latent vectors.
Consider the system of equations
Note:
a11x1 + a12x2 + a13x3 = b1
a21x1 + a22x2 + a23x3 = b2  (1) 1. The characteristic roots of a matrix and its transpose
a31x1 + a32x2 + a33x3 = b3 are the same.
2. 0 is the characteristic roots of a matrix, if the matrix
In matrix notation, (1) can be written as is singular.
AX = B  (2) 3. The characteristic roots of a triangular matrix are just
the diagonal elements of the matrix.
a11 a12 a13 x1 b1 4. If K is any scalar, the characteristic roots of matrix
KA are K times the characteristic roots of matrix A.
where A = a21 a22 a23 , X = x2 and B = b2
a a32 a33 x b 5. If a1, a2, a3, .... an are characteristic roots of matrix
31 3 3 A and K is a scalar, then the characteristic roots of
matrix A KI are a1 K, a2 K, an K.

Chapter 01.indd 10 8/29/2015 9:00:59 AM


Chapter 1 Linear Algebra | 2.11

6. If l is a characteristic root of a non-singular matrix, Note:Here, P is called the modal matrix and D is the spec-
then l1 is a characteristic root of A1. tral matrix of the matrix A
7. If the eigen values of A are l1, l2, .... ln then the eigen Power of a matrix by using its diagonal form If D is the
values of A are l1, l2, ..... ln. diagonal form of a square matrix A, then for any positive
integer n, we have An = P Dn P-1
Caley-Hemilton Theorem Where P is the modal matrix of A
Every square matrix satisfies its characteristic equation.
Solved Examples
Inverse by Cayley Hamilton Theorem
Example 1: Solve the system of equations
Let A be non singular square matrix of order n
x1 + x2 + x3 = 1, 3x1 + x2 3x3 = 5 and
Let the characteristic equation of A be
x1 2x2 5x3 = 10 by LU decomposition method.
|A - lI| = (-1)n ln + C1 ln - 1 + C2 ln - 2 + + Cn - 1 l
+ Cn = 0 1 1 1 x1 1

Where C1, C2, , Cn are all scalar constants Solution: AX = B 3 1 3 x2 = 5
Then, by cayley Hamilton theorem 1 2 5 x3 10
(-1)nAn + C1An - 1 + C2An - 2 + + Cn - 1A + CnI Step 1: LU = A
=0 (1)
Multiplying (1) throughout by A-1, we have 1 0 0 u11 u12 u13 1 1 1

A-1[(-1)n An - 1 + C1 An - 1 + C2 An - 2 + + Cn - 1 A + l21 1 0 0 u22 u23 = 3 1 3
CnI] = A-1.0 l l32 1 0 0 u33 1 2 5
31
 (-1)n An - 1 + C1 An - 2 + C2 An - 3 + + Cn - 1 I + Cn A-1
1 Expanding and on solving we get, u11 = 1, u12 = 1, u13 = 1,
 A-1 = [(-1)n An - 1 + C1 An - 2 + C2 An - 3 + .. + Cn - 1 I]
Cn u22 = 2, u23 = 6, u33 = 3, l21 = 3, l31 = 1, l32 = 3
2
Note: Similarly, we can find A-2, A-3, .for the matrix AX Step 2: Now, LUX = B
provided A is non singular. Step 3: Let UX = Y
Power of a matrix by Cayley-Hamilton theoremThe Step 4: LY = B
Cayley-Hamilton theorem is also helpful in finding higher
powers of a square matrix with least possible number of 1 0 0 y1 1

matrix multiplications. This is explained in Examples 1 and 2 3 1 0 y2 = 5
Reduction to diagonal form If A is a square matrix of order 3 y 10
1 1 3
n with n linearly independent eigen vectors, then A can be 2
reduced to a diagonal matrix, called diagonal form of A On solving, y1 = 1, y2 = 2 and y3 = 6
Procedure to reduce a square matrix into diagonal
Step 5: UX = Y
form Let A be a square matrix of order n that can be reduced
to diagonal form 1 1 1 x 1 1

1. Find the eigen values and their corresponding eigen 0 2 6 x 2 = 2

vectors of A 0 0 3 x 3 6
Let l1, l2, l3, .., ln be the eigen values and let
x1, x2, x3, , xn be their corresponding eigen vectors On solving we get x1 = 6, x2 = 7 and x3 = 2
that are linearly independent The sol is (6, 7, 2)
2. Form the matrix P with x1, x2, x3, ., xn as its columns
Example 2: Find the value of
i.e. P = [x1 x2 x3 xn] it can be easily observed that
P is invertible a + b + 2c a b
3. Find the inverse of P (i.e. find P-1) c b + c + 2a b
4. The diagonal form of A is given by D = P-1 AP c a c + a + 2b
l1 0 ...... 0
0
Solution: c1 c1 + c2 + c3
Where D = 0 l2
0 ...... 0 is a diagonal
0 0 ...... ln
0 2 (a + b + c) a b

2 ( a + b + c ) b + c + 2a b
matrix with eigen values of A as its principal diagonal
elements 2 (a + b + c) a c + a + 2b

Chapter 01.indd 11 8/29/2015 9:01:02 AM


2.12|Engineering Mathematics

1 a b R2 R2 -3R1, and R3 R3 - 2R1


= 2(a + b + c) 1 b + c + 2a b
1 1 1 6
c + a + 2b
~ 0 5 4 22
1 a
R2 R2 - R1 0 1 4 10
R3 R3 - R1
R1 R1 + 1/5R2 and R3 R3 + 1/5 R2
1 a b
2(a + b + c) 0 a + b + c 0 1 0 1/5 8 /5
0 0 a+b+c ~ 0 5 4 22
0 0 24 /5 72 /5
1 a b
= 2(a + b + c)3 0 1 0 = 2(a + b + c)3 5 5
R2 R3 + R2 ; R3 R
0 0 1 6 24 3

Example 3: How many solutions are for the system of linear 1 0 1/5 8 /5
equations x + 2y + z = 0, 3x + 2y - z = 0 and 4x + y - 3z = 0? ~ 0 5 0 10
Solution: Determinant of the coefficient matrix of the 0 0 1 3
1 2 1 1 1
given equation is 3 2 1 = 1(-6 + 1) -2 (-9 + 4) + 1 R1 R3 and R2 R2
5 5
4 1 3
(3 - 8) = 0 1 0 0 1
The system has infinite number of solutions. R2 -1/5R2 ~ 0 1 0 2

0 0 1 3
Example 4: Solve 3x + 2y - z = 0, 4x + y + 2z = 0 and x - 5y
+ 7z = 0 Solution is x = 1, y = 2 and z = 3
Solution: Determinant of the coefficient matrix of the Example 6: Determine the eigen values and eigen vectors
3 2 1
2 4
equations when written in matrix form is 4 1 2 of A =
3 3
1 5 7
Solution: Characteristic equation of the given matrix is
= 3(7 + 10) -2 (28 - 2) -1 (-20 - 1)
2l 4
= 51 - 52 + 21 = 20 A lI = 0 =0
3 3l
The given system of equations have only one solution
i.e. x = y = z = 0 l2 - 5l - 6 = 0
(l - 6)(l + 1) = 0
Example 5: Solve: x + y + z = 6, 3x - 2y - z = -4 and l = -1 and l = 6 are the eigen values. Eigen vector
2x + 3y -2z = 2 corresponding to l = -1 is obtained as follows.
1 1 1 2 4 1 0 x1 0
Solution: A = 3 2 1 =1(7) - 1(-4) + 1(9 + 4) 0 +1 =
3 3 0 1 x2 0
2 3 2
3 4 x1 0
The set of given equations are non homogeneous = 3x1 + 4x2 = 0
and the number of equations is equal to the number of 3 4 x2 0
variables.
3x1 + 4x2 = 0 x1 = -4/3x2
The given system of equations are consistent and has
a unique solution. Eigen vector corresponding to l = -1 is x =
Augmented matrix 1 1 1 6 4
x1 x2 4 / 3
[A B] is 3 2 1 4 =
x 3 = x 2 1

2 3 2 2
2
x2

Chapter 01.indd 12 8/29/2015 9:01:08 AM


Chapter 1 Linear Algebra | 2.13

Similarly, eigen vector corresponding to l = 6 is obtained Example 9: Solve the following equations by using Gauss
as follows elimination method.
2 4 1 0 x1 0 2 x1 x2 + 3 x3 = 9
6 =
3 3 0 1 x2 0 x1 3 x2 2 x3 = 0
3 x1 + 2 x2 x3 = 1
4 4 x1 0
= Solution: The augmented matrix of the given system of
3 3 x2 0 equations is
-4x1 + 4x2 = 0 and 3x1 - 3x2 = 0
2 1 3 9 1 3 2 0
x1 = x2 1 3 2 0 R
1
R2
2 1 3 9
eigen vector corresponding to l = 6 is
3 2 1 1 3 2 1 1
x x 1
x = 1 = 2 = x2 1 3 2 0
0 5 7 9
x
2 2x 1 R 2 2 R1 , R 3 3R1

Example 7: Find the eigen values of the matrix 0 11 5 1


6 2 2

A = 2 3 1 1 3 2 0
2 1 3 11
R3 - R2 0 5 7 9
5
52 104
Solution: Characteristic equation of the given matrix is 0 0
5 5
A lI = 0 From the above we get, x1 - 3x2 - 2x3 = 0 (1)
6l 2 2
5x2 + 7x3 = 9 (2)
2 3l 1 =0 x3 = 2 (3)
2 1 3l From (3) x3 = 2
From (2) 5x2 + 7x3 = 9 5x2 = -5 x2 = -1
l3 - 12l2 + 36l - 32 = 0 From (1) x1 - 3x2 - 2x3 = 0 x1 -3(-1) -2(2) = 0 x1 = 1
The required unique solution is x1 = 1, x2 = -1 and x3 = 2
l = 2, 2, 8
Example 10:Find whether the vectors given below are
Eigen values are 2, 2, 8. linearly dependent or independent {(1, 3, 2), (1, -4, 1),
3 1 2 (-1, 2, 5)}
Example 8: Find the rank of the matrix 2 0 1 Solution: Let x, y, z R such that x(1, 3, 2) + y(1, -4, 1)
1 4 1 + z(-1, 2, 5) = (0, 0, 0)
x +y z = 0
3 1 2 1 4 1
x 4 y + 2z = 0
3  (1)
olution: Given 2 0 1 R1 R3
S 2 0 1
2 x + y + 5z = 0

1 4 1 3 1 2
The above system of equations when expressed in deter-
minant form we have
R2 R2 R1 and R3 R3-3R1
1 1 1 1 1 1 1 1 1 1
R 2 3R1 , R 3 2 R1 R3 R 2
3 4 2 0 7 5 7
0 7 5
1 4 1 1 4 1 2 1 5 0 1 7 44
0 8 3 11 0 0
~ R3 R3 8 R2 0 8 3 7
0 11 5 18 Rank = 3 = number of unknowns
0 0
5 there exists a unique solution x = 0, y = 0 and z = 0
which is a row echelon form. The number of non zero x (1, 3, 2) + y (1, -4, 1) + z(-1, 2, 5) = (0, 0, 0) and
rows = 3. x = 0, y = 0, z = 0
The rank of the matrix = The number of non-zero rows The set of vectors are linearly independent.
in it = 3 Example 11: Show that the set of vectors {(2, 3, 9), (3, -2,
Rank of the matrix = 3. -6), (-1, 5, 15)} are linearly dependent

Chapter 01.indd 13 8/29/2015 9:01:14 AM


2.14|Engineering Mathematics

Solution: Let x, y, z R such that 1 0 0 0


where I = and O =
x (2, 3, 9) + y (3, -2, -6) + z (-1, 5, 15)
0 1
0 0
= (0, 0, 0)
A2 = 2I  (2)
2x + 3y z = 0
3x 2 y + 5 z = 0 Now, A16 = (A2)8 = (2I)8 (From (2))
9 x 6 y + 15 z = 0
= 28 I8 = 256 I
The above system when expressed in matrix form we
have the coefficient matrix 1 0
= 256
2 3 1 0 1
A = 3 2 5
9 6 15 256 0
A16 =
2 3 1 0 256
3 2 5 = 0
9 6 15 2 0 3
Example 13: If A = 0 4 5 ; then find the value of the

2 3 0 1 0
as R3 = 3R2 and 0
3 2 matrix polynomial
Rank of A = 2 < the number of variables which is 3. 3A9 - 18A8 + 39A7 - 32A6 + 12A5 - 26A4 + 16A3 + 24A2
The system will possess a non zero solution i.e. - 50A + 40I.
2x + 3y z = 0
Solution: The characteristic equation of
3x 2 y = 5 z = 0
x y z 2 0 3
= = = k (say )
15 2 3 10 4 9 A = 0 4 5
x = 13k, y = -13k and z = -13k 0 1 0
Let k = 1
x = 13, y = -13, z = -13 Is |A - l I| = 0
There exists a non zero solution such that x, y, z R
2 l 0 3
x (2. 3. 9) + y (3, -2, -6) + z (-1, 5, 15)
0 4 l 5 = 0
= (0, 0, 0)
0 1 l
The set of given vectors are linearly dependent.
4 2 (2 - l) {(4 - l)(- l) + 5} = 0 = (2 - d)
Example 12: If A = , then find A by using
16

7 4 {(l2 - 4l + 5)} = 0
Cayley-Hamilton theorem. 2l2 - 8l + 10 - l3 + 4l2 - 5l = 0.
-l3 + 6l2 - 13l + 10 = 0
Solution: The characteristic equation of
l3 - 6l2 + 13l - 10 = 0 (1)
4 2
A= is By Cayley-Hamilton theorem, the matrix A will satisfy
7 4 its characteristic equation (1)
|A - l I| = 0
A3 - 6A2 + 13A - 10I = O,
4l 2
=0 1 0 0
7 4 l
where I = 0 1 0 and
(4 - l) (-4 - l) + 14 = 0
0 0 1
-16 - 4l + 4l + l2 - 14 = 0
0 0 0
l2 - 2 = 0  (1)
O = 0 0 0
By Cayley-Hamilton theorem, the matrix A satisfies its 0 0 0
characteristic equation (1)
A2 - 2I = O A3 - 6A2 + 13A - 10I = 0  (2)

Chapter 01.indd 14 8/29/2015 9:01:19 AM


Chapter 1 Linear Algebra | 2.15

Now, consider the given matrix polynomial = 3A6 0 - 2A3 0 - 4 0 + 2A (From (2))
3A9 - 18A8 + 39A7 - 32A6 + 12A5 - 26A4 + 16A3 + = 2A
24A2 - 50A + 40I
= 3A9 - 18A8 + 39A7 - 30A6 - 2A6 + 12A5 - 26A4 + 2 0 3 4 0 6
20A3 - 4A3 + 24A2 - 52A + 2A + 40I = 2 0 4 5 = 0 8 10
= 3A6 (A3 - 6A2 + 13A - 10I) - 2A3 (A3 - 6A2 + 13A 0 1 0 0 2 0
- 10I) - 4(A3 - 6A2 + 13A) + 2A

Exercises
Practice Problems 1 a11 A11 + a21 A12 + a23 A32 =
(A)
Directions for questions 1 to 65:Select the correct alternative a11 A11 + a12 A12 + a13 A13 =
(B)
from the given choices. a21 A12 + a23 A32 + a12 A21 =
(C)
a12 A21 + a21 A12 + a31 A13 =
(D)
1. Which of the following is false?
(A) Every diagonal matrix is a square matrix. 2 3 -3
(B) Every unit matrix is a scalar matrix.
7. The determinant value of 1 -2 2 is
(C) Every square matrix is a diagonal matrix.
7 4 -4
(D) Every scalar matrix is a diagonal matrix.
2. If the trace of the matrix (A) 0 (B) 10 (C) -10 (D) 15
1 a12 .... a1 n
n! ( n + 1)! ( n + 2)!
a21 2 .... a2 n is 55, then the value of n is 8. The value of ( n + 1)! ( n + 2)! ( n + 3)! is
.... .... .... ....
( n + 2)! ( n + 3)! ( n + 4)!
an1 an 2 .... n
(A) 10 (A) 2n! (n + 1)!
(B) 11 (B) 2n! (n + 1)! (n + 2)!
(C) 9 (C) (2n)! (n + 1)! (n + 2)!
(D) Cannot be determined (D) 2n! (n + 3)!
3. Which of the following statement is/are false? x +1
x
C0 x
C1 C1
(A)  AT. BT always defined for square matrices of same order.
(B) AT. B is defined for matrices of the same order. 9. If f(x) = 2 x C1 2 C2 x
2 ( x +1)C2 , then f (200) is
(C)  tr(AT) + tr(BT) is always defined for matrices A, B of same 6 x C2 6 x C3 6 ( x +1)C3
order.
(D)  AT + BT is always defined for matrices A, B of same (A) 200 (B) -200 (C) 0 (D) 2001
order.
4. Consider the following statements about two square matrices 2 3+ i 1
A and B of the same order: 10. The determinant 3 i 0 1 + i is
P: (A + B)2 = A2 + 2AB + B2 1 1 i 1

Q: (A + B) (A - B) = A2 - B2 Then,
(A) purely imaginary (B) zero
(A) both P and Q are true.
(C) real (D) 10
(B) both P and Q are false
(C) both P and Q are true if A and B commute 11. The value of
(D) P is true but Q is false.
1 0 0 0 0 .... .... 0
2 1 2 -2 x 3 x 2! 1 0 0 0 .... .... 0

5. If 1 0 1 x -2 0 = I3 3, then x = 3! 2! 1 0 0 .... .... 0
2 2 1 2 -2 x - x 4! 3! 2! 1 0 .... ..... 0 is

-1
(A) (B) 1 (C) 1/2 (D) 2 .... .... .... .... .... .... .... ....
.... .... .... .... .... .... .... ....
a11 a12 a13
( n 1)! ( n 2)! .... .... .... .... 1 0
6. If = a21 a22 a23 , then which of the following is true?
n! ( n 1)! .... .... .... .... 2 1 nn
a31 a32 a33
(Here, Aij is the cofactor of the element aij) (A)
n! (B)
n (C) 0 (D) 1

Chapter 01.indd 15 8/29/2015 9:01:22 AM


2.16|Engineering Mathematics

x y z II. If a non-singular matrix A is symmetric, then A1 is also


symmetric.
12. If A = 2 x y 3 z , then |A| =
x/2 Which of the following is correct?
y / 2 z / 2
(A) Both I and II true.
(A) 10xyz (B) Both I and II false.
(B) 1 (C) I is true, II is false
(C) 0 (D) I is false, II is true
(D) 1/2(x3 + y3 + z3 - 3xyz) 20. If A is a matrix of order n n, then which of the following is
13. If the elements of a row or column of a given square matrix false?
is multiplied by 2, then the value of determinant is _______ (A)(AT)T = A
times the original determinant. (B)adj(adjA) = A (when A is unimodular)
(A) 1/2 (B) 1 (C) 2 (D) 4 (C) det (KA) = Kn-1 det A
|adj (adj A)| = (det A)( n1)
2
(D)
14. If A is a square matrix of order k and det(KA) = 27 det(A),
then k = 21. A is a third order matrix. If the value of the square of the
(A) 9 (B) 1 (C) 2 (D) 3 determinant of the matrix of co-factors of A is 28561, then |A|
equals
15. If A and B are two square matrices of order 4 such that |A| = -2 (A) 25 (B) 13 (C) 120 (D) 169
and |B| = 5, then |4AB| is
(A) -80 (B) -160 22. If A is a square matrix of order 3, then the product of A and its
(C) -2560 (D) -256 transpose is
(A) unit matrix (B) zero matrix
16. I. (a b), (b c), (c a) are factors of the determinant (C) identity matrix (D) symmetric matrix
1 1 1 23. If A and B are two skew-symmetric matrices of the same
a b c . order, then AB is skew-symmetric if and only if
(A) AB + BA = O (B) AB - BA = O
a2 b2 c2
(C) AB + BA = I (D) AB - BA = I
II. If the elements of a determinant are functions of x and its 2 1+ i 3
two rows or columns become identical (i.e. determinant
24. The matrix 1 - i 6 i is
equals zero,) when we substitute x = k, then (x k) is a
3 -i 4
factor of the determinant.
Which of the following is correct? (A) symmetric.
(A) Both I and II true. (B) skew-symmetric.
(B) Both I and II false. (C) hermition matrix.
(C) I is true, II is false (D) skew-hermition matrix.
(D) I is false, II is true
25. Which of the following matrices do not satisfy the relation
17. A lower triangular matrix A = (aij)n n is singular if and only if A2 = I?
(A) aii = 0 for all i = 1, 2, n (A) Involutory matrix
(B) aii = 0 for atleast one i = 1, 2, n (B) Identity matrix
(C) aii 0 for all i = 1, 2, n (C) Idempotent matrix
(D) aii 0 for atleast one i, i = 1, 2, n (D) None of these
2 -1 0 1 -2 -3

18. Inverse of the matrix 1 2 3 is 26. The matrix -1 2 3 is
-4 1 -1 1 -2 -3

-5 -11 9 -5 -1 -3 (A) idempotent. (B) nilpotent.
(C) involuntary. (D) orthogonal.
(A)
-1 -2 2 -11 -2 -6
(B)
-3 -6 5 9 2 5
1 2 3
27. Rank of the matrix A = is
5 11 9 5 1 3 4 5 6

(C)
1 2 -2 (D)
11 2 6
(A) 1 (B) 2 (C) 3 (D) 4
3 6 -5 9 -2 -5
2 -1 -3

28. The rank of the matrix -4 2 6 is
1 2 3 4 5
2 -10 5 15
3 4 5 6
19. I. If A = 3 4 5 6
1
7 , then A is symmetric. (A) 0 (B) 1 (C) 2 (D) 3

4 5 6 7 8 29. If A = (aij)m n and aij = 3 for all i, j, then r(A) is
5 6 7 8 9
(A) 0 (B) 1 (C) 2 (D) 3

Chapter 01.indd 16 8/29/2015 9:01:27 AM


Chapter 1 Linear Algebra | 2.17

1 40. If the number of variables in the linear homogeneous system


AX = O is n, then the system will have exactly one solution
30. If A = (1 2 3) and B = 2 , then r(AB) is
3 X = O, if the rank of the matrix A is

(A) 1 n (D)
(B) < n (C) n
(A) 0 (B) 1 (C) 2 (D) 4
31. Which of the following matrix is row echelon form? 41. If the equations 2x - y - z = 0, kx - 3y + 2z = 0 and -3x + 2y + kz
= 0 have a non-zero solution, then the value of k is
1 0 -1 2 (A) 2 (B) 1
0 0 1 2

(A) 1 0 3 (B)
1 0 -1 (C) 7 (D) Both 1 and 7
0 0 1 -2
0 1 0 42. The system of equations x - ky - z = 0, kx - y - z = 0 and
0 0 0 0 x + y - z = 0 has non trivial solutions. Then, the possible
values of k are
1 0 0 0 0 0 1
0 (A) 1 (B) 2 (C) 0 (D) 4
0 3 0 0

(C)
2 1

(D) 0
1 43. The system of equations a + 3y + 5z = 0, 2x - 4ay + az = 0,
0 0 1 4 0 1 0
0 -4x + 18y + 7z = 0 has only trivial solution if a is

(A) -1 or 3
0 0 0 0 1 0 0
2
(B) 1 or 3
32. Which of the following set of vectors are linearly dependent?
(C) not equal to 1, 3
(A) (2, 3, 3), (3, -1, 3), (4, -2, 5)
(D) not equal to -1 and 3
(B) (3, 4, -1), (-1, 3, 1), (-2, -7, -2)
(C) (2, 1, 4), (1, -2, 2), (-3, 1, -6) Linked answer for questions 44 to 46: Suppose x is the weight in
(D) (1, 3, -5), (-5, -1, 3), (4, -2, -2) kg, y is the height in cm and z is the waist measurement in cm. A
33. For system of m linear equations with n unknowns the dietician wants to study the relationship between x, y and z assuming
Crammers rule is applicable only when that there is a linear relationship between them. From the data available
(A) m=n in 3 days, he obtains the following equations satisfied by x, y, and z
(B) mn 2x + py + 6z = 8
(C) m = n and the coefficient matrix is singular. x + 2y + qz = 5
(D) m = n and the coefficient matrix is non singular. x + y + 3z = 4
34. The system of equations 2x - y + 3z = 9, x + y + z = 0 and where p and q are some constants.
x - y + z = 0 has/is 44. The values of p and q for which the above equations have a
(A) unique solution. unique solution are
(B) infinite solutions. (A) p = 2 ; q = 3 (B) p = 2 ; q 3
(C) only zero solution. (C) p 2; q = 3 (D) p 2 ; q 3
(D) inconsistent.
45. The system does not have a solution if
35. The system of equations 6x + 7y + 8z = 1, 13x + 14y + 15z = 2 (A) p = 1 (B) q = 3
and x + 2y + 3z = 2 is (C) p 2 , q = 3 (D) None of these
(A) consistent with unique solution. 46. The system will have infinite number of solutions if
(B) consistent with infinite solutions. (A) p = 2 (B) q = 3
(C) inconsistent. (C) p 2; q 3 (D) p 2; q = 3
(D) None of these
47. Solve the given system of equations by Gauss elimination
36. If the system of equations 2y + z = 2, x + ky + 3z = 1 and x - z = -1 method.
has a unique solution, then the value of K is 2 x + y - z = 4;
(A) equal to 8 (B) not equal to 8 3x - 2 y + 4 z = 8
(C) not equal to 7 (D) equal to 7
x - 3 y + 2 z = 1,
37. If the system of equations 4x + 2y - 3z = 7, x + 3y + kz = 4 and (A) (2, -1, 2) (B) (-1, 2, -2)
x + y - z = 5 have no solution, then the value of k is (C) (2, 1, 1) (D) (-1, 1, 2)
(A) -2 (B) -3 (C) 2 (D) 3
38. The value of l for which the following system of equation 2 -1 0
does not have a solution is 48. The eigen values of 0
1 -1 is

x+y+z=6 0 0 3

4x + 3ly - 3lz = 0 (A) 0, 0, 0 (B) 0, 1, 0
3x + 2y - 4z = -8 (C) 2, 1, 3 (D) -2, -1, -3
(A) 3 (B) -3 (C) 0 (D) 1
1 2 3
39. If the system 2x 3y + z = p, x + 2y + 3z = q, 5x 8y z = r 49. If A = 4 5 6 , then characteristic root of A is
has a solution, then
7 8 9
(A) p, q, r are in AP (B) p, q, r are in GP
(C) p, q, r are in HP (D) q, p, r are in AP (A) 3 (B) 2 (C) 1 (D) 0

Chapter 01.indd 17 8/29/2015 9:01:30 AM


2.18|Engineering Mathematics

2 3 4 0 2 4 -6
(A) 4 -5 (B)
50. The minimum eigen value of the matrix 3 5 -6 is
7 -9


1 1 14
-2 6 2 6
(C) 7 -3 (D)
-4 3
(A) 0 (B) 1 (C) 2 (D) 3
51. The characteristic roots of the inverse of the matrix Common data for questions 59 and 60:
2 2 1
2 5 4
1 3 1 are Consider the matrix A = 0 1 0 let B = A-1
1 2 2
0 -3 -2
(A) -1, -1, 5 (B) 1, 1, 5
(C) 1, 1, 1/5 (D) -1, -1, 1/5 59. B = ____________

52. The sum and product of the eigen values of the matrix -1 1 2
(A) [A2 - A - 4I] (B) [A - A - 4I]
2 0 -1 4 4
0
4 -2 are respectively 1
(C) [A2 + A - 4I] (D)
-1 2
[A - A + 4I]
1 3 -5 4 4
60. B2 = ____________
(A) 0, 24 (B) 1, -24
(C) 2, 20 (D) 4, -24 1 -1
(A) [A + I - 4B] (B) [A - I + 4B]
4 4
2 0 1
53. The eigen values of a matrix A = 0 2 p are 1, 2, and 3. -1 1
(C) [A - I - 4B] (D) [A - I - 4B]
1 0 q 4 4

Then, the values of p and q are _____ 2 3


61. If A = , then A =
15

(A) p = 0, q = 0 4 6
(B) p = any real number, q = 2
(C) p = 2, q = 0 (A) 814A (B) 815A (C) 816A (D) 15A
(D) p = 2, q = 2 2 0 0
0 -1 2 -3
1 0 4 6 62. If A = 3 6 7 , then the value of the matrix polynomial

54. The eigen values of the matrix is
9 0 1
-2 -4 0 5

3 -6 -5 0 2A10 - 18A9 + 40A8 - 25A7 + 9A6 - 20A5 + 13A4 - 9A3 + 20A2
- 10A is ______
(A) Real only (B) Imaginary
(C) Zero only (D) Imaginary or zero 2 0 0 4 0 0
(A) 3 6 7 (B)
6 12 14
5 2
55. The number of linearly independent eigen vectors of 9 0 1 18 0 2
is _____ -2 1
(A) 0 (B) 1 1 0 0 0 0 0
(C) 2 (D) infinite (C) 0 1 0 (D)
0 0 0

0 0 1 0 0 0
56. Which of the following is an eigen vector for the matrix
1 4 3 -1 -1
?
2 -1 63. For the matrix A = -1 3 -1 , consider the following
1 -1 3 2 statements 1 1 -1
(A)
(B) 1 (D)
(C) 2
3
1 P: The characteristic equation of A is l3 - 5l2 + 4l = 0
6 -6 2 -2 Q: A-1 exists
R: The matrix A is diagonalisable
57. For a matrix A = -6 5 -4 , x = 2 is an eigen vector.
2 -4 1 -1 Which of the above statements are TRUE?
(A) P, Q and R (B) P and R but not Q
The corresponding eigen value is _______ (C) P and Q but not R (D) Q and R but not P
-2
(A) (B) 1 (C) 2 (D) 13 64. If P is a modal matrix and D is a spectral matrix of a diago-
58. Let A be a 2 2 square matrix with l1 = -2 and l2 = -3 as its nalizable matrix A, then which of the following relations is
-4 6 NOT TRUE among A, P and D?
eigen values and x1 = , x2 = as its eigen vectors, then (A) PD = AP (B) DP-1 = P-1A
-4 7
A is given by (C) A P = PD (D)
2 2
DP = PA

Chapter 01.indd 18 8/29/2015 9:01:40 AM


Chapter 1 Linear Algebra | 2.19

65. If A is a 3 3 square matrix with eigen values 0, 2, 3 with P (A) 0, 2, 3 (B) 0, 4, 6


as its modal matrix, then the eigen values of the matrix P-1 AP 1 1 1 1
(C) 0, , (D) 1, ,
are _______ 2 3 2 3

Practice Problems 2 (C) The determinant of a square matrix is always defined


Directions for questions 1 to 70: Select the correct alternative 4 4
from the given choices. (D) 2
3 2
1. Which of the following is false?
(A) comparable matrices are always equal a11 a12 a13
(B) unequal matrices may be comparable 8. If = a21 a22 a23 , then which of the following is true?
(C) equal matrices are always comparable
(D) uncomparable matrices cannot be equal a31 a32 a33

2. Which of the following statements is false? (Aij is cofactor of aij)


(A)  A + B is always defined for matrices A, B of same order a11A11 + a12A21 + a13A31 =
(A)
(B)  A, B are always defined for square matrices A, B of a21A21 + a22A22 + a23A32 =
(B)
same order a13A13 + a23A23 + a33A33 =
(C)
(C) A . B is defined for matrices of same order a31A32 + a32A33 + a33A31 =
(D)
(D) tr(A) + tr(B) is always defined for matrices A, B of
same order a11 a12 a13
a 0 0 9. If = a21 a22 a23 , then which of the following is false?
3. If A = 0 b 0 , then A10 is a31 a32 a33

0 0 c a13A13 + a23A23 + a33A33 = 0
(A)
a12A11 + a22A21 + a32A31 = 0
(B)
(A) 0 (B)(abc) 10
a21A31 + a22A32 + a23A33 = 0
(C)
a10 0 0 a10 b c a21A11 + a22A12 + a23A13 = 0
(D)

(D)
(C)
0 b10 0 c b
10
a
a11 a12 a13
0 0 10
c a b c
10
10. If = a21 a22 a23 , then which of the following is false?
4. If A and B are diagonal matrices of order n with diago- a31 a32 a33
nal elements a1, a2, , an and b1, b2, , bn, then for [Here, Aij is the cofactor of the element aij]
k 1, (A + B)k is a
a11 A11 + a21 A21 + a31 A31 =
(A)
(A) diagonal matrix with diagonal elements (a1 + b1)k, (a2 + b2)k,
a12 A21 + a23 A32 + a13 A31 = 0
(B)
.. (an + bn)k
a21 A21 + a22 A22 + a23 A23 = 0
(C)
(B) diagonal matrix with diagonal elements (a1 b1)k, (a2 b2)k, a31 A13 + a32 A23 + a33 A33 = 0
(D)
.. (an bn)k
(C) diagonal matrix with diagonal elements (a1k + b1k), .. 2 0 -1 4
(ank + bnk)
0 -1 3 -2
(D) None of these. 11. The value of is
0 0 3 1
5. Let A and B be two matrices of order n with trace (A) = 10 and 0 0 0 -2
trace (B) = 2. Then, trace (AB) is
(A) 12 (A) 10 (B) -12 (C) 12 (D)0
(B) 8
(C) 20 x
Cx -r y
Cy-r z
Cz -r
(D) Cannot be determined with the given data
12. If D = x C y
C y - r -1 z
C z - r -1 and
1 x - r -1
1 0 2

x
Cx - r - 2 y
Cy -r -2 z
Cz - r - 2
6. The trace of the matrix 3 -1 4 is
0 2 5 ( x +1) ( x + 2)
x
Cr C r +1 Cr + 2
(A) 5 (B) 7 (C) -5 (D)1 D 2 = Cr
y ( y +1)
C r +1 ( y + 2)
Cr + 2 , then 1 : 2 is
( z +1) ( z + 2)
z
Cr C r +1 Cr + 2
7. Which of the following statement is false?
(A) If A is not a square matrix, then |A| cannot be defined (A) 1 : 1 (B) 2 : 1
(B) The value of a determinant can never be negative (C) 1 : 2 (D) None of these

Chapter 01.indd 19 8/29/2015 9:01:45 AM


2.20|Engineering Mathematics

m m m (A) both P and Q are true


13. The value of the determinant (B) both P and Q are false
n
C1 n +1
C1 n+ 2
C1 is equal to
n +1 n+ 2
(C) P is true while Q is not
n
C2 C2 C2 (D) Q is true while P is not
(A) m (B) mn 22. The trace of a skew-symmetric matrix is
(C) 0 (D) n (n - 1) (A) 1 (B) 1
1 2 3 (C) 0 (D) depending on its order.
1 1 1 1
14. If A = 1 2 4 , then A201 - A200 is

1 1 5 1 1 -1 1 -1
23. If A = , then A is
2 1 1 -1 -1
(A) 0 (B) 1 (C) 201 (D) 200
-1 -1 1 1
15. If the elements of a row or column are divided by 1/4, then
the value of the new determinant is ______ times the original (A) orthogonal (B) nilpotent
determinant. (C) symmetric (D) none of these
(A) 4 (B) 1/4 (C) 1/2 (D) 1 24. If A is a involutory matrix, then which of the following is
16. If A is a square matrix of order 5, then |5A| is equal to true?
(A) A need not be a square matrix
A
(A) 25 |A| (B) 5|A| (C) 55|A| (D) (B) A = I
5 (C) A = O
17. If A is a nonsingular matrix of order n, then |adj (D) A = A
(adjA)| is 1 -3 -4

|A|2 (B)
(A) |A|4 25. The matrix -1 3 4 is
1 -3 -4
(C) A
n 21
(D)
A
( n 2 n + 1) 2

(A) idempotent (B) involuntory
1 3 -2 (C) orthogonal (D) nilpotent
18. The inverse of -3 0 -5 is
26. If A is an involutory orthogonal matrix, then it is
2 5 0
(A) symmetric (B) skew-symmetric
25 -10 -15 (C) null (D) unit
(A) 1
-10 4 11 5+ 2i 4i 5- 2i
5
-15 1 9 27. If z = i - 8 -i -i - 8 , then z is a
25 -10 -15 1+ i 3 i 1- i
(B) 1 (A) real number
-10 4 11
25 (B) pure imaginary number
-15 1 9
(C) complex number with positive imaginary part
1 -2 -3 (D) complex number with negative imaginary part

-2 4 / 5 11/ 5
(C) 4 1- i 7
-3 1/ 5 9 / 5
28. The matrix 1 + i 6 -i is
7 i 5
1 -2 -3
(D) -2 4 / 5 11/ 5
1 (A) symmetric
5 (B) skew-hermitian
-3 1/ 5 9 / 5 (C) hermition but not symmetric
0 6 8 (D) unitary
19. The matrix -6 0 5 is a + 3i b - 2i
2
-8 -5 0
29. a - 3i 4

c - 8i is a
(A) skew-symmetric (B) symmetric b - 2i c + 8i 6

(C) hermitian (D) orthogonal
(A) skew-symmetric matrix
20. If A is a square matrix, then A AT is
(B) symmetric matrix
(A) symmetric (B) skew-symmetric
(C) hermitian matrix
(C) singular (D) null
(D) skew-hermitian matrix
21. Consider the two statements
1 6 7 -1
P: If A is a skew-symmetric matrix, then trace (A) is zero. 30. The rank of the matrix A = is
Q: If trace (A) is zero, then A is a skew-symmetric matrix, 5 8 3 4
then (A) 1 (B) 2 (C) 3 (D) 4

Chapter 01.indd 20 8/29/2015 9:01:51 AM


Chapter 1 Linear Algebra | 2.21

1 3 3 k1 -3 and k2 is any real number


(A)
k1 = -3 and k2 is any real number
(B)
31. The rank of 4 5 6 is
7 9 9 k1 -3 and k2 = 5
(C)
k1 = -3 and k2 = 5
(D)
(A) 2 (B) 3 (C) 1 (D) 0
I 0 Linked answer for questions 42 and 43: Suppose x is the
32. If a matrix A can be reduced to the normal form 6 , weight in kg, y is the height in cm and z is the waist measurement
then rank of A is 0 0
in cm. A dietician wants to study the relationship between x, y and
(A) 3 (B) 12 (C) 6 (D) 0 z assuming that there is a linear relationship between them. From
the data available in 3 days, he obtains the following equations
33. The system of linear equations is said to be consistent, if
satisfied by x, y, and z
there exists
(A) at least one solution. (B) no solution. 2x + py + 6z = 8
(C) exactly one solution. (D) infinite solutions. x + 2y + qz = 5
34. If the number of variables in the non homogeneous system x + y + 3z = 4
AX = B is n, then the system is consistent if [r(A) is a rank of where p and q are some constants.
coefficient matrix and r(AB) is Rank of augmented matrix]
42. For p = 1, q = 1, the ratio x : y : z equals.
(A) r(A) = r[A B] (B) r(A) < r[A B]
(A) 11 : 0 : 1 (B) 3 : 5 : 2
(C) r(A) > r[A B] (D) All the above
(C) 1 : 1 : 1 (D) 2 : 5 : 1
35. If the number of variables in the non-homogeneous system 43. Suppose he had wrongly recorded the third equation, which
AX = B is n, then the system is in consistent if should be x + 3y + 3z + 4 = 0, the values of p and q giving
(A) r(A) = r[A : B] (B) r(A) < r[A : B] unique solution for the original system is
(C) r(A) > r[A : B] (D) All the above (A) p = 4; q = 3 (B) p = 6; q = 4
x + 2 y + 3z = 2 (C) p 6; q 3 (D) p = 6; q = 3
36. The system of equation 6 x + 7 y + 8 z = 1 are 44. If the number of variables in the linear homogeneous system
13 x + 14 y + 15 z = 2 AX = 0 is n, then the system will have exactly one solution
(A) inconsistent X = 0 if
(B) consistent with unique solution (A) r(A) = 1 (B) r(A) < n
(C) consistent with infinite solutions (C) r(A) n (D) r(A) = n
(D) None of these
45. The system of equations 2x y + 3z = 0, x + y + z = 0, x y +
37. The equations x + 4y + 8z =16, 3x + 2y + 4z = 12 and z = 0 is
4x + y + 2z = 10 have (A) homogeneous and admits no solution
(A) only one solution (B) homogeneous and admits infinite solution
(B) two solutions (C) homogeneous and admits zero solution only
(C) infinitely many solutions (D) None of these
(D) no solution
2x + z = 2 46. Find the value of k for which of the following set of equa-
tions have non-zero solutions:
38. If the following system of equations, kx + y + 3 z = 1, has a
4x 2y 2z = 0
unique solution, then x - y = -1
kx 6y + 4z = 0
k = 5
(A) (B) k 5 (C) k 1 (D) k = 1 6x + 4y + kz = 0
39. The values of a and b for which the following set of equa- (A) 5 (B) 2
2 x + ay + 5 z = 3 (C) 14 (D) Either 2 or 14
47. The general value of x2 + y2 + z2 where x, y, z represent a non
tions have infinite number of solutions 3 x + y + 2 z = 4
zero solution of x + 2y 3z = 0, 2x 3y + z = 0, 3x y 2z = 0
x - 7 y + 8 z = b
is (where l is an arbitrary constant)
a = 3, b = 2
(A) (B) a = 3, b = 2 (A) 3 (B) 3l (C) 9l2 (D) 3l2
a = 2, b = 3 (D)
(C) a = 2, b = 3
48. The value of 4(l1 + l2 + l3) where l1, l2, l3 are the values
40. If no two of a, b, c are equal and
of l for which the system of equations (3 l)x + y + 4z = 0;
ax + a2y + (a3 + 1) z = 0 (2 l)y + 6z = 0; (5 l)z = 0 has non trivial solutions, is
bx + b2y + (b3 + 1) z = 0 (A) 40 (B) 10 (C) 4 (D) 400
cx + c2y + (c3 + 1) z = 0, has non-trivial solution, then
49. Which of the following is the characteristic equation of
(A) abc = 1 (B) abc = 1
a 0 0 0
(C) abc = 0 (D) a = 1, b = 2, c = 3
0 a 0 0 ?
41. Find the values of k1 and k2 for which the non homogeneous 0 0 a 0
linear system, 3x - 2y + z = k2; 5x - 8y + 9z = 3; 2x + y +
0 0 0 a
k1z = -1 has unique solution.

Chapter 01.indd 21 8/29/2015 9:01:53 AM


2.22|Engineering Mathematics

4 56. The characteristic values of a skew-hermitian matrix are


( -1) .4 Ck .ak - 4 .l k = 0
k
(A)
k =1
(A) all real.
4
(B) all purely imaginary.
(B) 4 Ck . a k - 4 . l k = 0 (C) some real and some imaginary.
k =0 (D) purely imaginary or zero.
4

( -1) .4 Ck .a4 - k .l k = 0
k
(C) 57. Which of the following is an eigen vectors for the matrix
k =1 3 1

6 2
4

( -1) .4 Ck .a4 - k .l k = 0
k
(D)
k =0
1 2
1 0 0 (A)
(B)

2 1
50. The eigen values of 2 1 0 is
3 2 0 1
(C)
(D) Both (A) & (C)
3
(A) 0, 0, 0 (B) 2, 0, 0 (C) 1, 1, 0 (D) 3, 1, 0
58. The eigen values and their corresponding eigen vectors of a
51. The characteristic values of the upper triangular matrix
2 2 square matrix P are given below
5 4 -7
Eigen values Eigen vectors
A = 0 -3 1 are
0 0 6 -1
l1 = 1 x1 =
1
(A) 2, 5, 1 (B) 5, 3, 6
(C) 5, 4, 1 (D) 7, 3, 0 3
l2 = -4 x2 =
3 -2 -8 -8

52. For the matrix 0 3 4 , Then, the matrix P is given by
0 0 7
-5 6
-1 3
(A) all the characteristic values are real and negative 5 4 (B)
(A) -1 2
(B) one characteristic value is negative and the other two are
complex 4 3 3 4
(C) one characteristic value is positive and the other two (C)
-8 -7 (D)
5 -6
characteristic values are complex
(D) all the characteristic values are real and positive
1 3 2
6 2 2
59. If A = 3 2 1 , then
53. The characteristic roots of the inverse of 2 3 1 is 2 1 3
2 1 3
(A) A3 + 6A2 + 3A 18I = 0

(A) 1/2, 1/2, 1/8 (B) 1/2, 1/3, 1/8 (B) A3 6A2 3A + 18I = 0
(C) 1/4, 1/2, 1/8 (D) 1/2, 1/2, 1/8 (C) A3 6A2 3A - 18I = 0
A3 6A2 + 3A 18I = 0
(D)
33 00 00
0 0 1
54. The eigen values of the inverse of the matrix - 1 -2 00
-1 -2 60. If A = 0 1 0 , then A4 - A3 - A2 =
is 4 -1 11
1 0 0
4 -1 22

1 1 1 1 - 2A
(A) - A
(B) 2A (C) (D) A
(A) 3, -2, (B) , ,
2 3 2 2
1 1 -1 61. If A is a nonsingular matrix satisfying A3 - 7A 6I = 0, then
(C) , 2, -2 (D) , ,2 A-1 is given by
3 3 2
1
55. If the characteristic values of A =
3 -1 (A) ( A2 + 7 I ) (B)
A2 7I
are l1 and l2 and 6
1 2 5 6
that of B = are 1 and 2, the equation whose roots 1
-1 5 (C) ( A2 + 7 I ) (D)
A2 + 7I
6
1 1 1 1
are + and + is
l1 l2 m1 m 2
5 8
62. If A = , then A =
17
(A) 201x2 161x + 54 = 0 -2 -3
(B) 161x2 201x + 54 = 0
(C) 201x2 + 161x 54 = 0 (A) 16A - 17I (B) 16A - 15I
(D) 161x2 + 201x 54 = 0 (C) 17A + 16I (D) 17A - 16I

Chapter 01.indd 22 8/29/2015 9:02:03 AM


Chapter 1 Linear Algebra | 2.23

Linked answer for questions 63 and 64: x 2


66. For the matrix A = the sum of the eigen values is 9
The characteristic equation of a matrix y 5
and product of the eigen values is 0. Then, the values of x and
0 1 -1
y are ________
A = 1 0 1 is l3 + l - 2 = 0. Let B = A-1
(A) x = 9, y = 0 (B) x = 4, y = 0
1 -1 0 (C) x = 3, y = 0 (D) x = 4, y = 10
63. The matrix B is given by _____ 11 1
67. The trace and determinant of 3 3 matrix are and
5 5
1 1 -1 -1 1 1 respectively. The eigen values of that matrix are _______
1
(A) -1 1 1 (B) 1 -1 1
1
2 2 3 -2 7 1
1 -1 1 1 1 -1 (A) 2, , (B) 2, ,
5 5 5 7
1 -1 -1 1 1 1 1 5 6 2
1 1
(C) 1 1 1 (D) 1 1 -1 (C) 1, 1, (D) , ,
2 2 5 3 5 5
1 -1 1 -1 1 -1
68. One eigen value of the matrix
64. B = _____
2
1 6 5

3 1 -1 2 4 1 A = 1 -1 5 is 13.775 one of the other eigen values is
1 1
(A) 1 3 1 (B) 1 3 -1 2 7 10
4 4
1 -1 1 1 -1 3
(A) 10 (B) -13.775
3 1 (C) -3.775 (D) -16.775
1 1 4 1
1
1 1 (D)
1
(C) 3 3 1 -1
4 4 Linked answer for questions 69 to 70: Find the values of k1 and
1 1 1 1 -1 -1
k2 for which the non homogeneous linear system, 3x - 2y + z = k2;
5x - 8y + 9z = 3; 2x + y + k1z = -1 has
2 3
65. If P = , then the value of the matrix polynomial 69. No solution
1 5
(A) k1 = -3, k2 = 1/3
2P9 - 14P8 + 14P7 - 3P6 + 21P5 - 22P4 - 7P3 + 11P2 + 3P - 2I
(B) k1 = 3, k2 1/3
1 0 (C) k1 = -3 and k2 1/3
where I = is _____
0 1 (D) k1 = 3 and k2 = 1/3

31 43 32 93 70. Infinite number of solutions.


(A) 32 95 (B)
31 125
(A) k1 = 3, k2 = -1/3
(B) k1 = -3, k2 = 1/3
27 95 37 93 (C) k1 = -3 and k2 = 1/3
(C) 39 125 (D)
35 129
(D) k1 = 3 and k2 = 1/3.

Previous Y
ears Questions
1 3 0 -1 -2
(A)
(B)

1. Given that the determinant of the matrix 2 6 4 is -12, 1
9
-1 0 2 1
2
(D)
(C)
2 6 0 -1
1
the determinant of the matrix 4 12 8 is [2014] 3. Consider a 3 3 real symmetric matrix S such that two of its
-2 0 4 eigen values are a 0, b 0 with respective eigen vectors
x1 y 1
(A) -96 (B) -24
(C) 24 (D) 96 x 2 , y 2 If a b, then x1y1 + x2y2 + x3y3 equals  [2014]
x y
-5 2 3 3
2. One of the eigenvectors of the matrix is [2014]
-9 6 (A) a (B) b (C) ab (D) 0

Chapter 01.indd 23 8/29/2015 9:57:42 AM


2.24|Engineering Mathematics

4. Which one of the following equations is a correct identity (A) Any real number
for arbitrary 3 3 real matrices P, Q and R? [2014] (B) 0
(A) P(Q + R) = PQ + RP (C) 1
(B) (P - Q)2 = P2 - 2PQ + Q2 (D) There is no such value
(C) det(P + Q) = det P + det Q 1 2
(D) (P + Q)2 = P2 + PQ + QP + Q2 12. The eigenvectors of the matrix are written in the form
1 1 0 2
a and b . What is a + b?
5. The eigen values of a symmetric matrix are all [2013] [2008]
(A) complex with non-zero positive imaginary part.
(B) complex with non-zero negative imaginary part. 1
(A) 0 (B) (C) 1 (D) 2
(C) real. 2
(D) pure imaginary. 13. If a square matrix A is real and symmetric, then the eigen
5 3 values [2007]
6. For the matrix A = , ONE of the normalised eigen (A) are always real
1 3 (B) are always real and positive
vectors is given as[2012]
(C) are always real and non-negative
1 1 (D) occur in complex conjugate pairs
2 2

(A) (B) 14. The number of linearly independent eigen vectors of
3 -1
2 1
2 2 [2007]
0 2 is
3 1
(A) 0 (B) 1
10 5
(D)
(C) (C) 2 (D) Infinite
-1 2

10 5 3 2
15. Eigen values of a matrix S = are 5 and 1. What are
7. Eigenvalues of a real symmetric matrix are always. [2011] 2 3
(A) positive (B) negative the eigen values of matrix S2 = SS? [2006]
(C) real (D) complex (A) 1 and 25 (B) 6 and 4
2 2 (C) 5 and 1 (D) 2 and 10
8. One of the eigen vectors of the matrix A = is
1 3 16. Match the items in columns I and II [2006]
 [2010]
Columns I Columns II
2 2 4 1
(A)
(B)
(C)
(D)
P. Singular matrix 1. Determinant is not defined
-1 1
1 -1
Q. Non-square matrix 2. Determinant is always one
3 4 R. Real symmetric matrix 3. Determinant is zero
5 5
9. For a matrix [M] = , the transpose of the matrix is S. Orthogonal matrix 4. Eigen values are always real
x 3
5. Eigen values of not defined
5
equal to the inverse of the matrix [M]T = [M]-1. The value of (A) P - 3 Q - 1 R - 4 S-2
x is given by [2009] (B) P - 2 Q - 3 R - 4 S-1
(C) P - 3 Q - 2 R - 5 S-4
4 3
- (B)
(A) - (D) P - 3 Q - 4 R - 2 S-1
5 5
17. Multiplication of matrices E and F is G. matrices E and G
3 4
(C) (D) are
5 5
cos q - sin q 0 1 0 0
1 2 4
E = sin q cos q 0 and G = 0 1 0 . What is the
10. The matrix 3 0 6 has one eigen value equal to 3. The 0 0 1 0 0 1
1 1 P 
matrix F? [2006]
sum of the other two eigen values is. [2008]
(A) P (B) P-1 cos q - sin q 0 sin q cos q 0
(C) P - 2 (D) P - 3 (A) sin q cos q - cos q sin q 0
0 (B)

11. For what value of a, if any, will the following system of 0 0 1 0 0 1
equations in x, y and z have a solution?
cos q sin q 0 sin q - cos q 0
2x + 3y = 4 - sin q cos q cos q
x+y+z=4 (C)
0 (D)
sin q 0
x + 2y - z = a [2008] 0 0 1 0 0 1

Chapter 01.indd 24 8/29/2015 9:02:25 AM


Chapter 1 Linear Algebra | 2.25

18. A is a 3 4 real matrix and A x = b is an inconsistent system (A) zero (B) 2 units
of equations. The highest possible rank of A is[2005] (C) 50 units (D) 100 units
(A) 1 (B) 2 (C) 3 (D) 4
1 1 3
19. Which one of the following is an eigen vector of the matrix.
21. The sum of the eigen values of the matrix 1 5 1 is
5 0 0 0 3 1 1
0
5 0 0  [2005]  [2004]
?
0 0 2 1
(A) 5 (B) 7 (C) 9 (D) 18
0 0 3 1
22. For which value of x the matrix given below becomes
1 0 1 1 singular? [2004]
-2 0 0 -1
(B)
(C)
(D)
(A)
0 1 2 8 0
0 4 0 2

0 0 -2 1
12 6 0
20. With a 1 unit change in b, what is the change in x in the
solution of system of equations x + y = 2, 1.01x + 0.99y = b? (A) 4 (B) 6 (C) 8 (D) 12
[2005]

Answer Keys
Exercises
Practice Problems I
1.C 2.A 3.C 4.C 5.B 6.B 7.A 8.B 9.C 10.C
11.D 12.C 13.C 14.D 15.C 16.A 17.B 18.B 19.D 20.C
21.B 22.D 23.A 24.D 25.C 26.B 27.B 28.B 29.B 30.B
31.A 32.C 33.D 34.A 35.C 36.B 37.A 38.A 39.D 40.D
41.D 42.A 43.C 44.D 45.C 46.A 47.C 48.C 49.D 50.A
51.C 52.B 53.B 54.B 55.B 56.B 57.D 58.B 59.A 60.C
61.A 62.B 63.B 64.D 65.A

Practice Problems 2
1.A 2.D 3.C 4.A 5.D 6.A 7.B 8.C 9.A 10.C
11.C 12.A 13.A 14.A 15.B 16.C 17.D 18.B 19.A 20.B
21.C 22.C 23.D 24.B 25.D 26.A 27.A 28.C 29.C 30.B
31.B 32.C 33.A 34.A 35.B 36.A 37.C 38.B 39.A 40.B
41.A 42.A 43.C 44.D 45.C 46.D 47.D 48.A 49.D 50.C
51.B 52.D 53.D 54.D 55.B 56.D 57.A 58.C 59.B 60.C
61.A 62.D 63.D 64.C 65.B 66.D 67.C 68.C 69.C 70.B

Previous Years Questions


1.A 2.D 3.D 4.D 5.C 6.B 7.C 8.A 9.A 10.C
11.B 12.B 13.A 14.B 15.A 16.A 17.C 18.B 19.A 20.C
21.B 22.A

Chapter 01.indd 25 8/29/2015 9:02:28 AM


Chapter 2
Calculus
LeARNING OBJeCTIVeS

After reading this chapter, you will be able to understand:


Limit of a Function Cauchys Mean Value Theorem
Continuous Functions Taylors Theorem
Algebra of Continuous Functions Maclaurins Theorem
Intermediate - Value Theorem Partial Differentiation
Uniform Continuity Indefinite Integrals
Types of Discontinuity Definite Integrals
Derivatives Applications of Integration
Product Rule Rectification
Quotient Rule Improper Integrals
Second Derivative Triple Integrals
Application of Derivatives Change of Variables

limit oF A FunCtion Now, let us see what happens when x is greater than 2.
when x = 2.1, f(x) = 9.261
Let y = f(x) be a function of x and let a be any real number.
when x = 2.01, f(x) = 8.12
We must first understand what a limit is. A limit is the value,
when x = 2.001, f(x) = 8.01
a function approaches, as the variable within that function (usu-
when x = 2.0001, f(x) = 8.001
ally x) gets nearer and nearer to a particular value. In other
words, when x is very close to a certain number, what is f(x) very As x decreases and approaches 2, f(x) still approaches 8. This is
close to? called right-hand limit and is written as lim
x 2+

Meaning of x a x 2 2 x
Let x be a variable and a be a constant. If x assumes values nearer
and nearer to a, then we say that x tends to a or x approaches a We get the same answer while finding both, left and right hand
and is written as x a. By x a, we mean that x a and x may limits. Hence, we write that lim x3 = 8.
x 2
approach a from left or right, which is explained in the example
given below. Meaning of the Symbol
lim f ( x ) = 
Let us look at an example of a limit What is the limit of the x a

function f(x) = x3 as x approaches 2? The expression the limit as Let f(x) be a function of x where x takes values closer and closer to
x approaches to 2 is written as : lim Let us check out some values a ( a), then f(x) will assume values nearer and nearer to . Hence,
x 2 we say, f(x) tends to the limit as x tends to a.
of lim as x increases and gets closer to 2, without even exactly
x 2 The following are some of the simple algebraic rules of limits.
getting there.
when x = 1.9, f(x) = 6.859 1. lim k f(x) = k lim f(x)
x a x a
when x = 1.99, f(x) = 7.88 2. lim [f(x) g(x)] = lim f(x) lim g(x)
when x = 1.999, f(x) = 7.988 x a x a x a

when x = 1.9999, f(x) = 7.9988 3. lim [f(x) g(x)] = lim f(x) lim g(x)
x a x a x a
As x increases and approaches 2, f(x) gets closer and closer to
8, and as x tends to 2 from left this is called left-hand limit and f ( x ) lim f (x )
4. lim = x a (lim g ( x ) 0)
is written as lim . x a g ( x ) lim g ( x ) x a
x 2 x a

Chapter 02.indd 26 8/28/2015 6:57:13 PM


Chapter 2 Calculus | 2.27

Note: (c, M) (c, M)


1. If the left hand limit of a function is not equal to the y
right hand limit of the function, then the limit does
not exist.
2. A limit equal to infinity is not the same as a limit that
does not exist.

Continuous Functions (d, m)

Let f: A B be any given function and let c A. We say f is x


a d c c b
continuous at c, if given > 0, there exists d > 0 such that
|f(x) - f (c)|< whenever |x - c| < d
Note: The converse may not be true as
In words, this means that, if x is very close to c in domain,
then f(x) is very close to f(c) in range. 1; 0 < x 1
f(x) = is bounded on [1,2] but it is not con-
Equivalently, f is continuous at c. If lim f (x) = f (c) 1; 1 < x 2
x c
We observe tinuous at x = 1.
c A ie f(c) must exist
1.
2.
lim f(x) exists
Intermediate Value Theorem
x c
If f is continuous on [a, b] and f(a) f(b) then f takes every
3.
f(c) and lim f(x) are equal. value between f(a) and f(b).
x c

If any of these three conditions fail, then f is discontinu- Equivalently, if f is continuous on [a, b] and f(a) < k < f(b)
ous at x = c. or f(b) < k < f(a), then there exists c (a, b) such that
f(c) = k.
Algebra of Continuous Functions Equivalently, If f(a) and f(b) are of opposite signs then
there exists c (a, b) such that f(c) = 0.
If f, g be two continuous functions at c, then f + g, f - g, fg
are also continuous at x= c. y
To solve a problem of continuous functions at a point a,
you can take the following approach.
1. Find the value f(x) at x = a. If a is in the domain of f,
f(a) must exist. If a is not in the domain, then f(a) does 0 x
a c b
not exist. In such a case, f is not continuous at x= a.
2. Find lim f(x). For this you have to first find lim f(x) = 1
x a x a
(say) and lim f(x) = 2 (say). If 1 2 then lim f(x) does
x a +
x a
not exist and so f is not continuous at x = a. If 1 = 2, f(a) < 0 and f(b)>0, clearly f(c) = 0.
then lim f(x) exists. Note:
x a

3. If lim f(x) exists and also f(a) exists. Then, verify 1. If f(x) is continuous in [a, b] then f takes all values
x a
whether lim f(x) = f(a). between m and M at least once as x moves from a to b,
x a where M = Supremum of f on [a, b] and m = infimum
If lim f(x) = f(a). Then f is continuous, otherwise it is not of f on [a, b].
x a
2. If f(x) is continous in [a, b], then | f | is also continuous
continuous at x = a.
on [a, b], where | f | (x) = | f(x)| x [a, b].
Problems on continuous functions can be grouped into
3. Converse may not be true
the following categories.
1;
(a) Using , d notation. For instance, f(x) = 1; 0 < x 3
1;
(b) Using existence of right and left hand limits. 1; 3 < x 5
(c) To find the value of the unknown in f(x) when f is is not continuous at x = 3, but | f |(x) = 1 x [0,5],
given to be continuous at a point. being a constant function is continuous [0, 5].
(d) To find f (a) when f is given to be continuous at x = a. Inverse-function theorem If f is a continuous one-to-one
For functions that are continuous on (a, b) the following function on [a, b] then f -1 is also continuous on [a, b].
holds:
Uniform continuity A function f defined on an interval I is
(i) f is bounded and attains its bounds at least once on [a, b] said to be uniformly continuous on I if given > 0, there
i.e. for some c, d [a, b], exists a d > 0 such that if x, y are in I and |x - y| < d, then
M = supremum of f = f(c) and m = Infimum of f = f(d) | f(x) - f(y) | < .

Chapter 02.indd 27 8/28/2015 6:57:18 PM


2.28|Engineering Mathematics

Note: Continuity on [a, b] implies uniform continuity whereas For example the function
continuity on (a, b) does not mean uniform continuity. 3 1
x sin ; x 0
f(x) = x has the derivative function as
Types of Discontinuity 0; x=0
If f is a function defined on an interval I, it is said to have
2 1 1
(TD1) a removable discontinuity at p I, if lim f(x) exists, 3 x sin x cos ; x 0
xp
f(x) = x x
but is not equal to f(p). 0; x=0
(TD2) a discontinuity of first kind from the left at p if lim
xp
However, lim f ( x ) doesnt exist.

f(x) exists but is not equal to f(p). x 0


(TD3) a discontinuity of first kind from the right at p if
Example 1: Discuss the continuity of the function at
lim+ f(x) exists but is not equal to f(p).
xp x = 1 where f(x) is defined by
(TD4) a discontinuity of first kind at p If lim f(x) and lim+ 3x 2
f(x) exists but they are unequal.
xp xp f (x) = for 0 < x 1
x
(TD5) a discontinuity of second kind from the left at p if sin ( x 1)
= for x > 1
lim f(x) does not exist as x approaches p from left.
xp ( x 1)
(TD6) a discontinuity of second kind from the right at p Solution: Consider the left and right handed limits
if lim+ f(x) does not exist as x approaches p from right.
xp 3x 2
Lt- f (x) = Lt =1
(TD7) a discontinuity of second kind at p if neither lim x1 x1 x
xp
f(x) nor lim+ f(x) exist. sin ( x 1)
xp
Lt+ f (x) = Lt
Examples for each type is presented in the following x1 x1 x 1
table: sin ( x 1)
= Lt = 1 and f (1)
( x -1) 0 ( x 1)
Type Example Point of discontinuity 3 (1) 2
TD1 x2 1 x=1 = =1
f(x) = ,x1 1
x 1
f(1) = 3
Lt- f (x ) = Lt+ f (x) = f (1)
x1 x1

TD2 f(x) = x + 3 for 0 < x < 1 x=1 f is continuous at x = 1.


f(x) = 5 for x 1
Example 2: If f (x) =
( 2 x 1)
2

for x 0 and
sin 2 x log (1 + x )
TD3 f(x) = x + 3, for x > 2 x=2
f(x) = 8 for x 2
f(x) = log 2 for x = 0, discuss the continuity at x = 0.
TD4 x + 3; x > 2 x=2

f (x ) = 7; x =2
Solution: Lt f (x) = Lt
( 2 x 1)2
x 3; x < 2
x0 x0 sin 2 x log (1 + x )

TD5 f(x) = tan x for x < p/2 p 2 x 1


2
x=
f(x) = 1, for x p/2 2 x
= Lt
TD6 f(x) = 1, for x p/2 x = p/2 x0 sin 2 x log (1 + x )
f(x) = tan x for x > p/2 ( 2)
2x x
TD7 f(x) = 1/x at x 0 f(0) = 3 x=0 2
at x = 0 2 x 1
x
= Lt
x0 sin 2 x 1
Note: 2 log (1 + x ) x
1. Every differentiable function is continuous, but the 2x
converse is not true. 2
2x - 1
The example of a function which is continuous Lt
1 x 0
x
but not differentiable at a point f(x) = |x - 3| for x R = 1
is continuous at x = 3, but it is not differentiable at 2 sin 2 x
Lt log Lt (1 + x ) x
x = 3. 2 x 0 2x x 0

2. The function may have a derivative at a point, but the 1


= (log2)2.
derivative may not be continuous. 2

Chapter 02.indd 28 8/28/2015 6:57:31 PM


Chapter 2 Calculus | 2.29

But given f (x) = 2 log 2 at x = 0 Then, lim f(x) = lim+ f(x)


x 0 x 0

Lt f (x) f (0) = lim f(x) = 0


x0 x 0
f (x) is not continuous at x = 0. Thus, the function is continuous at the origin.
Example 3: Find the value of k if
2 x 3 5 x 2 + 4 x + 11 Derivatives
f (x) = , for x - 1 In this section, we will look at the simplistic form of the
x +1
definition of a derivative, the derivatives of certain standard
and f (- 1) = k is continuous at x = - 1.
functions and application of derivatives.
Solution: Given f (x) is continuous at x = - 1 For a function f(x), the ratio [f(a + h) - f(a)]/h is the rate
of change of f(x) in the interval [a, (a + h)].
Lt f (x) = f (- 1) = k.
x-1 The limit of this ratio as h tends to zero is called the
2 x 3 5 x 2 + 4 x + 11 derivative of f(x). This is represented as f (x) i.e.,
Lt f (x) = Lt
x-1 x-1
x +1 f (a + h ) f (a )
lim = f (x)
( x + 1) ( 2 x 2
7 x + 11) h0 h
= Lt
x-1 x +1 d { f ( x )}
The derivative f (x) is also represented as or
= 2(- 1) - 7 (- 1) + 11
2
d dx
= 2 + 7 + 11 = 20 {f(x)}
dx
k = 20
Hence, if y = f(x), i.e., y is a function of x, then dy/dx is
x4 the derivative of y with respect to x.
Example 4: If f (x) = + a, for x < 4, = a + b
x4 Note:
at x = 4,
1. dy/dx is the rate of change of y with respect to x.
x4 2. If the function y can be represented as a general curve,
= + b, for x > 4
x4 and a tangent is drawn at any point where the tangent
makes an angle q with the horizontal (as shown in the
and f (x) is continuous at x = 4, then find the values of a
figure), then dy/dx = tanq. In other words, derivative
and b. of a function at a given point is the slope of the curve
at that point, i.e., tan of the angle, the tangent drawn
x4
Solution: Lt - f (x) = Lt - +a to the curve at that point, makes with the horizontal.
x 4 x 4 x4
Y y = f (x)
x4+a
= Lt - =-1+a
x 4 ( x 4 )


x4
Lt + f (x) = Lt + +b
x 4 x 4 x4 O X
x4
= Lt + +b = 1 + b
x 4 ( x 4 )
As given f (x) is continuous at x = 4 Standard Results
Lt f (x) = f (4) = Lt - f (x) If f(x) and g(x) are two functions of x and k is a constant,
x 4 - x 4 then
-1 + a = a + b = 1 + b a = 1, b = - 1 d
1. (c) = 0 (c is a constant)
Example 5: Examine the continuity of the given function dx
xe1 x d d
;x 0 2. k f(x) = k f(x) (k is a constant)
at origin where f ( x ) = 1 + e1 x dx dx
0 ; x = 0 d d d
3. (f(x) g(x)) = f(x) g(x)
xe 1 x dx dx dx
Solution: lim f ( x ) = lim =0
x 0 x 0 1+ e 1 x Product rule
x d
lim f ( x ) = lim+ =0 4. {f(x) g(x)} = f (x)g(x) + f(x)g(x)
x 0+ x 0 e 1 x
+1 dx

Chapter 02.indd 29 8/28/2015 6:57:43 PM


2.30|Engineering Mathematics

Quotient rule d
(c) tanh x = sech2 x
d g ( x ) . f ( x ) f ( x ) .g ( x ) dx
5. {f(x)/g(x)} =
( g ( x ))
2
dx d
(d) coth x = - cosech2 x
Chain rule dx
6. If y = f(u) and u = g(x) be two functions, then dy/dx = d
(e) sech x = - sech x tanh x
(dy/du) (du/dx). dx
d
Derivatives of Some Important Functions (f) cosech x = - cosech x coth x
dx
1. (a)d/dx (xn) = n xn-1
(b) d/dx [1/xn] = -n/xn+1 d 1
1 10. (a) sinh-1 x =
(c) d/dx (x) = ;x0 dx 1+ x2
2 x
2. d/dx [axn + b] = an xn-1 d 1
(b) cosh-1 x =
3. d/dx [ax + b]n = n a(ax + b)n-1 dx x2 1
4. d/dx [eax] = a eax d 1
5. d/dx [logx] = 1/x; x > 0 (c) tanh-1 x =
6. d/dx [ax] = ax loga; a > 0 dx 1 - x2
7. (a)d/dx [sinx] = cosx d -1
(b) d/dx [cosx] = -sinx (d) coth-1 x = 2
dx x -1
(c) d/dx [tanx] = sec2x
(d) d/dx [cotx] = -cosec2x d -1
(e) sech-1 x =
(e) d/dx [secx] = secx tanx dx x 1 - x2
(f) d/dx [cosecx] = -cosecxcotx
d -1
Inverse rule If y = f(x) and its inverse x = f -1(y) is also (f) cosech-1 x =
dx x x2 + 1
dy 1
defined, then = .
dx dx dy
Successive Differentiation
Second derivative If y = f(x), then the derivative of
If f is differentiable function of x and the derivative f is also
derivative of y is called as second derivative of y and is
a differentiable function of x, then f is called the second
d2 y
represented by 2 . derivative of f. Similarly, the 3rd, 4th ............nth derivative
dx of f may be defined and are denoted by f, f , ............... f
d2 y d dy dy n
or y3, y4 ............ yn.
= f (x) = where is the first derivative of y.
dx 2
dx dx dx
d 1 Application of Derivatives
8. (a) sin-1x =
dx 1 - x2 1. Errors in measurement: Problems relating to errors
d -1 in measurement can be solved using the concept of
(b) cosec-1x = derivatives. For example, if we know the error in meas-
dx x x2 -1 urement of the radius of a sphere, we can find out the
d -1 consequent error in the measurement of the volume of
(c) cos-1x =
dx 1 - x2 the sphere. Without going into further details of theory,
d 1 we can say dx = error in measurement of x and dy =
(d) sec-1x = consequent error in measurement of y, Where y = f(x).
dx x x2 1
Hence, we can rewrite dy/dx = f(x) as dy = f (x) dx.
d 1 Thus, if we know the function y = f(x) and dx, error
(e) tan-1x =
dx 1+ x2 in measurement of x, we can ind out dy, the error in
d 1 measurement of y.
(f) cot-1x =
dx 1+ x2 Note:
d (a) An error is taken to be positive when the measured
9.
(a) sinh x = cosh x value is greater than the actual value and negative
dx
d when it is less.
(b) cosh x = sinh x (b) The percentage error in y is given by (dy/y) 100.
dx

Chapter 02.indd 30 8/28/2015 6:57:55 PM


Chapter 2 Calculus | 2.31

2. Rate of change: While defining the derivative, we Another Form


have seen that the derivative is the rate of change. If f is defined on [a, a + h] such that
This can be applied to motion of bodies to determine
their velocity and acceleration. (i) f is continuous on [a, a + h].
Velocity: If we have s, the distance covered by a body (ii) f is differentiable on (a, a + h) then there exists atleast
expressed as a function of t, i.e. s = f(t), then rate of one q (0, 1) such that f(a + h) = f(a) + hf (a + qh).
change of s is called velocity (v). v = ds/dt = f (t).
Acceleration: Rate of change of velocity is defined as Cauchys Mean Value Theorem
acceleration. As v = f (t) itself is a function of t, we can Let f and g be two functions defined on [a, b] such that
write v = f (t). a = dv/dt = d2s/dt2, i.e. acceleration is (i) f and g are continuous on [a, b]
the second derivative of the function s = f(t). (ii) f and g are differentiable on (a, b)
3. Maxima and Minima: A function takes a maximum (iii) g(x) 0 for any x (a, b) then there exists
value or a minimum value when the slope of the tan- atleast one real number c (a, b) such that
gent of the curve at that point is zero, i.e. when the f (b) f ( a) f (c )
first derivative of the function is zero. If y = f(x), = .
g (b) g ( a) g (c )
then y is maximum or minimum at the point x = x1 if
dy Taylors Theorem
= 0.
dx x = x
1 Let f be a real-valued function defined on [a, a + h] such that
Thus, we can find the value of x1 by equating dy/dx = 0. f n -1 is continuous on [a, a + h]
1.
As mentioned above that y can have a maximum or a f n-1 is derivable on (a, a + h), then there exists a
2.
minimum value at x = x1. Whether y is a maximum value or number q (0, 1) such that
minimum is governed by the sign of the second derivative. h2
The function y has a minimum value if the second derivative f(a + h) = f(a) + hf (a) + f (a) +.......
is positive. In other words, y is maximum at x = x1 if d2y/dx2 2!
< 0 at x = x1. y is minimum at x = x1 if d2y/dx2 > 0 at x = x1. hn1 n - 1
+ f (a) + Rn.
dy ( n 1)
= 0. in both the cases discussed above.
dx x = x hn f n ( a + qh)
1
where Rn =
n!
(a) If f (c) = 0 and f (c) is negative, then f(x) is
maximum for x = c (Lagranges form of remainder)
(b) If f (c) = 0 and f (c) is positive, then f(x) is minimum hn (1 q )
n 1
f n ( a + qh)
for x = c Rn =
(c) If f (c) = f (c) = ......... = f r - 1(c) = 0 and f r(c) 0, then
( n 1)!
(Cauchys form of Remainder)
(i) If r is even, then f(x) is maximum or minimum
for x = c according as f r(c) is negative or positive. Maclaurins Theorem
(ii) If r is odd, then there is neither maximum nor a
Let f:[0, x] R such that
minimum for f(x) at x = c.
1.
f n-1 is continuous on [0, x],
Rolles Theorem 2.
f n-1 is derivable on (0, x).
Let f be a function defined on [a, b] such that Then, there exists a real number q (0, 1) such that
(i) f is continuous on [a, b]; x2
(ii) f is differentiable on (a, b) and f(x) = f(0) + xf (0) + f (0) + ..........
2!
(iii) f (a) = f(b), then there exists c (a, b) such that
x n1 (n - 1)
f (c) = 0 + f (0) + Rn.
( n 1)!
Lagranges Mean Value Theorem Where Rn =
xn n
f (qx)
Let f be a function defined on [a, b] such that n!
(Lagranges form of remainder)
(i) f is continuous on [a, b],
x n (1 q ) f n (qx )
n 1
(ii) f is differentiable on (a, b) then there exists c (a, b)
Rn =
f (b) f ( a) ( n 1)!
such that f (c) = .
ba (Cauchys form of remainder)

Chapter 02.indd 31 8/28/2015 6:57:59 PM


2.32|Engineering Mathematics

Maclaurins Series Example 6:For the function f(x)=x(x2 1) test for the
Let f(x) be a function which posses derivatives of all orders applicability of Rolles theorem in the interval [1, 1] and
in the interval [0, x], then hence find c such that 1 < c < 1
x2 x n1 (n -1) Solution: Given f(x) = x(x2 - 1)
f(x) = f(0) + xf (0) + f (0)+.......... + f (0) +
2! ( n 1) (i) f is continuous in [1, 1]
xn n (ii) f is differentiable in (1. 1)
f (0) + is known as Maclaurins infinite series. (iii) f(1) = f(1) = 0
n!
Meaning of the sign of the derivative f(x) satisfies all the properties of Rolles theorems
we can find a number c such that f (c) = 0 i.e., f (x) = 3x2 1
Sign of f(x) on [a, b] Meaning 1
f(c) = 0 3c2 1 = 0 +
f(x) 0 f is non-decreasing 3
f(x) > 0 f is increasing 1
c=
f(x) 0 f is non-increasing 3
f(x) < 0 f is decreasing Example 7: If f(x) = 2x2 + 3x + 4, then find the value of q
in the Mean Value theorem.
f(x) = 0 f is constant
Solution: f(a) = 2a2 + 3a + 4
Example: The function f, defined on R by f(a + h) = 2(a2 + 2ah + h2) + 3a + 3h + 4
f(x) = x3 - 15 x2 + 75 x - 125 is non-decreasing in every f(a + h) f(a) = 4ah + 2h2 + 3h
interval as f (x) = 3 (x2 - 10x + 15) = 3 (x - 5)2 0 = 2(2ah + h2) + 3h
Thus, f is non-decreasing on R. f ( a + h) f ( a )
= 2(2a + h) + 3
h
Series expansions of some s
tandard
h
= 4 a + +3  (1)
functions 2
Now, f (x) = 4x + 3, f 1(a + qh)
x 2 x3 xn = 4a + 4hq + 3 (2)
(a) ex = 1 + x + + + ........ + + ......
2 ! 3! n! h
Comparing (1) & (2) we have 4 a + + 3
(b) sin x = x -
x3 x5
+ -........ +
( 1) x 2 n+1 + .......
n
2
3! 5! ( 2n + 1)! = 4a + 4hq + 3 a + hq = a +
h
2
(c) cos x = 1 -
x2 x4
+ - +
( -1) x 2 n + ........
n
q=
1
2! 4 ! ( 2n ) ! 2

(d) sin hx = x +
x3 x5
+ + +
x 2 n +1
+
Partial Differentiation
3! 5! ( 2n + 1)! Let u be a function of two variables x and y. Let us assume
x2
x 4
x 2n the functional relation as u = f(x, y). Here, x alone or y alone
(e) cos hx = 1+ + + + + or both x and y simultaneously may be varied and in each
2! 4 ! ( 2n )! case a change in the value of u will result. Generally, the
x 2 x3 x 4
(f) log (1 + x) = x - + +
( 1) x n n 1
change in the value of u will be different in each of these
2 3 4 n three cases. As x and y are independent, x may be supposed
to vary when y remains constant or the reverse.
(g) (1 + x)-1 = 1 - x + x2 - x3 + ............................ The derivative of u w.r.t. x when x varies and y remains
(h) (1 - x)-1 = 1 + x + x2 + x3 + ......................... constant is called the partial derivative of u w.r.t. x and is
(i) (1 + x)-2 = 1 - 2x + 3x2 - 4x3 + .................... denoted by u/x
2 u u 2 u u
x 1.3 2 1 3 5 3 = , = .
(j) (1 - x)-1/2 = 1 + + x + x + x 2
x x xy x y
2 2.3 2 46
x3 x5 ( 1) x 2 n 1 x2n- 1 + ... n 1 Total Differential Coefficient
(k) tan x = x -
-1
+ +

3 5 ( 2n 1) If u be a continuous function of x and y and if x and y receive


small increments x and y, u will receive, in turn, a small
1 x 3 1 3 x 5 increment u. This u is called total increment of u.
(l) sin-1 x = x + + +
2 3 2 4 5 u = f(x + x, y + y) - f(x, y)

Chapter 02.indd 32 8/28/2015 6:58:06 PM


Chapter 2 Calculus | 2.33

In the differential form, this can be written as Maxima and minima for function of two variables A
u u function f(x, y) is said to have a local maximum at a point
du = dx = dy. (a, b), if f(a + h, b + k) f(a, b) for all small values of h and
x y
k, i.e. f(x, y) has a local maximum at (a, b), if f(a, b) has a
du is called the total differential of u.
highest value in a neighbourhood of (a, b).
If u = f(x, y, z) then
Similarly, f(x, y) is said to have a local minimum at a
du u x u y u z point (a, b), if f(x, y) has least value at (a, b) in a neighbour-
= + +
dt x t y t z t hood of (a, b).
Implicit function If the relation between x and y be given Procedure to obtain maxima and minima Let f(x, y) be a
in the form f(x, y) = c where c is a constant, then the total function of two variables for which we need to find maxima
differential coefficient w.r.t. x is zero. and minima.
Homogeneous functions Let us consider the function f f
(i) Find fx = and fy =
f(x, y) = a0xn + a1xn-1y + a2xn-2y2 + ................. + anyn. In this x y
expression the sum of the indices of the variable x and y in (ii) Take fx = 0 and fy = 0 and solve them as simultaneous
each term is n. Such an expression is called a homogeneous equations to get pairs of values for x and y, which are
function of degree n. called stationary points.
Eulers theorem If f(x, y) is a homogeneous function of 2 f 2 f 2 f
(iii) Find r = f xx = s = f = and t = f =
f f x 2 xy
x y yy
y 2
degree n, then x +y = nf. and find rt - s2.
x y
(iv) At a stationary point, say (a, b)
This is known as Eulers theorem on homogeneous function.
(a) If rt - s2 > 0, then (a, b) is called an extreme point
The nth derivatives of some special functions:
of f(x, y) at which f(x, y) has either maximum or
dn n minimum which can be found as follows.
(a) x = n!
dx n Case (i): If r < 0, then f(x, y) has a local maximum
dn m! at (a, b)
(b)  n xm = xm-n s(m being a positive inte- Case (ii): If r > 0, then f(x, y) has a local minimum
dx ( m n )! at (a, b).
ger more than n)
(b) If rt - s2 < 0, then (a, b) is called as saddle point
d n ax
(c) e = an eax of f(x, y) where f(x, y) has neither maximum nor
dx n minimum at (a, b).
d n 1 ( 1) n !
n

(d) = ;x-a Example 8: Find the stationary points of the function f(x, y)
dx n x + a ( x + a )n +1 = x2 y + 3xy - 7 and classify them into extreme and saddle
points.
dn
(e) log (x + a) =
( 1) ( n 1)! ; (x + a) > 0
n 1

dx n
( x + a )n Solution: Given f(x, y) = x2 y + 3xy - 7
dn np f f
(f) sin (ax + b) = an sin + ax + b fx = = 2 xy + 3 y and f y = = x 2 + 3x
dx n 2 x y
dn np Now, fx = 0 2xy + 3y = 0 and fy = 0
(g) cos (ax + b) = an cos + ax + b x2 + 3x = 0
dx n 2
3
d n ax y = 0 and x = ; x(x + 3)x = 0 and x = -3
(h) (e sin bx) 2
dx n 3
= (a + b2)n/2 eax sin (bx + n tan-1 b/a)
2
But for x = , fy 0
2
dn
(i)  n (eax cos bx) = (a2 + b2)n/2 eax cos (bx + n tan-1 b/a) The stationary points of f(x, y) are (0, 0) and (-3, 0)
dx Now, r = fxx = 2y; s = fxy = 2x + 3 and t = fyy = 0
d n 1 ( 1) n ! n+1 And rt - s2 = 2y 0 - (2x + 3)2 = - (2x + 3)2
n

(j)  n 2 = sin qsin(n + 1)q where rt - s2 < 0 at (0, 0) as well as (-3, 0)


dx x + a 2 an+ 2
q = tan-1 (x/a) Hence, the two stationary points (0, 0) and (-3, 0) are saddle
points where f(x, y) has neither maximum not minimum.
dn
(k)  n (tan-1x) = (-1)n-1 (n - 1)! sinnq . sinnq where
dx Example 9: Find the maximum value of the function f(x, y, z)
q = cot-1x = z - 2x2 - 3y2 where 3xy - z + 7 = 0.

Chapter 02.indd 33 8/28/2015 6:58:16 PM


2.34|Engineering Mathematics

olution: Given f(x, y, z) = z - 2x2 - 3y2 


S (1) x
16. cosecx dx = log (cosecx - cotx) + c = log tan +c
Where 3xy - z + 7 = 0  (2) 2
 z = 3xy + 7 (3), Substituting the value of z in (1), we 1
17. dx = sin-1x + c or - cos-1x + c
have f = 3xy + 7 - 2x2 - 3y2 1 x2
f 1
fx = = 3y - 4x and fy = f = 3x - 6y 18. dx = tan-1x + c or -cot-1x + c
x y 1+ x2
fx = 0 3y - 4x = 0 and fy = 0 3x - 6y = 0 1
19. dx = sec-1x + c or - cosec-1x + c
fx = 0 and fy = 0 only when x = 0 and y = 0 x x2 1
The stationary point is (0, 0) 20. sinh x dx = cosh x + c
2 f 2 f 21. cosh x dx = sinh x + c
Now, r = fxx = = 4; s = f = =3 22. sech2 x dx = tanh x + c
x 2 xy
x y
23. cosech2 x dx = -coth x + c
2 f 24. sech x tanh x dx = - sech x + c
and t = f yy = = 6 rt - s2 = (-4)(-6) - 32 = 24 - 9
y 2 25. sech x coth x dx = - cosech x + c
= 15 > 0 and r = -4 < 0 26. K f(x) dx = K f(x)dx + c
f has a maximum value at (0, 0) 27. (f(x) g(x))dx = f(x)dx g(x)dx + c
For x = 0, y = 0, from (3), z = 3 0 0 + 7 z = 7
The maximum value exists for f(x, y, z) at (0, 0, 7) f (x)
28. dx = log [f(x)] + c
and that maximum value is f(x, y, z)at(0,0,7) = 7 - 2 02 - 3 f (x)
f ( x )
n +1
02 = 7
29. f(x) . f (x)dx =
n
+c
Indefinite integrals If f(x) and g(x) are two functions of x n +1
such that g(x) = f(x), then the integral of f(x) is g(x). Further, dx x
g(x) is called the antiderivative of f(x). 30. = sin-1 a + c
The process of computing an integral of a function is a2 x 2
called Integration and the function to be integrated is called dx x
31. = sinh-1 + c or log x + a 2 + x 2 + c
Integrand. a +x2 2 a
An integral of a function is not unique. If g(x) is any one dx x
integral of f(x), then g(x) + c is also its integral, where c is 32. = cosh-1 + c or log x + x 2 +a 2 +c
x 2 a2 a
any constant termed as constant of integration.
1 1
33. 2 dx = tan-1(x/a) + c
x + a2 a
Some Standard Formulae 1 1 xa
x n+1 34. 2 dx = log +c
1. xn dx = + c(n -1) x a2 2a x+a
n +1
1 1 a+ x
(ax + b)n+1 35. dx = log +c
2. (ax + b)ndx = + c ( n -1) a x
2 2
2a ax
(n + 1) a x a2 + x 2 a2 x
1 36. a 2 x 2 dx = + sin-1 + c
3. dx = logx + c 2 2 a
x
1 log ( ax + b ) x a2 x 2 a2 x
4. dx = +c 37. a 2 + x 2 dx = + sinh-1 + c
ax + b a 2 2 a
ax x x 2
a 2
a 2
5. axdx = +c 38. x 2 a 2 dx = - cosh-1 x + c
log a 2 2 a
6. exdx = ex + c
39. logx dx = x(logx - 1) = x log(x/e) + c
7. sinxdx = -cosx + c
8. cosx dx = sinx + c 40. ex [f(x) + f(x)] dx = ex f(x) + c
9. sec2xdx = tanx + c Definite integrals The difference in the values of an
10. cosec2x dx = -cotx + c integral of a function f(x) for two assigned values say a, b of
11. secx tanx dx = secx + c the independent variable x, is called the Definite Integral of
12. cosecx cotx dx = -cosecx + c b

13. tanx dx = log(secx) + c f(x) over the interval [a,b] and is denoted by f ( x ) dx
14. cotx dx = log(sinx) +c a
p x The number a is called the lower limit and the number
15. secx dx = log(secx + tanx) + c = log tan + + c
4 2 b is the upper limit of integration.

Chapter 02.indd 34 8/28/2015 6:58:31 PM


Chapter 2 Calculus | 2.35

Fundamental Theorem of Integral b


b f ( x ) dx, if f ( x ) 0, a x b
Calculus If f(x) is a function of x continuous in [a, b], then a
b A= f ( x ) dx = b
f ( x )dx = g(b) - g(a) where g(x) is a function such that a f x dx, if f ( x ) 0, a x b
( )
a
a
d
g(x) = f(x)
dx y
y = f (x ) y
Properties of Definite Integrals
x=a x=b
1. If f(x) is a continuous function of x over [a, b], and o x
b c
A
c belongs to [a, b], then
b
f ( x) dx = f ( x)dx
a a
+

f ( x)dx .
a
o x=a x=b x y = f(x )

2. If f(x) is continuous function of x over [a, b], then 2. Similarly, the area enclosed by the curve x = g(y), the
b b d

K f(x)dx = K f(x)dx lines y = c and y = d and the y-axis is A = g ( y ) dy


a a c

3. If f(x) is continuous function of x over [a, b], then 3. when f(x) 0 for a x c and f(x) 0 for c x b, then
a b the area enclosed by the curve y = f(x), the lines x = a
f(x)dx = - f(x) dx c b

b a
an x = b and the x-axis is A = f ( x ) dx f ( x ) dx
a c
4. If f(x) is continuous in some neighbourhood of a, then
a y y = f (x)

b
f(x)dx = 0
b

5. If f(x) and g(x) are continuous in [a, b], then [f(x) A


(c, 0) x=b
b b a
o
+ g(x)]dx = f(x)dx + g(x)dx
a a
x=a x

b b b

6. f(x) dx = f(z)dz = f(t)dt


4. The area enclosed by the curves y = f(x) and y = g(x)
a a a
and the lines x = a and x = b is given by
a a

7.
0
f(x)dx =
0
f(a - x)dx
y f(x )

a
A
8. f(x) = 0, if f(x) is odd
a g(x )
a a o x=a x=b x
9.
a
f(x)dx = 2 f(x)dx if f(x) is even
0 y g(x )
2a a

10. 0
f(x)dx = 2 f(x)dx, if f(2a - x) = f(x) = 0
0 
A

if f(2a - x) = - f(x) f (x )
o x=a x=b x
na a

11. 0
f(x)dx = n f(x)dx, if f(a + x) = f(x)
0 b
b

A = f ( x ) g ( x ) dx =
( f (x ) g (x ))dx , if f (x ) g (x ); a x b
Applications of integration a

(g (x ) f (x ))dx , if f (x) g (x ); a x b
a

Area as a Definite Integral


1. The area enclosed by a curve y = f(x), the lines x = a Example 10: Find the area enclosed by the curve y = x3, the
and x = b and the x-axis is given by line y= 2 and the y-axis in first quadrant?

Chapter 02.indd 35 8/28/2015 6:58:42 PM


2.36|Engineering Mathematics

olution: The area bounded by y = x3, y = 2 and the y-axis


S b

given by S = 1 + ( dy dx ) dx, provided dy/dx is


2
is the area OAB as shown in the figure.
So, the region OAB is bounded by the curve x = y1/3, a

the lines y = 0 and y = 2 and the y-axis and x = y1/3 0, continuous on [a, b].
y [0, 2] Note: If the equation of the curve is given in the
The required area form x = f(y), then the length of the arc between
2 2
y the points with y-coordinates c and d is given by
3
= y dy = y 4 3
13 b
y=2
1 + ( dx dy ) dy provided dx/dy is continuous
2
y=0
4 0 S=
A c
3 on [c, d]
= 24 3 o
4 x 2. Parametric equations: Let x = f(t) and y = g(t) be
3 parametric functions of t. The length of the arc
= 23 y = x3
2 between the points{f(t1), g(t1)} and {f(t2), g(t2)} is
3 t2
dx dy
= 3
4 given by
t

dt + dt dt provided dx/dt and dy/dt

1

Example 11: Find the area enclosed by the curve y = x and 2 are both continuous on [t1, t2].
line y = 4? 3. Polar equations: Let r = f(q) be a function of q,
Solution: The area enclosed by the curve y = x and the line2 the length of the arc between the points {f(q1), q1}
q2
y = 4 is the region OAB. r 2 + ( dr dq )

2
and {f(q2),q2} is given by S = dq
The region OAB is bounded by line y = 4 and the curve q1
y= x2 from x = -2 to x = 2 and 4 x2 for all x [-2, 2] provided dr/dq is continuous along the arc.
If the equation of the curve is given in the form
y
q = f(r), then the length of the arc between the points
r2

(r1, f(r1)), (r2, f(r2)) is given by S = 1 + r 2 ( dq dr ) dr


B (2, 4) 2
(2, 4) A y=4
r1
A
provided dq/dr is continuous along the arc.
y = x2

Theorems on Integration
O x
1. If f is a continuous function on [a, b], then there exists
b

c (a, b) such that f ( x ) dx = f (c )(b a)


a
2

The required area = ( 4 x 2 ) dx


x = 2
2. If f, g R[a, b ] and g keeps the same sign on [a, b],
then there exists m R lying between the infimum and
b b

f ( x ) g ( x ) = m g ( x ) dx
2
the suprimum of f such that
= 2 ( 4 x ) dx ( 4 - x as even)
2 2
a a
0
Note: This is called the first Mean Value theorem.
2
x3 3. If f, g R[ a, b], g is positive and decreasing on [a, b],
= 2 4 x
3 0 then there exists m [a, b] such that
b m

=
32
2
f ( x ) g ( x ) dx = g ( a) f ( x ) dx
a a

Note: This is known as Bonnet Mean Value theorem.


Rectification 4. If f, g R [ a, b ] and is monotonic on [a,b], then
b m
The process of determining the length of arcs of plane
curves is called Rectification. The length of the arcb can be there
m
exists m (a,b)
b
such that a ( ) ( )
f x g x dx = g ( ) f ( x ) dx + g ( x )
x
a

a ( ) ( ) ( ) ( ) ( ) ( )
calculated by any one of the methods given below. f x g x dx = g x f x dx + g x f x dx
m
1. Cartesian equations: a

Let y = f(x) be a function of x. The length of arc Note: This is known as second Mean Value theorem
between the points with x-coordinates a and b is or weierstrars theorem.

Chapter 02.indd 36 8/28/2015 6:58:49 PM


Chapter 2 Calculus | 2.37

b
p
Example 12: Prove that there exists m 0, such that 1. f ( x )dx = lim f ( x )dx
2 a
b
a
p 2

0
x cos xdx = m (Singularity at upper limit)
b b

Solution: Take f(x) = x and g(x) = cosx f is continuous 2.


-
f ( x ) dx = Lt
a f ( x) dx
a
p p
on 0, and g is integrable on 0, also b
2 2
d F
3
3. f (x )dx = Lt
-
b
a a
f (x )dx
g(x) 0 in
du 3 or
By the first Mean Value theorem, 0 b

f ( x) dx
p 2 p 2
4. f ( x ) dx = Lt f ( x ) dx + Lt
x cos xdx = m cos xdx = m
0 0
-
a
a
b
0
r
p 2
p
There exists m 0, such that cos xdx = m
or = Lt
r f ( x ) dx
2 0
r

Convergent If the limits of the above integral exists or


Example 13:Verify the second Mean Value theorm for finite then the integral is said to be converge.
f(x) = x2 and g(x) = x2 on [-1, 1].
Divergent If the limits does not exists then they are said to
Solution: Given f(x) = x2 and g(x) = x2 on [-1,1] both f and g
be Divergent.
are continuous and integrable on [-1,1] but g is a decreasing
function on [-1,0] and increasing function on [0,1] g is Note 1: Geometrically, for f(x) 0, the improper integral

not monotonic.
1 1
f ( x ) dx denotes the area of an unbounded region lying
f(x)g(x)dx = x2.x2dx a

1 1 between the curve y = f (x) the ordinate x = a and x - axis.


1
x5 1 1 2 Note 2: Let f(x) and g(x) be non-negative functions and 0
= = + =  (1)
5 1 5 5 5
But by the second Mean Value theorm,
f(x) g(x) for x a. If g ( x ) dx converges then f ( x ) dx is
a a
a
m
also converges and f ( x ) dx g ( x ) dx .
b b

f ( x) g ( x)dx = g (a) f ( x)dx + g (b) f ( x)dx


a a m
a

g ( x ) dx diverges then
m
1 1
Similarly, let 0 g(x) f(x) if
x 4 dx = g ( 1) x 2 dx + g (1) x 2 dx a
1
m
1
1
m
f ( x ) dx also diverges.
2 a
=
1
x dx + x dx = x dx = 3
2

m
2 2
(2) i.e. The convergent or divergent of an improper integral
 by comparing it with a simple integral.
As (1) and (2) are not equal the Mean Value theorem does
not hold. Improper integral of the second kind

Improper Integrals
b

b
Consider f ( x) dx 
a
(1)

Consider definite integral f ( x ) dx


a
 (1)
If both the limits of (1) are finite f(x) is undefined or dis-
If f(x) is a function defined in a finite interval [a,b] and continuous at a point in between a and b, then (1) is known
f(x) is continuous for all x which belongs to [a,b] as Improper integral of second kind.
Then, (1) is called Proper integral. This can be evaluted as follows.
If f(x) is violated, at least one of these conditions then Let f(x) be undefined at a point c which belongs to (a, b),
the integral is known as improper integral. These improper then
c
integrals are classified into three kinds. b b

f ( x ) dx = Lt
0 f ( x ) dx + Lt
0 f ( x ) dx
Improper integral of the first kind a a c

In a definite integral if one or both limits of integration are If these limits exists then it is convergent otherwise it is
infinite, then it is an improper integral of first kind. divergent.

Chapter 02.indd 37 8/28/2015 6:58:59 PM


2.38|Engineering Mathematics

Improper Integral of Third Kind Case (ii): If the limits of x are function of y, say x1 = g1(y)
If the limits of the integral are infinite or f(x) may be dis- and x2 = g2(y) and the limits of y are constants, say y1 = c and
continuous or both then the improper integral is known as y2 = d, then integrate w.r.t x first treating y as constant and
third kind. then integrate w.r.t y.
y = d x = g (y )

That is, f ( x , y ) dR = f ( x , y ) dx dy
2 2 2
1
Note 1:1 x p dx is convergent when p > 1 and it is divergent R y = c x = g (y )
1 1 1

when p 1. This result is used in comparison test for testing


Case (iii): If both the variables x and y have constant limits,
the convergence or divergence of the integral of first kind. then one can follow any order of integration.
c
1 Change of order of integration The evaluation of some of
Note 2: dx is convergent for p < 1 and is
a a ( x c)
p the double integrals can be made simple by changing the
order of integration. In the change of order of integration,
divergent for p 1. This is used for convergence or
we take the limits of the variables for the given region
divergence of an improper integral of second kind.
of integration in such a way that the order of integration

reverses.
dx
Example 14:Examine for convergence / divergence. x2
xp x1 x
1
Example 15: Evaluate e y
dydx
x =0 y =x y 2

k
k
dx
k
x p +1
Solution: Consider = x p dx = if p 1 and x yx
2
1 x
x p + 11
p
1 1 Solution: Let I = e dydx (1)
x =0 y =x y 2

[ log x ]1 if p = 1
k

dx
k Y
Case (i):if p = 1, = logk - log 1
x A (1, 1)
1
= logk when k it does not tend to a finite limit. y = x2
A
it is divergent.
k p
dx 1
Case (ii):If p 1 p =
x 1 p
[ k 1 p ] it converges a
1 X
if p >1 and diverges if p 1. O

Multiple Integrates y=x


Double Integrates
Integration of f(x, y) over a region R in xy-plane is called a The evaluation of this integral can be made simple by
double integral. changing the order integration.
From the limits of x and y given, the region of integration
x2 y2
is the region bounded by the line y = x and the parabola y = x2
f ( x , y ) dR = f ( x , y ) dxdy as shown in figure.
R x = x1 y = y 1
Now, by changing the order of integration, we first inte-
grate w.r.t x, along the horizontal strip PQ from P(x = y) to
Order of Integration in a Double Integral
The order of integration depends on the nature of limits of ( )
Q x = y , and then
the variables. We integrate w.r.t y from 0(y = 0) to A(y = 1)
1 x x 2
x
Case(i): If the limits of y are function of x, say y1 = f1(x) l = e y
dxdy
x =0 y =x y
and y2 = f2(x) and the limits of x are constants, say x1 = a and
2

x2= b, where a and b are constants, then integrate w.r.t y first 1 y


x
x
2

treating x as constant and then integrate w.r.t x. l = e y dx dy (2)


y =0
x =y y 
y = f (x )
x 2 =b
x2 2x x 1
That is, f ( x , y ) dR = f ( x , y ) dy dx
2 2

put =t dx = dt dx = dt
R x = a y = f (x )
1 1 1 y y y 2

Chapter 02.indd 38 8/28/2015 6:59:05 PM


Chapter 2 Calculus | 2.39

( y) 3. Volume under the surface as a double integral: The


2
y2 volume V of the solid under the surface Z = f(x, y) and
x=yt= = y and x = y t = =1
y y above the xy-plane with the projection of Z = f(x, y)
(2) becomes on xy plane as its base is
Volume = D f ( x, y ) dx dy
1 1 1
l= y = 0 t =y e t 2 dt dy Z
Z = f (x, y)
1

(e )
1
= t
t=y
dy
y=0 V
O Y
1

= [ e 1
+e y
] dy = -ye - e -1 -y
D
y=0
X C
e2
= (-e-1 - e-1) - (0 - e-0) = 1 2r 1 = 4. Volumes as a Triple Integral: The volume of the
e
3-dimensional region V is given by V dx dy dz
Triple Integrals Example 16: Find the volume under the surface x + 2y + z = 4
Integration of a function f(x, y, z) over a 3-dimensional and above the circle x2 + y2 = 4 in the xy-plane.
region V is called the triple integral.
Solution: Given surface is x + 2y + z = 4
x2 y2 z2

v f ( x , yx , z ) dv = f ( x , y , z ) dxdydz z = 4 - x - 2y  (1)
x = x1 y = y 1 z = z 1
Let D be the region bounded by the circle x2 + y2 = 4 in xy-
Like double integrals, in triple integrals also the order plane
of integration depends on the nature of the limits of the
In D, y varies from y = - 4 x 2 to y = 4 x 2 and x
variables.
varies from x = -2 to x +2.
The volume under the surface x + 2y + z = 4 and above the
Applications of double and triple circle x2 + y2 = 4 in xy-plane is
integrals 2 4 x2

1. Area of the region r in xy-plane is given by V = zdxdy =


D

x = 2 y = 2 4 x 2
( 4 x 2 y ) dxdy (2)

Area of r = R dxdy
The evaluation of this double integral can be made simple
y by changing it into polar coordinates. In polar coordinates,
x = r cos q, y = r sin q and
R
x x
( x, y )
r q cos q r sinq
o x J= = =
( r , q ) y y sin q r cosq
2. Volume of the solid Revolution:
r q
(a) The volume of the solid of revolution obtained by
J = r, Also, in the circle x2 + y2 = 4, r varies from r = 0 to
revolving the area A about x-axis is
r = 2 and q varies from q = 0 to q = 2p
Volume = V = A 2p y dx dy
From (2),
y
2p 2
V = ( 4 x 2 y ) dxdy = ( 4 r cos q 2r sin q) J dr
A
D q = 0 r=0
2p 2
= ( 4 r cos q - 2 r sin q ) rdrdq
q =0 r=0
o x
2p
2
= ( 4 r r 2 cos q - 2r 2 sinq ) dr dq
(b) The volume of the solid of revolution obtained by q =0 r=0
revolving the area A about y-axis is 2p 2
r3 2r 3
= 2r 2 cos q sin q dq
Volume = v = R 2p x dx dy q =0 3 3 r=0

Chapter 02.indd 39 8/28/2015 6:59:13 PM


2.40|Engineering Mathematics

2p
8 16 x x x
= 8 cos q sin q dq
q =0 3 3 u J w
( x , y , z ) y y y
8 16 where J = =
= 8q - sin q + cos q ]q =0 = 16p
2p
(u ,J,w ) u J w
3 3
z z y
Example 17: Find the volume generated by the revolution u J w
of the rectangle formed by the lines x = 2, x = 5, y = 4 and
Jacobian of x, y and z w.r.t u, J and w and Ruw 1
is the
y = 6 about x-axis.
region of integration in u, J, w, coordinate system corre-
Solution: The volume of the solid generated by revolving sponding to the region Rxyz in xyz coordinate system.
the rectangle ABCD about x-axis = V = pydxdy

Y
R
Vector Calculus
y=6 If r is the position vector of a point P, having coordinates
D C
(x, y, z), then r = xi + y j + zk where i, j , k are unit vectors
x=2 R x=5 along OX, OY, OZ respectively, then
A y=4 B
r = xi + y j + zk = x2 + y2 + z2
O X
Given any vector v = ai + b j + ck its direction ratios are
5
5
6
6 a, b, c and its direction cosines are given by
= 2pydxdy = dx 2pydy
x =2 y =4 a b c
x =2 y =4 = ,m = ,n = and  2 + m 2 + n2 = 1
v v v
( )( )
= x ]x = 2 py 2 ]y = 4 = 3 20p = 60p
5 6

Linear combinations A vector r is said to be a linear


Change of variables The evaluation of some of the double combination of the vectors a, b, c ... etc. if there exist
(or) triple integrals can be made simple by changing the scalars x, y, z such that r = xa + yb + zc + .....
variabls.
Test of collinearity Three points A, B, C with position
1. 
In a double integral: Let a double integral vectors a, b, c respectively are collinear if there exist scalars
f ( x, y )dx dy in x and y be converted into the x, y, z not all zero such that xa + yb + zc = 0 where x + y
Rxy

variables u and J where x = f(u, J) and y = (u, J). +z=0


Then, Test of coplanarity Four points A, B, c and D with position
f ( x , y ) dxdy = f (f (u , J ) , y (u , J )) J dudJ vectors a, b, c, d are coplanar if there exist scalars x, y, z
R xyz Ru1J
and u (not all zero) such that xa + yb + zc + ud = 0 where
x+y+z+u=0
x x
( x , y ) u J Linear dependence and independence A system of
Where J = =
( u , J ) y y vectors a, b, c ... is said to be linearly independent (L.I.) if
u J xa + yb + zc + .... = 0
I s the Jacobian of x and y w.r.t u and J and Ru1 is the x = y = z .. = 0
region of integration in u, J-plane corresponding Rxy If a, b, c .... is a system of vectors which is not L.I., then
in xy-plane they are linearly dependent (L.D) and for such system of
2. In a triple integral: Let a triple integral vectors there exist scalars x, y, z ...(not all zeros) such that
f ( x, y, z ) dxdydz in x, y and z be converted into xa + yb + zc + ....... = 0
Rxyz

the variables u, J and w, where x = f(u, J, w), y = Note:


(u, J, w) and z = h(u, J, w) Every non-zero vector is L.I.
Then, Every set of four or more vectors is L.D.
Every pair of non-zero non-collinear vectors is L.I.
f ( x , y , z ) dxdydz = f (f (u , J , w ) , y (u , J , w )) Every pair of collinear vectors is L.D.
R xyz Ru1Jw
Three non-coplanar vectors are L.I.
J dudJ dw
Three coplanar vectors are L.D.

Chapter 02.indd 40 8/28/2015 6:59:23 PM


Chapter 2 Calculus | 2.41

Multiplication of vectors 4. The angle between two vectors:


Scalar or dot product a b
sinq =
If a and b are two non-zero vectors and q is the angle a b
between them (0 q p), then their dot or scalar product is
5. A unit vector perpendicular to the plane of a and b
given by a b = a b cosq . a b is a scalar.
ab
is given by n where n =
Note: a b
1. If one or both of a b are 0, then a b = 0 6. The area of parallelogram whose adjacent sides are a
a b = a (scalar components of b along a)=|b| (scalar
2. and b is given by a b
component of a along b ), 7. When the diagonals are given, the vector area of
parallelogram ABCD is 1/2 ( AC BD )
3. ab = ba
8. The vector area of the triangle ABC = 1/2 AB AC
(
4. If a, b, c are any three vectors, then a b + c = a b + a c ) ( )
( )
a b + c = ab + ac 9. If a = a1 i + a2 j + a3 k and b = b1 i + b2 j + b3 k ,
5. Two non-zero vectors a and b are perpendicular if
i j k
ab = 0 then a b = a1 a2 a3
6. i j = j i = j k = k j = j k = k i = 0 b1 b2 b3
7. a b is positive, negative or zero according as 0 q < 90, 10. The vector product is distributive with respect to
90 < q 180 or q = 90
8. The square of a vector is the square of its modulus (
vector addition a b + c = a b + a c)
()
2 2 2 2 2
i.e., a = a i = k = j = 1
Triple products
9.
m is a scalar, then
Scalar triple product The Scalar triple product of three
( ) ( )
m a b = ma b = a mb ( ) ( )
vectors a , b, c is a b c denoted by [ a b c ]
The Scalar triple product of orthonormal right handed
10. If a = a1 i + a2 j + a3 k and b = b1 i + b2 j + b3 k , then
vector triad i , j , k is equal to unity,
a b = a1b1 + a2 b2 + a3 b3 and angle between the
i.e. i j k = j k i = k i j = 1 .
vectors is
a b a1 b1 + a2 b 2 + a3 b3 1. The volume of a parallelepiped having a , b, c as co-
cosq = =
a b terminous edges = [ a b c ].
a12 + a2 2 + a33 b12 + b 2 2 + b32
2. If three vectors are coplanar then [ a b c ] = 0
11. Work done = F S 3. If two of the three vectors are equal, then their scalar
triple product is zero. That is, a a b = 0
Vector or Cross Product
a b =| a || b | sin q n where q (0 q 180) is the angle 4. If a = a1i + a2 j + a3 k , b = b1i + b2 j + b3 k , c = c1i + c2 j + c3 k ,
between a and b , and n is a unit vector such that it is per- a1 a2 a3
pendicular to both a and b. c = c1i + c2 j + c3 k , then a b c = b1 b2 b3
a , b and n (in the same order) are in the right handed c1 c2 c3
orientation (i.e., the rotation of a right handed screw from
a to b advances it in the direction of n ). 5. The volume of a tetrahedron with co-terminous edges
a , b, c is 1/6 a b c cubic units.
Note:
1. a b b a but a b = b a 6. a ( )
b c = a b c = a b c ( )
2. If a and b are parallel, then a b = 0
Vector triple product If a, b , c are three vectors, then the
3. i j = k , j k = i , k i = j and
( )
triple product a b c is called the vector triple product.
j i = k , k j = i , i k = j If a, b , c are any three vectors, then a (b c) = ( a c) b ( a b) c

a (ab bc)==0]
i i = j j = k k = 0 [In particular ( a c ) b ( a b) c

Chapter 02.indd 41 8/28/2015 6:59:49 PM


2.42|Engineering Mathematics

Vector variable A variable of the form r = x i + y j + z k Differential vectors


is called a vector variable and x, y, z are scalar variables. 1. If F = F1 i + F2 j + F3 k , then
Scalar function If t is a scalar variable on a range a t b
d F = dF1i + dF2 j + dF3 k
and a function f defined as f = f(t) for t [a, b] is called a
scalar function of t . d ( F G ) = F dG + d F G
2.
Examples: d ( F G ) = F dG + d F G
3.
f(t) = 9t3 + 4t2 + 7,
f(t) = sint + 5cost + et, etc. 4. If G = G ( x, y, z ) then

Vector function If t is a scalar variable defined on a domain G G G


dG = dx + dy + dz
[a, b], and a function F (t ) = x (t ) i + y (t ) j + z (t ) k is called x y z
a vector function of the scalar variable t.
Vector Differential Operators
Note: t is generally taken as time. is to be read as del or nabla
Differentiation If F (t ) is a continuous single valued
= i + j + k
vector function of the variable t, then the derivative of F (t ) x y z
dF F (t + t ) F (t ) 2 2 2
is defined as = Lt where t is a 2 = + 2 + 2 is called Laplacian.
dt t 0 t x 2
y z
small increment in t.
One can also look at the second and higher order deriva- Gradient of a scalar function If f(x, y, z) is a scalar
tives in a similar way. f f f
function, then i +j +k is known as the gradient of
Differentiation formula x y z
1. The derivative of a constant vector with respect to any f and is denoted by grad f. One can also write the gradient of
scalar variable is 0. f f f
f using the operator as grad f = i +j +k =f
d dF dG x y z
2. F (t ) G (t ) =
dt dt dt Now, f denotes a vector field.
d dF ds Note:
3. s (t ) F (t ) = s (t ) . + .F 1. If f is a constant, then f = 0
dt dt dt
dF dF du 2. If a vector G (x, y, z) is defined at all points in a region
4. Chain Rule: = , where F = F ( u ). we say G is a vector field. A vector field is said to be
dt du dt
5. Dot and cross products: irrational if G = grad f for some scalar function f.
3. The gradient can be used in finding the directional
d
dt
( F .G ) = F . dG
dt
+
dF
dt
.G derivative. (An example is discussed in worked
examples section)
d dG dF 4. f also gives the normal to the surface f (x, y, z) = C.
dt
( F G) = F.
dt
+
dt
G. 5. If 2f = 0, the function is called the harmonic
function.
6. Partial Derivatives: If F is vector function
dependent on F = F (x, y, z), then partial Divergence of vector F (x, y, z) be a vector field which is
derivative of F with respect to x is defined as differentiable at each point (x, y, z) in some region of space,
F F ( x + D x, y, z ) - F ( x, y, z ) i.e. F is differentiable vector field. The scalar product of the
= Lt . Like wise, one vector operator and F gives a scalar which is termed as
x Dx 0 Dx
divergence.
F F
and
can also define can be defined. F F F
y z F = i + j +k
It is also possible to define higher order partial x y z
derivatives as
Note: If div( F) or F = 0, then F is called solenoidal
2 F F 2 F F
= , 2 = .
x 2
x x y y y Curl of a vector Let F (x, y, z) is a vector field defined for
all (x, y, z) in a certain region of space and is differentiable,
2 F F i.e. F is a differential vector field. The cross product of the
= , etc
x z x z vector operator with the vector F is termed as curl F.

Chapter 02.indd 42 8/28/2015 7:00:05 PM


Chapter 2 Calculus | 2.43

1 0
i j k = (x - x2 - 1 - x2 + 2x) dx + y2dy
0 1

curl F = ; F = F1i + F2 j + F3 k 1 1

x y z = (- 2x2 + 3x - 1) dx - y2 dy
0 0
F1 F2 F3 = (- 2/3 + 3/2 - 1 - 1/3) = - 1/2.

Note: If curl F = 0, then F is said to be irrotational. Surface Integral


Standard Results Let S be a closed surface, then the normal surface integral
F N ds is called the flux of F over S.
1. div (f f ) = f div f + fgrad f or ff = f f + f f s
2. curl (f f ) = f f + f curl f

3. div ( f g) = f curl g - g curl f Cartesian form Let F ( r ) = F1 i + F2 j + F3 k where, F1, F2,


4. f = div (grad f ) or f = 2 f F3 are continuous and differentiable functions of x, y, z. If
5. curl (grad f) = 0 or (f ) = 0 i.e., curl of a gradient cosa, cosb and cosg be the direction cosines of the unit
equals 0. normal N, then
6. div (curl f ) = 0 or ( f ) = 0
N = i cos a + j cos b + k cos g
7. curl (curl f ) = grad (div f ) - 2 f (or)
( f ) = ( f ) - 2 f S S
(
F.N ds = F1 cos a + F2 cosb + F3 cos g ds )
But then ds cosa, ds cosb and ds cosg are the projections
Integration of ds on yz, zx and xy planes. If dx, dy, dz are the differen-
tials along the areas, then ds cosa = dy dz; ds cosb = dz dx;
Line Integral ds cosg = dx dy.
Let F (x, y, z) be a vector function defined on a region of
(
F .N ds = F1dy dz + F2 dz dx + F3 dx dy )
space and let C be curve in that region, the integral F dr S
S

is called the line integral. x = b c Note: If R1 is the projection of S on xy-plane, then


For Riemann Integration, f dx the limits of integration dx dy
x=a FNds = FN
are along the line segment joining (a, 0), (b, 0), where a < b. s R1 cosg
Here, instead of line, we integrate along the curve C. dx dy
= FN.
(|N k |= cosg)
Circulation The line integral around a closed curve C | N.k |
i + y 2 by
j 
denoted Fdr is called circulation of F around C. Equivalently,
Example 18: Evaluate Fdr, where F = xyi + y 2 j along
 the dx dy dz dx
F .N ds = F .N
= F .N
S R2 R3
triangle x = 0, y = 0 and x + y = 1 in the first quadrant. | N.i | | N. j |
Solution:
y Volume Integral
x2 y2 z 2
C2 f(x, y, z) dz dy dx
C3 x1 y1 z1

x
0 C1
x2 y2 z2
dy dx
= x f (x, y, z) dz
Fdr =
c

C1
xy dx + y dy + 2

C2
xy dx + y dy +
2

C3
xy dx + y dy
2
1 y1 z1

C1 C2 C3
y=0 y=1x x=0 Gauss divergence theorem If F is continuously
0<x<1 0<x<1 dx = 0 differentiable vector function in the region bounded by a
dy = 0 dy = dx 1<y<0 surface S, then the F .N ds = div F dv where N is the unit
S V
normal to the surface.
1 1 0

[x (0 ) dx + 0 + x (1 x ) dx + (1 x ) ( dx ) + y 2 dy
2
= Greens theorem If P and Q are scalar point functions,
x=0 x=0 1 possessing continuous derivatives of the first order, in a

Chapter 02.indd 43 8/28/2015 7:00:16 PM


2.44|Engineering Mathematics

region S of the xy plane bounded by a closed curve C, then 2 f


= 6x - 0 = 6x
Q P x 2
Pdx + Qdy = dxdy .
C S x y f
= -12xy - 9xz
Stokes theorem If S is an open surface bounded by a closed y
curve C and f is a continuously differentiable vector point 2 f f
= = (- 12xy - 9xz)
function, then F .dr = cul F .N ds , where N is unit normal x y x x x
C S
drawn at any point on the surface. = - 12y - 9z
Example 3: If f x3 + y3 + z3 - 3xyz, then find the value of
Solved Examples grad f at (2, 1, 1).
f f f

Solution: grad f = i +j + k (by definition)
Example 1: If A = x 3 i + x 2 j + x k and B = x i + x 2 j + x 3 k , x y z
then find the values of
d d = i (x3 + y3 + z3 - 3xyz) + j (x3 + y3 + z3 - 3xyz) +
(i) (A B) and (ii) (A B) x y
dx dx
k (x3 + y3 + z3 - 3xyz).
z
d d d
Solution: (i) (A B) = A (B) + B (A) = 3[ i (x - yz) + j (y - xz) + k (z - xy)]
dx dx dx
d grad f at (2, 1, 1)
= (x3 i + x j + x k ) (-x i + x j + x3 k ) +
dx = 3 [ i (4 - 1) + j (1 - 8) + k (1 - 2)]
d 3 = 9 i - 21 j - 3 k
(- x i + x j + x3 k ) (x i + x j + x k )
dx
then find the value of b if
Example 4: If f = i t2 + 2bt j + k,
= (x3 i + x j + x k ) (- i + 2x j + 3x k ) + (- x i + x
j + x3 k ) (3x i + 2x j + k ) (i) magnitude of f is constant
(ii) direction of f is constant
= - x3 + 2x3 + 3x3 - 3x3 + 2x3 + x3 = 4x3.
Solution: (i) When the magnitude of f is constant,
d dB dA df
(ii) (A B) = A + B f. = 0.
dx dx dx dt
d
d (-x i + x2 j + x3 k ) + ( i t + 2bt j + k ) (i t + 2bt j + k ) = 0
= (x3 i + x j + x k) dt
dx 2t3 + 4b2t = 0 b2 + t2 = 0 b = 0.
d 3 2 df
(x i + x j + x k ) (- x i + x j + x3 k ) (ii) when the direction of f is constant, f = 0.
dx dt
d
= (x3 i + x j + x k ) (- i + 2x j + 3x k ) ( i t + 2bt j + k ) ( i t + 2bt j + k ) = 0
dt
+ (3x i + 2x j + k ) (- x i + x j + x3 k ) i j k
t 2 2bt 1 = 0
i j k i j k
2t 2b 0
= x x
3 2
x + 3x 2 2x 1
1 2 x 3 x 2 x x 2 x3 - 2 i b - j (- 2t) + k (2bt - 4bt) = 0
b = 0.
= i (5x4 - 3x) - j (6x5 + 2x) + k (5x4 + 3x)
Example 5:If p = x y i - x3 j +xyz2 k , then find div p and
Example 2: If f = x3 - 6xy - 9xyz is a scalar function, then curl p.
2 f 2 f Solution:
find 2 , .
x x y (i) div p = p
Solution: f = x3 - 6xy - 9xyz
= (x2y) - (- x3) + (xyz2)
f x y z
= 3x - 6y - 9yz
x = 2xy - 0 + 2xyz = 2xy (1 + z)

Chapter 02.indd 44 8/28/2015 7:00:45 PM


Chapter 2 Calculus | 2.45

i j k Let n1 = grad f at (1, - 2, 1) and n2 = grad g at (1, - 2, 1)


respectively
(ii)curl p = p = x y z
x 2 y x 3 xyz 2 n1 = ( grad f ) (1, 2,1)

= i [(- 2) 1 - 3] + j [2 (A) (- 2) 1] + k [1 (- 2) - 2(A)]


= i ( xyz ) ( x 3 ) - j ( xyz 2 ) ( x 2 y )
2

y z x z = i - 4 j + 2 k
n2 = (grad g ) (1, - 2, 1)
+ k ( x 3 ) ( x 2 y )
x y
= i [6 (A)] + j [- 2 (- 2)] + k (B) = 6 i + 4 j + 2 k
= xz i - yz j + k (-3x- x) = xz i - yz j - 4x k

Let the angle between the normals n1 and n2 be q.
Example 6:Find the value of r if p = xy 2 i + xyz 2 j + ( r 2) xyz 3 k
So, n1 .n2 = |n1| |n2| cos q 6 - 16 + 4
p = xy 2 i + xyz 2 j + ( r 2) xyz 3 k is solenoidal at (1, - 1, 1).
Solution: p is solenoidal div p = 0 p = 0
(
= 1 + 16 + 4 )(
36 + 16 + 4 cos q )
p1 p2 p3 6 3 3
+ + =0 cos q = = =
x y z 21 56 7 6 7 6

xy + (xyz) + [(r - 2) xyz3] = 0
x y z 3
y + xz + (r - 2) 3xyz = 0 at (1, -1, 1), div p = 0 q = cos-1
7 6
(- 1) + (1) + (r - 2) 3 (1) (- 1) (1) = 0
1 + r - 3r + 6 = 0 r = 7/2.
Example 9:If F = ( x 2 + y 2 ) i 2 xyj evaluate  F.dr
Example 7: Find the value of a, if P = (y + 2xz) i + (z + along the straight line C from (0, 0, 0) to (1, 2, 3).
2xy) j + (x + ayz) k is irrotational.
Solution: The equation of the line joining (0, 0, 0) and (1,
Solution: The vector P is irrotational x y z
2, 3) is = = = (t).
curl P = 0 P = 0 1 2 3
i j k Then, along the line C, x = t, y = 2t, z = 3t.
x y z = 0 r = xi + yj + zk = ti + 2tj + 3tk dr
y 2 + 2 xz z + 2 xy ayz + x 2
2
= i +2 j + 3 k

= i ( x 2 + ayz ) ( 2 xy + z 2 ) Given F = ( x 2 + y 2 ) i 2 xyj
y z
And along c F = t 2 + ( 2t ) i 2t ( 2t ) j = 5t i - 4t j
2


j ( x 2 ayz ) ( y 2 + 2 xz ) Fdr = (5t - 8t + 0) dt = - 3t dt
x z
at (0, 0, 0), t = 0 and at (1, 2, 3), t = 1.

+ k ( z 2 + 2 xy ) ( y 2 + 2 xz ) = 0
1 1
3t 3
x y Fdr = - 3t dt = =-1
c t =0
3 0
i (az - 2z) + j (2x - 2x) - + k (2y - 2y) = 0
i z (a - 2) = 0 = 0 i z (a - 2) = 0 Example 10: If F = 3x i - z2 k , evaluate  F.dr,
f .dr where the
z = 0 or a - 2 = 0 a = 2 curve c is the rectangle in the xz bounded by z = 0, z = 2, x = 0,
x = 3.
Example 8: Find the angle between the surfaces xy z = 3x
+ z and 3x - y + 2z = 1 at (1, - 2, - 1). Solution: As the integration takes place in xz-plane (y = 0)
Solution: Let f = xyz - 3x - z = 0 and g = 3x - y + 2z F . dr = f1 dx + f2 dz = 3x dx - z dz
- 1 = 0. c c c

(i)Along OP:
grad F = i (yz - 3) + j (2xyz) + k (xy - 2z)
z = 0, dz = 0 and x varies from 0 to 3
grad g = i (6x) + j (-2y) + k (B) 3 3
3x 2 27
But, angle between two surfaces at a point is equal to angle F dr = 3xdx = =
between the normals to the surfaces at that point. 0 2 0 2

Chapter 02.indd 45 8/28/2015 7:01:08 PM


2.46|Engineering Mathematics

2 Example 12:By applying Gauss theorem, evaluate


z

R (0, 2) Q(2, 3) ( x dydz + x


s
3 2
y dzdx + x 2 z dx dy ) , where S is the closed
surface consisting of the cylinder x + y = a and the circular
discs z = 0 and z = b.

O P X Solution: We have
(3, 0)
F1 = x3; F2 = xy; F3 = xz
(ii)Along PQ: F1 F2 F3
= 3x 2 , = x2 , = x2
x = 3, dx = 0 and z changes from 0, 2. x y z
2 2
z3 8 F1 F2 F3
Fdr = 0 -z dz = 3 = + + = 3x + x +x = 5x
0 3 x y z
(iii)Along QR: Using Gauss theorem,
y = 2, dy = 0 and x changes from 3 to 0
0
x2 27
0 F dy dz + F
1 2
dz dx + F3 dx dy
Fdr = 3x dx = 3 =
2 3 2
s

F F F
= 1 + 2 + 3 dx dy dz
3

(iv)Along RO: x y z
x = 0, dx = 0 and y varies from 2 to 0. x 3 dydz + x 2 dzdx + x 2 dxdy
0 0
z 3 8 s
Fdr = - z dz = - 3 =
3
a a2 x 2 b

27 8 27 8
2 2 = 5x 2 dxdydz = 20 x 2 dx dy dz
Thus,  F.dr = + =0
2 3 2 3
x x=0

a
y=0

a2 x 2
z=0

Example 11: Evaluate by Greens theorem (C) (xy + y)dx = 20 x 2 b dx dy


+ x dy, where C is the closed curve of the region bounded x=0 y=0

by y = x and y = x. a

= 20b x 2 a 2 x 2 dx
P
Solution: Here, P = xy + y = x + 2y 0
y
Q [Let x = a sin q; dx = a cosq dq
Q = x = 2x
x Upper limit : x = a a sin q = a q = z
Hence, by Greens theorem, Lower limit : x = 0 a sinq = 0 q = 0]

( xy + y ) dx + x dy = ( 2 x x 2 y ) dx dy
2 2 p 2

C s
= 20b a sin q a 2 (1 sin 2 q ) a cosq dq
0
1 x2 p 2
= ( x 2 y ) dx dy = ( x 2 y )dy dx
s x=0 y = x
= 20a4 b
0
sinq cosq dq

y p 2

= 20a4b
0
1/4 sin 2q dq
(1, 1) p 2
1 cos 4q
(0, 0)
= 5a4b
0
2
dq
x p
5a4b sin 4q 2
= q-
y=x 2 4 0
y = x2
5a 4 b p 5p 4
= 0 0 = ab
1 x2 1 2 2 4
= [ xy y ]
x=0
2
y= x
=
x=0
(x3 - x4)dx
Example 13: Evaluate F.dr by Stokes theorem, If F =
1
x 4 x5 c
(x + y) i - 2xy j , where c is the rectangle formed x = a,
= = 1/4 - 1/5 = 1/20
4 5 0 y = 0 and y = b.

Chapter 02.indd 46 8/28/2015 7:01:23 PM


Chapter 2 Calculus | 2.47

Solution: F = ( x 2 + y 2 ) i 2 xyj 4 y dx dy
R
By Stokes theorem,
As N k ds = dx dy
( F ) Nds = F dr c
And R is the region bounded by the rectangle.
a b

i j k
= ( 4 y ) dx dx
x=a y=0

a b
F = = 4 yk 4 y 2
x y z = 2 0 dx
a
x2 + y2 2 xy 0 a

=-2 (b 2
0 ) dx
(
F Nds ) a

= - 2b2 [ x ] a = - 4ab2.
a

= (- 4ky) N ds = 4 y ( N .K ) ds

Exercises
Practice Problems 1 9. Lt |x - 2| + [ x - 2] =
x 2
Directions for questions 1 to 105: Select the correct alternative
(A) 0
from the given choices.
(B) only left limit exists
x2 - 1 (C) only right limit exists
1. Lt , x -1
x +1
x -1 (D) limit does not exist
(A) 2 (B) 0 10. Statement I
(C) 2 (D) Does not exist
sin x - e x + 1
lim exists
x
{
2. Lt 3 x - 9 x 2 - x = ______. }
x 0
x
sin x ex - 1
1 and lim = 1, lim = 1 and
(A) (B) 3 x 0 x x 0 x
6
(C) 6 (D) None of these Statement II
If lim ( f ( x ) - g ( x ) ) exists then, both lim f ( x ) and
24 cos x - 24 + 12 x 2 - x 4
3. Lt
xa xa

24 x 6 = lim g ( x ) exist.
x 0
xa

1 -1 (A) Statement I true, Statement II false


(A) (B)
720 120 (B) Statement I true, Statement II true
(C) Statement I false, Statement II true.
1 -1
(C) (D) (D) Both are false
120 720
11. Statement I
4. If 0 < x < y, Lt ( y n + x n )
1
n = f(x) = [x]2 - 5[x] + 3, 1 < x < 4, where [ ] represents the
n
greatest integer function is not continuous at x = 2 and 3 but
(A)
e (B)
x (C)
y (D)
nxn-1 is bounded. and
5. Evaluate Lt (x [x]), where [x] is the greatest integer less Statement II
x 2.7
than equal to x) A continuous function in a closed interval I is bounded.
(A) 0.3 (B) 0.7 (C) 4.7 (D) 2 (A) Statement I true, Statement II false
1 (B) Statement I true, Statement II true
6. Evaluate Lt 189 . (C) Statement I false, Statement II true
x 0 x
(A) 0 (B) (D) Both are false
(C) (D) None of these 12. Statement I
1x
2 +3
7. Lt
x x cos x
= lim =1
x 0
2 x x
(A) 1 (B) 3 (C) 6 (D) 2 and
Statement II
8. Evaluate Lt ( 3x + 4 x ) , (x 1)
1x
tan x
x
lim =1
(A) 4 (B) 3 (C) 2 (D) 1 x 0 x

Chapter 02.indd 47 8/28/2015 7:01:35 PM


2.48|Engineering Mathematics

(A) Both are false 20. Let f(x) = max (1 - x, x2 -1). Then f is
(B) Both are true (A) not continuous at x = 1, -2
(C) Statement I true, Statement II false (B) continuous and differentiable everywhere
(D) Statement I false Statement II true (C) not differentiable at x = -2, 1.
13. Let the function f(x) = [x]. where [x] is the greatest integer (D) continuous but not differentiable at x = 1, -1
less than or equal to x. Which of the following is /are true? 21. Which of the following functions is differentiable at x = 0?
(A) f(x) has jump discontinuity at all x Z (A) sin |x| + |x| (B) sin |x| - |x|
(B) f(x) has removable discontinuity at all x Z (C) cos |x| + |x| (D) cos |x| - |x|
(C) f(x) is continuous at all irrational values. 1 1
(D) both (A) and (C) 22. Consider the function f(x) = + defined in the
x -1 3 - x
interval [1, 3]
5x - 4 0 < x 1
14. f(x) = at x = 1 (P) f is continuous on [1, 3]
4 x - 3x 1 < x < 2
2
(Q) f is differentiable on (1, 3)
(A) Left hand continuous at x = 1 (R) there exists c (1, 3) such that f 1(c) = 0 which of the
(B) Right hand continuous at x = 1 above statements are true?
(C) continuous at x = 1 (A) P, Q only (B) Q, R only
(D) None of these (C) P, R only (D) P, Q, R
x sin x 23. The derivative of 5th root function of f(x) = x3
15. The function f ( x ) = is
( x 2 + 2) (A) x 3 5 (B)
3 25
x
(A) continuous for all x 5
(B) discontinuous for all x 3
(C) (D) None of these
5
(C) constant function 5 x2
(D) discontinuous only at x = 2 24. A function f: R R is such that f(x + y) = f(x) . f(y) for all x,
16. Check the continuity of the following function y in R and f(x) 0 for any x in R. If f(x) is differentiable and
sin 2 ax f (0) = 2, then
, when x 0
f(x) = x 2 at x = 0 (A) f (x) = 2f(x) (B) f(x) = 2f (x)
a 2, (C) f(x) = f (x) (D) f (x) = -f(x)
when x = 0
1 p
(A) continuous at x = 0 25. The function f(x) = x(1 - x cot x) - in 0, has
x 2
(B) discontinuous at x = 0 (A) only one minimum
(C) discontinuous of first kind (B) only one maximum
(D) None of these (C) no extrema
7 x < 5, (D) one maximum and one minimum

17. If f(x) = ax + b 5 < x < 7, is continuous on R then the 26. Which of the following statement(s) is/are true?
11 x>7 (A) y = x2 has a minimum value at x = 0
,
(B) y = x 3 has a minimum value at x = 3
values of a and b are 1
(A) a = 2, b = 3 (B) a = 2, b = 3 (C) The maximum value of the function y = is 1
1 + x2
(C) a = 3, b = 2 (D) a = 2, b = 3 (D) all the above
cos 3x - 1 27. The maximum and minimum values of f(x) = 3 sin2 x + 4 cos2 x is
, x0
18. If f ( x ) = 5 x 2 + 1 - 1 , find the value of l so that (A) {-4, -3} (B) {7, 3}
(C) {4, -3} (D) {4, 3}
l , x = 0
28. If the function f(x) = 2x3 - 9ax2 + 12a2 x + 1, where a > 0,
f(x) is continuous at x = 0.
attains its maximum and minimum at x = p and x = q, respec-
(A) 9/5 (B) 5/9 (C) -5/9 (D) -9/5
tively, such that p2 = q, then the value of a is
19. Find values of a, b, c so that 1 1
(A) 2 (B) (C) (D) 4
a cot -1 ( x - 3) , 0 x<3 4 8

bx, x=3
Linked answer for Questions 29 and 30: The sum of the
f(x) = -1
1 hypotenuse and one side of a right angled triangle is given as a units.
c tan x - 3 , 3< x < 4
29. When the area is maximum the ratio of the side and the
cos -1 ( 4 - x ) + ap , 4 x5 hypotenuse is ______.
is continuous in the interval [0, 6]. (A) 2 : 1 (B) 1 : 3 (C) 1 : 2 (D) 2 : 3
(A) a = 2/3, b = p/9, c = 2/3 30. When the area is maximum, find the angle between the hypot-
(B) a = -2/3, b = -p/9, c = -2/3 enuse and the other side is ______.
(C) a = 3/2, b = p/9, c = -2/3 (A) 60 (B) 30
(D) a = -3/2, b = -p/9, c = 2/3 (C) 45 (D) None of these

Chapter 02.indd 48 8/28/2015 7:01:41 PM


Chapter 2 Calculus | 2.49

31. Consider f(x) = |x2 - 3|, 0 x 6 and g(x) = f f f


41. x +y +z =
3 , 0 x 1,
x x y z
. Then Rolles theorem can be applied in (A) tan 2 f (B) cot 2 f
4 - x, 1 < x 3
(C) cos 2 f (D) sin 2 f
the respective intervals
(A) to both f(x) and g(x) 42. The stationary points of the function f(x, y) = x3 + y4 27x +
(B) only to f(x) 32y + 100 is/are
(C) only to g(x) (A) (3, 2), (3, 2) (B) (3, 2), (3, 2)
(D) neither to f(x) nor to g(x) (C) (3, 2), (3, 2) (D) (3, 2), (3, 2)
32. If the function f(x) = px2 + qx2 + rx + s on [0, 1], satisfies the 43. For the function f(x, y) = 2x2 + 4y2 + 4xy + 2x + 10y + 7
Mean Value theorem, then the value of c in the interval (0, 1) is (A) Local maximum exists, but no local minimum
1 1 2 (B) Local minimum exists, but no local maximum
2
(A) (B) (C) (D) (C) Neither local minimum nor local maximum exists
2 3 3 3 3 (D) Both local minimum and local maximum exists
33. In the interval [1, 2] the value of q to the function 44. For the function xyz, if x + y + z = 3, then the local maximum
f(x) = 2x3 5x + 3 is equal to occurs for xyz at the point _____
2 7 1 1
(A) (B) (A) (B) (5, -1, -1)
3 3 4, 2 , 2

7 4 (C) (1, 1, 1) (D) (7, -3, -1)
(C) 1 (D)
3 5 45. The ratio of the dimensions of a rectangular box of volume
x2 64 cubic and open at the top that requires least material for its
34. f(x) = increases in construction is
x +1
(A) 2 : 2 : 1 (B) 2 : 4 : 5
(A) (- 2, 0) (B) [- 4, -2]
(C) 2 : 3 : 4 (D) 1 : 2 : 3
(C) (- , - 2] [0, ) (D) (- , - 2) (0, )
46. Which of the following function/s is/are integrable but not
35. Let f(x) = eax and g(x) = eax be two functions defined in [p,
continuous on (0, 10)?
q], If the functions satisfies Cauchy Mean Value theorem then
(A) f(x) = [x] (greatest integer function)
the value of c is ______.
(B) f(x) = |x - 3|
p+q (C) f(x) = |x - 5| + |x - 2|
(A) p + q (B)
2 (D) f(x) = x2 + 5x + 9
(C) 2(p + q) (D) None of these 1 2n
z ( e + e 4 n + + e 2 n n ).
47. Evaluate Lt
n n
36. If x = cos (z + y2), then =
y 1
(A) e2 - 1 (B) ( e - 1)
(A) 1 (B) y - 2y
(C) 2y (D) 2
1 2 1 2
37. The function f(x, y, z) = x y 2 y z + 5 x z 3 xyz , is a (C) ( e - 1) (D) ( e + 1)
2 2
2 2 2 2 2

x 4 + y 2 z 2 + xy 2 z 2 2

48. Evaluate Lt
homogeneous function of order _______. n n n n
+ + + ......... + 2
(A) 1 (B) 2 (C) 4 (D) 0 n 1 + n 2 4 + n2 9 + n2 2n

6
4 x+4 y p p p p
, then x u + y u =
(A) (B) (C) (D)
38. If u = 4 3 6 2
6 x+6 y x y
49. sec3 x dx =
u 4
(A) (B) (C) 4u (D) 6u sec x tan x
2 u (A) + log ( sec x + tan x )
3
u u
( )
39. If u = log x + x 2 + y 2 , then eu
y
-y
x
= (B)
sec 2 x tan x 1
3
p
+ log tan + x
3 4
(A) 0 (B) 1 (C) -1 (D)
u
sec x tan x 1 p x
(C) + log tan +
Linked answer for Questions 40 and 41: 2 2 4 2
x5 2 + y5 2 + z5 2 (D) None of these
Let u = tan f =
x1 2 + y1 2 + z 1 2 p 2
50. sin 4 x cos6 xdx = ______.
u u u 0
40. The value of x +y +z is equal to
x y z 3p 2p 3p 3p
(A) 2 tan f (B) tan f (A) (B) (C) (D)
128 425 2560 512
(C) 2 sin 2 f (D) sin 2 f

Chapter 02.indd 49 8/28/2015 7:01:55 PM


2.50|Engineering Mathematics

51. Area bounded by the curve y2 = x and the line x = 3 is 62. I converges to
________ sq units 2 1 1
(A) (B) (C) (D) 2q
(A) 2 3 (B) 4 3 (C) 6 3 (D) 8 3 q q q2
52. Area bounded by the curve y = 3x2, x = 2 and the two coor- p /4 p /4
dinate axes is ______ sq units 63. Evaluate (3 cosq + 4 sin q ) dqdf ______.
(A) 2 (B) 3 (C) 6 (D) 8 0 0

53. The volume of the solid obtained by revolving the area 2 -1 ( 4 2 - 1)p
bounded by the parabola y2 = x - 4, x - axis and the lines
(A)
p (B)
2 4 2
x = 4 and x = 7, about x - axis is _______ cubic unit
9 11 13 15 ( 4 2 - 1)p 4 2 -1
(A) p (B) p (C) p (D) p (C) (D)
2 2 2 2 2 4 2
54. The length of arc of the curve y = ln (cos x) from x = 0 to p q
dxdy
p
x = is _______
64.
1 2
xy
= _______.
4
( )
(A) ln 1 + 2 (B) ln ( 2 - 1) (A) logp.log
q
(B) logp logq
2
(C) ln ( 2 + 3 ) (D) ln ( 2 - 3 ) p q
(C) log (D) log

1 2q 2p
55. dx = ______ .
1.0001
1x
1 1- x
2

dxdy
(A) 1000 (B) 100000 65. Evaluate
1- x 2 - y 2
.
(C) 10000 (D) 1000000 0 0

3
1 p p
56. dx = ______ . (A) (B) 0 (C) (D) 1
4 2
( x - 2)
45
0

log t x
(A) 5 - 21/5 (B) 5 + 21/5 66. e x + y dxdy ______.
(C) 5(1-2)1/5 (D) 5 1 + 21 5 0 0

0
dx t2 1 t2 - t
57. converges to ______. (A) - t + (B)
2 2 2
(1 - 4 x )
2
-
t -1
2
t + t -1
2
1 3 1 4 (C) (D)
(A) (B) (C) (D) 2 2
2 4 4 5
2 e2
dydx
1
1 67. The value of is
58. 2 5 dx = ______ 0 ex ny 2
-1 x 1 2
e2 - 1
(A) (B) (e - 1)
10 5 5 2
(A) 5 (B) (C) (D)
3 2 3 1 2
e2 + 1
(C) (D) (e + 1)
59. Which of the following converges? 2

1 dx 68. By changing the order of integration, the integral

I. dx II.
x-2
x4 + 1 0 4 x5 + 1
f ( x, y ) dxdy becomes _______
0

(A) only (I) (B) only (II) 2 0

(C) both (I) and (II) (D) neither (I) nor (II) y+2
(A) f ( x, y ) dxdy (B)
f ( x, y ) dxdy

4x + 7 0 2 0 y+2
60. dx is ______ .
3 x 6 + 10
1
(C) f ( x, y ) dxdy (D)
f ( x, y ) dxdy
(A) Convergent 0 0 1 y+2
(B) divergent
dxdy
(C) cannot be determined 69. By changing the variables in the double integral ,
(D) none of these xy R

where x = eu+ and y = u, it changes to f ( u, v ) dudv then


Linked answer for Questions 61 and 62: R
f(u, ) is

Consider I = e
0
- qx
dx
(A) R
eu + v
(B)
(e )(uv )
u+v uv
61. I converges if
(A) q 0 (B) q > 0 1 1 1 1
(C) + (D) -
(C) q < 0 (D) q 0 v u v u

Chapter 02.indd 50 8/28/2015 7:02:17 PM


Chapter 2 Calculus | 2.51

70. By changing the variables from x, y to u, where x = u + 2 81. The acute angle between the surfaces f and g at (1, -1, 2) is
and y = 4u + 3, the given integral f ( x, y )dxdy changes to
15 15
R (A) cos-1 (B) cos-1
f ( u + 2v, 4u + 3v )Y ( u, v ) dudv then Y (u, ) is ______ 390 390
R
(C) 60O (D) 30O
(A) 5 (B) -5 (C) 1/5 (D) -1/5
82. The directional derivative of F = x3 y + y3 z + z3 x in the direc-
71. By changing a triple integral f ( x, y, z ) dxdydz in rec- tion of i + 2 j + 2k at (0, 1, - 1) is
R
tangular coordinates x, y and z to cylindrical coordinates
-4 -5
p, f and z we get f ( x, ( p, f , z ) , y ( p, f , z ) , z )Y ( p, f , z )
5 4
(A) (B) (C) (D)
R 3 3 3 3
f ( x, ( p, f , z ) , y ( p, f , z ) , z )Y ( p, f , z ) dpdfdz, then the
R 83. The magnitude of maximum directional derivative of f = 2xy2
value of (p, f, z) is - xyz + y2 z in the direction from the point (1, - 1, 1) is
(A) p (B) cos f
(C) z (D) p cos f (A) 62 (B) 52 (C) 62 (D) 56
72. The area bounded by the circle x2 + y2 = 6 and the parabola y 84. The values of constants a, b, c respectively such that the
= x2 is given by directional derivative of the scalar function f = ax3 + by3 + cz3
at (1, - 1, -1) has maximum magnitude of 3 in the direction
2 x2 6 2 6 x2
(A)
dydx (B) dydx parallel to y - axis is
x = 2 y = x x= 2 y= x (A) 0, 1, 0 (B) 1, 0, 1
2 x2 -6 2 6 - x2 (C) 0, 1, 1 (D) 1, 1, 0
(C)
x = -2 y = x
( x 2 + y 2 ) dydx (D) x = -
2 y= x
( y - x 2 ) dxdy 85. The directional derivative of a scalar point function is a
function of
73. The volume of the region under the plane 12x + 4y - 3z = 0, (A) only direction (B) only position
z 0, and above the region bounded by x = 1, x = 3, y = x2 (C) either A or B (D) both A and B
and y = 4x2 is _____ cubic units
86. The values of div r and curl r respectively when r = 2x
(A) 240 (B) 484 (C) 724 (D) 860
i - y j + 3z k is
74. The volume of the solid generated by revolving the triangle
formed by the lines y = 1, y = x and x + y = 4 about x - axis is (A) 4; i (B) 0, 0 (C) 4, 4 k (D) 4, 0
_____cubic units 87. The necessary and sufficient condition that the force field
2p 4p 8 F (x, y, z) is conservative is
(A) (B) (C) 2p (D) p
3 3 3 (A) (curl F ) = - F (B) div F = 0
75. The volume of the solid bounded by the planes x = 0, (C) curl F = F (D) curl F = 0
y = 0, z = 0 and x + y + z = 4 is ___ cubic units 88. Which of the following is/are true?
32 64 (A) . ( r a ) = 0
(A) (B) (C) 32 (D) 64
3 3 (B) Grad (r . a ) = a
76. The acute angle between the vectors 3i + j + 2k and i - j + k is
(C) (r a )=-2a
q, then the value of cosq is
(D) All the above
8 8 8
(A) (B) (C) 21 8 (D) 89. If F = (x + y + 1) i + j - (x + y) k , then the value of F .
21 21 21
curl F =
77. If r is the position vector of a particle which passes along the (A) 0 (B) 3 (C) 2 (D) 1
curve x = 3 sin 4t, y = 3 cos 4t, and z = 5t (t > 0). The magni-
90. Compute the value of div (f f).
tude of its velocity and acceleration respectively are
(A) f curl (f) (B) f curl (f)
(A) 13, 45 (B) 12, 48 (C) 13, 48 (D) 12, 45
(C) curl (f f) (D) 0
df
78. f (t ) be a vector function and f = 0 implies 91. For what value of p the vector f = (2x + 3y) i + (z + 2y) j +
dt
(A) f is a vector function with constant magnitude (x - pz) k is solenoidal?
(B) fis a vector function both in direction and magnitude (A) 4 (B) - 4 (C) 2 (D) 0
(C) f is a vector function of constant direction 92. For what values of p, q and r the vector f = (x + ry - z) i
(D) (D) Either A or C + (3x - y + qz) j + (px + y - z) k is irrotational?
(A) p =1, q = -1, r = 3
79. If r = xi + y j + z k and | r | = r, then rn =
(B) p = -1, q = 1, r = 3
(A) n (n - 1) rn-1 r (B) n (n - 2)rn-2 r (C) p = -1, q = 1, r = - 3
(C) n.r n -2
r (D) n(n - 1) r (D) p =1, q =1, r = - 3
Linked answer for Questions 80 and 81: Two equations f = xy2 93. If f = yz i + zx j + xy k , then f (x, y, z) =
z - 2y + z2 and g ; x2 + yz - x - 2 represents two surfaces (A) xyz + f(y, z); f constant
80. Find normal vector to g at (1, -1, 2) (B) xyz + g(x, z); g constant
(A) i + 2j + 2k (B) i + 2j - k (C) xyz + h(x, y); h constant
(C) 2 i - j - k (D) i - j - 2k (D) xyz + k; k is a constant

Chapter 02.indd 51 8/28/2015 7:02:41 PM


2.52|Engineering Mathematics

94. If F = ( 5 xy - 6 x 2 ) i + ( 2 y - 4 x ) j, compute the line integral 100. Find the area of the region in the first quadrant bounded by
x
the curves y = 4x, y = 1/x and y =
F dr where C y = x3 in the xy-plane joining (1, 1) and (2, 8).
C
4
using Greens theorem.
1
(A) 35 (B) - 32 (C) 12 (D) 18 (A) log 2 (B) log 2
2
95. Compute x y ds around the circle x = cos t and y = sin t.
S
2 2 (C) log 4 (D) log 16

p/4
(A) (B) 0 p
(C) p/2 (D) 101. Evaluate F nds where F = 2xz i - yz j + yx k where
s
96. If F = y2 i - 2xy j , compute the circulation F dr, where S is the cube bounded by x = 0, x = 3, y = 0, y = 3 and
C z = 0, z = 3.
C is the rectangle bounded by y = 0, y = 1, x = 0 and x = 2. 27 81 27 81
(A) 3 (B) 4 (C) - 4 (5) - 3 (A) (B) (C) (D)
2 4 4 2
Linked answer for Questions 97 and 98: A particle in the force
Linked answer for Questions 102 and 103: For the force field
field F = 2x2 i + (y - 3xz)j + 2z k is moving along a space curve
F = x2 i + xyj in the square region in the xy - plane bounded by the
defined by x = 2t, y = t2, z = 3t2- 2
lines x = 0, y = 0, x = 2, y = 2.
97. Find F . d r 102. Find the value of curl F
(A) (- 36 t4 + 38 t3 - 40 t2 + 24 t) dt (A) xi (B) yj (C)
yk (D)
xk
(B) (- 36 t4 + 38 t3 + 40t2 - 24 t) dt
(C) (36 t4 - 38 t3 + 40 t2 - 24 t) dt 103. Using Stokes theorem, find the value of F.dr .
(D) (3 t4 - 38 t3 + 40 t2 - 24 t) dt C
(A) 4 (B) 6 (C) 8 (D) 2
98. Find the work done by F in moving a particle along the
straight line from A (0,0,0) to B (2, 1, 1). 104. Evaluate the volume integral div N dv , where N is the
V
107 121 113 109 outward drawn normal to the surface described by x +
(A) (B) (C) (D)
30 30 30 30 (y - 5) + (z - 8) = 12.
(A) 8p (B) 12p (C) 48p (D) 24p
(x2ydx + xy2 dy) using greens theorem where C is
99. Evaluate 
C 105. If S is a closed surface and n is unit normal to the surface S
the triangle with vertices (0, 0), (2, 0) and (2, 1). then r . nds =
11 11 -11 11 S
(A) (B) (C) (D)
24 12 6 4 (A) 4V (B) 3V (C) 2V (D) V

Practice Problems 2 (A) n ! (B) (n - 1)!


Directions for questions 1 to 115: Select the correct alternative (C) (n !)1/n (D) n
from the given choices.
x-3
6. Lt = ______.
1. Evaluate Lt cos x sin x . x 3 x-3
x
p
p (A) 1
4 2x
2 (B) Only left limit exists
1 1 (C) only right limit exists
(A) 2 (B) (C) 2 (D)

2 2 (D) Limit does not exists

x 2 + 3x + 4
x
7. Evaluate Lt
[ x] + x .
2. Lt 2 = x 3 x
x
x + x -1 (A) 1 (B) 5/3
(A) e (B) e2 (C) e3 (D)
e4 (C) 2 (D) Does not exist
3. Evaluate Lt x 2 x . 8. If f(x) is continuous at x = a then which of the following is
x 0
(A) 0 (B) 1 true?
(C) 2 (D) None of these (A) Lt - f ( x ) exists and Lt + f ( x ) exists.
1 xa xa
4. The value of lim 1
is (B) Left limit and right limit of f(x) at x = a are equal.
x 0
5+3 x
(C) Limit of f(x) at x = a is equal to f(a)
(A) 5 (B) 3
(D) All the above.
(C) 0 (D) does not exist
1
1x + 2 x + ........ + n x x 3 x - 1, for x 0
9. The function f(x) = , f(x) is ______.
5. lim = 3 x + 1, for x > 0
x 0
n

Chapter 02.indd 52 8/28/2015 7:02:58 PM


Chapter 2 Calculus | 2.53

(A) continuous at x = 0 1 1
(A) 1 (B) -
(C) (D) 0
(B) Lt f ( x ) = Lt f ( x ) = 0 4 4
x 0- x 0+
(C) The limit does not exist 3 sin x - 2 x
18. The value of f(0) so that f(x) = is continuous at
(D) None of these tan x + 4 x
-p p
10. f(x) =
x2 - 4
if x 2 = 6 if x = 2 is each point of , is
x-2 2 2
(A) continuous at x = 2. 1 3
(B) continuous for all values of x. (A) 0 (B) (C) (D) 1
5 4
(C) discontinuous at x = 0.
1
(D) discontinuous at x = 2. 19. Find the point at which the function f(x) = is
1 + 3 tan x
4x2 - 9 discontinuous. Also mention the type of discontinuity.
11. If f (x) = , for x 3/2, is continuous at
2x - 3 (A) x = np, n Z; removable.
3
x = 3/2, then the value of f(x) at x = is (B) x = np/2, n is odd integer; second.
2 (C) x = np; first kind.
(A) 2 (B) 0 (C) 6 (D) 1 (D) x = np/2; removable.
12. If f(x) = [x] where [x] represents the greatest integer less than
20. Which of the following function/s have a removable disconti-
or equal to x, then which of the following statements is true?
nuity at x = 0?
p : f has jump discontinuity at all integers.
x cos (1 x 2 ) at x 0
q : f is continuous at all irrational points. f(x) =
I.
r : f has removable discontinuity at all integer points. 1 at x = 0
(A) p, q, r are all correct.
1
(B) p and r are correct. f(x) =
II. xR
(C) q and r are correct. x sin x
(D) p and q are correct. III. f(x) = 1/x; x 0
(A) I alone (B) I and II
1 + px - 1 - px (C) III alone (D) All the above
13. A function f is defined as f(x) = for -1
x 21. Find the set of points of discontinuities of the function f(x) =
2x + 1
x < 0 and f (x) = for 0 x 1. If f(x) is continuous on x - |x| x R.
x-2
[-1, 1], then p = (A) Z (B) ( , 0) (C) (- , 0) (D)
1 22. The function f(x) = 2x3 x2 x + 2 when x 1 and f(x) = 0
(A) 1 (B)
2 when x = 1 is ______.
-1 (A) Continuous at x = 1 (B) discontinuous at x = 0
(C) (D) 0
2 (C) continuous at x = 1 (D) can not say
14. The function f (x) = |x| + |x - 1| + |x - 2| is 23. The number of discontinuous points of the function f(x) = [x] + 3
(A) continuous only at x = 0, 1 for all negative integers of x.
(B) continuous everywhere (A) infinite (B) 1
(C) continuous only at x = 1, 2 (C) 0 (D) None of these
(D) continuous only at x = 0, 2 24. If f(x) = |x| sin x, then f is
15. If the function f (x) = ax - b, for x 1 (A) differentiable everywhere
= 3x, for 1 < x < 2 (B) not differentiable at x = np, nZ
(C) not differentiable at x = 0
= bx2 - a, for x 2 (D) not continuous at x = 0
is continuous at x = 1 and discontinuous at x = 2, then the
25. For the function defined as f(x) = x - k; k x < k + 1, k Z
relation between a and b is
which of the following statement/s is/are true?
(A) a = 3 + b (B) a = 3 - b, b 3
I. f is bounded on R.
(C) a = 3 + b, b 3 (D) a = 3 - b
II. f is discontinuous at all integer points.
3 x + 1, x 1. III. f is not differentiable at all integer points.
16. Let f(x) = . The value of a for which f(x) is (A) I alone (B) II and III
2 - ax x > 1
2

continuous is (C) I and III (D) All the above


(A) -2 (B) 6 (C) 1 (D) 2 x
26. The function is defined by f (x) = , then at what points f (x)
17. The value of k so that 1+ x
is differentiable?
sin ( 4 k - 1) x (A) All points of x. (B) Except at x = 1.
,x 0
f(x) = 3k is continuous everywhere in (C) Except at x = 0 (D) Except at x = 0, 1
tan ( 4 k + 1) x , 0 < x < p 27. How many real roots does the function f(x) = 5x3 - 2x2 + 3x
5x 2 - 4 have on (0, 1)?
its domain of definition is (A) 1 (B) 2 (C) 3 (D) 0

Chapter 02.indd 53 8/28/2015 7:03:06 PM


2.54|Engineering Mathematics

28. The maximum area that can be enclosed by a wire of length 38. The point at which the function f(x, y) = x4 - y4 - 2x2 + 2y2 has
24 cm by bending it in the form of a sector of the circle in neither maximum nor minimum is _____
sq.cm is. (A) (1, 1) (B) (1, 2) (C) (1, -1) (D) (-1, 1)
(A) 16 (B) 36 (C) 30 (D) 26 39. The number 30 is divided into three parts such that the prod-
29. Rolles theorem is applicable if a function f is defined on [a, uct of the first part, the square of the second part and the cube
b] such that of the third part is maximum. Then, the product of the three
(A) f is continuous on [a, b] parts is _____
(B) f is differentiable on (a, b) (A) 1140 (B) 672 (C) 208 (D) 750
(C) f(a) = f(b)
40. Evaluate Lt
(D) All the above 1 1 1
+ + ............ + .
n n + 2 + 4 3
30. Find the value of c (-p, p) such that f(c) = 0 where f(x) = n n
cosx. Using Rolles Theorem. 1
(A) log3 (B) log 3
(A) p/9 (B) p/4 (C) 0 (D) p/3 2
1
31. Why is Rolles Theorem not applicable on [0, 2] for the (C) log 3 (D) None of these
3
2 - x; 0 x 1
function f ( x ) = ? n
n2
3 - 2 x;1 < x 2 41. Evaluate Lt
n
( r + n)
r =1
3
.
I. f is not continuous at x = 1
II. f is not differentiable at x =1 3 7 5 5
(A) (B) (C) (D)
III. f (0) f (2) 8 8 8 16
(A) I alone (B) II alone n
1
(C) III alone (D) II and III alone 42. Evaluate Lt
n
na + r .
r =1
a +1
32. Let f(x) = 3x2 1 in the interval [1, 3] (A) log(a+1) (B) log
(A) only Mean Value theorem is applicable to f over [1, 3] a
(B) only Rolles theorem is applicable to f over [1, 3] 1
(C) log(a+1) (D) a log(a+1)
(C) Both Rolles theorem and Mean Value theorem are ap- a
plicable to f over [1, 3] n
1
43. Evaluate Lt .
(D) Neither Rolles theorem nor Mean Value theorem are n
r =1 n - r2
2

applicable to f order [1, 3]


p p p
x 2 x3 x 4 p (C)
(A) (B) (D)
33. Let f(x) = x - + - + ........... 4 2 8
2 3 4
The first derivative of the function f(x) becomes undefined 44. Evaluate tan 5 x dx
when x =
(A) 0 (B) 1 (C) 1 (D) - 1 tan 3 x
(A) + log cos x
3
2u
34. When u = f(x, y) then = tan 4 x tan 2 x
x 2 (B) - + cos x
5 3
f x ( x + dx , y ) - f x ( x , y )
(A) Lt tan 5 x tan 3 x
dx 0 dx (C) - + tan x
5 3
f x ( x , y + dy ) f x ( x , y ) tan 4 x tan 2 x
(B) Lt (D) log cos x
dy 0 dy 4 2
f y ( x + dx , y ) - f y ( x , y ) p 2
(C) Lt
dx 0 dx 45. sin m
x dx ( m is even ) = ______.
f y ( x , y + dy ) - f y ( x , y )
0

(D) Lt m m-2 1p
dy 0 dy (A) .......
m-2 m-4 22
2u m -1 m - 3 m - 5 1p
35. If u = exy then the value of
= (B) .........
2 x y m m-2 m-4 22
(A) 2u (B) exy xy
m -1 m - 3 2
(C) u (xy + 1) (D) u (xy - 1) (C) .........
m m-2 3
u u u (D) None of these
36. If u = log (x3 + y3 + z3) then x +y +z =
x y z 46. f(x) is a bounded real value function in the interval [a,b] then
a a
(A) 2 (B) 3 (C) 1 (D) 0
f ( x ) dx = 2 f ( x ) dx is true only when
37. If u = tan-1 then the value of uxx + uyy =
y -a 0


x (A)
f(x) is any real function (B) f(x) is an odd function
(A) 24 (B) 1/24 (C) 4 (D) 0 (C)
f(x) is an even function (D) Both (B) and (C)

Chapter 02.indd 54 8/28/2015 7:03:19 PM


Chapter 2 Calculus | 2.55

47. The area enclosed by the curve y = x2, the lines y = 6x, 1 t2 - t
(A) log 256 (B)
x = 2 and x = 6 is ________sq units 16 2
20 40 80 100
(C) log
(A) (B) (C) (D) 1 256 1 256
3 3 3 3 16 (D)
4
log 5
e5 e
48. The bounded by the curve y = 6 - x2, the lines x = -2,
9-3 y2
x = 1 and x - axis is ____ sq units 3

(A) 9 (B) 12 (C) 15 (D) 18


58. The value of
0 - 9-3 y2
ydxdy is _____

49. The volume of the solid generated by the revolution of the (A) 0 (B) 3 (C) 6 (D) 9
area bounded by the lines x + 2y = 6, y = 1, y = 2, and y - axis
about y - axis is ____ cubic units y
4 2
7
(A) p (B) p (C) p (D) p
14 21 28 59. By changing the order of integration, f ( x, y ) dxdy
3 3 3 3 becomes _____ 0 y2
8
3
50. The length of the curve 27 y = ( 3 x 2 + 2 ) 2 from x = 0 to 2 8x 4 x

x = 4 is (A) f ( x, y ) dydx (B)


0 2x
f ( x, y ) dydx
0 8x
(A) 68 (B) 30 (C) 10 (D) 2
8 8x 6 2x
0
51. Evaluate
32 x dx
(C) f ( x, y ) dydx (D)
0 2x
f ( x, y ) dydx 0
- 8x

1 1 60. The area bounded by the parabola y = x and the line x - y = 2


2
(A) (B)
log e 3 2 log e 3 can be expressed as _____
2 2 y+2 4 y+2
(C) (D)
2 log e 3
log e 3 (A)
y = -1 x = y 2
dxdy
dxdy (B)
y = -1 x = y 2
1
1 y+2 y+2
x
2 2
52. dx = ______.
0
0.999 (C) ( y 2 - x ) dxdy (D)
y = -1 x = y 2
( x - y ) dxdy y = -1 x = y 2
(A) 999 (B) 10000 (C) 100 (D) 1000
61. The volume of the solid below the surface x2 + y2 + z = 4

and above the triangle formed by the lines x = 0, y = 0 and x +
53. e
0
-2 x
dx converges to ______ .
y = 2 in xy plane is given by _____
1 2- y
1 1 1 1
(A) (B)
4 2
(C)
6
(D)
8
(A) y =0 x =0
(4 - x 2
- y 2 ) dxdy

54. Consider the following statements 2 4 - y2

1
dx
(B) ( x + y ) dxdy
A: The improper integral x-x
0
2
always converges. y = -2 x = - 4 - y 2

2 2- y
B: There exists any point of discontinuity at (0, 1). (C) (4 - x 2
- y 2 ) dxdy
Which of the following is/are true? y =0 x =0

(A) A and B are true 2 4 - y2




(B) A is true and B is false
(C) A is false and B is true
(D) ( x + y ) dxdy
y = 0 x = - 4 - y2
(D) A is false and B is also false
62. The volume of the solid generated by revolving the region R
1 2 2
shown in the figure about y - axis is given by
55. xyz dx dy dz = ______.
0 0 1
Y
1 3 5
(A) (B) (C) (D) 1
2 2 2 y=6
a a2 - x 2 a2 - x 2 - y 2
dxdydz R
= ______.
56.

0 0

0 a2 - x 2 - y - z 2
y=1 X
p ap a 2p a 2p 2
(A) (B) (C) (D) O
8 8 8 8
1 1 - x 1- x - y xy=3
(1 + x + y + z )
-3
57. Evaluate dxdydz = ______.
0 0 0 y 2 = 4x

Chapter 02.indd 55 8/28/2015 7:03:38 PM


2.56|Engineering Mathematics

6 y +3
22 2 (A) a and b are perpendicular
2p x dxdy
(A) 2p xdydx (B)
1 y2
(B) a and b are parallel
1 1
4 4 (C) a and b are neither parallel nor perpendicular
22 x 6 y +3
(D) none of these
2p ydxdy
(C) 2p ydydx (D) 71. A unit vector perpendicular to the plane of a and b is given
1 1 1 y2
4
by n , in then n =
4
a b
(B) a b
r (sin q + cosq ) 2q drdq
(A)
2
63. By changing the double integral 2
- sin
a b
r (sin q + cosq ) - sin 2q drdq in polar coordinates to cartesian coordinates,
2 R
2

R

(C)
2 a b ( 4 a b
(D)
) ( )
we get f ( x, y ) dxdy . Then, f (x, y) is equal to ______
a b a b
R

(A) ( x2 + y2 )
x2 + y2 (B) d 3F
72. If F = a cos ui + a sin u j + au k , then
at u = 0 is
1 1 du 3
(C) (D)
( x + y2 )
2
( x2 + y2 ) a i (B)
(A) - a j
a i + a j - k
a i - a j (D)
(C)
64. By changing the variables from x and y to u and . Where
x = ucot and y = utan, the double integral f ( x, y ) dxdy dF
=
73. If F = a sin nt + b cos nt , then F
f ( u cot v, u tan v ) Y ( u, v ) dudv then (u, ) is
R
changes to dt
R (A) a b (B) a b cosnt
(A) 4u sin 2 (B) 4u cosec 2 n b a (D)
(C) a sin nt b
(C) -4u sin 2 (D) -4 u cosec 2
74. If F = u 2i - uj + ( 2u + 1) k and a = (2u-3) i + j - u k , then
65. The volume bounded by the cylinder x = 9 - y2 and the planes
z = y, x = 0, z = 0 in the first octant is ____ cubic units ( F a ) at u = 1 is
81 81 81 (A) 7 i + 3 k (B) 7 j + 3 k
(A) 81 (B) (C) (D)
2 4 8
(C) i + j + k (D) 3 j + k
66. By changing a triple integral f ( x, y, z ) dxdydz in
d 2r
R
rectangular coordinates x, y and z to spherical polar 75. If r = aew t + be -w t , then = (a, b are constant vectors)
dt 2
coordinates r, q and f, we get f ( x, y, z ) dxdydz = (A) 0 (B) a w ewt
(C) - r (D) wr
f ( x ( r,q,f ) , y ( r,q ,f ) , z ( r,q ,f ))Y ( r,q ,f ) drdqdf
R

3 (fF )
R
76. If f (x, y, z) = xyz and F = xz i - xy 2 j + yz 2 k then
Then (r, q, f) is at (3, 1, 1) = x 2 z
(A) r sin q (B) r2 sin q
(C) r sin q (D)
2
r2 sin q cos f (A) 4 i - 2 j (B) i - 3 j + 5 k
67. If q is the acute angle between two vectors a and b then (C)
i + j -k
(D) 0
cos q =
77. If a = 2 t3 i - t2 j - (t - 1) k and b = cos 2 t i - sin t j + k
| a .b | a .b
(A) (B) d
| a || b | | a || b | then ( a . b ) at t = 0 is
dt
2| a b | | a .a | (A) 0 (B) - 1 (C) 1 (D) p/2
(C) (D)
| a || b | 2 | a || b |
78. If a = 5 t2 i - 3 t j - (2 t2 - 1) k and b = 3 t2 i - 5 t j +
68. Which of the following pair of vectors are orthogonal to each d
other? 4 t3 k then
dt
( )
a b at t = 1 is
(A) (1, - 1, 0), (- 1, 1, 2)
(B) (- 2, 1, - 1), (- 4, 2, 2) -(73 i
(A) + 118 j + 48 k )
(C) (4, - 1, 3), (3, 0, - 4) -73 i
(B) - 16 j + 48 k
(D) (- 2, - 3, 4), (2, 3, -4) -73 i
(C) + 118 j + 48 k
69. Let a and b be two vectors. Then which of the following is -73 i
(D) + 116 j + 48 k
not implied by a . b = 0? 79. If A and B are any two vector point functions of a scalar
(A) a and b are perpendicular variable t, then ( A B ) is ______.
(B) a and b both are null vectors
(C) a and b are parallel d2B d2 A
(A) A + 2 B
(D) At least one of a or b is null vector. dt 2 dt
70. Let a and b are two non zero vectors, and a b = 0 d2A dA dB d 2 A
(B) A + 2 B
implies dt 2 dt dt dt 2

Chapter 02.indd 56 8/28/2015 7:04:23 PM


Chapter 2 Calculus | 2.57

d2A dA dB d 2 B 90. Find the divergence of the curl of F if F = x 2 yi + xzj + y 2 zk .


(C) 2 + 2
dt dt dt dt 2 (A) 3 (B) 2 (C) 0 (D) 7/ 2
dA d 2 B dA dB d 2 A dB 91. Which of the following is / are true for a vector r ?
(D) 2 + 2
dt dt dt dt dt 2 dt (A) Solenoidal if div r = 0, irrotational if curl r = 0
80. If r = x i + y j + z k and c is a constant vector, then find (B) Solenoidal if curl r = 0 , irrotational if div r = 0
(C) Solenoidal if div r = 1, irrotational if curl r = 0
( c r ) i + ( c r ) j + ( c r ) k (D) Solenoidal if div r = 0, irrotational if curl r = i
x y z
92. Find the value of p if the vector
(A) 0 (B) c (C) 2 c (D) 4 c
F = ( x + z ) i + ( 3 x + py ) j + ( x 5 z ) k is solenoidal.

Common data for Questions 81 and 82:For a space curve (A) 2 (B) 1 (C) 3 (D) 4
curvature k and torsion t are given by
93. If F (x, y, z) is a vector field which is differentiable at every
2
3
point (x, y, z) and div ( F ) = 0, then F is called
and t =
dr d 2 r dr dr d 2 r d 3 r dr d 2 r
k= 2 .
dt dt dt dt dt 2 dt 3 dt dt 2 (A) irrational (B) harmonic
(C) solenoidal (D) irrotational
81. If r = (3t - t3) i + 3t j + (3t + t3) k , then k = 94. If F (x, y, z) is a vector field which is differentiable at every
point (x, y, z) and if curl F = 0 then F is called
3 1
(A) (B) (A) irrotational (B) solenoidal
(1 + t 2
) (1 + t )
2
(C) harmonic (D) irrational
1 1 95. If f is a scalar point function and f is a vector function, then
(C) (D)
3 (1 + t )
2
2
3 (1 + t2) which of the following is true?
(A) Div (grad f) = 2f (B) Curl (grad f) = 0
82. Find the value of t for the curve x = a cost, y = a sin t and (C) Div (curl f) = 0 (D) All the above
z = bt.
96. ( f ) =
b a
(A) 2 (B) (A) 2 f (B) (. f )
a + b2 a2 + b2
2 f
1 a2 (C) grad (div f ) - (D) div (curl f )
(C) 2 (D)
a +b 2
a + b2
2 97. Div ( f g ) =
83. What is the angle between the normals to the surface xy = z (A) f (curl g ) - curl f
at (1, 4, 2) and (- 3, - 3, 3)? (B) f . (curl g ) - g . (curl f )
(A) cos-1(1/4) (B) cos-1(1/3) (C) g . (curl g ) - f . (curl f )
-1
(C) cos (1/ 22 ) (D) cos-1(1/ 12 ) (D) None of these
84. Find the angle between the surfaces x + y + z = 9 and z = x 98. If f (x, y, z) be a scalar function and if 2f = 0, then f is
+ y at the point (2, - 1, 2). called
(A) cos-1 (1/ 21 ) (B) cos-1 (8/3 21 ) (A) irrational (B) harmonic
-1
(C) cos (1/3) (D) p/2 (C) solenoidal (D) irrotational
85. The directional derivative of any scalar point function f along 99. The Laplace operator applied on f is
any direction is obtained by multiplying ______ in that direc- (A) f (B) 2f
tion by ______ (C) (grad f)
2
(D) Curl (grad f)
(A) a unit vector, grad f Linked answer for Questions 100 and 101:
(B) a unit vector, curl f 100. Which of the following relation is true for which the scalar
(C) a vector, grad f function f = axy + byz + cxz satisfies laplace equation?
(D) a vector, div f (A) a - b + c = 0 (B) 2a + b - c = 0
86. Find the directional derivative of f = xyz + 4xz at (C) a - b - c = 0 (D) a + b + c = 0
(1, -2, -1) in the direction of 2 i - j - 2 k . 101. Which of the following is a laplace equation?
(A) 27 (B) 47/3 (C) 5/ 2 (D) 37/3 (A) 3xy - 4yz + xz (B) xy - yz + zx
87. Div f = (C) 2xy + xz + yz (D) 2xy - 3xz - 2yz
(A) f (B) . f 102. Which of the following is a vector?
(C) f (D) None of these (A) f u (B) (u v )
88. Curl f = (C) ( u v ) (D) f
(A) . F (B) f 103. Compute the value of 2 f( r ).
(C) f (D) None of these d 2 f df d 2 f df
(A) 2 + (B) + + f
89. If r = xi + yj + zk , then (div r , curl r ) = dr dr dr 2 dr
(A) (0, 3i) (B) (3, 3i) d 2 f 2 df d2 f df
(C) 2 + (D) + 2r
(C) (0, 0 ) (D) (3, 0 ) dr r dr dr 2
dr

Chapter 02.indd 57 8/28/2015 7:04:52 PM


2.58|Engineering Mathematics

1
111. To satisfy the conditions of Greens theorem which of the
104. Compute f ( t ) dt where F (t) = t i + 3t j + 2t k
following conditions a curve must posses?
0
(A) A simple closed curve
i + j +
(A) k (B) 1/3 ( i + j - k ) (B) Positive orientation
i - j +
(C) k (D) 1/3 i + j + k (C) Piece wise smoothness
(D) All the above
105. If F = 3x2 yi + 2y2 xj, then the value of F dr , when C is the
C 112. The theorem that is used in the reduction of surface integral
curve in the xy - plane y = 2x2 from (0, 0) to (3, 18) is to volume integral is
(A) 10829.3 (B) 10289.3 (A) Stokes theorem
(C) 10982.3 (D) 10928.3 (B) Greens theorem
(C) Gauss divergence theorem
106. If F (x, y, z) represents vector function defined on a closed
(D) None of these
curve C in that region, then F dr represents
C 113. To apply Gauss theorem, the vector point function given
(A) work done by F along C must be
(B) circulation of F about C (A) continuous
(C) area of the region bounded by C (B) differentiable
(D) volume of the content in the region R (C) monotonic
(D) continuously differentiable
107. Evaluate the circulation (3 x 2
- 8 y 2 )dx + ( 4 y - 6 xy ) dy
C 114. If P, Q are continuously differentiable functions of x, z on
where C is the closed curve given by y = x and y = x. the region S of the xz plane bounded by a closed curve C,
(A) 0 (B) 1/2 (C) 3 (D) 3/2 then which of the following corresponds to Greens theorem
in a plane?
108. Find the area bounded by a simple closed curve C given by P Q
1/2 xdy - ydx where x = 3 cos q, y = 4 sin q; 0 q 2p. (A) C (Pdx + Qdz) = C x - z dxdz
(Hint: Use Greens theorem)
(A) 2p (B) 3p (C) 4p (D) 12p P Q
(Pdz + Qdx) =
(B) z - z dxdz
F ds over the square surface 0 x a and 0 y a
C C
109. Evaluate
P Q
S
where F = curl r, r = x 2 + y 2 + z 2 . (Pdx - Qdz) =
(C)
C
x - z dxdz
S
(A) S (B) 0 (C) 3S (D) 4S Q P
(D) (Pdx + Qdz) = - dxdz
110. Evaluate the volume integral F dv , where F = 4 xyzi - y j + yzk 2
C S x z
V 115. The Stokes theorem is applied for
F = 4 xyzi - y 2 j + yzk over the cube bounded by 0 x, y, z 1. (A) Reduction of line integral to surface integral
(B) Reduction of surface integral to volume integral
(A) 2 i + 3 j - 4 k (B) i + j + k
(C) Reduction of line integral to volume integral
(C) i - j - k (D) 1/2 i - 1/3 j +1/4 k (D) None of these

Previous years questions


x - sin x e2 x - 1
1. Lt is [2014]
x 0 1 - cos x 4. Lt is equal to [2014]
x 0 sin ( 4 x )

(A) 0 (B) 1
(C) 3 (D) not defined (A) 0 (B) 0.5 (C) 1 (D) 2

2. Which one of the following describes the relationship among 2 2 2 2 3
5. Curl of vector F = x z i 2 xy zj + 2 y z k is[2014]
the three vectors, i + j + k, 2i + 3 j + k and 5i + 6 j + 4 k ? (A) (4yz3 + 2xy2) i + 2x2z j + 6y2z2 k
[2014]
(B) (4yz3 + 2xy2) i - 2x2z j - 2y2z k
(A) The vectors are mutually perpendicular
(B) The vectors are linearly dependent (C) 2xy2 i - 4xyz j + 6y2z2 k
(C) The vectors are linearly independent (D) 2xy2 i + 4xyz j + 6y2z2 k
(D) The vectors are unit vectors
6. The best approximation of the minimum value attained by
3. The integral  C ( ydx - xdy ) is evaluated along the circle e-x sin (100 x) for x 0 is _________ [2014]
1
x + y = traversed in counter clockwise direction. The
2 2
7. If a function is continuous at a point,[2014]
4
integral is equal to [2014] (A) the limit of the function may not exist at the point.
p (B) the function must be derivable at the point.
(A) 0 (B) -
4 (C) the limit of the function at the point tends to infinity.
p p (D) the limit must exist at the point and the value of limit
(C) - (D)
2 4 should be same as the value of the function at that point.

Chapter 02.indd 58 8/28/2015 7:05:19 PM


Chapter 2 Calculus | 2.59

8. The divergence of the vector field x 2 zi + xyj - yz 2 k at q2 q4 q3 q5


(1, -1, 1) is[2014] 1-
(A) + (B) q- +
2! 4 ! 3! 5!
(A) 0 (B) 3 (C) 5 (D) 6 q2 q3 q3 q5
1+q
(C) + + ......... (D) q + + + ....
( x - 1) sin ( x - 1)
2
2 2! 3! 3! 5!
9. The value of the integral 0 ( x - 1)2 + cos ( x - 1) dx is
lim sin q
19. What is equal to? [2011]
 [2014] q 0 q
(A) 3 (B) 0 (C) -1 (D) -2 q
(A) (B) sinq (C) 0 (D) 1
2 x
20. If f(x) is an even function and a is a positive real number
10. The value of the integral e x+ y
dydx is[2014) a
0 0 then f ( x ) dx equals [2011]
1 1 2 -a
(A) ( e - 1) (B) ( e - 1)
2

2 2 (A) 0 (B) a
a
1 1 1
2
(C) 2a (D) 2 f ( x ) dx
(C) ( e 2 - e ) (D) e-
2 2 e 0

11. Choose the CORRECT set of functions, which are linearly 21. The parabolic arc y = x , 1 x 2 is revolved around the
dependent.[2013] x-axis. The volume of the solid of revolution is [2010]
(A) sin x, sin2x and cos2x (B) cos x, sin x and tan x p p 3p 3p
(A) (B) (C) (D)
(C) cos2x, sin2x and cos2x (D) cos2x, sin x and cos x 4 2 4 2
12. The following surface integral is to be evaluated over
dx
a sphere for the given steady velocity vector field 22. The value of the integral 1+ x 2
is[2010]
F = xi + yj + zk defined with respect to a Cartesian coordi- -

nate system having i, j and k as unit base vectors. p p


p (B)
(A) p
- (C) (D)
2 2
1
4 ( F. n) dA
S
23. The function y = |2 3x| [2010]
(A) is continuous x R and differentiable x R
Where S is the sphere, x2 + y2 + z2 = 1 and n is the outward (B) is continuous x R and differentiable x R ex-
unit normal vector to the sphere. The value of the surface cept at x = 3/2
integral is[2013] (C) is continuous x R and differentiable x R ex-
(A) p (B) 2p (C) 3p/4 (D) 4p cept at x = 2/3
13. The area enclosed between the straight line y = x and the (D) is continuous x R except at x = 3 and differenti-
parabola y = x2 in the x-y plane is[2012] able x R
(A) 1/6 (B) 1/4 (C) 1/3 (D) 1/2 24. The divergence of the vector field 3 xzi + 2 xyj - yz 2 k at a
14. Consider the function f(x) = x in the interval point (1, 1, 1 ) is equal to [2009]
-1 x 1. At the point x = 0, f(x) is [2012] (A) 7 (B) 4 (C) 3 (D) 0
(A) continuous and differentiable.
25. A path AB in the form of one quarter of a circle of unit
(B) non-continuous and differentiable.
radius is shown in the figure. Integration of (x + y)2 on path
(C) continuous and non-differentiable.
AB traversed in a counterclockwise sense is[2009]
(D) neither continuous nor differentiable.
y
1 - cos x
15. lim is [2012]
x 0
x2 B
(A) 1/4 (B) 1/2 (C) 1 (D) 2
16. At x = 0, the function f(x) = x3 + 1 has[2012] A x
(A) a maximum value (B) a minimum value
p p p
(C) a singularity (D) a point of inflection (A) - 1 (B) + 1 (C) (D) 1
2 2 2
17. For the spherical surface x + y + z = 1, the unit outward
2 2 2

26. The distance between the origin and the point nearest to it
1 1 is given by[2012]
normal vector at the point , ,0 on the surface z2 = 1 + xy is [2009]
2 2
3
1 1 1 1 (A) 1 (B)
(A) i + j (B) i- j 2
2 2 2 2 (C) 3 (D) 2
1 1 1
k (D) 27. The area enclosed between the curves y2 = 4x and
(C) i+ j+ k
3 3 3 x2 = 4y is [2009]
16 32
18. A series expansion for the function sin q is[2011] (A) (B) 8 (C) (D) 16
3 3

Chapter 02.indd 59 8/28/2015 7:05:36 PM


2.60|Engineering Mathematics

28. In the Taylor series expansion of ex about x = 2, the f Y f Y


coefficient of (x - 2)4 is[2008] (A) = ; =
y x y y
1 24 f Y f Y
(A) (B) (B) = ; =
4! 4! y x x y
e2 e4
(C) (D) 2f 2f 2Y 2Y
4! 4! (C) 2 + 2 = 2 + 2 = 1
x y x y
x1 3 - 2
29. The value of lim is[2008] f f 2Y 2Y
x 8 ( x - 8)
(D) + = + =0
x y x y
1 1 1 1
(A) (B) (C) (D)
16 12 8 4 38. If y = x + x + x + x + ..... , then y ( 2 ) = [2007]
30. The divergence of the vector field (x - y) i + (y - x) j + (A) 4 or 1 (B) 4 only
(x + y + z) k is[2008] (C) 1 only (D) undefined
(A) 0 (B) 1 (C) 2 (D) 3 39. The area of a triangle formed by the tips of vectors a , b
31. Consider the shaded triangular region P shown in the fig- and c is [2007]
ure. What is xydxdy ?[2008] 1 1
P
( )
(A) a - b ( a - c ) (B)
2 2
a - b (a - c ) ( )
y 1 1
(C) a b c (D)
2 2
a -b c ( )
x2
1 e x - 1 + x +
2
40. lim = [2007]
P x 0 x3
0
(A) 0 (B) 1/6 (C) 1/3 (D) 1
2 x
41. Let x denotes a real number. Find out the INCORRECT
statement. [2006]
1 2 7
(A) (B) (C) (D) 1 (A)  S = {x : x > 3} represents the set of all real numbers
6 9 16 greater than 3
32. The directional derivative of the scalar function (B) S = {x : x2 < 0} represents the empty set
f (x, y, z) = x2 + 2y2 + z at the point P = (1, 1, 2) in the direc- (C)  S = {x : x A and x B} represents the union of set
tion of the vector a = 3 i - 4 j is [2008] A and set B
(D)  S = {x : a < x < b} represents the set of all real num-
-4 (B)
(A) -2 (C)
-1 (D) 1
bers between a and b, where numbers between a and
33. Which of the following integrals is unbounded [2008] b, where a and b are real numbers.
p
4
42. Equation of the line normal to function
1 2
(A) 0 tan x dx (B)
0 x 2 + 1 dx F(x) = ( x - 8) 3 +1 at P(0,5) is:[2006]
1 (A) y = 3x - 5 (B) y = 3x + 5
1
(C) 0 xe - x dx (D)
0 1 - x dx (C) 3y = x + 15 (D) 3y = x - 15
2x2 - 7x + 3
2 3 43. If f ( x ) = , then lim f(x) will be[2006]
34. The length of the curve y = x 2 between x = 0 and 5 x 2 - 12 x - 9 x 3
3 -1 5 2
x = 1 is [2008] (A) (B) (C) 0 (D)
(A) 0.27 (B) 0.67 (C) 1 (D) 1.22 3 18 5
2 f 44. Stokes theorem connects [2005]
35. Let f = y x . What is at x = 2, y = 1?[2008] (A) a line integral and a surface integral
x y
(A) 0 (B) ln 2 (B) a surface integral and a volume integral
1 (C) a line integral and a volume integral
(C) 1 (D) (D) gradient of a function and its surface integral
ln 2
36. The minimum value of function y = x2 in the interval [1,5] a

is [2007] 45. (sin


-a
6
x + sin 7 x ) dx is equal to[2005]
(A) 0 (B) 1 a a
(C) 25 (D) undefined (A) 2 sin 6 xdx (B)
2 sin 7 xdx
37. If f(x, y) and y(x, y) are functions with continuous second 0 0

derivatives, then f(x, y) + iy(x, y) can be expressed as an a


(C) 2 (sin 6 x + sin7 x )dx (D) zero
analytic function of x + iy(i = -1 ), when[2007]
0

Chapter 02.indd 60 8/28/2015 7:05:57 PM


Chapter 2 Calculus | 2.61

46. Changing the order of the integration in the double integral (C) is -1
8 2 S Q
(D) Cannot be determined without specifying the path
I = f ( x, y ) dxdy leads to I = f ( x, y ) dxdy. What is q?
dy
0 x
[2005]
4
r P 50. If x = a(q + sinq) and y = a(1 - cosq), then will be
dx
(A) 4y (B) 16y2 (C)
x (D) 8 equal to [2004]
v q q
47. By a change of variables x(u, v) = uv, y(u, v) = in a (A) sin (B) cos
u 2 2
double integral, the integrand f(x, y) changes to f uv,
v q q
(C) tan (D) cot
u 2 2
f(u, v). Then, f(u, v) is [2005] 51. The angle between two unit-magnitude coplanar vectors
v P(0.86, 0.500, 0) and Q(0.259, 0.956, 0) will be[2004]
(A) 2 (B) 2uv (C) v2 (D) 1
u (A) 0 (B) 30
48. The right circular cone of largest volume that can be (C) 45 (D) 60
enclosed by a sphere of 1 m radius has a height of
52. The volume of an object expressed in spherical
 [2005]
coordinates is given by
1 2 p
(A) m (B) m 2p 3 1
3 3
2 2 4
V= r
0 0 0
2 sin fdrdfdq
(C) m (D) m
3 3 The value of the integral is[2004]
49. The line integral V d r of the vector function V ( r ) = p
(A) (B)
p
2xyz i + x2z j + x2y k form the origin to the point P(1,1,1) 3 6
2p p
[2005] (C) (D)
(A) is 1 3 4
(B) is zero

Answers Keys
Exercises
Practice Problems 1
1.C 2.A 3.D 4.C 5.B 6.D 7.C 8.A 9.D 10.A
11.C 12.B 13.D 14.C 15.A 16.A 17.D 18.D 19.B 20.C
21.B 22.B 23.C 24.A 25.C 26.D 27.D 28.A 29.C 30.B
31.D 32.A 33.C 34.D 35.B 36.D 37.B 38.A 39.A 40.A
41.D 42.D 43.B 44.C 45.A 46.A 47.C 48.A 49.C 50.D
51.B 52.D 53.A 54.A 55.C 56.D 57.C 58.B 59.C 60.A
61.B 62.B 63.B 64.A 65.C 66.A 67.B 68.B 69.D 70.A
71.A 72.B 73.C 74.D 75.A 76.D 77.C 78.C 79.C 80.B
81.A 82.D 83.C 84.A 85.D 86.D 87.D 88.D 89.A 90.D
91.A 92.B 93.D 94.A 95.A 96.C 97.B 98.D 99.C 100.C
101.D 102.C 103.A 104.C 105.B

Practice Problems 1I
1.B 2.B 3.B 4.D 5.C 6.D 7.D 8.D 9.C 10.D
11.C 12.D 13.C 14.B 15.C 16.A 17.A 18.B 19.B 20.A
21.D 22.C 23.A 24.A 25.D 26.A 27.A 28.B 29.D 30.C
31.D 32.A 33.D 34.A 35.C 36.B 37.D 38.B 39.D 40.B
41.A 42.B 43.C 44.D 45.B 46.C 47.C 48.C 49.D 50.A
51.B 52.D 53.B 54.D 55.B 56.A 57.A 58.C 59.A 60.A
61.C 62.B 63.B 64.B 65.C 66.B 67.A 68.C 69.C 70.B
71.A 72.B 73.C 74.B 75.D 76.A 77.B 78.A 79.B 80.B
81.C 82.A 83.C 84.B 85.A 86.D 87.B 88.B 89.D 90.C
91.A 92.D 93.C 94.A 95.D 96.C 97.B 98.B 99.B 100.D

Chapter 02.indd 61 8/28/2015 7:06:12 PM


2.62|Engineering Mathematics

1 01.A 102.C 103.C 104.D 105.B 106.B 107.D 108.D 109.B 110.D
111.D 112.C 113.D 114.D 115.A

Previous Years Questions


1.A 2.B 3.C 4.B 5.A 6. -1k0 - 0.94 7.D 8.C 9.B 10.B
11.C 12.A 13.A 14.C 15.B 16.D 17.A 18.B 19.D 20.D
21.D 22.D 23.C 24.C 25.B 26.A 27.A 28.C 29.B 30.D
31.A 32.B 33.D 34.D 35.C 36.B 37.B 38.B 39.B 40.B
41.C 42.B 43.B 44.A 45.A 46.A 47.A 48.4/3 49.A 50.C
51.C 52.A

Chapter 02.indd 62 8/28/2015 7:06:12 PM


Chapter 3
Differential Equations
LEARNING OBJECTIVES

After reading this chapter, you will be able to understand:


Differential Equations Methods to Find the Integrating Factors
Order of a Differential Equation Linear Equations
Degree of a Differential Equation Bernoullis Equation
Linear Differential Equation Cauchys Homogenous Linear Equations
Solution of a Differential Equation Method of Variation of Parameters
First Order Equation Partial Differential Equation (PDE)
Second Order Equation Classication of Second Order Homogenous Linear
Homogeneous Differential Equations Equation

Familiarity with various methods used in evaluating indefinite inte- 2u 2u


grals or finding antiderivatives of functions [or, in other words, 8. + = 10
y 2 x 2
evaluating f(x) dx] is a prerequisite.
2u 2u
9. = 25 2
y 2
x
dIfferentIaL EquatIons
An equation involving derivatives of a dependent variable with 4u 4u 4u
10. + 6 2 2 + 4 = e 3 xy
respect to one or more independent variables is called a differen- x 4
x y y
tial equation. The equation may also contain the variables and/or
their functions and constants. If there is only one independent vari- We note that in the examples above, equations (1) to (6) are ordinary
able, the corresponding equation is called an ordinary differential differential equations, while equations (7) to (10) are partial differ-
equation. If the number of independent variables is more than one, ential equations. We refer to these examples later on in this chapter.
the corresponding equation is called a partial differential equation.
Certain Geometrical Results may also be
Examples:
dy
Expressed as Differential Equations
1. = x4 + e x + y Illustration 1 Consider a family of parallel lines. All these lines
dx
2 have the same slope. If k represents this slope, we may interpret
d2 y dy
2. x 2 2 + 3 + 3 y 4 x = sin x + 6 the family of parallel lines as curves having the same slope. As
dx dx dy
dy represents the slope of the tangent to a curve at any point
3. + 5y = x 3 tan x dx
dx dy
(x, y), we may say that the differential equation = k represents
d2 y dx
4. + 4y = 0 a family of parallel lines.
dx 2
d 3y
2
dy
4 dy
5. 3 + 5 +e2xy = 6 Illustration 2 The differential equation y = k (a constant)
dx dx dx
may be said to represent the family of curves having the length of
d y
3
d y 3dy
2
subnormal equal k at every point (x, y) on the curve. (We may note
6. +8 2 + + 9 y = 16 x 2
dx 3 dx dx that the family of curves is the family of parabolas). Our study
u u is confined to ordinary differential equations. In what follows,
7. x +y = 8u
x y differential equation means ordinary differential equations.

Chapter 03.indd 63 8/29/2015 9:06:37 AM


2.64|Engineering Mathematics

Order of a differential equation equation) throughout the interval where the equation is
The order of the differential equation: It is defined as the valid, and is such that the equation becomes an identity
order of the highest derivative present in the equation. dy d 2 y
when y , , , ... are replaced by f(x), f(x), f(x),
Examples (1), (3) are first order; (2), (4) are second order dx dx 2
and (5), (6) are third. respectively.
dy d 2 y
Degree of a differential equation [In the case of F (x, y) = 0 one has to get , ,... by
dx dx 2
The degree of a differential equation is defined as the degree successive differentiation of F(x, y) = 0 with respect to x].
of the highest order derivative present in the equation. (It is
assumed that the various order differential coefficients or Examples:
derivatives present in the equation are made free from frac- dy
1. y = e7x is a solution of = 7 y , as on substitution of y =
tional powers). dx
Examples (1), (2), (3), (4), (6) are first degree while e7x, both left and right sides of the differential equation
example (5) is second degree. 1 7 x
become identical. We find that y = e 7 x, 3e 7 x, e or,
Consider the differential equation, 2
5 in general, y = Ce , where C is an arbitrary constant
7x

dy 2 2 d 3y dy
1 + = 4 3 represents solutions of = 7 y.
dx
dx dx
2. y x = 4 is a solution of the differential equation
2 2

Taking the square on both sides (to free it from fractional dy x


= Also, y2 x2 = 5, y2 x2 = 10, ... or, in
powers) the differential equation is dx y
5
dy 2 d3 y
2 general, y2 x2 = C where C is an arbitrary constant
1 + = 16 3 dy x
dx dx represents solutions of = .
dx y
This is a third order second degree differential equation. In both the above examples, we could represent the solu-
tions of the differential equations which involve an arbitrary
Linear differential equation constant denoted by C. We now define the general solution
If, in a differential equation, the dependent variable and the of a first order differential equation.
derivatives appear only in the first degree and there is no The general solution of a first order differential e quation
term involving products of the above or containing func- is a relation between x and y involving one arbitrary constant
tions of the dependent variable, it is called a linear differ- such that the differential equation is satisfied by this relation
ential equation. or, the general solution of a first order differential equation
dy is a one parameter family of curves where the parameter
1. + Py = Q (where P and Q are functions of only x) is is the arbitrary constant. By assigning particular values to
dx
an example of a first order linear differential equation. the arbitrary constant, we generate particular solutions of
d2 y dy the equation.
2. 2 + P + Qy = R where P, Q, R are functions of In example (1) above, y = Ce7x represents the general
dx dx
d2 y dy dy
only x; + a1 + a2 y = f(x) where a1, a2 are con- solution of the differential equation = 7 y and the solu-
dx 2
dx dx
tions y = e , y = 3e , are its particular solutions. The
7x 7x
stants and f (x) is a function of x are examples of sec- general solution represents a family of exponential curves.
ond order linear differential equation. In example (2) above, y2 x2 = C represents the general
Similarly, we can have nth order linear differential equation. dy x
solution of the differential equation = and the solu-
dx y
Dny d n 1 d n 2 dy
P0 + P1 n 1 + P2 n 2 + ..... + Pn 1 + Pn y = Q tions y2 x2 = 4, y2 x2 = 5, are its particular solutions.
dx n
dx dx dx The general solution in this case represents a family of
where P0, P1, P2, Pn, Q are functions of x or constants. If an rectangular hyperbolas.
equation is not linear, it is called a non-linear differential equa- 3. y = 2e3x + 5e6x is a solution of the second order differ-
tion. In examples 1, 3, 4, 6 are linear differential equations, d2 y dy
while examples 2, 5 are non-linear differential equations. ential equation 2 - 3 - 18 y = 0.
dx dx
(which can be verified by actual substitution). Also,
Solution of a differential equation y = 4e3x 10e6x, e3x + e6x, or, in general, y = Ae3x +
A function y = f(x) or F(x, y) = 0 is called a solution of Be6x where A and B are arbitrary constants represents
a given differential equation if it is defined and differenti- d2 y dy
solution of - 3 - 18 y = 0.
able (as many times as the order of the given differential dx 2
dx

Chapter 03.indd 64 8/29/2015 9:06:43 AM


Chapter 3 Differential Equations | 2.65

4. y = 2 cos 4x + 3 sin 4x or, in general, y = A cos 4x + B Example 2: Obtain the differential equation of all the cir-
sin 4x where A and B are arbitrary constants represents cles in the first quadrant, which touch the co-ordinate axes.
d2 y
solutions of + 16 y = 0. Solution: The equation of any circle in the first quadrant,
dx 2
In example (3), the general solution is y = Ae3x + Be6x and in which touches the co-ordinate axes may be represented as
example (4), the general solution is y = A cos 4x + B sin 4x. (x h)2 + (y h)2 = h2 differentiating with respect to x,
By assigning particular values to the arbitrary constants dy
one can generate particular solutions. 2 (x h) + 2 (y h) =0
dx
From examples (3) and (4), we infer that the general dy
solution of a second order differential equation is a relation x+ y
dx
between x and y involving two arbitrary constants such that or h=
dy
the differential equation is satisfied by this relation or the 1 + dx

general solution of a second order differential equation is
a two-parameter family of curves where the parameters are Substituting the above expression for h in the equation of
the arbitrary constants. the circle
2 2 2
To sum up, the general solution of an nth order differ- dy dy dy
x+y x+y x+ y
ential equation is a relation between x and y involving n dx + y dx = dx
arbitrary constants, such that the differential equation is sat- x dy dy

dy

1+ 1+ 1+
isfied by this relation or the general solution of an nth order dx dx dx
differential equation is an n-parameter family of curves 2 2

( x - y ) + ( x - y ) = x + y
2 dy dy
where the parameters are the arbitrary constants. For the or
first and second order differential equations, we have dx dx
dy 2 dy
2

( x - y)
2
First Order Equation or 1 + = x + y .
dx dx
One parameter family of curves:
Initial value problemsA first order differential equation
Representation: Relation between x and y involving one
with a condition that y = y0 when x = x0 [written as y (x0) = y0)]
arbitrary constant, say C.
is known as an initial value problem for e.g.,
Eliminate: Eliminate C to obtain a D.E representing the dy x
given curve. 1. = ; y (0) = 1
dx y
dy
Second Order Equation 2. + 2 xy = x 3; y(1) = 6
dx
Two-parameter family of curves: dy 3 y
3. + = e x ; y(0) = 4
Representation: Relation between x and y involving two dx x
arbitrary constants, say A and B. To solve such problems, we first obtain the general solution
Elimination: Eliminate A and B to obtain a D.E represent- and find that particular value of the arbitrary constant in the
ing the two-parameter family of curves. general solution which satisfies the condition y (x0) = y0.
This means that the solution of an initial value problem is a
We shall work out a few examples to illustrate the forma- particular solution of the given differential equation.
tion of differential equations:
Example 1: Form the differential equation representing the First Order First Degree Equations
one-parameter family of curves dy
The general form of the equation will be = f ( x, y ).
x3 Ay = 0 dx
In the example, we explain how to solve equations of the
Solution: Given, x3 Ay = 0  (1)
above type (or how to obtain the general solutions of such
Ay = x3 equations).
dy 3x 2
A = 3x 2 A = (2)
dx dy Separable Equations (or V
ariables
dx  Separable Type)
Substituting A in the equation (1), Here, the given differential equation can be reduced to the
we have dy
form f(y)dy = g(x)dx. [Recall that may be thought as the
3x 2 dy dx
x3 .y = 0 x - 3y = 0 ratio of the differential of y to the differential of x]. Direct
dy dx
dx integration of the relation with respect to the variable on each

Chapter 03.indd 65 8/29/2015 9:06:48 AM


2.66|Engineering Mathematics

side gives general solution or, in other words, the general Given: When x = 1, y = 0;
solution of the differential equation above may be written 1 1 2
as f(y) dy = g(x) dx + C, where C is an arbitrary constant. - e = + c, c = .
3 3 3
1+ y 2 1 x 3
2
dy The solution is e y = .
3

Example 3: Solve: = 3 3 3
dx 1+ x 2
x3 + e y 2 = 0.
3

dy 1+ y2
Solution: =
dx 1+ x2 Homogeneous Differential Equations
1 1 The given differential equation will be of the form f(x, y)
dy = dx
1+ y 2
1+ x 2 dy = g (x, y) dx, where f (x, y) and g (x, y) are homogeneous
functions in x and y of the same degree.
Integrating on both sides,
1 1 Definition A function F(x, y) in x and y is a homogeneous
dy = dx function in x and y of degree n (n, a rational number), if
1+ y 1+ x
y x
sinh1y = sinh1x + c F(x, y) can be expressed as xn f or yn y .
x y
dy
Example 4: Solve: (x xy2) + (y + x2y) = 0 4 y y3
dx 1. x3 + 4x2 y y3 = x3 1 + x - x 3 is a homogeneous

dy
Solution: (x xy2) + (y + x2y) = 0 function in x and y of degree 3.
dx
(x xy2) dy + (y + x2y) dx = 0 y
2. x 3 tan is a homogeneous function in x and y of
x(1 y2) dy + y(1 + x2) dx = 0 x
degree 3.
1- y2 1+ x2 x+y
dy + dx = 0 3. is a homogeneous function in x and y of
y x 2x 3y
Integrating on both sides, degree 0. We change the dependent variable y to v by
1 1 dy dv
- y dy + - x dx = 0 the substitution y = vx. Then, = v + x . On substi-
y x dx dx
dy
y2 x2 tution y and in the given homogeneous equation, it
logy + log x + = logC dx
2 2 reduces to the variables separable form.
y -x 2 2

xy y 2 - x 2 xy
loge = = e 2 dy
C 2 c Example 6. Solve: x 2 = x2 + 7xy + 9y2
dx
y2 - x2

xy = c e 2 dy
Solution: x2 dx = x2 + 7xy + 9y2
dy
Example 5: Solve the initial value problem y 2 = x 2 e y . y (1) = 0
3

2
dx dy 7y y
2 dy = 1 + + 9
y = x e . y (1) = 0
2 y x
3

dx dx x
dy dy xdv
Solution: Given: y2 dx = x e
2 y 3
Put y = xv =v+
dx dx
y 2 e - y dy = x 2 dx.
3
xdv
v+ = 1 + 7v + 9v 2
dx
y 2 e - y dy = x 2 dx
3

dv
Let e - y = t e - y . - 3 y 2 dy = dt
3 3 x = 9v 2 + 6 v + 1
dx
1 1 1
- dt = x 2 dx dv = dx
3 9v 2 + 6 v + 1 x
-1 x3
t= +c Integrating on both sides,
3 3
1 x3 1 1
e- y =
3
+ c. dv = dx
3 3 9v + 6 v + 1
2
x

Chapter 03.indd 66 8/29/2015 9:07:02 AM


Chapter 3 Differential Equations | 2.67

1 1 Integrating on both sides,


dv = dx
( 3v + 1)
2
x 1 3 1
- dv + dx = 0
1 v 5v + 3 x
- = log x + log c 3
3 ( 3v + 1) log v - log (5v + 3) + log x = log c.
1 5
= - = log e cx 5 logv 3log (5v + 3) + 5 logx = 5log c.
3y
3 + 1 v5
x log x 5 = log C 5
( 5v + 3 )
3
x
= = loge cx
9 y + 3x y5
3
= c1, where c1 = c5
where c is an orbitary constant. y
5 + 3
dy y x
Example 7: Solve x = y + x sin y5 x3
dx x = c x3y5 = c1(5y + 3x)3
( 5 y + 3x )
3

dy y
Solution: Given: x = y + x sin
dx x Exact Differential Equations
dy y y If M as well as N is a function in x and y, then the equation
= + sin (1)
dx x x Mdx + Ndy = 0 is said to be an exact differential equation if

dy dv there exist a function f(x, y) such that
Put y = vx., =v +x . d(f(x, y)) = Mdx + Ndy.
dx dx
Substituting in (1) we get, f f
i. e., dx + dy = Mdx + Ndy
dv x y
v + x = v + sinv
dx Example: 3x2ydx + x3dy = 0 is an exact differential
xdv 1 1 equation, as there exists a function x3y such that
= sinv = dv = dx
dx sin v x d(x3 y) = 3x2ydx + x3dy
1 The necessary and sufficient condition for an equation of the
= cosecv dv = dx
x M N
log (cosecv cotv) = logx + logc form Mdx + Ndy = 0 to be an exact equation is =
y x
cosecv cotv = cx.] The solution of the exact differential equation Mdx +
y y Ndy = 0 is U + f(y)dy = C
cosec cot = cx. u
x x where U = Mdx and f(y) = N
y
Example 8: Solve 3y2 dx + (2xy + 3x2) dy = 0 x
or Mdx + (terms of N not containing x)dy = C
Solution: 3y2 dx + (2xy + 3x2) dy = 0. x
Here, Mdx denote integration of M with reference to x
dy -3 y 2
= treating y as a constant.
dx 2 xy + 3 x 2
dy dv Example 9: Find the solution of
Put y = vx =v+x
dx dx (3x 2y + 5)dx + (3y 2x + 7)dy = 0.
dv -3v 2 Solution: M = 3x 2y + 5, N = 3y 2x + 7
v+x =
dx 2v + 3
M N M N
dv -3v 2 = 2 = 2. =
x = -v y x y x
dx 2v + 3 The given equation is exact
dv -3v 2 - 2v 2 - 3v The solution is
x = x
dx 2v + 3 Mdx + (the terms of N not containing x) dy = C
2v + 3 1
dv = dx x

-5v 2 - 3v x ( 3 x - 2 y + 5 ) dx + ( 3 y - 7 ) dy = C
2v + 3 1 3x 2 3y2
dv = dx = 0 - 2 yx + 5 x + + 7y = C
v ( 5v + 3) x 2 2

Chapter 03.indd 67 8/29/2015 9:07:14 AM


2.68|Engineering Mathematics

Example 10:Find the solution of (ey + 1) cotxdx + x


x + 2y
U = M1dx , where M1 =
ey log(sinx)dy = 0. x2 + y2
x
Solution: Given (ey + 1)cotxdx + ey log (sin x)dy = 0 x + 2y
Let M = (ey + 1)cotx and N = ey log (sin x)
= x
2
+y2
dx

M N x 1
= ey cot x and = ey cot x = 2 dx + 2 y 2 dx
y x x +y 2
x +y2
1 1 x

M N
= = log ( x 2 + y 2 ) + 2 y tan -1
y x 2 y y
The given equation is exact 1 x
x = log ( x 2 + y 2 ) + 2 tan -1
2 y
The solution is Mdx + As in N1 there is no term independent of x, the solution is
(the terms of N not containing x)dy = C 1 x
x log ( x 2 + y 2 ) + 2 tan -1 = C
( e y + 1) cot xdx + 0dy = C 2 y

(ey + 1) log (sin x) = C Method 2:


If the differential equation Mdx + Ndy = 0 is of the form y
1
f(x, y)dx + x g(x, y)dy = 0 and Mx Ny 0, then is
Integrating factors Mx - Ny
Let us say M(x, y)dx + N(x, y)dy = 0 be a non-exact differen- an integrating factor of Mdx + Ndy = 0
tial equation. If it can be made exact by multiplying it by a Method 3:
suitable function (x, y), then (x, y) is called an integrating 1 M N
In the equation Mdx + Ndy = 0, if = f(x),
N y x
factor.
f ( x )dx
then e is an integrating factor of the given equation.
Methods to Find the Integrating Factors
1 N M
Similarly if = g ( y ), then eg(y)dy is an
Method 1: M x y
If Mdx + Ndy = 0 is a homogeneous differential equation integrating factor of the given equation
1
and Mx + Ny 0, then is an integrating factor of Example 12: Find the solution (x2 y2)dx + 2xy dy = 0.
Mx + Ny
Mdx + Ndy = 0
Solution: Given (x2 y2)dx + 2xy dy = 0 (1)
Example 11: Find the solution of (x + 2y)dx + (y 2x)dy = 0. M = x2 y2 and N = 2xy
Solution: Here, M = x + 2y and N = y 2x M N
= 2 y and = 2y
M N y x
= 2 =2
y x M N

M N x

y x 1 M N 1
N y
=
x 2xy
[ 2y 2y ]
The above equation is not an exact equation.
But M and N are homogeneous functions -2
= = f(x) Integrating factor (I.F)
1 x
The integrating factor =
Mx + Ny = ef(x)dx
2 1
(x + 2y)x + ( y 2x)y = x2 + y2 dx log 1
= e x = e 2 log x = e x = 2
2

x
1
Now, by multiplying (1) by it is reduced to 1
x + y2
2
Multiplying the given equation with , we get
become an exact equation. x2
x -y
2 2
2 xy
x + 2y y + 2x x 2 dx + x 2 dy = 0
2 2
dx + 2 2
dy = 0
x +y x +y
x2 - y2 y
x 2 dx + 2 x dy = 0 (2)
The solution is U + f(y)dy = C


Chapter 03.indd 68 8/29/2015 9:07:24 AM


Chapter 3 Differential Equations | 2.69

x2 - y2 2y Integrating the above equation gives


M1 = and N1 =
x2 x log y = Pdx + logC or y = CePdx (3)
M 1 2 y N 1 2 y This represents the general solution of (2).
= 2 and =
y x x x2 Equation (3) may also be written as yePdx = c.
d
M1 N1 Now,
dx
( ye Pdx ) = 0,
=
y x dy
i. e. e Pdx + ye Pdx P = 0 or
(2) is an exact equation and its solution is x m1dx + (the dx
terms of N1 not containing x)dy = C
dy
x - y2
x 2
e Pdx + Py = 0.
dx + 0 dy = C dx
x2
x
y2 y2 This means that if we multiply both sides of (2) by ePdx,
1 - 2 dx = C x + 2 = C the product
x x
dy d
Example 13: Find the solution of xy dx + (y + y2)dy = 0. ( ye Pdx ). The factor ePdx is called an
2
e Pdx + Py is
Solution: Given xy2dx + (y + y2)dy = 0  (1) dx dx
integrating factor of (2).
Mdx + Ndy = 0
Suppose we multiply both sides of (1) by ePdx, it is reduced
M = xy2; N = y + y2
d Pdx
M
= 2xy and
N
=0 to
d
dx
(
ye Pdx =
d
dx
) (
Qe Pdx dx as Qe dx = Qe .
dx
Pdx
)
y x
Hence, we get the general solution of (1) as
M N

ye = C + Qe
pdx pdx
y x dx.
1 N M 1 dy
M x
=
y xy 2
[ 2xy ] Example 14: Solve sinx + y cos x = 1
dx
= 2/y = g(y) dy
g(y)dy
Solution: Sinx + y cos x = 1
Integrating factor is e dx
2 1
dy log 1 dy
=e y
= e 2 log ydy = e y2
= + cos x y = cos ec x .
y2 dx
2 This is a linear equation in y
dy log
1
1 xy 2 dx y + y 2
Here, P = cotx, Q = cosecx.
e y
=Multiplying
e 2 log ydy = e (1)y by
2
= , we get + dy = 0
y 2
y2 y
2

pdx = cotx dx = log (sinx)
1
xdx + + 1 dy = 0 I. F = e = e
pdx
= sinx.
logSinx

y The general solution is Y.I.F = QIF dx + c


x2 y sinx = cosecx.sinxdx + c
Integrating on both sides we get + log y + y = C
2 y sinx = dx + c
Linear Equations y sinx = x + c
Consider the linear differential equation dy
Example 15: Solve: (1 + x4) + 4 x3y = sin3x
dy dx
+ Py = Q  (1)
dx dy
Solution: Given: (1 + x4) + 4 x 3 y = sin 3 x
where P and Q are functions of only x. We explain below, dx
how such equations can be solved. Consider the equation dy 4 x 3 sin 3 x
dy + y=
+ Py = 0 (2) dx 1 + x 4
1+ x4
dx It is a linear differential equation in y.

The equation (2) is called the homogeneous linear equa- 4 x3 sin 3 x
tion corresponding to (1). We find the general solution of (2). Here, P = and Q =
1 + x4 1+ x4
Equation (2) is a variables separable type. We write it as
dy 4 x3
- Pdx. Pdx = 1 + x dx = log (1 + x 4 )
y 4

Chapter 03.indd 69 8/29/2015 9:07:36 AM


2.70|Engineering Mathematics

I.F. = epdx = e log (1+ x ) = 1 + x 4


4
dy du
Let y1 = u - y -2 =
General solution y.I.F = Q I.F dx + c. dx dx
sin 3 x du
y(1 + x4) = (1 + x 4 ) dx + c The equation (1) becomes
dx
+ xu = 3x
1+ x4
3 sin x sin 3x du
= sin 3 xdx + c = dx + c xu = 3x
4 dx
cos 3 x 3 The above equation is a linear differential equation in u.
y(1+x4) = - cos x + c
12 4 x 2
I.F = e = e = e 2
pdx xdx
12y(1 + x4) = cos3x 9cosx + c
solution is u. I.F = QI.Fdx
dy x 2 x 2
Example 16: Solve x2 + y = 4x2 + 8 2y u. e 2 = 3x e 2 dx .
dx
-x 2
dy = - 3e -t dt when t =
Solution: Given: x + y = 4x2 + 8 2y
2
2
dx -3e -t
= = 3e -t
dy 8 2y -1
+ y = 4+ 2 - 2
dx x x x 2 x 2
u. e 2 = 3e 2 +C
dy 2 8
+ y 1 + 2 = 4 + 2 1 x 2

dx x x = 3 + Ce 2
y
2 8 1
Here, P = 1 + 2 Q = 4 + 2 y=
x x x 2

2 2 3 + Ce 2
Pdx = 1 + x 2 dx = x - x dy y y
+ log y = 3 ( log y )
2
2
Example 18: Solve
x- dx x x
I.F = epdx = e x

dy y log y y ( log y )
2
General solution is y.I.F = Q . I.F dx + c Solution: Given + =
dx x x3
x-
2
8 x- 2
ye x = 4 + 2 e x dx + c
x 1 dy 1 1 1
+ . = 3 (1)
2 x- 2 y ( log y )
2
dx x ( log y ) x
= 4 1 + 2 e x dx + c 
x 1
x-
2 Let = u Differenting w.r.to x
(Put e x =t log y
-1 1 dy du
x-
2
2 . =
e 1 + x 2 dx = dt ) ( log y ) y dx dx
x 2


(1) becomes
= 4 dt + C = 4t + c
-du 1 1 du 1 -1
x-
2
x-
2 + u= 3 - u= 3
ye x = 4e x +c dx x x dx x x
It is a linear equation in u.
Bernoullis Equation -1 -1
dy Here, P = and Q = 3
An equation of the form + Py = Qy n is called Bernoullis x x
dx 1
equation, where P, Q are continuous functions in x. I.F = e = e x = e - log x = 1/ x
pdx - dx

dy solution is u . I F = QIFdx + c
Example 17: Solve + xy = (3xy 2 )
dx 1 -1 1
u = 3 . dx + c
dy x x x
Solution: Given + xy = (3xy 2 )
dx 1
u = x -4 dx + c
Throughout the equation dividing with y2 we get x
dy 1 1
y -2 + xy -1 = -3 x  (1) = 3 +c
dx ( log y ) x 3x

Chapter 03.indd 70 8/29/2015 9:07:50 AM


Chapter 3 Differential Equations | 2.71

Equations of First Order but Not logy = [log(secx + tanx) + logsecx] + logc
logy + log(secx + tanx) secx = logc
First Degree y (secx + tanx) secx = c
dy
Let =p The solution is
dx
The differential equation of the form Pn + P1(x, y) Pn1 + (y (1 + sinx)c1)(y (secx + tanx)secx c) = 0
P (x, y) Pn2 + . + Pn(x, y) = 0 is called the first order equa-
2
Solvable for y Consider Pn + p1(x, y) Pn 1 + Pn(x, y) = 0 (1)
tion of degree n. If the above equation cannot be reduced in to factors of p.
Solvable for p When the differential equation is in the Then, the above equation is solvable for y. The method is
form f(x, y, p) = 0. Then, the equation was n linear factors explained by the following example.
and hence we have n solutions for p. The solutions can be
Example 21: Solve: y = xp2 + p
termed as F1(x, y, c1) = 0, F2(x, y, c2) = 0, ....... Fn(x, y, cn) = 0
2 Solution: Given y = xp2 + p
dy dy As the above equation cannot be split in to factors. So we
Example 19: Solve: + 7 x + 10 x 2 = 0
dx dx solve for y differentiating w. r. to x.
dy dy dp dp
Solution: Let =p = x 2 p + p2 +
dx dx dx dx
p2 + 7xp + 10x2 = 0
We can easily find the factors of the equations dp dp
p = 2 px + p2 +
p2 + 5xp + 2xp + 10x2 = 0 dx dx
p(p + 5x) + 2x(p + 5x) = 0 dp
(p + 5x)(p + 2x) = 0
p p2 = ( 2 px + 1)
dx
p = 5x, p = 2x dx 2 px + 1
=
dy
= 5x or
dy
= 2x dp p (1 - p )
dx dx
dy = 5xdx, or dy = 2xdx dx 2p 1
= x=
-5 x 2 dp p (1 - p ) p - p2
y= + c y = x2 + c
2 dx 2x 1
5x 2 = =
Solution in y = + c (y + x 2 c ) = 0 dp 1 - p p (1 - p )
2
Example 20: Solve p2 + 2py tanx = y2 The above equation is a linear D.E in x
2
p -1dp
Solution: p2 + 2py tanx = y2 IF= e = e 2 log ( p -1)
p2 + 2py tanx + y2 tan2x = y2 + y2 tan2x
= e log ( p 1) = (p 1)2
2

(p + y tanx)2 = y2(1+ tan2x)


(p + y tanx)2 = (y secx)2 Solution is x I.F. = Q.I.F dp + c
p + y tanx = y secx) 1
x (p 1)2 = ( p - 1) .
2
dp + c
p = y secx y tanx) p ( p - 1)
p = y(secx tanx), p = y(secx + tanx)
p -1
dy = - dp + c
= y (secx tanx) p
dx
dy 1
or = y(secx + tanx) x(p 1)2 = - 1 - dp + c
dx p
1 x(p 1) = p + log p + c and y xp p = 0
= dy = ( sec x + tan x ) dx
y
Solvable for x The equation cannot be reduced in to factors
1
or dy = (sec x + tan x)dx and it is a linear equation in x, then we solve the equation for
y x. The method is explained by the following example.
Integrating on both sides
Example 22: Solve xp3 = 5 + 4p
logy = log(secx + tanx) log secx + logc1
sec x + tan x 1 Solution: Given xp3 = 5 + 4p (1)
y = c1 or dy 5 4
sec x y x= + 2  (2)
= (secx + tanx) dx p 3
p

Chapter 03.indd 71 8/29/2015 9:07:58 AM


2.72|Engineering Mathematics

Differentiating (2) w. r. t y on both sides, equation (1). y = u (x) is called the complementary function
of (1) and y = v (x) is called a particular integral of (1). The
dx -15 dp 8 dp
= 4 - general solution of (1) is given by y = u (x) + v (x).
dy p dy p3 dy = [complementary function] + [particular integral]
1 15 8 dp = C.F + P.I (in short).
= 3
dy /dx p 4 p dy To find the complementary function of (1)
-15 8 or to obtain the general solution of the
dy = 3 - 3 dp
p p homogeneous equation (2)
Integrating on both sides, dy
As y = emx is a solution of - my = 0, we assume y = emx
-15 8 dx
dy = p 3 - p 3 dp (for some value of m) to be a solution of (2).
d2 d
15 8 Then, a0 2 ( e mx ) + a1 ( e mx ) + a2 e mx must be equal to
y= + + c  (3) dx dx
2 p2 p
zero (or) emx {a0 m2 + a1 m + a2} = 0
Its not possible to eliminate p from (1) and (3). As emx cannot be equal to zero,
So the general solution of (1) is a0 m2 + a1 m + a2 = 0  (5)
15 8 (5) is called the auxiliary equation corresponding to (1) [or
y= + +c
2 p2 p (2)]. (5) is quadratic in m and gives two values for m, which
may be real or complex.
and xp3 4p 5 = 0
Case (i):
Second order linear differential equations Let the roots of (5) be real and distinct, say m1 and m2.
with constant coefficients (m1 m2). Then, y = e m1x and y = e m 2 x are two distinct solu-
The standard form of a second order linear differential tions of (2) or y = C1e m1x + C2 e m 2 x  (6)
equation with constant coefficients is (C1 and C2 are arbitrary constants) is the general solution
of (II) or the complementary function of (I).
d2 y dy
a0 + a1 + a2 y = F ( x )  (1) Case (ii):
dx 2
dx
Let the roots of (5) be real and equal and each equals to m1.
where a0, a1, a2 are real constants and F(x) is a function of d d2
only x. The second order equation, Let D, D2 .
dx dx 2
d2 y dy Then, (II) may be expressed as (a0 D2 + a1 D + a2) y = 0.
a0 + a1 + a2 y = 0  (2)
dx 2 dx As the roots of the auxiliary equation are equal and each
represents the corresponding homogeneous equation. equal to m1, this is reduce to
Let y = u (x) represent the general solution of (2) a0 (D m1)2 y = 0 or (D m1)2 y = 0 (7)
[u(x) will contain two arbitrary constants]. This means that As a0 0
d 2u du Let
a0 2 + a1 + a2 u = 0 (3) (D m1) y = Y1 (8)
dx dx  Then, (7) becomes
Let y = v (x) represent a particular solution of the given (D m1) Y1 = 0. (9)
equation of (1). We have, then,
d 2v dv dY1
a0 2 + a1 + a2 v = F ( x )  (4) Now, (9) is reduced to - m1Y1 = 0 giving Y1 = C1 e m x as
1

dx dx dx
the solution.
Substituting y = u (x) + v (x) in (1), dy
Substituting in (8), m1 y = c1e m x is a linear
1

d2 d dx
a0 2 ( u + v ) + a1 ( u + v ) + a2 ( u + v ) equation. The general solution is given by
dx dx
ye
m x
1
= c 2 + c1e m x e m x dx = c2 + c1x
1 1

d 2u du d 2v dv
= a0 2 + a1 + a2 u + a0 2 + a1 + a2 v
dx dx dx dx or y = c 2e m x + c1xe m x = e m x c 2 + c1x
1 1 1
( )
= 0 + F(x), by (3) and (4) = F(x). where c1 and c2 are arbitrary constants.
We infer that y = u (x) + v (x) is the general solution of Case (iii):
the given equation (1). Thus, the general solution of (1) is Let the roots of (5) be complex. Let us assume the roots as
the sum of the general solution of the corresponding homo- the conjugate pairs a ib. (The coefficients a0, a1, a2 being
geneous equation (2) and a particular solution of the given real, roots occur in conjugate pairs).

Chapter 03.indd 72 8/29/2015 9:08:06 AM


Chapter 3 Differential Equations | 2.73

The general solution is y = c1e (a + i b )x + c 2e (a i b )x The complementary function is given by xy = e3x(C1cosx


+ C2sinx)
= c1e ax (cos bx + i sin bx ) + c 2e ax (cos bx i sin bx )
= e ax {(c + c ) cos bx + i (c c ) sin bx }
1 2 1 2
To find a particular integral of (1) or to find
a particular solution of the equation (1)
= eax {A1 cos bx + A2 sin bx)
d2 y dy
where A and B are arbitrary constants. We may now sum- a0 + a1 + a2 y = F ( x )
dx 2
dx
marise the nature of the complementary function of (1) as
We may write the above as (a0 D2 + a1 D + a2) y = F(x) or
follows:
f(D) y = F(x) where f (D) stands for (a0 D2 + a1 D + a2).
Roots of the auxiliary Complementary function of Particular integral y is that function of x independent of
equation a0m2 + a1m + a2 = 0 (1), or general solution of (2) arbitrary constants such that f (D) on y or f (D) y yields F(x).
1
1 Roots, real and distinct, say
m1, m2
y = c1e m x + c 2e m x
1 2
This is symbololically represented as y = {F ( x )}.
f (D )
2 Roots, real and equal, say y = (c1 + c 2 x )e m x
1 Case (i):
each equals m1 F (x) = ekx where k is a constant.
3 Roots, complex, say a ib y = eax {c1 cos bx + c2 sin bx} We have D (ekx) = kekx, D2 (ekx) = k2 ekx or, in general,
4 Roots, complex and y = eax[(c1 + c2x) cos bx + g (D) (ekx) = g (k) ekx where g (D) is a polynomial in D. in
repeated, say m1 = m2 = a + ib (c3 + c4x) sinbx 1
particular, f (D) {ekx} = f (k) ekx. As e kx is that func-
and m3 = m4 = a ib f (D)
tion of x which when operated by f (D) gives ekx, it is clear
Example 23:Obtain the complementary function of the 1 1
that e kx = e kx provided f (k) 0. f(k) reduces to
d 2 y 7dy f (D) f (K )
equation - + 6 y = x4
dx 2 dx zero when one or both the roots of the auxiliary equation
d 2 y 7dy a0 m2 + a1 m + a2 = 0, is k.
Solution: - + 6 y = x4
dx 2 dx (i) Suppose one of the roots is k. Then, f(D) = a0 (D k)
(D2 7D + 6) y = x4 (D m0) where m0 k.
Particular integral
Auxiliary equation is m2 7m + 6 = 0
1
m = 1, 6. = e kx
a0 ( D - k ) ( D - m0 )
The Complementary function of the given equation.
y = c1 ex + c2 e6x 1 1
= e kx
D - k a0 ( D - m0 )
Example 24: Obtain the general solution of the equation
d2 y dy 1 1
- 10. + 25 y = 0. = e kx
dx 2 dx a0 ( k - m0 ) ( D - k )
d 2y dy 1
Solution: Given: 10 + 25y = 0 Let e kx = X 1
dx 2 dx
(D - k)
(D2 10D + 25)y = 0 dX 1
Then, (D k) X1 = ekx or - kX = ekx
Auxiliary equation is m 10m + 25 = 0
2
dx 1

The roots are (m) = 5, 5 This is a linear equation and the particular s olution
The general solution of the equation is (c1 + c2x)e5x of the above equation is xekx. Therefore, particular
1
integral = xe kx .
a0 ( k - m0 )
Example 25:Obtain the complementary function of the
d2 y dy
equation 2 - 6. + 10 y = e 3 x . (ii) Suppose both the roots of the auxiliary equation are k.
dx dx
Then, particular integral
d2 y dy 1
Solution: Given: - 6. + 10 y = e 3 x =
dx 2 dx e kx
2
a0 ( D - k )
(D2 6D + 10)y = e3x
1 1
Auxiliary equation is m2 6m + 10 = 0 = e kx
a0 ( D - k ) ( D - k )
6 36 - 40 6 2i 1
m= = =3i = xe kx ,
2 2 a0 ( D - k )

Chapter 03.indd 73 8/29/2015 9:08:16 AM


2.74|Engineering Mathematics

1 1 1
Use the result in (i) above. Now, let ( xe kx ) = X 2 P.I = e 6x = e6 x
D-k (D 12D + 36 ) ( D - 6)
2 2
2

We have, therefore, (D k) X2 = xekx Or


dX 2 x 6x 2

- kX 2 = xe kx which is a linear equation. = e


dx 2!
x 2 kx General solution is y = CF + P.I
Particular solution is X2 = e or, particular inte-
2 x 2 6x 0
x 2 kx = (C1 + C2x)e6x + e
gral in this case is given by y = e . 2!
2
Case (ii):
Example 26: Solve the differential equation.
F(x) = sin kx or cos kx where k is a constant.
(D2 + 5D + 6)y = e4x We have D {sin kx} = k cos kx
Solution: (D2 + 5D + 6)y = e4x D2 {sin kx} = k2 sin kx
Auxiliary equation is m2 + 5m + 6 = 0. Similarly, D2 {cos kx} = k2 cos kx
(m + 3) (m + 2) = 0. If g (D2) is a polynomial in D2,
roots are m = 3, 2. g(D2) {sin kx or cos kx} = g (k2) sin kx or g(k2) cos kx.
Complementary function c1 e3x + c2 e2x.
1 1 1 1
1 Hence, sin kx = sin kx and cos kx = cos kx
Particular integral = 2 .e -4 x g (D )
2
g (k )
2
g (D )
2
g (k 2 )
D + 5D + 6
1 1
1 e -4 x 2 cos kx = cos kx , provided g ( k2) 0.
= = g (D ) g (k 2 )
-4 x
e
( -4 ) + 5 ( -4 ) + 6
2
2
We shall illustrate the above technique by considering
General solution is two examples.
e -4 x
y = c1e3x + c2e2x + . Example 29: Find the particular integral of the equation
2 (D2 + 16)y = cos3x.
Example 27: Solve (3D2 D 10)y = 6e2x 1 1
Solution: P.I = cos 3 x = cos 3 x
D + 16 - ( 3) + 16
2 2
Solution: Given (3D2 D 10)y = 6e2x
Auxiliary equation 3m2 m 10 = 0 cos 3 x
=
7
m = 2, 5/3.
Example 30: Find the particular integral of the equation
Complementary function is (D2 5D + 6)y = sin3x.
CF = c1e2x + C2 e 3 x
5 1
Solution: P.I = 2 sin 3 x
D - 5D + 6
P.I = 1 1 1
6e 2 x = sin 3 x = sin 3 x
3D D 10
2
-32 - 5 D + 6 -5 D - 3
1 5D - 3
= 6e 2 x = sin 3 x
( D - 2 ) ( 3D + 5 ) ( 5 D + 3) ( 5 D - 3)
1 1 1 1 2x 5D - 3 3 - 5D
= 6 e2 x = 6 e = sin 3 x = sin 3 x
D - 2 3D + 5 D - 2 11 - ( 25 D - 9 )
2
25 -9 - 9
6 1 6 2x 1
= e2 x = xe = ( 3 - 5 D ) sin 3 x
11 D - 2 11 -234
General solution is 1
y = c1e2x + c2 e x
6 2x 5
x = [3 sin 3x 5D sin 3x ]
xe3
+ 234
11 1
Example 28: Solve (D2 12D + 36)y = e6x =
234
[3 sin 3x 15 cos 3x ]
Solution: Given: (D2 12D + 36) y = e6x 15 cos 3 x 3 sin 3 x
P.I = -
Auxiliary equation is m2 12m + 36 = 0. 234 234
m2 12m + 36 = 0. Suppose g (k2) = 0.
m = 6, 6 Let us discuss the technique of finding particular integral
Complementary function (C.F) = (c1 + c2x)e6x in this case.

Chapter 03.indd 74 8/29/2015 9:08:27 AM


Chapter 3 Differential Equations | 2.75

Suppose we have to find


1
[sin kx ] . Cauchys Homogenous Linear
D2 + k 2
By Eulers formula, eikx = cos kx + i sin kx or sin kx =
Equations
imaginary part of eikx An equation of the form
1
Particular integral = 2 [sin kx ] dn y d n -1 y
+ p1 x n -1 n -1 + .... pn y = Q ( x )  (1)
D + k2 xn
dx n
dx
1
= Imaginary part of 2 ( eikx ) where p1, p2 pn are constants is called cauchys linear equa-
D + k2
1 tion. To convert the above equation into linear differential
= Imaginary part of e ikx equation with constant coefficients, we substitute x = ez = or
( D - ik ) ( D + ik )
z = logx
1 e ikx z = logx i.e. z = logx
= Imaginary part of
D - ik 2ik dz 1
=
xe ikx dx x
= Imaginary part of
2ik dy dy dz
= .
x
= Imaginary part of (cos kx + i sin kx ) dx dz dx
2ki dy dy 1
x = .
= Imaginary part of ( -i cos kx + sin kx ) dx dz x
2k
-x cos kx dy dy
= . =x
2k dz dx
1 dy 2 d dy d 1 dy
Similarly, if we have to find 2 [cos kx ] . = = .
D + k2 dx 2 dx dx dx x dz
1
We write it as the real part of 2 ( ei kx ) =
-1 dy 1 d dy -1 dy 1 d dy dz
+ = +
D + k2
1 x 2 dz x dx dz x 2 dz x dz dz dx
= Real part of ( eikx )
( D - ik ) ( D + ik ) d 2 y -1 dy 1 d 2 y
= +
x dx 2 x 2 dz x 2 dz 2
= Real part of ( -i cos kx + sin kx ) d dy
2k d 2 y d 2 y dy
x sin kx x2 = 2 - = - y
= . dx 2
dz dz dz dz
2k dy dy d y
2

1 -x let = qy x = q , x 2 2 = q (q - 1) y
sin kx = cos kx dz dx dx
D2 + k 2 2k d 3
y
1 x Similarly x 3 3 = q (q - 1) (q - 2 ) y and so on. Then,
cos kx = sin kx dx
D2 + k 2 2k (1) is changed into a linear differential equation. We solve
this by methods discussed earlier.
Example 31: Solve the equation (D2 + 16) y = sin4x.
d2 y dy
Solution: Given: (D2 + 16)y = sin4x Example 32: Solve:. x 2 + 3x - 3 y = 0
dx 2
dx
Auxiliary equation m2 + 16 = 0
Solution: Let x = ez or z = logx
m = 4i dy d 2y
Then, x = q y ; x 2 2 = q (q 1) y
CF = C1 cos4x + C2 sin4x dx dx
1 The above equation becomes
P.I = sin 4 x.
D + 16
2
q (q 1) + 3q 3 y = 0
-
x
sin4 \ 2
1
sin kx = -
x
cos kx [q 2 + 2q 3] y = 0
2.4 D +k 2
2kx
A . E = m2 + 2m 3 = 0
x
= - cos4x = (m + 3)(m 1) = 0
8
m = 3, 1
General solution is y = C.F + P.I
x y = c1e3z + c2ez
= c1cos 4x + c2 sin4x - cos4x. Y = c1x3 + c2x
8

Chapter 03.indd 75 8/29/2015 9:08:40 AM


2.76|Engineering Mathematics

d3 y 2 d y
2
dy d 2y dy
Example 33: Solve x 3 + 6 x + 8 + 2 y = x 2 log x. u2 +u 25y = 8u 2
dx 3 dx 2 dx du 2 dx
Solution: Put x = e or z = logx. Then,
z dy d2 y
Let u = ez, u = 0; x 2 2 = q (q - 1)
dy d2 y dx dx
x = qy, x 2 2 = q (q - 1) y , y q (q - 1) + q - 25 y = 8e 2 z
dx 3 dx
3 d y
x = q (q - 1) (q - 2 ) [q 2
25] y = 8e 2 z
dx 3
The given equation becomes AE = m2 25 m = 5
[q(q 1) (q 2) + 6q(q 1) + 8q + 2]y = e2z. z CF = C1e5z + C2e5z
(q3 + 3q2 + 4q + 2)y = e2z.z 1 1 -8 2 z
PI = 2 8e 2 z = 8 e 2 z 2 = e
A E = m3 + 3m2 + 4m + 2 = 0 q - 25 2 - 25 21
(m + 1)(m2 + 2m + 2) = 0 Y = C F + P I = C1e5z + C2e5z 8 / 21e2z
m = 1 or m = 1 ie 8 2
= C1 u5 + C2 u5 u where u = (2x 1)
C F = C1e + e (C2cosz + C3sinz)
z z
21
1
P. I = 3 .e2z z Method of Variation of Parameters
q + 3q 2 + 4q + 2
d2 y dy
1 An equation of the form + P ( x ) + Q ( x ) y = R ( x ),
= e2z z dx 2
dx
(q + 2 ) + 3 ( q + 2 ) + 4 (q + 2 ) + 2
3 2
where P(x), Q(x) and R(x) are real valued functions of x, is called
1 .z the linear equation of the second order with variable coefficients.
= e2z
q 3 + 9q 2 + 28q + 30 The above equation is solved by the method of variation
-1
e2 z q 3 + 9q 2 + 28q of parameters.
30 1 + 30 z The method is explained below

e 2z
q + 9q + 28q
3 2 d2 y dy
= 1. Find the solution of + p + Qy = 0 and let the
30 1 - 30 z dx 2
dx

solution be Yc = C1U(x) + C2 V(x)
e2z 28 2 z
= z- e 2. Write particular solution as follows yp = A U(x) + B V(x)
( 30 )
2
30
-VR
Where A = dx
y = C.F + P.I W
e2z 28 2 z UR
= C1ez + ez(C3 cos z + C2 sin z) + z- e B= dx
( 30 )
2
30 W
C 1 U V
=  1 + (C3 cos(logx) + C2 sin(logx)) + dV dU
x x Where W = dU dV = U V is called the
x 2 log x 28 2 dx dx dx dx
x
30 900 wronskian of U and V.
3. Then, the solution is yc + yp
d 2y dy
+ 2 ( 2x 1) 100 y = 32 ( 2x 1)
2
Example 34: (2x 1) y = C1U(x) + C2V(x) + AU(x) + B V (x)
dx 2 dx
dy
+ 2 ( 2x 1) 100 y = 32 ( 2x 1)
2
Example 35: Solve the equation (D2 + 4)y = sec 2x by vari-
dx
ation of parameters.
du
Solution: let 2x 1 = u, 2 = Solution: Given (D2 + 4)y = sec2x
dx
dy dy du dy AE = m2 + 4 = 0 m2 = 2i
= =2 CF = yc = C1cos2x + C2 sin 2x
dx du dx du
d 2 y d dy d dy U(x) = cos 2x; V(x) = sin 2x
= = 2
dx 2 dx dx dx du Yp = AU(x) + B V(x)
d dy dy d2 y dV dU
=2 = 22 W=U V
dy du dx du 2 dx dx
d d
The given equation becomes = cos2x (sin 2x ) sin 2x (cos 2x )
d 2y dy dx dx
22 u2 2 + 2.2u 100 y = 32u 2 = 2cos22x + 2 sin2 2x = 2
du dx

Chapter 03.indd 76 8/29/2015 9:08:52 AM


Chapter 3 Differential Equations | 2.77

A=
VR yd = yc + yp
dx
W = A U (x) + B v (x ) + C1 e2x + C2 xe2x
sin 2 x.sec 2 x
= dx e 2 x x 3e 2 x 3x 2e 2 x
2 = C1e2x + C2xe2x 4x + +
tan 2 x 1 4 4 16
= dx = log ( cos 2 x ) 6 xe 2 x 3e 2 x x 4 e 2 x 3x 3e 2 x 3 3 2x
2 4 = + + x 2e 2 x e x
UR 64 3128 4 16 32 128
B= dx
W 1 3 9 xe 2 x 3 2x
= c1e 2 x + c 2 xe 2 x x 3e 2 x x 2e 2 x + e
cos 2x .sec 2x 1 16 32 128 128 128
= dx = x
2 2
1 1
yp = log ( cos 2 x ) cos 2 x + x sin 2 x
Partial Differential Equation (P.D.E)
4 2 An equation involving two or more independent variables
y = yc + yp = C1 cos 2 x + C2 sin 2 x + x, y and a dependent variable z and its partial derivatives is
called a partial differential equation.
1 1
log ( cos 2 x ) cos 2 x + x sin 2 x z z
4 2 f (x, y, z, , ..... ) = 0.
x y
Example 36: Solve the equation y + 4y + 4y = x3e2x Standard Notation
Solution: Given equation z z
= p = zx, = q = zy
(D2 + 4D + 4) y = x3 e2x x y
m2 + 4m + 4 = 0 z
2
2 z
= r = zxx, 2 = t = zyy
(m + 2)2 = 0 m = 2 x 2
y
yC = C2 e2x + C1 e2x 2z
=z =s
U(x) = e2x and V(x) = xe2x xy xy
Yp = AU(x) + BV(x) Formation of Partial Differential Equations
VR UR Partial differential equation can be formed by two ways.
A= dx, B = dx
W W
1. By eliminating arbitrary constants.
dv du d d 2 x 2. By eliminating arbitrary functions.
W= u
dx
v
dx
= e 2 x
dx
( xe 2 x ) xe 2 x
dx
(e )
Formation of P.D.E by eliminating arbitrary con-
= e 2 x e 2 x 2xe 2 x + 2xe 2 x e 2 x = e4x stants. Let the functions f (x, y, z, a, b) = 0 where a, b, are
arbitrary constants.
UR
A= dx Differentiating this partially w.r.to, x and y eliminate a, b,
udv vdu from these equation we get as
-
dx dx f (x, y, z, p, q) = 0 which is partial differential equation of
xe -2 x 3.e 2 x first order.
= - dx
e -4 x
Example 37:z = ax2 by2, a, b are arbitrary constants
e4 x e4 x
= x 4 e 4 x dx = x 4 - 4 x3 dx
4 4 Solution: Given z = ax2 by2  (1)
Differentiating z partially w.r.to x,
4 e4 x 3 e4 x 3 2 4 x
= x - x - x e dx z p
4 4 4 = 2ax p = 2ax a =
x 2x
e4 x e
4x
3 e4 x e
4x
differentiate z partially w.r.to y,
= x4 - x3 + x2 - 2 x dx
4 4 4 4 4
z
= 2by i.e, q = 2by
e 4x
x e
3 4x
x e
2 4x
xe 4x
e 4x y
= -x4 + -3 +6 -6 -q
4 4 16 16 4 16 16 b=
2y
UR e 2 x x 3e 2 x
B= W dx =
e 4 x
dx = x3e4xdx Substitute the values of a and b in (1) we get
p 2 q 2
z= x + ,y
e 4x 3 2 e 4x xe 4 x e 4 x 2x 2y
= x3 x 2 +
4 4 4 16 32 2z = p x + q y which is a partial differential equation of order 1.

Chapter 03.indd 77 8/29/2015 9:09:06 AM


2.78|Engineering Mathematics

Formation of P.D.E by eliminating arbitrary Linear Equation of First Order


function Linear equation of first order is Pp + Qq = R. This is also
Consider z = f (u) (1) called Lagranges equation. where P, Q, R are the functions
Z is an arbitrary function in u and u is function in x, y, z in x, y, and z.
Now, differentiate (1) wr x, y partially by chain rule we get
Procedure for solving lagrange equations
z f . u f . u . z Take the auxiliary equation as
= +  (2)
x u x u z x dx dy dz
= = .
z f . u f . u . z P Q R
= +  (3)
y u y u z y Solving any two equations and take the solutions as u and v.
The complete solution is f (u, v) = 0 or u = f (v).
by eliminating the orbitary functions from (1), (2), (3) we
get a P. D. E. of first order Example 39: (z y) p + (x z) q = y x.
Solution: Auxiliary equation
Formation of P.D.E. two arbitrary functions dx dy dz
are involved = = .
z y x z y x
When two arbitrary functions are involved, we differentiate Using the multipliers of x, y, z we get
the given equation two times and eliminate the two arbitrary
functions from the equation obtained. xdx + ydy + zdz
x (z y ) + y (x z ) + z ( y x )
f ( x)
Example 38: z = = xdx + ydy + zdz = 0
g ( y) x2 + y2 + z2 = 0
f ( x) and also
dx + dy + dz
=0
Solution: Given z = z y +x z +y z
g ( y)
dx + dy + dz = 0, x + y + z = 0.
f 1 (x )
p = zx =  (1) The required solution is x2 + y2 + z2 = f (x + y + z).
g (y )
f ( x ) Non-Linear Equation of First Order
q = zy = .g(y)  (2)
g ( y )
2
There are four types of non linear equations of first order.
f ( x) Type I:
z=
g ( y) f (p, q) = 0.
The given equation contain only p and q, then the solution
z f 1 ( x )
= is taken as z = ax + by + c.
x g ( y )
Example 40: Solve 2p + 3q = 5
2 z f 1 ( x )
S= = .g(y) (3) Solution: Given 2p + 3q = 5
x y g ( y )
2

Z = ax + by + c.
f ( x ) -f ( x ) . g ( y ) 5 - 2a
(1) (2) = pq = = s. z Where 2a + 3b = 5, b =
g (y ) gy 2 3
pq + sz = 0 5 - 2a
The solution is z = ax + y + c.
3
Forming P.D.E by the elimination of arbitrary
Type II:
function of specific functions
f (z, p, q) = 0
Consider F (u, v) = 0
Where u, v are the functions in x, y, z, When the equation is not containg x and y, then to solve the
dz dz
Differentiate the above equation w. r. t x and y by chain rule equation assume u = x + ay and substitute p = ,q= a
du du
F F Solve the resulting equation and replace u by x + ay
and eliminate the , and convert them in the form
u v
Pp + Qq = R. which is a first order linear P.D.E. where P, Q, Type III:
R functions x, y, z. f(x, p) = f(y, q).

Chapter 03.indd 78 8/29/2015 9:09:17 AM


Chapter 3 Differential Equations | 2.79

The equation not containing z. The laplace equation


Assume f (x, p) = a and f (y, q) = a.
2u 2u
Solve the equations for p and q, and then write the solution + = 0 is elliptic (B2 4AC < 0)
x 2
y 2
Example 41: Solve p2 q2 = x2 y2.
Method of separation of variables Consider a PDE
Solution: p2 q2 = x2 y2 involving a dependent variable u and two independent
p2 x2 = y2 + q2 variables x and y. In the method of separation of variables,
Let p2 x2 = a2 = y2 + q2 we find a solution of the PDE in the form of a product of a
function of x and a function of y,
p2 = a2 + x2q2 = y2 + a2
i.e. we write u(x, y) = X(x). Y(y)  (1),
p= a2 + x 2 q = a2 + y 2
u u
Take dz = pdx + qdy then = (xy) = x1y; = (xy) = xy1
x x y y
Integrating on both sides, dz = pdx + qdy
2u 2u 2u
= x11
y, = x1y1, = xy11 and so on
x 2 x y y 2
z= a2 + x 2dx + a2 + y 2dy
dX dY d2 X d 2Y
x 2 a2 x y 2 Here, x1 = ; y1 = ; x11 = ; y11 =
= a + x2 + sin1h + a + y2 dx dy dx 2
dy 2
2 2 a 2
Substituting these in the given PDE, separating x and its
a2 y derivatives from y and its derivatives, finding solutions for
+ sin1h + b.
2 a x and y and substituting them in (1), we get the solution of
the given PDE
Type IV: This is best explained through the examples given
z = px + qy + f (p, q) Example 42: Solve xp + yq = 0 by the method of separation
The equation in the above form is clairant equation. The of variables
solution is z = ax + by + f(a, b).
Solution: For the PDE
Classification of second order homogenous linear equa-
xp + yq = 0  (1)
tion A second order linear homogeneous PDE of the form
Let z = X(x) . Y(y)  (2)
2f 2f 2f f f Be the solution
A + B + C +D +E + Ff (x, y)
x 2
x y y 2
x y z z
=0 (1) p= = x1y and q = = xy1
x y
Where A, B, C, D, E and F are either functions of x and y Substituting these in (1)
only or constants is called x X1 Y + y X Y1 = 0
1. a parabolic equation, if B2 4AC = 0 xX1Y = y XY1
2. an elliptic equation, if B2 4AC < 0 X1 Y1
3. a hyperbolic equation, if B2 4AC > 0 x =y  (3)
X Y
For example: In (3), as LHS is a function of x alone and RHS is a
function of y alone, they are equal only if each of them is
1. consider the one dimensional heat equation
equal to some constant
u 2 u
= c2 X1 Y1
t x2 x =y = k (say)  (4)
X Y
2 u u Where k is a constant
c2 =0
x2 t X1
From (4), x = k xX1 = kX
Comparing it with (1), we have X
dX
A = c2, B = 0 and C = 0 x = kX
dx
B2 4AC = 02 4 c2 0 = 0 dX dX
One dimensional heat equation is parabolic = k.
dx dx
Similarly, it can be easily observed that
2. one dimensional wave equation Integrating on both sides we have
2 y 2u dX dX
t 2
= c2 2 is hyperbolic (B2 4AC > 0) and (3).
t
X
= k
X

Chapter 03.indd 79 8/29/2015 9:09:32 AM


2.80|Engineering Mathematics

log x = k log x + log C1 T1 = (3 k)T


log x = log xkc T1 (3 k) T = 0  (6)
x = c1 xk  (5)
Which is a linear equation with its auxiliary equation being
Y1 m (3 k) = 0
Again from (4), y =k
Y m=3k
y Y = kY
1
The solution of (6) is T = C2 e(3 k)t  (7)
dY
y = k Y Substituting (5) and (7) in (1), we get the general solution
dy of given PDE (2) as
dY dy U = X.T = (C1ekx) (C2e(3 k)t)
= k
dy y
= C1C2ekx+(3k)t
Integrating on both sides,
dy dy u = c ekx+(3k)t; where c = c1 c2
y = k y u(x, t) = cekx+(3k)t  (8)
log y = k log y + log C2 Given u(0, t) = 4et
log y = log yk C2 From (8), u(0, t) = c e(3-k)t = 4et
y = C2 yk (6) Comparing on both sides, we get
Substituting (5) and (6) in (2), we get the solution of (1) as C = 4, 3 k = 1
c = 4; k = 2
Z = (C1xk) (C2yk)
= C1C2 xkyk Substituting these in (8), we get the required solution of
k
(2) as
x
Z = C where C = C1C2 U(x, t) = 4e2x+t
y
Example 43: Solve the PDE ux + ut = 3u; u(0, t) = 4et by the
method of separation of variable. Transforms
Solution: Let u = X(x). T(t) (1) be the solution of the Laplace Transform
PDE Let f(t) be a given function defined for all t 0. The Laplace
Ux + ut = 3u  (2) transform of F(t) is denoted by L{f(t)} or L{f} and is defined as
u
U = x T ux = = X1T and ut F ( t ) dt = f ( s )
x L {F(t)} = e - st

u 0
= = XT1 Here, L is Laplace transform operator. F(t) is the determin-
t
ing function depends on it. F(s) is the function to be deter-
Substituting these in (2), we get
mined called generating function. e - st is called kernel of the
X1T + XT1 = 3XT transform.
Dividing throughout by XT, we have Some standard results of Laplace transforms are given
below.
X1 T1
+ =3 1
X T 1. L {e at } = ,s>a
s a
X1 T 1
= +3=k (3), (say) 1
X T L {e at } =
2. ,
s+a
X1 k
From (3), = k X1 = KX 3. (a) Let k be a constant L {k } =
X s
X1 KX = 0  (4) 1
(b) L {1} = , s > 0
Which is a linear equation with its auxiliary equation being s
n!
mk=0m=k 4. L {t } = n +1 , s > 0
n

Hence, its solution is x = C1 ekx  (5) s


s
-T 1 5. L {cos at } = 2 ,s>0
Again from (3), +3=k s + a2
T
T 1 a
=3k 6. L {sin at } = 2 ,s>0
T s + a2

Chapter 03.indd 80 8/29/2015 9:09:41 AM


Chapter 3 Differential Equations | 2.81

s p
7. L {cos h at } = ,s > a = e st sin 2t dt
s - a2 2
0
a
8. L {sin h at } = 2 ,s > a e - st
[ - s sin 2t - 2 cos 2t ] 0
p
s a2 =
s +4
2
n!
9. L {t n e at } = 2 (1 - e -ps )
( s - a)
n +1
=
s2 + 4
10. L f (t ) = ........... F (s )ds
1
t n s (n times ) s Example 47: Find the Laplace transform of the function
F(t) = (sint + cost)2
Example 44: Find the Laplace transform of the function Solution: L{sin t + cos t)2} = L{1 + sin2t} = L{1} + L{sin 2t}
F(x) = 5e2x + 7e3x 1 2
= + 2
s s +4
Solution: L {F ( x )} = L (5e 2 x + 7e 3x )
Some important (theorems) properties of
= 5L (e2x) + 7L(e3x) laplace transform
1 1
L {F (t )} = 5 + 7 1. Linear property: Let f and g be any two functions of
s2 s+3 t and a1, a2 are constants, then L{a1f(t) + a2g(t)} = a1L
5 7 {f(t)} + a2L{g(t)}
= +
s-2 s+3 2. Shifting property: If L{f(t)} = F(s), then L{eat f (t) =
F(s a)
Example 45: Find L {F (t)} where F(t) = 0, 0 < t < 1 s-a
= 1, 1 < t < 2 Example: L{eat cos ct} =
( s - a ) + c2
2
= t, t > 2
3. Change of scale property: If L{f(t)} = F(s), then
Solution: As the given function is not defined at t = 0,
1 s
1 and 2 L{f(at)} = F
a a
L {F (t )} = e st F (t )dt Example: We know L {e at } =
1
= F (s )
0 s a
1 2 1 s 1 1 1 b
= e - st 0 dt + e - st 1dt + e - st tdt Then, L {be at } = F =
b b b s b s - ab
0 1 2 -a
2
b
= e - st dt + e - st tdt 4. Differentiation theorem: If derivatives of f(t) are
1 2 continuous and L{f(t)} = F(s), then L {f(t)} = sF(s)
2 f(0) and
e - st e - st e - st
= t
-s 1 -s 2 2 -s - dt L{fn(t)} = snF(s) sn 1 f(0) sn3 f11(0) . fn1(0)
n 1
= snF(s) s n 1 r f r (0) (fr represents rth derivative

e 2s e s 2e 2s 1 e st r =0
= + + + of f)
s s s s s

2 5. Multiplication theorem: If L{f(t)} = F(s), then


e 2s e s e 2s 1 2s L{t . f(t)} = F1(s)
= + +2 + 2e dn
s s s s and L{tn.f(t)} = ( 1) n n [ F (s ) ]
ds
e 2s 1 e s
= F(s), then L f (t) = F (s )ds
= 1 + + 1
s s s 6. Division theorem: If L{f(t)}
t s
1
Example 46: Find the Laplace transform of the functionL f (t) = F (s )ds
t s
F(t) = sin 2t, 0 < t < p
7. Transforms of integrals (theorem)
= 0, t > p
1
t

Solution: L{F(t)} = e F (t )dt
st If L{f(t)} = F(s), then L f (u )du = F (s )
0 s
0
p Example 48: Find the Laplace transform of t e2t sin2t.
= e st
sin 2tdt + e st
0dt 1 1 1 s
0 p Solution: L {sin 2 t} = L {1 - cos 2t} = - 2
2 2 s s + 4

Chapter 03.indd 81 8/29/2015 9:09:53 AM


2.82|Engineering Mathematics

d 1 1 s 1
L {t sin 2 t } = ( 1) 1. L1 = 1
ds 2 s s 2 + 4 s
(Using multiplication theorem) 1 tn
2. L1 n +1 = where n is a positive integer
s n!
-1 -1 ( s 2 + 4 ) - s ( 2 s )
L{tsin2t} = - 1 t n 1
2 s2 ( ) or L1 n =
2
s 2
+ 4 s ( n 1)
= 1 4 - s 2
+ 1
2 s 2 ( s 2 + 4 )2 3. L1 = e at
s a
4 - ( s + 2)
2
1 1 e at t n 1
L {e -2t t sin 2 t} = + 4. L1 =
2 ( s + 2) n
(s a) ( n 1)!
2 2
( s + 2 ) + 4
2


(Using shifting property) 1 1
5. L1 2 = sin at
1 4s + s2 s + a2 a
=
2 ( s + 2 ) ( s 2 + 4 s + 8) s
2 2
6. L1 2 = cos at
s + a2
sin 2t cos 2t s
Example 49: Find the Laplace transform of . 7. L1 2 = cosh at
t s a2
2 s 1 1
Solution: L{sin 2t cos 2t} = 2 8. L1 2 = sinh at
s +4 s +4
2
s a2 a

sin 2 t cos 2 t 2 s 1
L = 2 ds 9. L1
1
= e at sin bt
t s s + 4 s2 + 4 (s a) + b 2 b
2

(using division property)


2 s 1
sa
= tan 1 log ( s 2 + 4 ) s

10. L1 = e cos bt
at

2 2 s 2 ( s a ) 2
+ b 2

p s 1
= tan 1 + log ( s 2 + 4 ) 1 1
2 2 2 11. L1 = (sin at at cos at)
( s + a )
2
2 2 2a 3
s 1
= cot 1 + log ( s 2 + 4 )
2 2 1 1
12. L1 = t sin at
(s + a )
2
t
2 2 2a
sin 2u
Example 50: Find the Laplace transform of
0
u
du
To find the inverse Laplace transform we use the following
methods.
2 sin 2u 2
Solution: L {sin 2u} = and L = 2 ds 1. Using the following properties
s +4 2
u 0 s +4
(Using division theorem) (a) If L1 {F (s )} = f (t ), then L1 {F (s a)} = e at f (t )

2 1 s p s s (b) If L-1 {F (S )} = f (t ) and f (0) = 0;
tan = tan 1 = cot 1
2 2 s 2 2 2
d
sin 2u 1 1 s
t (f (t ))
then (i) L1 {sF (s )} =
dt
L dt = cot
0 t s 2 dn
(ii) L1 {s n F (s )} = n (f (t ) ) if f(0) = f1(0) =
(using transform of integration theorem) dt
= f n1(0) = a
Inverse Laplace Transforms F (s )
t

(c) If L1 {F (s )} = f (t ), then (i ) L1 = f (t )dt


If F(s) is the Laplace Transform of the function f(t) i.e., s 0
L{f(t)} = F(s), then f(t) is called the inverse Laplace trans- F (s )
t
1
{F (s )} =
form of the function F(s) and is written as f(t) = LL1{F(s)}. f (t ), then((i)i ) L1 = f (t )dt
1
s 0
Here, L is called inverse Laplace transformation operator. t t
(ii) L1
F (s )
Some important standard results for inverse Laplace
transform.

s2 0 0
= (t )dt } dt

Chapter 03.indd 82 8/29/2015 9:10:07 AM


Chapter 3 Differential Equations | 2.83

2. Convolution theorem: Let f(t) and g(t) be two func- Note:


tions and L1 {F (s )} = f (t ) and L1 {G (s )} = g (t ) , 1. The above result is also known as Bromwichs
t integral formula
then L1 {F (s ) G (s )} = f ( x ) g (t x ) dx 2. The integration is to be performed along a line s = r
0 in the complex plane where s = x + iy. The real
It is denoted by f(t) * g(t) here * represents convolution. number r is chosen so that p = r lies to the right of
3. Unit step function: This function is defined as U(t a) all the singularities.
0 t < a
H(t a) = the Laplace transform of H(t a) 10. The gamma function: If n > 0, then the gamma
1 t a
= L {H(t a)} function is defined by (n) = u n -1e u du

e as 0
= e st u (t a) dt = 11. Exponential integral: The exponential integral is
0
s denoted by
u
Note: This is also called as Heavisides unit function e
Ei(t) = du
4. Periodic function: If f(t) is a periodic function with t
u
period a i.e. f(t + a) = f(t), then
a e 2 3 s
e -st f (t )dt Example 51: Evaluate L1 52

L{f(t)} = 0 ( s + 2)
1 - e - sa
1 1
G ( s) Solution: We have L1 52
= e 2t L1 5 2
5. Using partial fractions: If F(s) is of the from
H ( s) ( s + 2) s
5 3
1
where G and H are polynomials in S, then break F(s) t2 2t 2 e 2t
into partial fractions and manipulate term by term. = e 2 t
=
5 3 p
6. Heavisides expansion formula: Let F(s) and G(s)
2
be two polynomials in s where F(s) has degree less
than that of G(s). If G(s) has n distinct zeros ar, r = 1, e 2 3 s e 3 s
L1 52
= e 2 L1 52
2, 3, . n ( s + 2) ( s + 2)
i.e, G(s) = (s a1)(s a2).(s an), then

(t 3)3/ 2 e 2(t 3) , t > 3
4
F (s ) n F a r a t ( ) = e 2
L1 = 1 e r 3 p
G ( s ) r 1 G a r ( ) 4
= (t 3)3/ 2 e 2(t 4) , t > 3 = 0, t < 3
3 p
Transform of Special Functions 4
= (t 3)3/ 2 e 2(t 4) , H (t 3)
7. Bessel function: 3 p
x x 2
x 4 6 (when expressed in terms of Heaviside unit step function)
J0 (x) = 1 + 2 2 2 2 2 + ....
2 2 4 2 4 6 3s + 7
1 Example 52: Evaluate L1 2
then L{J0 (x)} = s 2s 3
s2 + 1
8. Error function: 3 ( s 1) + 10
Solution: L1
( s 1) 4
Error function is denoted as erf (t) 2

( x) =
x
2
erf
p
e t 2
dt , 3 ( s 1) 10
0 = L1 +
( s 1) 4 ( s 1) 4
2 2

then L {erf ( x) = s
s +1
1
s 1
1
1
9. Complex inversion (theorem) formula: If f (t) has a = 3L1 + 10 L
( s 1) 4 ( s 1) 4
2 2

continuous derivative and is of exponential order and


L{f (t)} = F(s), then L1{F (s)} is given by s 1
1
1
r + i = 3etL1 s 2 22 + 10 L s 2 22 et.
e st F (s ) ds, t > 0 and f(t) = 0 for t < 0
2pi r i
f (t) =
= 3et cosh 2t + 5et sinh 2t = 4e3t et

Chapter 03.indd 83 8/29/2015 9:10:17 AM


2.84|Engineering Mathematics

applying Laplace transform we get


1
Example 53: Evaluate L1 2 L{y11} + 6L{y1} + 9L{y} = 9L{e3t}
s ( s + 4 )
2
9
or s2 L{y} sy(0) y1(0) + 6[sL{y} y(0)] + 9L{y} =
1 s s+3
Solution: L1 2 9
or s2 L{y} + s 9 + 6s L{y} 6 + 9L{y} =
s ( s 2 + 4 )2 s+3

1 s 9
Let F1 (s) = 2 and F2 (s) = so that = (s2 + 6s + 9) L{y} = s+3
s+3
s ( s + 4 )2
2
18 s 2
1 (s + 3)2 L{y} =
L1 {F1 (s)} = L1 s 2 = t = f1 (t) s+3

18 s 2
s L{y} =
and L1 {F2 (s)} = L1 2 2 ( s + 3)3
( s + 4 )
9 ( s + 3)2 + 6 ( s + 3)
t sin 2t y = L 1

= = f2 (t) (say)
4 ( s + 3)3
By convolution theorem, we have
9 s2 + 6s
1 = e 3t L1
L1 2 .
s
2 { }
= L1 F1 (s ) F2 (s ) s3

s ( s 2
+ 4 ) 9 1 1
t t = e 3t L1 3 L1 + 6 L1 2
x s s s
= f 2 ( x ) f1 (t x ) dx = sin 2 x (t x ) dx
0 0
4 t 2

t t y = e3t 9 1 + 6t
t 1 2!
=
40 x sin 2 xdx x 2 sin 2 xdx
40 The required solution is
t e 3t
=
t x 1
cos 2 x + sin 2 x
y=
2
(9t 2 + 12t 2)
4 2 4 0

t
Solving of simultaneous ordinary differential equations
1 x2 x 1 using the Laplace transform
cos 2 x + sin 2 x + cos 2 x
4 2 2 4 0
Example 55: Solve (D + 2)x + (D 1)y = 6e3t
1
= (1 t sin 2t cos 2t ) (2D + 3)x (D + 3)y = 6e3t,
16 x = 3, y = 0 when t = 0
Application of laplace transforms to solutions Solution: Taking Laplace transform on both sides of the
of differential equations given two equations we get
Solution of ordinary differential equations with constant L{x} + 2 L{x} + L{y} L{y} = 6 L{e3t} and
coefficients: 2L{x} + 3L{x} L{y} 3 L{y} = 6 L{e3t}
Consider a Linear differential equation with constant 6
or s x x (0) + 2 x + s y y (0) y = and
coefficients s+3
(Dn + C1Dn1 + C2Dn2 +.....+ (Cnt)y = F (t)  (1) 6
2s x 2x (0) + 3 x s y + y (0) 3 y =
where F (t) is a function of the independent variable t s+3
6
Let y (0) = A1, y1 (0) = A2, ......., yn1 (0) = An1 (2) or x (s + 2) + y (s 1) = 3 and
be the given initial or boundary conditions where A1, A2... s+3
An1 are constants. as x (0) = 3 and y (0) = 0
By taking the lapalce transform on both sides of (i) and Solving the above two equations for x and y we have
using the conditions (2), we obtain an algebraic equation 4 s 2 8s 4 s 2 + 8s + 11
known as subsidiary equation from which y (s) = L{y (t)} is x = and y =
determined. The required solution is obtained by finding the ( s + 3) ( s + 1) 2
( s + 3) ( s + 1)2
inverse Laplace transform of y (s) 2 2 4
i.e x = + and
Example 54: Solve (D + 3) y = 9 e , y (0) = 1 and y (0) = 9
2 3t 1 s + 3 s + 1 ( s + 1)2
Solution: The given equation can be written as 1 2 2
y = + +
(D2 + 6D + 9)y = 9e3t s + 3 s + 1 ( s + 1)2

Chapter 03.indd 84 8/29/2015 9:10:34 AM


Chapter 3 Differential Equations | 2.85

1 Solution: Taking Laplace transform on both sides of the


2 1 1 u 2u
x = L1 + 2 =
1
s + 3
2L 4L
s + 1 ( s + 1) equation
t
=2 2
x
2e3t 2et + 4tet u 2u
L = 2L 2
1 1 t x
and y = L1 s + 3 + 2L1 s + 1 d 2u
s u u (x, 0) = 2. 2
dx
1 2u s
2L1 2 = e
3t
+ 2et + 2tet or u = 4sin 2px as u (x, 0) = 8sin 2px
( s + 1)
x 2 2
The general solution of the above equation is u
Heat equation 4 sin 2p x
The heat flow in a body of homogenous materia L is gov- = Ae S 2X
+ Be S 2X

s
( 2p )
2
erned by the heat equation
2
u 2u 2u 2u k or
= c 2 2 + 2 + 2 where c2 = 8 sin 2px
t x y z sr u = Ae S 2X
+ Be S 2X
+ (1)
and u (x, y, z, t) is the temperature in a body, k is the thermal 8p 2 + s 
conductivity, s is specific heat of the body, r is the density But u(0, t) = 0 = u(4, t)
of the material and c2 the constant is called the diffusivity u (0, s) = 0, u(4, s) = 0
of the body. If the heat flow is in X-direction only, then u
From (1) we have A + B = 0
depends only on X and t, then the heat equation becomes
u 2u
4
s
4
s
8 sin 8p
= c 2 2 which is known as one dimensional and 0 = Ae 2 + Be 2 +
t x 8p 2 + s
s s
heat equation 4 4
Ae 2 + Be 2 = 0
Wave equation Solving we get A = B = 0
8sin 2px
The one dimensional wave equation of a vibrating elastic From (1) we have u =
string is given by 8p 2 + s
2u 2u T 8
= c 2 2 where c2 = y = L1 8p 2 + s sin 2px
t 2
x r
2
Laplace equation i.e. y = 8 e 8p t sin 2p x
When the temperature in a homogenous material are in
steady state and the temperature does not vary with time, Example 2: Solve the wave equation of a stretched string g iven
then the heat conduction equation becomes 2u 2u
by = 9 2 satisfying the boundary conditions u (x, 0)
2u 2u 2u t 2
x
+ + = 0 and this is known as Laplaces equa-
x 2 y 2 z 2 = 0, ut (x, 0) = 0, x > 0 and u (0, t) = F (t), Lt u ( x, t ) = 0, t 0
x
tion in cartesian system While solving the boundary value
problems the following results may be used 2u 2u
Solution: Given = 9 2
If u (x, t) is a function of x and t t 2
x
u Taking Laplace transform on both sides of the equation with
L = su ( x, s ) u ( x, 0 )
1. the boundary conditions we have
t
2u 2u
2u L 2 = 9L 2
L 2 = s 2 u ( x, s ) su ( x, 0 ) ut ( x, 0 )
2. t x
t
d 2u d 2u s2
u du or s2u (x, s) su (x, 0) ut (x, 0) = 9. 2 or u
L =
3. dx dx 2 9
u dx =0 (1)
2u d 2 y
= where L {u (x, t)} = u (x, s)
F (t ) e dt = F (s ) and u(x, s) = 0 as x
4.
L Also u (0, s) = st
x 2 dx 2
0
5 5
x x
Solved Examples The G.S of (1) is u(x, s) = c1e 3 + c2 e 3

Example 1:Solve the one dimensional heat equation as u (x, s) = 0 x c1 = 0


u 2u and u (0, s) = F(s) = C2
= 2 2 satisfying the boundary conditions u (0, t) = 0 =
t x
sx
u (4, t) and u (x, 0) = 8sin 2px Hence u(x, s) = F (s) e 3

Chapter 03.indd 85 8/29/2015 9:10:53 AM


2.86|Engineering Mathematics

sx when expressed in terms of Heavisides unit step function.


u (x, t) = L1 e 3 F ( s ) x x
u(x, t) = F t H t
3 3
x x
F t 3 , t > 3 as L {F (s ) = F (t )} ,

=
0, t < x
3

Exercises
Practice Problems 1 dy
9. Solve = x
Directions for questions 1 to 80:Select the correct alternative dx
from the given choices. x2 x2
y=
(A) y=
+ c (B) +x+c
d2y 2 2
1. The order and degree of the D.E. 2 = n2y respectively are
dx -x x xx
y=
(C) y=
+ c (D) +c
(A) 1, 2 (B) 1, 1 (C) 2, 2 (D) 2, 1 2 2
2. The order and degree of the D.E dy
3
10. Solve (x + y)2
= k2.
dy 2 d2y dx
x+ y
1 + = 2 respectively is y = tan1 (x + y) (B)
(A) y = sin1 +c
dx dx k
x+ y x+ y
(A) 1, 2 (B) 3, 2 (C) 2, 2 (D) 2, 3 y = k tan1
(C) y = cot1
+ c (D) +c
k k
4
3. The differential equation whose solution is y = mx + , where dy
m is parameter is m 11. The general solution of the D.E. = (3x + y + 1)2 is
2 2 dx
dy dy dy dy
x - y + 4 = 0. (B) -
(A) + 4 = 0. (A) sec1 (3x + y + 1) = x + c
dx dx dx dx
1 3x + y + 1
(B) tan1 =x+c
2
dy dy dy
(C)
x - y + 4 = 0. (D)
x + + 4 = 0. 3 3
dx dx dx (C) tan1 (3x + y + 1) = x + c
4. Obtain the differential equation, whose solution is given by xy 2 2x - y + 1
= aex + bex, where a, b are arbitrary constants.
(D) tan1 =x+c
3 3
(A) xy1 2y1 + xy = 0 (B) xy2 + 2y1 = xy
(C) y2 + 2y1 = xy (D) xy2 2y1 = xy dy x - y
12. The general solution of = is
dx x + y
5. If y = c1 logx + c2 log c3 + c4 ex + c5 is the general solution of
a homogeneous linear differential equation, then the order of x2 + xy + y2 = k (B)
(A) x2 y2 = k
the equation is x 2xy y = k (D)
(C) 2 2
x2y2 = k
(A) 2 (B) 3 (C) 4 (D) 5 13. Solve
6. The differential equation of all rectangular hyperbolas with y y y y dy
asymptotes as coordinate axes is x cos x + y sin x y - x y sin x - x cos x dx = 0

(A) xy + xy = 0 (B) 1 + y = 0
(C) y + xy = 0 (D) x + yy = 0 (A) sec (x/y) = cxy (B) cos (x/y) = cxy
(C) cos (y/x) = cxy (D) sec (y/x) = cxy
7. Find the solution of tany sec x dx + tan x sec ydy = 0 when
2 2

p dy x - 2y +1
x=y= . 14. The general solution of = is
4 dx 2 x - 4 y + 3
(A) tan x tan y = 1 (B) cot x tan y = 1 x2 4xy 6y = c
(A)
(C) tan x cot y = 1 (D) cot x cot y = 1 x2 4xy + 4y2 + 2x 6y = c
(B)
8. The general solution of the D.E. x2 + 4xy + 4y2 + 2x 6y = c
(C)
(ex + 1) ydy = (y + 1) exdx is x2 + 4xy x + 6y = c
(D)
dy
(A) log (ex + 1) log (y + 1) + c = 0 + 3 x 2 sin y = x 2
15. Consider the differential equation cos y
(B) log (ex + 1) = y log (y + 1) + c dx
To convert the above equation into linear form the substi-
(C) log (ex 1) + log (y + 1) + c = 0
tuted variable is
ex (A) z = cosy (B) z = cosecy
(D) log =c
y +1 (C) z = siny (D) z = secy

Chapter 03.indd 86 8/29/2015 9:11:06 AM


Chapter 3 Differential Equations | 2.87

16. The general solution of yexy dx + (xe xy + 2y) dy = 0 is y = c1e2x + c2ex


(A) (B) y = (c1 + c2x) e4x
(A) ex + y2 = c (B) exy + y2 = c y = c1e2x + c2e4x
(C) (D) y = (c1 + c2x)e2x
(C) e y + xy = c (D) ey + xy = c
2

27. The solution of (aD2 + bD + c) y = 0 whose auxiliary equation


17. The integrating factor of the equation (x + xy y )dx + (xy
2 2
has its discriminant as zero and has 5 as one of its roots is
x2)dy = 0 is (A) y = c1e5x + c2e5x (B) y = c1ex + c2ex
(A) 1/x2 (B) 1/x3 (C) x2 (D) x3 (C) y = (c1 + c2x) e5x (D) y = c1 + c2 x
dy d3 y d2y
18. Consider the differential equation cos y + 3 x 2 sin y = x 2 28. Find the general solution of + 3 2 4y = 0.
dx dx 3
dx
The solution of the above equation is
y = (c1 + c2x)ex + c3e2x (B)
(A) y = (c1 + c2x)e2x + c3ex
(A) siny e - x = e x + c y ex = 3 ex + c
(B) sin
3 3 3 3

y = (c1 + c2x)e + c3e (D)


(C) 2x x
y = (c1 + c2x)ex + c3e2x
1
y = e x + c
(C) sin y ex = ex + c
(D) sin
3 3 3
d4 y d2y dy
3 29. Find the general solution of -4 4 y = 0.
dy dx 4
dx 2 dx
19. The solution of (1 + x) xy = 1 x satisfying the initial
dx y = (c1 + c2x)e2x + ex (c3 cosh 2x + c4sinh 2x )
(A)
conditions at x = 0 and y = 1 is y = (c1 + c2x)e2x + ex (c3 cosh 2x + c4sinh 2x )
(B)
(A) 1 + x = y + ex (B) y (1 + x) = x + ex
y = (c1 + c2x)ex + ex (c3 cosh 2x + c4sinh 2x )
(C)
(C) x + y = ex (D) x (1 + y) = cex
y = (c1 + c2x)ex + ex (c3 cos hx + c4sin hx)
(D)
dy
20. The general solution of x + y = y2Log x is
dx 30. A solution of the differential equation
(A) y = log x + cx (B) y = x + c log x d4 y d3 y d2y dy
8 + 24 2 32 + 16y = 0 is _____
1 1 dx 4
dx 3
dx dx
(C) = 1 + cx (D) = 1 + cx + log x
y y (A) y = (1 8x + 24x2 32x3) e2x
Linked answer for questions 21 and 22: (B) y = ex e8x + e24x e32x
dy (C) y = [(2 + 3x) cos2x + (5 4x) sin2x]
Consider the differential equation + y cot x = y 2 sin x (D) y = (1 + x)e2x + (x2 + x3) e2x
dx
21. The integrating factor of the above equation is 31. The general solution of the differential equation
(A) cosec x (B) sin x d4x d2x
+ 13 + 36x = 0 is _______
(C) cos x (D) sec x dt 4 dt 2
22. The solution of the above equation when (A) x = (c1 + c2t) cos2t + (c3 + c4t) sin3t
p (B) x = c1e2t + c2e2t + c3e3t + c4e3t
x = , y = 1 is (C) x = (c1 + c2t) e2t + (c3 + c4t) e3t
2
(D) x = c1 cos2t + c2 sin2t + c3cos3t + c4 sin3t
p +2 cosec x p +2
(A) y cosec x x = (B) +x=
2 y 2 32. The particular integral of (D2 4D + 3) y = e3x is
p -2 cosec x p +2 xe 3 x
y cosec x + x =
(C) (D) -x= (A) (B) e3x
2 y 2 2
1
dy (C) e3x (D) xe2x
23. The solution of the equation + 3 xy = y n x is 2
dx
33. The particular integral of (D3 4D2) y = 6 is
3( n -1)
1 x 2

y1- n = + c.e 2 (B)


(A) y1n = ce3(n+1)+2 3 2
3 (A) x2 (B) x
4
1
y1n =
(C) + ce ( n -1) 3 x (D) None of these 3 -x 2
2

6(1 - n) (C) x2 (D)


4 4
24. The solution of the differential equation 34. The particular integral of D.E. (D3 5D2 + 7D 3)y = 2ex
2xy dy + (x2 + y2 +1)dx = 0 is cosh 2x is
x3 + xy2 + 3x = c (B)
(A) x3 + 3xy2 + x = c e3 x + e - x e x - e -3 x
(A) (B)
x 3
4 4
(C) + xy2 + x = c (D) 3x2 + y2 + 2x = c
3 1 1
(C) {xe3x + ex} (D) {4xe3x ex}
25. The solution of the differential equation 16 16
(3xy + 2 y2)dx + (x2 + 2xy)dy = 0 is 35. The particular of integral of (D2 + 3D + 2)y = cos 2x is
x3y + x2y = c (B)
(A) x3y + x2y2 = c 3 sin 2 x - cos 2 x
x2y + xy2 = c
(C) (D) 2xy (x + y) = c (A) 3 sin 2x cos 2x (B)
20
d2y dy cos 2 x - 3 sin 2 x cos x - sin 2 x
26. The general solution of - 6 + 8 y = 0 is (C) (D)
dx 2 dx 10 40

Chapter 03.indd 87 8/29/2015 9:11:25 AM


2.88|Engineering Mathematics

36. The particular integral of (D2 D) y = x2 2x + 4 is 47. Solve (2p + 1) q = pz


(A) x3 8x + 4 (B) x3 + 4x 4 (A) py xq = z (B) py + xq = z
x 3
-x 3 (C) px + yq = z (D) px yq = z
(C) + 8x 4 (D) 4x 4
3 3 2 z
48. Solution of = cos (x y) is ______.
37. Solve (D D 2)y = e sin x.
2 2x
x 2
(A) y = c1ex + c2ex + e2x (3cos x + sin x) (A) y2 + cos (x y) = f (y) + z
(B) y = c1ex + c2e2x + e2x (cos x + 3sin x) (B) z = cos (x y) + x f (y)
(C) y = c1ex + c2e2x + e2x (cos x 3sin x) (C) z = cos (x y)
(D) None of these (D) None of these
38. If y1 = e2x and y2 = xe2x are two solutions of a second order 49. The solution of x2p + y2q = (x + y) z is _______.
Linear differential equation, then the Wronskian W of y1 and
xy x - y
y2 is _______ f (xy, x y) = 0.
(A) (B) f , =0
z z
e4x (B)
(A) xe4x (C) 2e4x (D) 2xe4x
f (zx, z x) = 0
(C) (D) None of these
39. The complementary function of the differential equation
d2y dy 50. The solution of x (x2 + 3y2) p y (3x2 + y2) q = 2z (y2 x2) is
+ 5 + 6y = 5e3x is yc = c1 e2x + c2e3x using the
dx 2
dx ______.
method of variation of parameters, its particular is found to
be yp = A(x) e2x + B(x) e3x. Then, A(x) = xy
(A) f , ( x 2 + y 2 ) z = 0 (B) f (xyz, x2 + y2) = 0
1 5x z
(A) 5e5x (B) e5x (C) e5x
e (D)
5 f (xy, (x2 + y2)z) = 0
(C) (D) f (xz, (x2 + y2)z) = 0
Linked answer for questions 40 and 41:
51. Form a P.D.E of z = (x y) f (x2 y2)
Consider the equation (D2 + 5D + 6)y = e5x (A) a log (z a) = x ay + b
40. Solving the above equation by using variation of parameters (B) 2a log (z + a) = ay + b
A can be calculated using the formula ______ (C) 2a log (z a) = x + ay + b
UR VR (D) a log (z + a) = 3x + ay + b
(A) A= dx (B) A= dx
dv du dv du
u -v u -v 52. The solution of q2 x (1 + y2) = py2 is ______.
dx dx dx dx
UR VR z = a (1 + y2)
(A)
(C) A= dx (D) A= dx ax 2
dv du dv du ( z=
B) a (1 + y2) + b
u -v u -v 2
dx dx dx dx
ax 2
41. In solving the above equation by variation of parameters z=
(C) + a (1 + y 2 ) + b
method B = 2
(A) e7x (B) e3x/3 (C) e7x/7 (D) e3x ax
z=
(D) + a (1 + y 2 ) + b
42. The solution of the D.E. (D + 1) y = 0 given x = 0,
2 2
y = 2 and x = p/2, y = 2 is 53. Solve pqz = q2 (yp + q2) + p2 (xq + p2).
(A) y = sin x cos x (B) y = 2 (cos x sin x)
(C) y = 2 cosx sin x (D) y = 2(ex + ex) a3 b3
z = ax + by +
(A) + (B) z = ax by
d y
2
dy b a
43. Solve the equation 3 x 2 2 + x - y = x 2
dx dx a b
z = ax + by + 3 + 3
(C) (D) None of these
(A) y = C1x3 + C2 x1 + x3/7 (B) y = C1x3 + C2x + x2/7 a b
(C) y = C1x + C2x + x/7 (D)
1/3 1
y = C1x1/3 + C2x + x2/7 2u
54. In the process of solving the partial differential equation 2
x 2d 2 y dy x
44. The PI of the equation + 7 x + 9 y = x4 log x is 2u
dx 2
dx + 5 2 = 0 by the method of separation of variables, the lin-
dy
(A) x4(log x 2) (B) x4/49
ear differential equation involving the independent variable
x 4 log x 2 4 x 4 log x X is ____ (Here, k is a constant)
(C) - x (D)
49 343 49
d2X
2 z z z
2
(A) 2 + k X(x) = 0
45. The order and degree of the + 3 xy + 5 = 8 are dx
x 2
x y d2X
(A) 1, 1 (B) 1, 2 (C) 2, 1 (D) 2, 2 (B) 2 k X(x) = 0
dx
46. The differential equation whose solution is z = (x a) (y b) d2X dx
(C) 2 +k + k2X(x) = 0
is ______. dx dx
(A) pq = 2z (B) pq = z d2X dx
(D) 2 k + 2k X(x) = 0
(C) p = 2zq (D) p = zq dx dx

Chapter 03.indd 88 8/29/2015 9:11:36 AM


Chapter 3 Differential Equations | 2.89


55. The second order partial differential equation e -4 t - e -8t
63. The value of dt is ______.
u 2
u 2
u u2
u t
3x2 6xy + 3y2 5 + 7 = 6x2y is 0
x 2 xy y 2 x y (A) log 2 (B) log 4 (C) log 8 (D) log 6
__________

(A) Elliptic equation 64. t e -2 t
sin 3tdt = ______.
(B) Parabolic equation 0

(C) Hyperbolic equation 5 10


(A) (B)
(D) Depends on the value of x and y. 169 169
56. Laplace transform of 2sin2 2t = ______. 6 12
(C) (D)
1 1 s 169 169
(A) + 2 (B)
s s + 16 s 2 + 16 65. The Laplace transform of the function defined by
1 s 1 1 2, 0 < t < 1
(C) - 2 (D) - f(t) = is ______.
s s + 16 s s 2 + 16 1, t >1
57. The Laplace transform of (t + 1)3 is ______.
2 - e-s 2 - e-s
(A) (B)
6 - 6 s + 3s - s
2 3
6 + 6 s + 3s + s 2 3
s 2
(A) (B)
s3 s 2 + e-s 2 + e-s
6 (1 + s + s 2 + s3 ) 6 + 6 s + 3s 2 + s 3 (C)
2
(D)
2
(C) (D) 4
s4 s
1
58. Laplace transform of e cos 2t is ______.
2t 2 66. The inverse Laplace transform of 9 2 is ____.
s
s 2 + 4 s + 12 4 s 2 + 12 16 t 7
8 t5
(A) (B) (A) (B)
( s + 2 ) ( s 2 + 4 s + 20 ) ( s + 2 ) ( s 2 + 4 s + 20 ) 105 p 15 p
s 2 - 4 s + 12 s 2 - 4 s - 12 16 t 8 t7
(C) (D) (C) (D)
( s - 2 ) ( s + 4 s + 20 )
2
( s + 2 ) ( s 2 - 4 s + 12 ) 35 p 105 p
59. The value of L{sin h3t cos 3t} ______. 8 4 + 2s
67. The value of L1 - is ______.
s 2 + 18 s 2 + 18 3s - 2 16 s 2 - 25
(A) 4 (B)
s + 81 s 4 + 324 8 5t
(A) sinh - cosh
5t
3 ( s - 18 )2
3 ( s + 18 ) 2 3 4 4
(C) 4 (D) 8 2 3t 5t 5t
s + 324 s 4 - 324 (B) e - sinh - cosh
3 4 4
60. The value of L{t2cos 3t} is ______.
8 1 5t 1 5t
(C) e 2 3t - sinh - cosh
s 2 - 27 2 s ( s 2 - 27 ) 3 5 4 8 4
(A) (B)
( s2 + 9) ( s2 + 9)
4 3
(D) None of these
s3 - 27 s ( s3 - 27 ) 68. The inverse Laplace transform of
1
is ______.
(C) (D) s 2 - 8s + 20
( s2 + 9) ( s2 + 9)
4 3

et e 4t
cos 4t (A) sin 2t (B) sin 2t
61. Laplace transform of ______. 2 2
t
64 16 (C)
e4t sin 2t (D)
e4t sin 4t
(A) 2 (B)
s + 16 ( s + 16 )
2
2 e -3 s
69. The inverse Laplace transform of when expressed in
8 ( s - 4)
5
(C) (D) Does not exist
( s 2 + 16 )
2
terms of Heaviside unit step function is ______.
1 4 4 ( t - 3) 1
( t - 3) e 4 H ( t - 3)
4
62. If f(t) = t; 0 < t < 3 and f(t + 3) = f(t), then L{f(t)} is (A) t e H(t 3) (B)
6 24
1 1 1
(A) 2 1 + e 3 s + e -3 s (C) (t - 3) 4 e 4 (t -3) H (t - 3) (D) t 4 e 4 t H ( t - 3)
s (1 - e -3 s ) 24 24
1
(B) 1 - e -3 s + se -3 s 1
s (1 - e -3 s ) 70. The inverse Laplace transform of is
s3 ( s 2 + 4 )
1 1
(C) 1 - e -3 s - 3se -3 s (A) ( 2t 2 + cos 2t - 1) (B) 2t2 cos2t 1
s (1 - e -3 s )
2
16
1 1 1
(D) 1 - e -3 s - se -3 s (C) (1 - cos 2t - 4t 2 ) (D) ( 2 + cos 2t - 4t 2 )
s (1 - e -3 s ) 16 8

Chapter 03.indd 89 8/29/2015 9:12:02 AM


2.90|Engineering Mathematics

2 s3 - 13s 2 + 34 s - 15 -1 1
71. The inverse Laplace transform of is y=
(C) + ( cosh 2t - sin t )
16 32
( s 2 - 1) ( s - 3)
2
______.
-1 1
et 2et + 3e3t (B)
(A) et 2et 3e3t y=
(D) + ( cosh 2t + cos 2t )
16 32
et + 2et e3t (1 3t)
(C) (D) None of these
77. Solve (D2 5D + 6)y = 1 + e2t, y = 1, y = 0 when t = 0.
s - 4
72. The value of L-1 log is 1 -2t 11 -2t 59 3t
s + 3 y=
(A) e + e - e
1 4t 20 4 30
e 4 t - e -3t (B)
(A) ( e - e -3t ) 1 1 -2t 11 2t 28 3t
t y= - e + e - e
(B)
1 -3t
(C) ( e - e 4t ) (D)
t ( e -3t - e 4 t ) 6 20 4 15
t 1 1 2t 11 -2t 59 3t
y= - e + e + e
(C)
1 s - 4 6 20 4 30
73. The value of L-1 log is
s s + 3 1 1 2t 11 -2t 59 3t
y= - e - e + e
(D)
t
e 4 x - e -3 x t 6 20 4 30
0 x dx (B)
(A) 0 ( e 4 x - e -3 x ) 78. Solution of the one dimensional heat equation
u 2u
= x > 0,
t x 2
t
e -3 x - e 4 x
t
t > 0 satisfying the boundary condition u (0, t) = 1, u(x, 0) =
0 x ( e -3 x - e 4 x ) (D)
(C) 0 x dx 0 is ______.
t
x x
74. Using convolution theorem, the value of sin x cos ( t - x ) dx (A) (B)
erf
2 t
erf
0 2 t
is _______.
1 x
(C) erf (D)
erf
t t
(A) cos t (B) sin t 2 t t
2 2
t t 79. A string is stretched between two fixed points follows the
(C) t sin (D) t cos 2 y 2 y
2 2 equation 2 = a 2 2 (t > 0, x > 0) satisfying the boundary
t x
75. The value of 2 * 2 * 2 *.* 2 upto n times is conditions y(x, 0) = 0, x > 0 and y(0, t) = t Lt y(x, t) = 0, t 0,
x
2n -1 t n -1 2n t n -1 Find y (x, t) in terms of Heavisides unit step function.
(A) (B)
n -1 n -1
(A) (t x) H (t x) (B) x x
2n t n -1 2n t n t - a H t - a
(C) (D)
( n - 1)! n! (C) (t xa) H (t xa) (D) None of these
76. Solve (D4 16)y = 1, y = y = y = y = 0. 80. The one dimensional wave equation is ______.
-1 u u 2u 2u
y=
(A) - [cosh 2t + sinh 2t ] (A) = c (B) = c2
16 t x t 2
x 2
1
y=
(B) (1 - cosh 2t + cos 2t ) 2u 2u 2u
(C) 2 = c2 2 (D) = c2
u
32
x t t 2 x

Practice Problems 2 4. Form the differential equation representing the two-parame-


ter family of curves y = Ae2x + Bex. Where A and B are the
Directions for Questions 1 to 85: Select the correct alternative
parameters.
from the given choices.
4 (A) y2 + y1 + 2y = 0 (B) y2 y1 2y = 0
d 3 y dy (C) y2 y1 + 2y = 0 (D) y2 + y1 2y = 0
1. The order of the D.E. + = 2x is
dx dx
3
5. The D.E. of all circles which touch x - axis at origin is
(A) 2 (B) 3 (C) 4 (D) 1
dy dy
2 (A) (x y) = 2xy (B) (x2 y2) = 2xy
d 2 y dy dx dx
2. The degree of the D.E. + = 0 is
dx 2 dx (C) (y x)
dy
= 2xy
d2y
(D) (x2 y2) 2 = 2xy
(A) 1 (B) 2 (C) 3 (D) 4 dx dx

3. Find the order and degree of the DE 6. The D.E. of the family of parabolas having vertex at origin
23
and foci on y-axis is
dy 2 d2y . dy dy
1 + = (A) = y (B) = 2y
dx dx 2 dx dx
dy d2y
(A) 2, 3 (B) 3, 2 (C) 2, 1 (D) 1, 2 (C) x = 2y (D) = y2
dx dx 2

Chapter 03.indd 90 8/29/2015 9:12:24 AM


Chapter 3 Differential Equations | 2.91

7. Obtain the differential equation of all conics whose axes (A) 1/M x + N y (B) Mx+Ny
coincide with coordinate axes. (C) M x N y (D) 1/M x N y
(A) xyy2 xy12 = 0 (B) xy y2 + xy12 = yy1 16. Solve ex (x2 + y2 + 2x) dx + 2yex dy = 0.
(C) xyy2 + xy1 + y = 0
2
(D) None of these (A) ex (x2 + y2) = c (B) ex (x2 y2) = c
8. The D.E. whose general solution as y = ae2x + be3x is (C) e (x y ) = c (D)
x 2 2
ex (x2 + y2) = c
d 2 y dy d 2 y dy 17. The integrating factor of the differential equation xy dx + (y2
(A) 2 + - 5 y = 0 (B) - + 5y = 0
dx dx dx 2 dx + x2 + y)dy = 0 is
(A) 1/y (B) ey (C) y2 (D) y
d2y dy d2y dy
(C) 2 - 5 + 6 y = 0 (D) 2 - 6 + 5 y = 0
dx dx dx dx 18. The integrating factor of the equation (x3 + y3 2x) dx + 3y2
dy = 0 is
9. The general solution of the D.E. (A) ex (B) x (C) x2 (D) ex
2

3ex tan y dx + (1 ex) sec2 ydy = 0 is dy


19. The general solution of (x + 2y3) = y is
(A) (ex 1)3 = c tan y (B)
ex 1 = tan y + c dx
(C) (e + 1) = c sec y (D)
x 3
e3x c tan y = 0 x y
(A) = y2 + c (B) = y2 + c
dy y x
10. The solution of = e x + y ( e x + e - y ) is
dx x y
(C) = x2 + c (D) = x2 + c
(A) ee e y = e x + c
x
y x
(B) e y e e = e x ( e x -1) + c dy
x

20. Solve 1 + (x tan y sec y) = 0.


(C) -e - y e e = e e ( e x - 1) + c dx
x x

(A) x tan y = sec y + c (B) sec y tan y = c


(D) None of these
(C) sec y + tan y = c (D) x sec y = tan y + c
11. The solution of the equation x dy + y dx = x dx is
1
x dy 1- log x
(A) x2 y = + c (B) 2x = x2y + c + y log x = e x x 2 is
21. The general solution of x
2 dx
xylog x = c (B)
(A) yxlog x = c
x2 x3
(C) xy = + c (D) xy = +c 1
log x
2 3 (C) yx 2 = ex + c (D)
xy = ex + c
dy x+ y-2 dy
12. The general solution of = is 22. The general solution of = y tan x 2 sin x is
dx 2x - y - 1 dx
(A) y = cos 2x + c (B) y cos x = sin 2x + c
(A) 2 3 tan1 2 y - x = log(x2 xy + y2) + c. cos x cos 2 x
3x (C) y= + c (D) y cos x = +c
2 2
3x dx
(B) 3 tan1 = log(x2 xy + y2) + c. = xy + x3y3 is
2 y - x 23. The general solution of
dy

2y - x +1 x2 = (y2 1) + c e - y (B)
(A) y2 = (x2 1) + c e y
2 2

(C) 2 3 tan 1
= log(x2 xy + y2) + c.
3 ( x + 1) -1 1
(C) = (y2 1) + c e - y (D) = (x2 1) + c e - x
2 2

x2 y2
2y - x -1
(D) 2 3 tan1 = log(x2 xy + y2 x y + 1) + c. dy tan y
3 ( x - 1) 24. Solve = (1 + x) ex sec y.
dx 1+ x
dy x + 2y +1 (A) sin y = (1 + x) (ex + c) (B) cos y = (1 x) (ex + c)
13. Solve= . (C) sin y = (1 x) (ex + c) (D) cos y = (1 + x) (ex + c)
dx 2 x + 4 y + 3
(A) 8y + 4x + log (4x + 8y + 5) = c 25. The system of confocal and coaxal parabolas represented by
(B) 8y 4x + log (4x + 8y + 5) = c y2 = 4a (x + a) are
(C) 4x 8y + log (4x 8y 5) = c (A) orthogonal. (B) not orthogonal.
(D) x + 2y + log (4x + 8y + 5) = c (C) self-orthogonal. (D) None of these
26. Find the orthogonal trajectories of the family of curves satis-
14. The general solution of dy y2 - x2
(a2 2xy y2) dx (x + y)2 dy = 0 is fying = .
dx 2 xy
y3 y3 (A) x2 y2 = c1y (B) x2 y2 + c1y = 0
(A) a2x x2y xy2 = c (B) x2 + y2 =c
3 3 (C) 3x2 + 2y2 = c1y (D) x2 + y2 = c1y
(C) a2x + x2y + xy2 = c (D) x + x2y + xy2 + y3 = c
27. The equation of orthogonal trajectories of the family of the
15. If the differential equation M dx + N dy = 0 is of the form y curves rn sin nq = an (a being the parameter) is
f(xy) dx + x g(xy)dy = 0, then the integrating f actor of M dx + (A) rn cos nq = c (B) r cos q = c
N dy = 0 is (C) rn sin q = c (D) rn sin n q = c

Chapter 03.indd 91 8/29/2015 9:12:44 AM


2.92|Engineering Mathematics

28. The solution of the differential equation 36. The general solution of the differential equation
y = 2 x p p2 is d4 y d2y
+ 8 2 + 16y = 0 is
dt 4
dt
2 p3 2 p2
x p2 =
(A) + c (B) xp= +c y = et[c1 + c2t] cos 2t + et[c1 + c2t] sin 2t
(A)
3 3
y = (c1 + c2t) sin 2t + (c3 + c4t) cos 2t
(B)
x p 4p3 + c = 0
(C) (D) None of these
y = (c1 + c2t) e2t + (c3 + c4t) e2t
(C)
2 y = e2t (c1 + c2t) cos t + e2t (c1 + c2t) sin 2t
(D)
dy dy
29. Solve: + 7 x + 10 x 2 = 0
dx dx 37. Find the general solution of
5x 2 (D2 4D + 4) y = e2x.
(A) (y + c) (y + x2 c) = 0 x 2x
2 y = (c1 + c2x) ex +
(A) e
2
(B) (y + 5x2 + c) (y x2 + c) = 0
x 2 2x
(C) (y + x2 c) (y 2x2 + c) = 0 (B) y = (c1 + c2x) e2x + e
(D) (y 5x2 c) (y x2 c) = 0 2
x 2 3x
30. The solution of the DE p2 + 5px + 4x2 = 0 is (C) y = (c1 + c2x) e2x e
3
(D) None of these
y2 - x2
(A)
- c = 0
2 38. Find the general solution of
x2 (D3 4D2 3D + 18) y = excos h2x.
(B) y+ + c ( y + 2x2 + c) = 0
2 1 2 3x 1 x
(A) y = (c1+ c2x)e3x + c3e2x + xe + e
y - x2 20 32
(C) ( y - 2 x 2 - c )
- c = 0 1 3 2x 1 x
2 (B) y = (c1+ c2x)e3x + c3e2x + xe + e
(D) None of these 20 23
(C) y = (c1+ c2x)e3x + c3e2x + x2e3x
31. The solution of x = p2 + 3p is (D) None of these
3 p2 2 p 39. Find the general solution of
y=
(A) + y = 6p3 + 7p2 + c
+ c (B)
2 3 (D4 + 3D2 4) y = sin 2x.
3 p2 2 p3 3 p 2
y=
(C) + c (D) y= + +c y = c1ex + c2ex + c3 cos 2x + c4 sin 2x
(A)
5 3 2 1
y = c1ex + c2ex + c3cos 2x + c4 sin 2x +
(B) x cos 2x
32. Solve (D2 + 2D 15) y = 0. 20
(C) y = c1e + e + c3 cos x + c4 sin x + x cos 2x
2x 3x
(A) y = c1e3x + c2e5x (B) y = c1e5x + c2e3x
(D) None of these
(C) y = c1e3x + c2e5x (D) y = c1e3x + c2e5x
40. Find the general solution of
33. Find the general solution of (D2 3D + 2)y = x2 + x.
d y 2
dy x2
4 + 7y = 0. (A) y = c1 ex + c2e2x +
dx 2
dx 2
y = c1e2x (c1cos 3x + c2 sin 3x)
(A) x2 5
(B) y = c1 ex + c2e2x + + 2x +
y = c1ex (c1cos 3x + c2 sin 3x)
(B) 2 2
y = e2x (c1cos 3 x + c2 sin 3 x)
(C) (C) y = c1 e2x + c2ex + x2 + 4x + 5
y = c1ex (c1cos 3 x + c2 sin 3 x)
(D) (D) None of these

d4 y d3 y dy d2y dy
34. The general solution of -2 - 2 y = 0 is 41. The solution of + 3 = e 2 x x is
dx 4
dx 3 dx dx 2 dx
y = (c1 cos 2x + c2 sin 2x ) + ex (c3 cos h 2 x + c4 sin h 2 x)
(A)  (A) y = c1 + c2 e3x + e2x
(B) y = (c1 + c2x) cos x + ex (c3 + c4x) sin x (B) y = c1 + c2 e3x + e2x/10
y = (c1 cos 2x + c2 sin 2x) + ex (c3 cos 2 x + c4 sin 2 x)
(C)  (C) y = c1e3x + c2x + e2x/5
(D) None of these
y = (c1cos 2 x + c2 sin 2 x) + ex (c3 cos x + c4 sin x)
(D)
42. Solve (x2D2 +xD +1)y = x3ex.
d4 y d3 y d2y dy
35. The general solution of +3 3 +3 2 + = 0 is y = c1cos(log x)
(A)
___________ dx 4
dx dx dx y = c1 cos (log x) + c2 sin(log x) + x2 + 6 cos x 3
(B)
(A) y = c1 ex + c2 e3x + c3 e3x + c4 x3
(B) y = (c1 + c2x) e3x + (c3 + c4x) e3x y=
(C) ( 9 log x - 6 )
81
(C) y = (c1 + c2x + c3x2) ex + c4 x3
y = c1 cos(log x) + c2 sin(log x) +
(D) (10 log x 6)
(D) y = (c1 + 3c2x) ex + (c3 xc4)ex2 100

Chapter 03.indd 92 8/29/2015 9:13:02 AM


Chapter 3 Differential Equations | 2.93

43. Solve the equation (D2 + 4)y = sec 2x by variation of x x


parameters. f , 2x2 + y2 = 0
(A)
y
y
(A) c1cos 2x + c2 sin 2x + cos 2x + x sin 2x
(B) c1cos 2x + c2 sin 2x + x sin 2x xx x
f , 3x2 + y2 = 0
(B)
(C) c1cos 2x + c2 sin 2x + 1/4cos 2x 1/2x sin 2x y
y y
(D) None of these xx x
f , x2 + y2 z2 = 0
(C)
44. The Wronskian of the two solutions y1 = cos 3x and y
y y
y2 = sin 3x of a second order linear differential equation is x x 2 2 2 x
_______ f ,x +y +z =0
(D)
(a) cos2 3x (B) sin2 3x y y y
(C) 3 (D) 3 54. The solution of the equation px (x + y) = qy (x + y) (x y)
(2x + 2y + z) is _____.
45. The complementary function of the differential equation
d2y dy f (xy, x + y + z) = 0
(A)
x2 2 4x + 4y = 6x3 is yc = c1x + c2x4. if its p articular f (x + y + z, x + y) = 0
(B)
dx dx
integral, when evaluated by the method of variation of f (xy, x + y) = 0
(C)
parameters of yp = A(x) x + B(x) . x4, then the value of B(x) is f (xy, (x + y) (x + y + z)) = 0
(D)
___________ 55. The solution of pq = 3 is ______.
2 -2 (A) z = ax + yc (B) z = ax + y
2x (C)
(A) 2x (B) (D)
x x 3
46. Cauchys linear differential equation can be transformed into (C) z = ax ay (D) z = ax + y + c
a
linear differential equation by substituting x = 56. The solution of qz2 + p = 1 is ______.
(A) log z (B) z2 (C) e z (D) 2 sec z
az 3
(A) + z = x + ay + c (B)
az2 x + ay = c
d y
2
dy 3
47. Solve: x 2 + 3 x - 3 y = 0.
dx 2 dx z3 = x + ay (D)
(C) az3 + z = x + ay
y = c1x3 + c2x2
(A) (B) y = c1x3 + c2x2 57. The solution of zpq = p + q is ______.
y = c1x3 + c2x (D)
(C) y = c1x3 + c2x (A) z = 3x + by + 7
d2y dy z2 a + 1
48. The solution of the equation ( x - 1) + 3 ( x - 1) + y = 2 (B) = x + (a + 1) y + b
2 a
2

dx 2
dx
sin(log(x 1)) is (C) z2 = ax + by
(D) z2 = x + y + b
c1 + c2 log ( x - 1)
y=
(A) - cos log ( x - 1) 58. The solution of (p q) (z xp yq) = 1 is ______.
x -1
(B) y = c1 + c2 log(x 1) + cos(log(x 1)) 1
z = ax by +
(A) (B) z = ax by
(C) (x 1)y = c1 log(x 1) + cos(log(x 1)) a+b
1
(D) None of these z = ax + by
(C) (D) z = ax + by +
a-b
49. The partial differential equation of 59. If u(x, y) = X(x).Y(y) be the solution of the partial differential
z = f (x2 + y2) is _____. u u
equation 4 + 5 = 0, which is obtained by solving it by
py + qx = 0
(A) (B) px + qy = 0 x y
py qx = 0
(C) (D) px qy = 0 the method of separation of variables, then X(x) (the function
50. The partial differential equation of of x only in u(x, y)) is ______
z = f (x + at) g (x at) is _____. [Note: Here, c and k are arbitrary constants]
z z
2
z z
2 (A) X(x) = ce(4k/x)
X(x) = ce(kx) x2 (B)
(A) 2 = a 2 2 (B) =a X(x) = ce(k/4)x (D)
(C) X(x) = ce(5k)x2
t x t x
2 z 2 z 2 z 2 z 60. Which of the following second order partial differential equa-
(C) 2 = 2 (D) + =0
t x t 2 x 2 tions is an elliptic equation?
51. Eliminate the function from z = f (x y). 2u 2u 2u u u
(A) 3 +4 5 2 +7 = 7x2
(A) p q = 0 (B) p + q = 0 x 2
xy y x y
(C) p + 2q = 0 (D) 2p + q = 0
2u 2u 2u u u
(B) 3 4 + 5 + = 6x2y
52. Eliminate the arbitrary function from the equation z = x y f x 2 xy y 2 x y
(x2 + y2).
2u 2u 2u u u
(A) xp yq = x2 + y2 (B) py + xq = x2 + y2 3
(C) + 4 + 5 + 4x 7y =0
(C) xp + yq = x2 + y2 (D) qx py = x2 y2 x 2 xy y 2 x y
2u 2u 2u u u
53. The solution of z (xp + yq) = x2 + y2 is ______. (D) 2 + 2 + 2 +5 = 6xy2
x xy y 2 x y

Chapter 03.indd 93 8/29/2015 9:13:23 AM


2.94|Engineering Mathematics

61. The Laplace transform of sin 3t 3t cos 3t is ______. 1 s 3


70. The inverse Laplace transform of + - is
54 27 s
5
2 s2 - 9 s + 4
(A) (B)
( s2 + 9) s2 + 9
2
t3 2 4t 3 2
(A) + cosh 3t - e -4 t (B) + cosh 3t - 3e -4 t
54 27 p 3
(C) (D)
s2 + 9 ( s2 + 9)
2
4t 32
(C) + cosh 3t - 3e -4 t (D) cosh 3t e4t
62. Laplace transform of cos h23t is ______. 3 p
1
s 2 - 18 s 2 - 18 71. The value of L-1 is
(A) (B) 4
s - 36
2
s ( s 2 - 36 ) ( s - 4 )
s 2 + 18 e -4 t t 3 e 4t t 3
s 2 + 18 (A) (B)
(C) 2 (D) 4! 3!
s ( s + 36 ) s 2 + 36
e t t -3 e 4 t t -3
63. If n is a positive integer, then L{tn} = ______. (C) (D)
3! 3!
n n! n! n 3s 2 + 11s - 22
(A) (B) (C) (D)
s n+1
s n
s n +1
s n-1 72. The value of L-1 2
is
( s + 2 )( s - 2 )
64. L {t ne -3t } = 5e 2t + 2e -2t + 3e 2t (B)
(A) 5e 2t - 2e -2t + 3te 2t
n n! (C) 5e 2t + 2e 2t - 3te -2t (D)
5e -2t + 2e 2t - 3te -2t
(A) (B)
( ) ( 3)
n +1 n +1
s - 3 s + 5s + 3
73. The value of L-1 2 ______.
n! n! s - 6 s - 27
(C) (D)
( s - 3) ( s + 3)
n +1 n
(A) e3t (5cosh 6t + 3sinh 6t)
(B) 5cosh 6t + 3sinh 6t
e 5t t n -1 (C) e3t (5cosh 6t 3sinh 6t)
65. L =
( n - 1)! (D) e3t (3cosh 6t + 5sinh 6t)
4s - 3
1 1 74. The inverse Laplace transform of is ____.
(A) (B) 9 s 2 + 30 s + 25
( s + 5) ( s - 5)
n -1 n -1

(A) e 3 4 + t (B)
1 5t 29 1 -t 29
1 s e 4 - t
(C) (D) 9 3 9 3
( s - 5) ( s - 5)
n n
1 - 53 t 29
(C) e 4 - t (D) None of these
9 3
66. The Laplace transform of (t3e3t) is ______. 5s + 3
75. The inverse Laplace transform of the function 2 is
(A)
s
(B)
s2 s - 4 s + 13
( s - 3) ( s - 3)
4 4
e -2t
(A) ( cos 3t + 13 sin 3t )
6 6s 3
(C) (D) 1
( s - 3)
4
( s - 3)
4
(B) (15 cos 3t + 13 sin 3t )
3
e 2t
67. If f(t) = (
t - 2 ) , (C) (15 cos 3t - 13 sin 3t )
2
t>2
; then L { f(t) } = 3
0, 0<t <2
e 2t
(D) (15 cos 3t + 13 sin 3t )
2 e -5 e -2 s 2e -2 s 3
(A) 3 e -2 s (B) 3 (C) (D) 2
s 2s 2s s s 2 + 2s + 3
76. The inverse Laplace transform of 2 is
68. If L { f(t)} = F(s) and g(t) =
( s + 2s + 5) ( s 2 + 2s + 2 )
1 e -t
f (t - a ) , t > a (A) ( sin 2t + sin t ) (B) ( sin 2t + sin t )
, then L { g ( t )} = 3 3
0 t<a e t
-e -t
(C) ( sin 2t - sin t ) (D) ( sin 2t - sin t )
es F(s) (B)
(A) ea F(s) 3 3
(C) eas F(s)
eas F(sa) (D) s
77. The inverse Laplace transform of 4 is
69. The Laplace transform of Heavisides unit function s + 7 s 2 + 16
H (t a) is ______. 2 15 ht
e - as (A) sin t sin
(A) eas (B) 15 2 2
s
e as 1 15 ht
(C) eas (D) (B) sin t sin
s
15 2 2

Chapter 03.indd 94 8/29/2015 9:13:51 AM


Chapter 3 Differential Equations | 2.95

15 u 2u
(C) ht 83. Solve = 2 2 satisfying the conditions u(0, t) = 0 u(1, t) = 0,
sin t sin t x
2 2 u(x, 0) = 10sin 4px
2 15 ht u = 20sin 8p xe32t
(A)
(D) sin t cos
15 2 2 (B) u = 10 sin 4pxe -32p t
2


1
78. For a > 0, If L1 {F(s)} = f(t), then L1{F(as + b)} is (C) u= sin 4px + e -32p t
10
- bt
t 1 abt t 10
(A)
e a f (B) e f u = sin 4pxe -32p t
(D)
2

a a a 32
- bt
1 - bt t t 2u 2u
(C) e a f (D)
ae a f 84. Solve the wave equations = 4 2 - 3 for x > 0, t > 0
a a a t 2
x
satisfying the boundary conditions u(x, 0) = 0 = ut(x, 0), for x 0
79. Solve (D2 2D 8)y = 0, y(0) = 3 and y(0) = 6. and u(0, t) = 0, Lt tux ( x, t ) = 0, t 0
x
(A) y = e 2t - e -4 t (B)
y = e -2t - 2e -4 t
3x
(C) y = e -2t + 2e -4 t (D)
y = e -2t + 2e 4 t ( 4tx - x 2 ) , x 2t
u ( x, t ) = 8
(A)
0, x 2t
80. Solve ty + y + 9ty = 0, if y(0) = 0 and y(0) = 3.
1
(A) y = J 0 ( 3t ) (B) y = J 0 ( 3t ) -3
8 ( 4tx - x ) , x 2t
2
3
(B) u ( x, t ) =
(C) y = 3 J 0 ( 3t ) (D)
y = 9 J 0 ( 3t ) -3t 2
x 2t
2
81. Solve (D3 D)y = t, when y(0) = y(0) = y(0) = 0.
x
1 8 (12tx - x ) , x 2t
2
(A) y = t + e - t - e - t (B) y = {t + e t + e - t }
2 (C) u ( x, t ) =
3t
et - e -t x 2t
y = 2 ( t - e t + e - t ) (D)
(C) y= -t 2
2
x2
82. Solve (D2 D)y = t3 2t2, where y(0) = 2 and y(0) = 4 ( 4t - x ) , x 2t
(D) u ( x, t ) = 8
1 3t 2 x 2t
(A) y= ( 48 - 24t + 12t 2 - 4t 3 + 24et )
12
u 2u
1 85. Solve the heat equation = satisfying the boundary
y = ( 48 + 24t - 12t 3 - 4t 3 - 3t 4 - 24e t )
(B) t x 2
12
conditions u(x, 0) = sinpx, u ( 0, t ) = u (1, t ) = 0 .
1
(C) y= ( 48 - 24t - 12t 2 + 4t 3 - 3t 4 - 24et ) u = e4pt sinpx (B)
(A) u = e4pt sinpx
6
(C) u = e -4p t sin p x (D)
u = e -p t sin p x
2 2

1
y = ( 48 + 24t - 12t 2 - 4t 3 - 3t 4 - 24e t )
(D)
6

previous years questions


d y
2
1. If y = f(x) is the solution of = 0 with the boundary 3. Consider two solutions x(t) = x1(t) = x(t) = x2(t) of the differ-
dx d 2 x (t )
dy ential equation + x(t) = 0, t > 0, such that x1(0) = 1,
conditions y = 5 at x = 0, and = 2 at x = 10, f(15) = dt 2
dx dx1 ( t ) dx ( t )
= 0, x2(0) = 0, 2 = 1. The Wronskian W(t) =
______ [2014] dt t = 0 dt t = 0

dy x1 ( t ) x2 ( t )
2. The general solution of the differential equation = cos dx1 ( t ) dx2 ( t ) at t = p/2 is [2014]
dx
(x + y), with c as a constant, is [2014] dt dt
(A) 1 (B) 1 (C) 0 (D) p/2
x + y
y + sin (x + y ) = x + c
(A) (B) tan = y+c
dy
2 4. The solution of the initial value problem = 2xy; y (0) =
2 is dx [2014]
x + y x + y
(C) cos (D) tan (A) 1 + e - x (B) 2 e - x
2 2

= x+c = x+c (C) 1 + e - x


2
(D) 2 e - x
2

2 2

Chapter 03.indd 95 8/29/2015 9:14:16 AM


2.96|Engineering Mathematics

s d3 f f d2 f
5. Laplace transform of cos (wt) is . The Laplace 12. The Blasius equation, + = 0 is a
s + w2 2
dh3 2 dh 2
2t
transform of e cos (4t) is[2014] [2010]
s-2 s+2 (A) Second order nonlinear ordinary differential equation
(A) (B) (B) Third order nonlinear ordinary differential equation
( s - 2 ) + 16 ( s - 2 ) + 16
2 2
(C) Third order linear ordinary differential equation
s-2 s+2 (D) Mixed order nonlinear ordinary differential equation
(C) (D)
( s + 2 ) + 16 ( s + 2 ) + 16
2 2

1
13. The Laplace Transform of a function f(t) = . The
u u u 2 s 2 ( s + 1)
6. The partial differential equation +u = is a function f(t) is [2010]
t x x 2
 [2013] (A) t 1 + et (B) t + 1 + et
(A) linear equation of order 2 (C) 1 + et (D) 2t + et
(B) non-linear equation of order 1 1
14. The inverse Laplace transform of f(t) = 2 is [2009]
(C) linear equation of order 1 ( s + s)
(D) non-linear equation of order 2
(A) 1 + et (B) 1 et
d 2u du (C) 1 et (D) 1 + et
7. The solution to the differential equation -k =0
dx 2 dx
where k is a constant, subjected to the boundary condition dy 6
15. The solution of x + y = x4 with the condition y(1) = is
u(0) = 0 and u(L) = U, is [2013] dx 5
 [2009]
x 1 - e kx
u = U (B)
(A) u =U x4 1 4x4 4
L 1 - e kL y=
(A) y=
+ (B) +
5 x 5 5x
1 - e - kx 1 + e kx
(C) u =U (D) u = U x4 x5
1 - e - kL 1 + e kL y=
(C) y=
+ 1 (D) +1
5 5
8. The function f(t) satisfies the differential equation 16. Given that x + 3x = 0, and x (0) = 1, x (0) = 0, what is x (1)?
d2 f
+ f = 0 and the auxiliary conditions, f(0) = 0,  [2008]
dt 2 (A) 0.99 (B) 0.16 (C) 0.16 (D) 0.99
df
( 0 ) = 4 The Laplace transform of f(t) is given by 17. It is given that y + 2y + y = 0, y(0) = 0, y(1) = 0. What is
dt
 [2013] y(0.5)? [2008]
2 4 (A) 0 (B) 0.37 (C) 0.62 (D) 1.13
(A) (B)
s +1 s +1
18. The partial differential equation
4 2
(C) 2 (D)
s +1 s2 + 1 2f 2f f f
+ + + = 0 has [2007]
x 2 y 2 x y
9. The inverse Laplace transform of the function F(s)
1 (A) degree 1 order 2 (B) degree 1 order 1
= is given by[2012]
s ( s + 1) (C) degree 2 order 1 (D) degree 2 order 2
f(t) = sint (B)
(A) f(t) = et sint dy
f(t) = et (D)
(C) f(t) = 1 = et 19. The solution of= y 2 with initial value y(0) = 1 is bounded
dx
d2y dy in the interval [2007]
10. Consider the differential equation x 2 + x - 4y = 0
dx 2
dx (A) x (B) x 1
with the boundary conditions of y(0) = 0 and y(1) = 1. The (C) x < 1, x > 1 (D) 2 x 2
complete solution of the differential equation is [2012] 20. If F(s) is the Laplace transform of function f(t), then Laplace
px
(B) sin
t
(A) x2
2
transform of f ( r ) dt is[2007]
0
px px
(C) ex sin (D) ex sin 1
(A) F ( s ) (B)
1
F ( s) - f (0)
2
2 s s
dy
= (1 + y2)x. The gen-
11. Consider the differential equation (C) F ( s ) ds
sF(s) f(0) (D)
dx
eral solution with constant c is [2011] 21. The solution of the differential equation
dy
x 2
+ 2 xy = e - x with y(0) = 1 is: 
y = tan2 + c
x [2006]
2

(A) y = tan + tan c (B) dx


2 2
(A) (1 + X) e + x (B) (1 + X) e - x
2 2

2 x
y = tan + c
x2
(C) y = tan + c (D)
(C) (1 X) e + x (D) (1 X) e - x
2 2

2 2

Chapter 03.indd 96 8/29/2015 9:14:36 AM


Chapter 3 Differential Equations | 2.97

(A) p = 3, q = 3 (B) p = 3, q = 4
22. For d y + 4 dy + 3y = 3e 2 x, the particular integral is:
2

dx 2 dx (C) p = 4, q = 3 (D) p = 4, q = 4
 [2006]
1 1 2x (ii) 
Which of the following is a solution of the differential
(A) e 2 x (B) e equation
15 5
(C) 3e2x (D) C1ex + C2e3x d2y dy
+ p + ( q + 1) y = 0?
dx dx
2
dy 2InX
23. If x2 + 2 xy = , and y(1) = 0, then what is y(e)? e3x (B)
(A) xex
 dx x [2005] xe2x (D)
(C) x2e2x
1 1
(A)
e (B) 1 (C) (D) 25. A delayed unit step function is defined as u(t a)
e e2
0, for t < a
Linked answer for question 24(i) and 24(ii): = . Its Laplace transform is [2004]
1, for t a
The Complete solution for the ordinary differential equation as
e
d2y dy a.e3s (B)
(A)
+ p + qy = 0 is y = c1ex + c2e3x s
dx 2 dx
e as e as
(C) (D)
24. (i) Then, p and q are [2005] s a

Answer Keys
Exercises
Practice Problems I
1.D 2.C 3.A 4.B 5.B 6.C 7.A 8.B 9.D 10.C
11.B 12.C 13.D 14.B 15.C 16.B 17.B 18.D 19.B 20.D
21.A 22.B 23.A 24.C 25.B 26.C 27.C 28.B 29.C 30.A
31.D 32.A 33.C 34.D 35.B 36.D 37.D 38.A 39.B 40.B
41.C 42.B 43.D 44.C 45.C 46.B 47.B 48.D 49.B 50.A
51.C 52.C 53.A 54.B 55.B 56.C 57.D 58.A 59.C 60.B
61.D 62.C 63.A 64.D 65.A 66.A 67.C 68.B 69.C 70.A
71.C 72.C 73.D 74.B 75.C 76.D 77.B 78.A 79.B 80.B

Practice Problems 2
1.B 2.A 3.A 4.B 5.B 6.C 7.B 8.C 9.A 10.C
11.C 12.D 13.B 14.A 15.D 16.A 17.D 18.A 19.A 20.D
21.C 22.D 23.C 24.A 25.C 26.D 27.A 28.A 29.A 30.B
31.D 32.B 33.C 34.A 35.C 36.B 37.B 38.A 39.B 40.B
41.B 42.D 43.D 44.C 45.D 46.C 47.C 48.A 49.C 50.A
51.B 52.D 53.C 54.D 55.D 56.A 57.B 58.D 59.C 60.B
61.A 62.B 63.C 64.B 65.C 66.C 67.A 68.D 69.B 70.C
71.B 72.B 73.A 74.C 75.D 76.B 77.A 78.C 79.D 80.C
81.D 82.B 83.B 84.B 85.D

Previous Years Questions


1. 34 to 36 2.D 3.A 4.B 5.D 6.D 7.B 8.C 9.D
10.A 11.D 12.B 13.A 14.C 15.A 16.B 17.A 18.A
19.C 20.A 21.B 22.B 23.D 24.(i)C (ii)C 25.B

Chapter 03.indd 97 8/29/2015 9:14:43 AM


Chapter 4
Complex Variables
LEArNING OBJECTIVES

After reading this chapter, you will be able to understand:


Complex Number Complex Integration
Modulus and Argument of a Complex Number Cauchys Integral Theorem
Functions of a Complex Variable Moreras Theorem [Converse of Cauchys Theorem]
Continuity Taylors Series Expansion of f (z)
Differentiability Laurents Series Expansion of f (z)
Analytic Function Singular Points
Entire Function Cauchys Residue Theorem
Elementary Functions

CoMPlex NuMber Note: A complex valued function does not have to be defined on
the whole of C; it may be defined over a non-empty subset D of C.
A number of the form x + iy, where x, y R, the set of real num-
D is called the domain of definition of the function f .
bers, and i = 1 is called a complex number.

Note: Solved Examples


1. If z = x + iy, then x is termed as the real part of z denoted
by R(z) and y is termed as the imaginary part of z, denoted Example 1: Consider F(z) = z2 + iz, when expressed in terms of
by I(z). real and imaginary parts.
2. z = x + iy can also be written as an ordered pair notation (x, y). Solution: f(z) = (x + iy)2 + i(x + iy)
= x2 y2 + 2ixy + ix y
= x2 y2 y + i(x + 2xy).
Modulus and Argument of a Complex Number
Here, u(x, y) = x2 y2 y and v(x, y)
The modulus of a complex number z = x + iy is the real number
= x + 2xy.
x 2 + y 2 and is denoted by |z|.
The argument or amplitude of a complex number z = x + iy, Limits
when (x, y) (0, 0) is the value q Let w = f(z) be any function of z defined in a bounded closed
domain D. Then the limit of f(z) as z approaches zo is denoted by l
y and is written as lim f(z) = l i.e., for every > 0, there exists d >
= tan1 and is denoted by arg(z).
x z z 0

0 such that |f(z) l| < whenever |z zo|< d, where and d are


Note: The arg(z) satisfying the inequality p < q p is called the arbitrary small positive real numbers.
principal argument, denoted by Arg(z).
Note:
1. Here, z should approach zo through all possible curves.
Functions of a complex variable 2. l is the simultaneous limit of f(z) as z z0.
Complex functions are functions which assign complex numbers z 3 103 23 103
Example 2: Find zlim =
for complex numbers. Let C be the set of complex numbers. 10 z 10 2 10
A function f : C C is a rule which associates with z C, a z 3 103
unique w C, written as w = f(z). Solution: zlim = lim (z2 + 10z + 100)
10 z 10 z 10
As z = x + iy, w = u + iv, where u = u(x, y) and v = v(x, y),
= 100 + 100 + 100 = 300
u and v are functions of x and y.
(using standard limits as in real variable calculus).

Chapter 04.indd 98 8/28/2015 7:52:47 PM


Chapter 4 Complex Variables | 2.99

Example 3: Show that


x 2y Analytic function
lim
( x , y ) ( 0 , 0 ) x 4 + y 2
does not exist.
A function f(z) is said to be analytic at a point z0, if f is dif-
Solution: Path I ferentiable not only at z0, but also at every point in some
lim x2 y neighborhood of z0.
x 0, y 0 = lim 0 = 0.
x4 + y2 x 0 Note:
Path II 1. Analytic functions are also called as holomorphic,
regular or monogenic functions.
x2 y
lim 2 lim 0 = 0.
2. A function f(z) is analytic in a domain if it is analytic
y 0 x 0 x + y x 0
4
at every point of the domain.
Path III, along any straight line through the origin. Let 3. Sum difference product and quotient of analytic
y = mx. functions are also analytic.
lim x2 y
y mx , x 0 x 4 + y 2
Entire function
mx 3
lim lim
mx A function which is analytic everywhere (i.e., at every point
=x 0 4 = x 0 2 = 0.
x +m x
2 2 x + m 2 Z on the complex plane) is known as an entire function.
Path IV Eg: All polynomial functions are entire functions.
Let y = mx2
lim x2 y Standard results on Complex
y mx 2 , x 0 4
x + y 2 Differentiation
dc
lim mx 4 1. = 0, where c is a complex constant.
= x 0 4
x + m 2 x 4 dz
m d df dg
lim
= x 0 , which depends on the values of m. 2. [f g] = +
1 + m 2 dz dz dz
Therefore, the limit does not exist. d df
3. [c f(z)] = c.
dz dz
Continuity 4.
d
[f.g] = f.
dg
+ g.
df
A function w = f(z) defined in the bounded closed domain dz dz dz
D, is said to be continuous at a point Z0, if f(z0) is defined, df dg
g. f
lim lim d f dz dz
z z 0 f(z) exists and z z 0 f(z) = f(z0). 5. =
dz g g2
Note: d
6. zn = n.zn 1
1. A function f(z) is said to be continuous in a domain if dz
it is continuous at every point of the domain. 7. If f(z) = G (h (z))
2. A function f(z) is not continuous at z0 implies either d
lim
f(z0) does not exist or z z 0 f(z) does not exists or then, (f(z)) = G(h (z)).h(z) (termed as chain rule).
dz
Lim
z z0 f(z) f(z0). The Necessary Condition for f(z) to be
3. If f(z) = u + iv is continuous, then both u and v are Analytic
continuous.
If a function f(z) = u(x, y) + iv(x, y) is differentiable at any
4. If f(z) and g(z) are two continuous functions in D, then
point z = x + iy, then the first order partial derivatives of u
f + g, f g, fg and f/g (g(z) 0) are all continuous.
and v exist at this point and satisfy the equations
u v u v
Differentiability =
x y
(1) and
y
=
x
(2).
A function f(z) defined in a domain D is said to
be differentiable at a point z0, if the limit f(z0) = Equations (1) and (2) are referred to as Cauchy-Riemann
Equations (CR equations).
lim f (z 0 + z ) f (z 0 )
z 0
z The Sufficient Condition for f(z) to be
analytic
lim f (z ) f (z 0 )
=z z0 exists. A continuous function f(z) is analytic in a domain D if the
z z
0 u u v v
four partial derivatives , , and exist, are also
The limit f(z0) is known as the derivative of f(z) at z0. x y x y

Chapter 04.indd 99 8/28/2015 7:52:59 PM


2.100|Engineering Mathematics

continuous and satisfy the Cauchy-Riemann equations at Elementary Functions


each point on the domain D. 1. Complex exponential function ez:
ez = ex(cos y + isin y).
Laplace equation 2. Trigonometric functions:
An equation having the second order partial derivatives of 1
(a) cos z = (eiz + eiz)
2u 2u 2
the form 2 u = 2 + 2 = 0 is known as the Laplace equa-
x y 1
tion. 2 is called the Laplacian operator. (b) sin z = (eiz eiz)
2i
sin z
Harmonic function (c) tan z = , if cos z 0
cos z
A function u(x, y) is said to be a harmonic function if it
cos z
satisfies the Laplace equation i.e., 2u = 0. (d) cot z = , if sin z 0
sin z
Note: The real and imaginary parts of an analytic function 1
are harmonic. (e) sec z = , if cos z 0
cos z
1
Conjugate harmonic function (f) cosec z = , if sin z 0.
sin z
If f(z) = u + iv is an analytic function, then v is the conju-
gate harmonic function of u and vice versa. Note:
1. ez, sinz and cos z are entire functions.
Cauchy-Riemann equations in polar co-ordinates 2. tan z and sec z are analytic, except at the points
where cosz = 0.
u 1 v 3. cot z and cosec z are analytic except at the points
If f(z) = u(r, q) + iv(r, q) where Z = reiq, then = and
r r dq where sin z = 0.
v 1 u
= are called Cauchy-Riemann equations (C.R. 4. eiz = cos z + isin z.
r r q
equations) in the polar form. 3. Hyperbolic functions:
ez e z
Methods of constructing an analytic (a) sinh z =
2
function ez + e z
1. If the real part u(x, y) of the function f(z) is given, then (b) cosh z =
2
u u We can write the other hyperbolic functions like the
Step 1:find and
x y above.
u v 4. Relationship among trigonometric complexes and
Step 2:write f(z) = +i hyperbolic functions:
x x
u u (a) cosh(iz) = cos z (b) sinh(iz) = isin z
= i [using C.R. eqns.] (c) cos(iz) = cosh z (d) sin(iz) = isinh z
x y
Step 3: Replace x by z and y by 0. (e) tan(iz) = itanh z
Step 4:Integrate f(z) with respect to z.
Find the following limits:
 he above method is known as Milne-Thompson
T
method. Lt z
Example 4: (i) z
2. If the real part u(x, y) of an analytic function f(z) is given, 3+ i z

( )
to find the harmonic conjugate, find f(z) using Milne- Lt
Thompson method and then separate the real and z z + 4i z +i .
(ii)
imaginary parts by putting z = x + iy. Then, the imaginary
z
part v(x, y) is the harmonic conjugate of u(x, y). Solution: (i) Lt , Let z = 1/a a =1/z as
z 3 + iz
Complex potential function z ; a 0.
The given limit becomes
If w = f(x, y) + iY(x, y) is analytic, then it is called complex
1/a 1 1
potential function. Its real part f(x, y) is known as the Lt = Lt = = i.
a 0 3 + i /a a 0 3a + i i
velocity potential function and its imaginary part Y(x, y) is
known as the stream function.
(ii) Lt
( z + 4i z +i )( z + 4i + z +i )
Note: Both f and Y satisfy Laplace equation. z z + 4i + z +i

Chapter 04.indd 100 8/28/2015 7:53:05 PM


Chapter 4 Complex Variables | 2.101

( ) ( ) If we consider the path y = mx


2 2
z + 4i z+i
Lt
mx
z z + 4i + z +i Lt
x 0
x + x 2 m2
2

3i
= Lt mx
z + 4i + z +i
z = Lt
x 0
x 1+ m2
Let z = 1/a a = 1/z, as z , a 0
3i m
Lt =
a0
1 1 1+ m2
+ 4i + + i
a a As the limit value depends upon m, the limit does
not exist at z = 0. It is not continuous at z = 0.
3 ai
= Lt = 0. Re ( z 2 ) x2 y2
a0 1 + 4 ai + 1 / 1 + ai (ii) Lt = Lt 2
z 0 | z |2 y0 x + y 2
x0
Example 5: Show that the following limits do not exist: Considering the path y = mx,
(i) Lt
[Rez mz ] 2 x 2 m2 x 2 1 m2
z0 2 Lt =
z x 0 x 2 + m 2 y 2 1+ m2
1 which also depends on m. Hence, the limit does not
(ii) Lt iy exist and therefore f(z) is not continuous at z = 0.
z 0 1 e1/ x

Example 7: Show that the function
Solution: (i)If we consider any path which is a straight
line i.e. y = mx, x 3 (1 + i ) y 3 (1 i ),
z 0
Lt
[Re z mz ] 2

= Lt 2
( x y)2 f(z) = x2 +y 2
z0
z
2 z0 x + y 2 0, z =0

( x mx ) 2 satisfies Cauchy-Riemann equations at


= Lt x (1 m)
2 2
Lt z = 0, but f(0) does not exist.
x0 x 2 + m2 x 2 x 0 x 2 (1 + m 2 )

x3 y3
(1 m) 2 Solution: We have u(x, y) = and
= x2 + y2
1+ m2
x3 + y3
As the limit depends upon m, it is not unique. v(x, y) = ,
x2 + y2
The limit does not exist.
when (x, y) (0, 0).
1 Also, it is given that, f(0) = 0 u (0, 0) = V(0, 0) = 0.
(ii) Lt iy
z 0 1 e1/ x

u u ( x, 0) u (0, 0) x
If we consider the path y 0, followed by = Lt = Lt = 1
x x 0 x x0 x
x0
1 0, x > 0 u u (0, y ) u (0, 0) y
= Lt = Lt = 1
Lt 1 e1/ x = 1. x < 0. y y 0 y y 0 y
x0
The limit is not unique; it does not exist. v v ( x, 0) v (0, 0) x
= Lt = Lt = 1
x x 0 x x0 x
Example 6: Show that the function f(z) is not continuous
at z = 0, where v v (0, y ) v (0, 0) y
= Lt = Lt = 1.
m( z ) y y 0 y y0 y
z0
(i) f(z) = | z |2 C.R. equations are satisfied at the origin.
0, z= 0 f ( z ) f ( 0)
But Lt
Re ( z ) 2 z 0 z
, z0
(ii) f(z) = | z |2 x3 y3 + i ( x3 + y3 )
0, z = 0. = Lt
z 0 ( x 2 + y 2 ) ( x + iy )

Lt
Lt
Solution: (i) z
m ( z )
= y0 y
. (1 + i ) [ x 3 + iy 3 ] ( x iy )
0 |z| = Lt
x0 x2 + y2 z 0 ( x 2 + y 2 ) ( x + iy ) ( x iy )

Chapter 04.indd 101 8/28/2015 7:53:19 PM


2.102|Engineering Mathematics

(1 + i ) ( x 3 + iy 3 ) ( x iy ) Integrating (3) and (4) partially w.r.t. x, we get,


Lt u (x, y) = e-x [x sin y - y cos y] + 1 (y)
z 0 ( x 2 + y 2 )2
V (x, y) = e-x [y sin y + x cos y] + 2 (y) respectively,
We consider the path y = mx. We get,
where 1(y) and 2(y) are arbitary functions of z.
(1 + i ) ( x 3 + m3 x 3 ) ( x imx ) Substituting in equations (3) and (4), we obtain respectively,
Lt
x 0 ( x 2 + m2 x 2 )2 e-x [(1 - x) sin y + y cos y] = e-x [sin y + y cos y - x sin y] +
12 (y)
(1 + i ) (1 + im3 ) (1 im) x 4
Lt and e-x [(1 - x) cos y - y sin y]
x 0 (1 + m 2 ) 2 x 4 = e-x [a cos y - cos y + y sin y] + 11 (y).
(1 + i ) (1 + im3 ) (1 im) On simplifying, we get 12 (y) = 0 and 11 (y) = 0.
=
(1 + m 2 ) 2 which gives 11 (y) = k1 and 2 (y) = k2
which depends on the value of m. Therefore, the limit where k1 and k2 are arbitary constants.
does not exist. Hence f(0) does not exist. Hence we have,
For the given function f(z), though Cauchy-Riemann u(x, y) = e-x (x sin y - y cos y) + k1 and
equations are satisfied at the origin, it is not analytic at the V(x, y) = e-x (y sin y + x cos y) + k2.
origin. and f (z) = u(x, y) + iv(x, y)
z = e-x[x sin y - y cos y ] + k1 + i[e-x(y sin y + x cos y] + k2
Example 8: Show that the function f(z) = is analytic = e-x [(x + iy) sin y - i(a + iy) cos y] + k1 + ik2
at z = . z +1
= i(x + iy) e-x e -iy + k1 + ik2
= i(x + iy) e-(x + iy) + k1 + ik2
z = ize-z + k, where k = k1 + ik2 is a complex constant.
Solution: Given that f(z) =
z +1
Example 10: Show that the function u(r, q) = r2 cos2q is
1
harmonic. Find its conjugate harmonic function and the
1
f(1/z) = z = corresponding analytic function f(z).
1/ z +1 z +1
We know that f (1/z) is differentiable at Solution: We have z = reiq and given that u (r, q) = r2 cos2q.
z = 0 and at all points in its neighbourhood. u 2 u
= 2r cos2q, 2 = 2 cos2q.
f(1/z) is analytic at z = 0 r r
f(z) is analytic at z = . u
Also, = -2 r2 sin2q and
Example 9: If f(z) = u + iv is an analytic function of z = x q
+ iy and u v = ex [(x y) sin y (x + y) cos y] then find u, 2 u
v and the analytic function f(z). = - 4 r2 cos2q.
q 2
Solution: u v = ex [(x y) sin y (x + y) cos y]
2 u 1 u 1 2 u
u v = e-x [(x - y) sin y - (x + y) cos y] Since, + +
r 2 r r r 2 q 2
x x
+ e-x [sin y - cos y] 1 1 2 u
= e [(1 - x + y) sin y + (x + y - 1) cos y] (1).
-x = 2 cos2q + (2r cos2q) + 2
r r q 2
u v -x
Also, - = e [x cos y - (sin y + y cos y) + x sin y = 4 cos2q - 4 cos2q = 0,
y y 
- (cos y - y sin y)] u(r, ) satisfies the Laplace equation in the polar form
= e-x [(x + y - 1) sin y + (x - y - 1) cos y] (2). and therefore, it is harmonic.
u v From C.R. equations, we know that
Using C.R. equations, = and
x y v u
=r = r. (2r cos2q) = 2r2 cos2q.
u v q r
=-
y x v
and substituting in (1) and (2) we get, Integrating partially w.r.t. q, we get
q
u v v(r, q) = r2 sin2q + (r), where (r) is an arbitary
= ux = e-x[(1 - x)sin y + y cos y] = (3)
x y function of r.
v v 1 u
and = Vx = e-x[(1 - x)cos y - y sin y] Also using C.R. equations, =-
x r r q
u 1
=- (4). we get, 2r sin2 + 1 (r) = - (-2 r2 sin2)
y r

Chapter 04.indd 102 8/28/2015 7:53:32 PM


Chapter 4 Complex Variables | 2.103

1 (r) = 0 (r) = c a constant. x y


v(r, q) = r2 sin2q + c. = +i
x 2 + y 2 x 2 + y 2
f(z) = u(r, q) + iv (r, q)
= r2 cos2q + i[r2 sin2q + c] Using Milne-Thompson method, replacing x by z and y
= r2 [cos2q + isin2q] + ic = r2 e2iq + ic. by 0, we get
z 1
Example 11: Show that the given function u(x, y) = sinh x f (z) = 2 =
cos y is harmonic. Find the corresponding harmonic func- z z
tion v(x, y) and construct the analytic function f(z) = u + iv. On integrating, f(z) = log z + ic
Solution: u(x, y) = sinh x cos y. f(z) = log (reiq) + ic = log r + i q + i c
u 2u = log x 2 + y 2 + i[q + c]
= cosh x cos y, 2 = sinh x cos y.
x x 1
u = log (x2 + y2) + itan-1 (y/x) + ic
Also, = - sinh x sin y, 2
y u(x, y) = log (x2 + y2 )
2u
= - sinh x cos y. Example 13: Find the constants p, q, r such that the function f(z)
y 2 = x - 2 py + i (qx - ry) is analytic. Express f(z) in terms of z.
2u 2u Solution: From the given function, u (x, y) = x - 2 py and
+ = 0 u is harmonic.
x 2 y 2 v (x, y) = qx - ry.
As f(z) = u + iv As u and v are continuous every where, they will be ana-
lytic if C.R. equations are satisfied.
u iv u iu
f (z) = + = - u u v v
x x x y i.e. = 1, = -2p, = q and = -r
x y x y
[using C.R. equations] = cosh x cos y -i(-sinh x sin y).
Using Milne-Thompson method, replacing x by z and y by u v
= 1 = -r r = -1
0, we get f(z) = cosh z x y
Integrating w.r.t. z, we get f (z) = sinh z + ic u v
f(z) = sinh z + ic = sinh (x + iy) + ic and =- -2 p = -q
y x
= sinh x cos y + icosh x sin y + ic
where p is arbitary q = 2p.
v (x, y) = sin y cosh x + c.
The required analytic function is
Example 12: Show that v(x, y) = arg (z), z 0, is harmonic. f(z) = x - 2 py + i (q x - r y)
Find the corresponding conjugate harmonic function u(x, y) = x - 2 py + i (2px + y)
and constract the analytic function f(z) = u + iv. = [x + iy] + 2 pi [x + iy]
f(z) = z + 2 piz.
Solution: Given v(x, y) = arg (z)
v(x, y) = tan-1 (y/x).
Complex Integration
v 1 y x2 y y Let f(z) be a function of complex variable defined in a
= = =
x y 2 x 2 x 2 + y 2 x 2 x 2 + y 2 domain D. Also, let c be the closed curve in the domain D.
1+ 2
x Let f(z) = u(x, y) + iv(x, y), where z = x + iy
2 v ( x 2 + y 2 ) ( 0) + y ( 2 x ) 2x y (OR)
= = 2
x 2 (x + y )
2 2 2
( x + y 2 )2 f(z) = u + iv and dz = dx + idy.
v 1 .1 x 2
1 x
y
= = 2
y x x +y x x +y2
2 2
= 2 f ( z )dz = (u + iv ) dz
1+ 2 c c
x
= (u + iv ) ( dx + idy )
2 v ( x 2 + y 2 ) 0 x (2 y) 2 xy
= = 2 c
y 2 (x + y )
2 2 2
( x + y 2 )2
= (udx vdy ) + i (udy + vdx ).
2 v 2 v c c
2 + 2 = 0.
x y f ( z ) dz is determined as the contour integral.
v(x,y) satisfies Laplace equation and hence harmonic. c

As f(z) = u + iv
Example 14: Evaluate f ( z ) dz, where f(z) = z2 and c is the
u i v v i v
f(z) = + = + [using C.R. equation] c

x x y x straight line joining z = 0 and z = 4 + i.

Chapter 04.indd 103 8/28/2015 7:53:45 PM


2.104|Engineering Mathematics

4+i i.e., suppose c is the curve consisting of OA and OB,


Solution: If we integrate 0
z 2 dz formally, then

z3
4+i
(4 + i ) = 52 + 47i . 3 z dz = z dz + z dz .
we get = OA B OA AB

3 0 3 3 x =2 y =1

But, let us find the parametric equations of the curve c, = (x iy ) (dx + idy ) + (x iy ) (dx + idy )
i.e., the straight line joining z = 0 and z = 4 + i, that is O (0, 0) x =0 y =0

and A (4, 1). x=2 y =1

Slope of OA =
1 0 1
= .
= ( x ) (dx ) + (2 iy ) (idy )
40 4 x=0 y=0
1
Equation of OA = y - 0 = (x - 0) x2
2
iy 2 i
1

x = 4y. dx = 4 dy. 4 = + i 2y = 2 + i 2
2 0 2 0 2
Taking y itself as the parameter,
1 5
1 = 2i + 2 + = 2i + .
z 2 dz = ( x + iy ) (dx + idy )
2
2 2
y=0
1 Note: Therefore, it is very clear that the value of the
(4 y + iy ) (4dy + idy )
2
= integral depends on the contour c.
y=0

1
Cauchys Integral Theorem
=
y=0
(16 y 2
y 2 + 8iy 2 ) ( 4 + i ) dy
If f(z) is analytic in a simply-connected region R, then
1
f (z ) dz = 0 for every closed contour c contained in R.
= (4 + i)
y=0
(15 y 2
+ 8iy 2 ) dy c

Example 16:If c is any simple closed curve, evaluate


y 3 ( 4 + i ) (15 + 8i )
1

= (4 + i) [15 + 8i] = f (z ) dz , if f(z) =


3 0 3 c
(A) 2z2 + 18 (B) e 12 z
60 + 15i + 32i 8 52 + 47i (C) cos 8z (D) sin 4z + 8z5
= = .
3 3
Solution: All these functions are analytic everywhere and
Note: in particular, on and within any closed curve c.
1. The value of the integral in both the above cases is the Hence, by Cauchys theorem f ( z ) dz = 0.
same. c
2. But, we cannot expect such a thing to happen in all the
Note:
cases.
1. If f(z) is analytic in the region R, then the line integral
The value of integral f ( z ) dz depends on the contour c. of f(z) is independent of the path joining any two points
c
of that region.
Example 15: Evaluate zdz , where c is the straight line joining 2. If f(z) is analytic in a doubly-connected region R
c bounded by two simple closed curves c1 and c2 where
z = 0 and z = 2 + i.
c2 is in the inside of c1, then f ( z ) dz = f ( z ) dz .
1 c1 c2
Solution: z dz = ( x iy ) (dx + idy )
c 0
dz
1 Example 17: Evaluate , where c is the circle, z 3 = 2.
z
= ( 2 y iy ) ( 2dy + idy ) c

0 1
1 Solution: f(z) = is analytic at all points except the origin,
= ( 2 + i ) ( 2 i ) ( y ) dy z
0 but the circle z 3 = 2 does not pass through the origin and
1
1
y2 5 also O does not lie in c. Hence, f(z) is analytic on and in c.
= ( 4 i 2 ) ydy = 5 = . Hence, by Cauchys theorem.
0 2 0 2
Remark: Suppose, c is the path consisting of two line f (z ) dz = 0.
segments joining z = 0, z = 2 and z = 2 and z= 2 + i c

Chapter 04.indd 104 8/28/2015 7:53:56 PM


Chapter 4 Complex Variables | 2.105

z2 1 f (p)
Example 18:Evaluate z 5 dz, where c is the circle where an =
2pi  ( p a) n +1
dp .
z = 2. c
c

z
z2 Example 20: Expand e z 2 as a Laurent series about z = 1.
Solution: f(z) = is not analytic at z = 5. But this
z5
point lies outside the circle z = 2. Hence, f(z) is analytic Solution: Put z - 2 = w. Then,
at all points interior and on the closed curve c. Hence, by z 2+w 2 2

2n w n
= e 1 +
1+
Cauchys theorem f ( z ) dz = 0 . ez 2 = e =e = e1 e
n !
w w w

c n =1

Cauchys integral formula



= e 1 +

( z 2) n .
n!
If f(z) be analytic within and on a closed contour c inside a n =1

simply-connected domain and if zo is any point in the mid-
z2 1
Example 21: Expand f(z) = ; 2 z 3, as
dle of C, then f ( z o ) =
1

f (z)
dz , the integral being Laurents series. ( z + 1) ( z + 4)
2 p i c z zo
taken in the positive sense around c. z2 1
Solution: f(z) = ceases to be analytic only for
( z + 1) ( z + 4)
Generalisation of Cauchys integral formula z = -1 and z = -4.
If f(z) is analytic on and within a simple closed curve C and In the region 2 z 3, f(z) is analytic. Resolving f(z)
if zo is any point within c, then into partial fractions,

f n
( z ) = 2np!i f (z )
dz . f(z) = 1 -
5
.
(z z )
o n +1
c
o
z+4

z
Moreras theorem As z 3, < 1, hence
4
[converse of Cauchys theorem]:
If a function f(z) is continuous throughout the simple con- 1 1 z 1
1
z z2
nected domain D and if f ( z ) dz = 0 for every closed con- = 1 + 4 = 4 1 4 + 16 .....
z+4 4
c
tour c in D, then f(z) is analytic in D. z2 1 5 5 z z2
=1 = 1 1 + .....
Taylors Series Expansion of f(z)
( z + 1) ( z + 4) z+4 4 4 16

Let f(z) be analytic at all points within a circle co with centre which is the required Laurents expansion.
a and radius r. Then at each point z within co,

f (a) ( z a) 2 f n (a) Zero of an Analytic Function


f(z) = f(A) + f(A) (z-a) + ++ (z A azero
) n . of an analytic function f(z) is a value of z, such that
f n ( a) 2! n!
( z a) n . + .. . f(z) = 0.
n!
Example 19:Find the Taylor expansion for ez about the Zero of mth Order
point z = 3. If an analytic function f (z) can be expressed in the form f(z)

Solution: Put z - 3 = w, then ez = ew + 3 = e3 ew = e3


w = (z
= ez3

( z 3) .
- p)m n (z), where (p) 0 and (p) is analytic then,
n

n ! = p is ncalled
! zero of mth order of f(z).

wn
( z 3) n n= 0 n= 0


n= 0 n !
= e3
n= 0 n!
. Note: Zero of order one is known as simple zero.

(i) f(z) = (z - 2)5, then z = 2, is a zero of order


Example
Laurents Series Expansion of f(z) 5 of f(z).
If f(z) is analytic in the ring shaped region r2 < z a < r1 , (ii) f(z) = sinz, then z = 0, p, 2p,
and c is any simple closed curve around z = a in the region .. are simple zeros of f(z).
of analyticity, then 2
(iii) f(z) = , then z = is a simple
a ( z a) , 3 z
n
f(z) = n
n = zero of f(z).

Chapter 04.indd 105 8/28/2015 7:54:06 PM


2.106|Engineering Mathematics

Exercises
Practice Problems 1 x2 + y2 = 12x + 4
(C)
Directions for questions 1 to 40:Select the correct alternative x2 + y2 + 12x + 4 = 0
(D)
from the given choices. z+i
3 + 5i (v) = 2.
1. The multiplicative inverse of is z +1
2-i
(A) x2 + y2 + 8x - 2y + 3 = 0
(A) 1 + 13i (B) 1 - 13i (B) 3 (x2 + y2) + 8x - 2y + 3 = 0
1 - 13i 1 + 13i (C) x2 + y2 + 4x + 2y + 3
(C) (D)
34 34 (D) 3 (x2 + y2) - 8x + 2y + 3 = 0
4 + 7i 4 - 7i z+9
2. If - = x + iy, then (x, y) = (vi) is purely imaginary.
2 - 3i 2 + 3i z + 3i
(A) (2, 0) (B) (0, 2) (C) (4, 0) (D) (0, 4). (A) 2x2 + 2y2 + 9x + 3y = 0
12 (B) 2x2 + 2y2 = 9x + 3y
1+ i 3 (C) x2 + y2 = 9x + 3y
3. =
1 - i 3 (D) x2 + y2 + 9x + 3y = 0

(A) -1 (B) 1 (C) 36 (D) 312. 3- z p
(vii) Arg =
4. The square root of -9 + 40i is 3+ z 6
(A) (4 - 5i) (B) (5 + 4i) (A) (x2 + y2 + 9)2 = y2
(C) (4 + 5i) (D) (5 - 4i). (B)
(x2 + y2)2+ = 12y2
5. If 1, w, w2 are the cube roots of unity, then (5 + 5w (C) (x2 + y2 - 9)2 = 108y2
+ 8w2)9 + (9 + 12w + 9w2)9 = (D) (x2 + y2 - 9)2 = y2
(A) -39 (B) 39 (C) -2.39 (D) 0. 9. The real part of tanh z is _______.
6. Solve x5 + 1 = 0. sinh 2x
(A)
p cosh 2x + cosh 2 y
(A) cis (2k + 1) , k = 0, 1, 2, 3, 4
6 sinh 2x
p (B)
(B) cis (2k + 1) , k = 0, 1, 2, 3, 4 cosh 2x cosh 2 y
10
sinh 2x
p (C)
(C) cis (2k + 1) , k = 0, 1, 2, 3, 4 cosh 2x + cosh 2 y
5
p (D) none of these
(D) cis (2k + 1) , k = 0, 1, 2, 3, 4
3 10. The imaginary part of log sinz is:
7. log (logi) = (A) tan1(cot x sech y) (B) tan1(cot x tanh y)
cot x
p 2
(A) - log (B)
ip ip (C) tan1 (D) none of these.
- log tanh y
2 2 2 p
11. If z = x + iy, then cos z = __________.
2 ip 2
ip + log (D)
(C) + log (A) cos z (B) tanh z
p 2 p
(C) cos z (D) sec z
8. If z = x + iy and p represents a point in the argand plane, then
12. The principal value of log(i) is _____.
find the locus of p, when
p p
(i) |z - 1| = 3. (A) log1
i (B) log1 + i
(A) (x - 1)2 + y2 = 9 (B) (x + 1)2 + y2 = 3 2 2
(C) x2 + (y - 1)2 = 9 (D) x2 + (y + 1)2 = 3 p
log1 i
(C) (D) none of these
2
p
(ii) amp (z - 5) = .
3 13. The function f(z) = 5z2 is
(A) y2 = (x - 5)2 (B) y = 3 (x - 5)2 (A) continuous but not differentiable
(C) y = 3 ( x - 5) (D)
2 2
y2 = 3 (x - 5)2 (B) differentiable only at some points
(iii) Im (z2) = 10. (C) analytic only at z = 0
(A) xy = 10 (B) xy = 5 (D) entire function.
(C) xy = 6 (D) xy = 7 z+3
14. The function f(z) = ceases to be analytic at
(iv) 4Re (z + 3) = |z|2 z 2 ( z 2 + 1)
(A) x2 + y2 = 4x + 12 z = 0
(A) (B) z = i
(B) x2 + y2 + 4x + 12 = 0 z = i
(C) (D) all of these.

Chapter 04.indd 106 8/28/2015 7:54:15 PM


Chapter 4 Complex Variables | 2.107

xy 5 ix 2 y 4 (A) 1, 1 (B) 1, 1
15. The function f (z) = - 2 8 , for z 0, for z = 0 (C) 1, 1 (D) none of these
x +y
2 8
x +y
(A) does not satisfy Cauchy-Riemann equation 27. Find the function f(z) in terms of z using the above v alues of
(B) is analytic everywhere p and q.
(C) satisfies C-R equations but is not analytic at the origin (A) z(1 i) + c (B) z(1 + i) + c
(D) none of these. (B) z2(1 i) + c (D) z2(1 + i) + c
( z - 2i ) 4 28. If f(z) = u + iv is an analytic function of z = x + iy, and f is any
16. The rational function f(z) = 2
( z - 4 z + 5) 2 function of u and v with differential coefficient of the first and
(A) is analytic at every point second orders, then
(B) is not analytic at z = 3 2i
f
2
f
2
f 2 f 2 1
(C) is analytic everywhere except at z = 2 i (A) + = + |f (z)|
y
(D) none of these. x u v
17. The harmonic conjugate of u = y2 x2 x is
f f f f |f1(z)|2
2 2 2 2

(A) 2xy y (B) 2xy + y x y = u + v


(B) +
(C) 2xy y (D) 2xy + y.

f f f f |f1(z)|2
2
2y 2 2 2
18. Find the analytic function whose real part is . (C) + = -
x + y2
2 x y u v

2i
(A) + C (B) iz + C (D) none of these.
z
-1 1 2z - 1 p
(C) 2 + C (D) +C 29. The loci of the points z satisfying arg = is _______.
z z 2z + 1 3
19. The analytic function f = u + iv where u = 12xy 9x + 6 is (A) a circle (B) a straight line
(A) 3z + 2z2i + C (B) 9z + 6z2i + C (C) interior of a circle (D) exterior of a circle
(C) 9z 6z i + C
2
(D) none of these. 30. If z1, z2 and z3 are complex numbers on the complex -
20. Given that u = e 3( x 2 - y 2 )
cos6xy is a harmonic function, the cor- plane and if z12 + z22 + z32 = z1z2 + z2z3 + z3z1, then the triangle
responding analytic function is whose vertices are z1, z2 and z3 is ______.
(A) an isosceles right triangle
(A) e z + C (B) e2z + C
2

(B) an equilateral triangle


(C) e + C (D)
z
e 3 z + C.
2

(C) scalene triangle


21. If f(z) = ex(cosp y isinp y) is analytic, then the value of p is (D) none of these
(A) 1 (B) 0 ( 2, 5 )
1
(C) (D) none of these. 31. Evaluate
( 1, 2 )
(y2 - x) dx + (x2 + y) dy along the parabola x = t
22. If the imaginary part of an analytic function is a constant,
then its real part _________. and y = t2 + 1.
(A) must be a constant 1 11
(B) need not be a constant (A) 27 (B) 27
30 30
(C) can be any function
(D) none of these 3
(C) 27 (D) none of these
17
23. Which among the following cannot be the real part of an ana-
lytic function? cosp z 5
(A) x2 y2 (B) 2xy 32. Evaluate dz, where c is | z | = .
z - 2z - 32
2
(C) xy2 (D) none of these c
p i
(A) (B) 2 p i
24. Which among the following is an example for the entire p i/2
(C) (D) none of these
function?
z + 2z
(A) (B) log2z z-4d z
33. Evaluate , where c is a circle | z | = 2.
e2z
(C) (D) none of these c z 2
+ 4z +8
25. u2 is a harmonic function, only when u is ______ (A) p i (B) 2p i
(A) any harmonic function (C) 0 (D) none of these
(B) a constant harmonic function 34. If f (z) has a pole of order q at z = p and f(z) = (z - p)q
(C) not a constant function f(z), then ______.
(D) none of these.
1
(A) [Res f (z)]z = p = f(p)
Linked answer for questions 26 and 27: Let f(z) = (x2 2xy + py2) ( q -1)!
+ i(qx2 y2 + 2xy).
1
(B) [Res f (z)]z = p = f ( q - 1) ( p)
26. If f(z) is analytic, then the values of p and q are ________. ( q -1)!

Chapter 04.indd 107 8/28/2015 7:54:23 PM


2.108|Engineering Mathematics

(C) [Res f (z)]z = p = f ( q -1) ( p) 1 5 1 z + 3


n n
1
(A) - +
(D) none of these z + 3 n = 0 z + 3 7 n = 0 7 z+3
n n
1- e 2 z 1 5 1 z + 3 1
35. If f(z) = , then at z = , f(z) have ______. (B) + +
1+ e 2 z z + 3 n = 0 z + 3 7 n = 0 7 z+3
n n
(A) Pole 1 5 1 z + 3 1
(C) + -
(B) Removable singularity z + 3 n=0 z + 3 7 n=0 7 z + 3
(C) Isolated singularity
(D) none of these
(D) Non-isolated singularity
log z dz
2p 39. Evaluate , where c: |z - 3| = 2.
dz
c ( z - 2)
2
36. Evaluate
0 e - z ( 2 z +1)3
, where c : |z|= 3.
(A) - p i (B) pi
(C) 2 p i (D) none of these
pi pi
(A) (B) 40. The first five terms of the Laurents expansion of the function
e e
e3 z
about the singular point z = 1 are:
p ( z - 1) 4
(C) (D) None of these
e e3 3 e3 9 e3 27 e 3 81 e 3
(A) , , , ,
e z
z 3 ( z - 1) ( z - 1) 2 ( z - 1) 6 ( z - 1) 18
4 3 2
37. Evaluate 2 + 2
dz , where c : | z | = 3/2.
c z ( z i ) e3 2 e3 9 e3 27 e3 81 3
(B) , , , , e
-8 p i
(A) (B) 8 p i ( z - 1) ( z - 1) 2 ( z - 1) 6 ( z - 1) 24
4 3 2

-6 p i (D)
(C) -2 p i e3 3 e3 9 e3 27 e3 81 3
(C) , , , , e
( z - 1) ( z - 1) 2 ( z - 1) 8 ( z - 1) 24
4 3 2
38. Find the Laurent series expansion of the function
e3 3 e3 9 e3 27 e 3 81e 3
z 2 - 8z + 2 (D) , , , ,
f(z) = in the region 5 < |z + 3| < 7. ( z - 1) ( z - 1) 2 ( z - 1) 6 ( z - 1) 24
4 3 2
( z - 2 ) ( z - 4 ) ( z + 3)

Practice Problems 2 6. The polar form of Cauchy-Riemann equations is ________.


Directions for questions 1 to 40:Select the correct alternative u v v u
(A) = , =-
from the given choices. r q r q
1. 2 + i 2 in the modulus-amplitude form is u -1 v v 1 u
(A) cosp/3 - isinp/3 (B) = , =
r r q r r q
(B) 2(cosp/4 + isinp/4)
u 1 v v -1 u
(C) 2(cosp/4 - isinp/4) (C) = , =
(D) 2 (cosp/2 - isinp/2). r r q r r q
(D) none of these
2. The value of (1 + w) (1- w + w) where w is a complex cube
root of unity is 7. The function f(z) = logz ______
(A) 5 (B) 7 (C) 2 (D) 1. (A) does not satisfy Cauchy-Riemann equations
(B) is not analytic
x 1 (C) is everywhere analytic except at z = 0
3. The domain of the function f(z) = - i is:
y (1 - x ) (D) none of these.
(A) entire complex plane
(B) entire complex plane except at y = 0 and x = 1 8. The function f(z) = zz is:
(C) entire complex plane except at y = 1 and x = 0 (A) continuous but not differentiable
(D) none of these. (B) analytic everywhere
(C) differentiable but not analytic at z = 0
z 1
lim (D) none of these.
4. z 1 = ______.
log z 9. If u = 3xy and v = 2y, then f(z) = u + iv is:
(A) 1 (B) 0 (A) analytic function
(C) 1 (D) none of these (B) differentiable
5. The necessary condition for a function f(z) = u(x, y) + iv(x, y) (C) everywhere continuous, but not analytic
which is analytic in the region R is (D) none of these.
u -v u v u v u -v 10. The function f(z) = z 5z is
(A) = , = - (B) = , = (A) analytic
x y y x x y y x
(B) entire function
u 1 v u -1 v (C) not analytic anywhere
(C) = , = (D) none of these.
x v y y u x (D) none of these.

Chapter 04.indd 108 8/28/2015 7:54:35 PM


Chapter 4 Complex Variables | 2.109

11. Analytic function with constant absolute value _____. 23. An arbitrary pair of harmonic functions u and v ______.
(A) need not be a constant. (A) are conjugate, u + iv is not analytic
(B) is a constant (B) need not be conjugate though u + iv is analytic
(C) depends on the value of the analytic function (C) need not be conjugate unless u + iv is analytic
(D) none of these (D) none of these
12. An analytic function with a constant real part is 24. The sum of two harmonic functions is _____.
(A) a constant (A) a harmonic function
(B) depends on its imaginary part (B) need not be a harmonic function
(C) not a constant (C) a conjugate harmonic function
(D) none of these. (D) none of these
13. Which among the following is not an analytic function? 25. Analytic function of an analytic function ______.
(A) z2n (B) z3 z2 (A) need not be analytic
(C) sin z (D) none of these (B) is analytic
14. Which of the following function is analytic? (C) is an entire function
(A) f(z) = xy (D) none of these
f(z) = e - z
-4
(B)
26. If w = f(x, y) + i(x, y) is a complex potential function, then
(C) f(z) = z + 2z its real part is known as ______.
( f(z) = ex(cos y + isin y)
D) (A) stream function
15. The set of points at which the complex valued function f(z) (B) velocity potential function
ceases to be analytic are called _____. (C) non-Laplaces function
(A) zero points (B) null points (D) none of these
(C) singular points (D) none of these 27. The imaginary part of a complex potential function is called
16. For what values of z, is the function w defined by z = sinh u ______.
cos v + icosh u sinv, w = u + iv, ceases to be analytic? (A) velocity potential function
(A) z = 1 (B) z = i (B) stream function
(C) z = 2i 1 (D) None of these (C) periodic function
-y (D) none of these
17. The functions u = x2 y2 and v =
x + y2
2 28. The loci of the point z satisfying |z 1| 3 is ______.
(A) do not satisfy Laplaces equation (A) interior of the circle
(B) do not satisfy C.R. equations but Laplaces equation (B) exterior of the circle
(C) satisfy C.R. equations (C) exterior and boundary of the circle
(D) none of these. (D) none of these
18. The analytic function f(z) = u + iv for u = x2 y2 takes the 29. If z0 is the circumcentre of an equilateral triangle formed with
form: vertices z1, z2 and z3 on the complex plane, then 3z02 equals
(A) 2z3 + (complex constant) ______
(B) z2 + (complex constant) (A) z12 z22 + z32
(C) z2 z + (complex constant) (B) (z1 z2)2 + (z2 z3)2
(D) none of these. (C) z12 + z22 + z32
(D) none of these.
Linked answer for questions 19 and 20: v = ex(xsin y + ycos y)
30. Which among the following represents the Cauchy-integral
is the imaginary part of an analytic function f(z).
formula?
19. f(z) is equal to
zez + (imaginary constant) 1 f ( z)
(A)
(B) zez + (imaginary constant)
f(a) =
(A)
2p z - a dz
c

(C) zez + (imaginary constant) 1 f ( z)


(D) none of these. f (a) =
(B)
2pi z - a dz
c
20. In the above question, the real part of f(z) is _____. 2! f ( z)
(A) ex(xcos y ysin y) (B) ex(xcos y + ysin y) f (a) =
(C)
2pi ( z - a) 2
dz
e (xsin y ycos y)
x c
(C) (D) none of these
1 f ( z)
pi c ( z - a)3
21. The functions which satisfy Laplaces equations in a region R f (a) =
(D) dz
are called ______.
(A) analytic (B) harmonic
3 z 2 - 2 z -1
(C) conjugate (D) none of these 31. Evaluate
1
dz, where c: z - = 1. | |
c z (z - i )
3
3
22. Both the real and imaginary parts of an analytic function are
(A) harmonic (B) non - harmonic (A) -2p (B) -2pi
(C) periodic (D) non - periodic. (C) 2pi (D) none of these

Chapter 04.indd 109 8/28/2015 7:54:38 PM


2.110|Engineering Mathematics

z 2 dz 7 sin h z
32. Evaluate , where c is |z|=
c ( z - i )(16 - z )
2
2
37. Evaluate
c

z6
dz, where c is the boundary of the square

whose sides lie along the lines x = 1, y = 1 and c is


2p i pi described in the positive sense.
(A) (B)
17 17
pi pi pi pi
- 2 pi (A) (B) (C) (D)
(C) (D) none of these 120 30 8 60
17
z 4 dz
1 5e 3 zp 38. Evaluate where c is |z| = 3/2.
c ( z + 1) ( z - 4)
2p i c z 2 + 1
33. Evaluate: dz, p > 0 and C is the circle |Z| = 4. 2

38p i 3p i
(A) sin 3p (B) 5 sin 3p (A) (B)
(C) 3 sin 5p (D) none of these 25 25
25p i
(C) (D) none of these
6 [sin p z 2 + cos p z 2 ]
34. Evaluate dz, where C is the circle |z| = 38
5. c z ( z - 3) z+2
39. The Laurent series expansion of the function f(z) = for
the domain |z| > 2 is ________ . z -2
(A) 4pi (B) 8pi n
(C) -8pi (D) none of these
2n
2
(A) 1 + 4
n =0 z n +1
(B) 1 + 4
n =0

z
1
35. Evaluate dz, where c is the circle |z| = 3.
2
n +1

z ( z -5)
7
(C) 1 + 4
n =0

z
(D) none of these
2p i - 2p i
(A) 7 (B)
5 57 ( z - 2)( z + 2)
40. Expand f (z) = in the region |z| < 1.
pi -p i ( z + 1)( z + 4)
(C) 5 (D)
7 57
(A) 1 + (-1)n+1 [ 1 + 4- n ] z n
e 2 z cos 2 z -1 1 n =1
36. Evaluate dz, where c is |z| = .
c z ( z + 3) 3 (-1)n+1 [ 1 + 4- n ] z n
2 2
-1 +
(B)
n =1
4p i 2p i
(A) (B)
9 9 1 +
(C) (1)n [ 1 + 4- n ] z n
n =1
pi (D) none of these
(C) (D) none of these
9

previous years questions 3i


dz
1+ i 4. If z is a complex variable, the value of
is [2014]
1. The argument of the complex number where i = -1, is 5
z
1- i
 [2014] (A) 0.511 1.57i (B) 0.511+57i
p (C) 0.511 1.57i (D) 0.511+1.57i.
(A) p (B) -
2 5. The product of two complex numbers 1 + i and 2 5i is
p
(C) (D) p.  [2011]
2 (A) 7 3i (B) 3 4i
2. An analytic function of a complex variable z = x + iy (C) 3 4i (D) 7 + 3i.
is expressed as f (z) = u(x, y) + iv(x, y), where i = -1.
3 + 4i
If u(x, y) = 2 x y, then v(x, y) must be [2014] 6. The modulus of the complex number is [2010]
1 - 2i
x2 + y2 + constant
(A) (B) x2 y2 + constant 1 1
x2 + y2 + constant
(C) (D) x2 y2 + constant. (A) 5 (B) 5 (C) (D) .
5 5
3. An analytic function of a complex variable z = x + i y is
7. An analytic function of a complex variable z = x + iy is
expressed as f(z) = u(x, y) + iv(x, y), where i = -1. If u(x, y)
expressed as f(z) = u(x, y) + iv(x, y) where i = -1.
= x2 y2, then the expression for v(x, y) in terms of x, y and
If u = xy, the expression for v should be [2009]
a general constant c would be[2014]
( x + y)2 x2 - y 2
x2 + y 2 (A) + k (B) +k
(A) xy + c (B) +c 2 2
2
y 2 - x2 ( x - y)2
( x2 + y 2 ) (C) + k (D) + k.
(C) 2xy + c (D) + c. 2 2
2

Chapter 04.indd 110 8/28/2015 7:54:59 PM


Chapter 4 Complex Variables | 2.111

f(z ) dz evaluated around the unit circle on the


8. The integral 
cos z 3 1 3 1
complex plane for f(z) = is  [2008] (A) + i (B) -i
z 2 2 2 2
(A) 2p i (B) 4pi (C) 2pi (D) 0.
p 1 3 1 3
(C) + i (D) + i 1 -
2
3
9. Assuming i = -1 and t is a real number, eit dt is: 2 2 2
0
 [2006]

Answer Keys
Exercises
Practice Problems 1
1.C 2.D 3.B 4.C 5.C 6.C 7.B 8.(i)A(ii)D(iii)B
(iv)A(v)B(vi)D(vii)C 9.D 10.B 11.C 12.A 13.D 14.D
15.C 16.C 17.A 18.A 19.C 20.D 21.C 22.A 23.C 24.C
25.B 26.B 27.D 28.B 29.A 30.B 31.D 32.C 33.C 34.B
35.D 36.D 37.A 38.B 39.B 40.D

Practice Problems 2
1.B 2.C 3.B 4.C 5.B 6.C 7.C 8.C 9.C 10.C
11.B 12.A 13.D 14.D 15.C 16.B 17.B 18.B 19.B 20.A
21.B 22.A 23.C 24.A 25.B 26.B 27.B 28.C 29.C 30.D
31.A 32.C 33.B 34.C 35.B 36.A 37.D 38.A 39.A 40.B

Previous Years Questions


1.C 2.C 3.C 4.B 5.A 6.B 7.C 8.A 9.A

Chapter 04.indd 111 8/28/2015 7:55:02 PM


Chapter 5
Probability and Statistics
LEARNING OBJECTIVES

After reading this chapter, you will be able to understand:


Probability Some Special Continuous Distributions
Definition of Probability Measures of Central Tendencies
Addition Theorem of Probability Measures of Dispersion
Conditional Probability Correlation
Advanced Probability Regression
Mathematical Expectation Covariance
Some Special Discrete Distributions

ProbAbility Example: Two drawings of one ball each time are made from a
bag containing balls.
The word PROBABILITY is used, in a general sense, to indicate
Here, we have two events: drawing a ball first time (E1) and
a vague possibility that something might happen. It is also used
drawing a ball second time (E2). If the ball of the first draw is
synonymously with chance.
replaced in the bag before the second draw is made, then the out-
come of E2 does not depend on the outcome of E1. In this case, E1
Random Experiment
and E2 are independent events.
If the result of an experiment conducted any number of times under If the ball of the first draw is not replaced in the bag before the
essentially identical conditions, is not certain but is any one of the second draw is made, then the outcome of E2 depends on the out-
several possible outcomes, then the experiment is called a trial or come of E1. In this case, events E1 and E2 are dependent events.
a random experiment. Each of the outcomes is known as an event.
Compound events: When two or more events are in relation with
Example:
each other, they are known as compound events.
1. Drawing 3 cards from a well shuffled pack is a random
experiment, while getting an Ace or a King are events. Example: When a die is thrown two times, the event of getting
2. Throwing a fair die is a random experiment, while getting 3 in the first throw and 5 in the second throw is a compound
the score as 2 or an odd number are events. event.

Mutually exclusive events: If the happening of any one of the Definition of Probability
events in a trial excludes or prevents the happening of all other,
If an event E can happen in m ways and fail in k ways out of a total
then such events are said to be mutually exclusive.
of n ways and each of them is equally likely, then the probability of
Example: The events of getting a head and that of getting a tail happening E is m/(m + k) = m/n where, n = (m + k).
when a fair coin is tossed are mutually exclusive. In other words, if a random experiment is conducted n times
Equally likely events: Two events are said to be equally likely when and m of them are favourable to the event E, then the probability
the chance of occurrence of one event is equal to that of the other. of the occurrence of E is P(E) = m/n. As the event does not occur
(n m) times, the probability of non-occurrence of E is P ( E ).
Example: When a die is thrown, any number from 1 to 6 may be
got. In this trial, getting any one of these events are equally likely. nm m
P( E ) = = = 1 P(E)
Independent events: Two events E1 and E2 are said to be n n
independent, if the occurrence of the event E2 is not affected by the
occurrence or non-occurrence of the event E1. Therefore, P(E) + P ( E ) = 1.

Chapter 05.indd 112 8/28/2015 7:48:23 PM


Chapter 5 Probability and Statistics | 2.113

Note: Probability of picking three red balls


1. Probability [P(E)] of the happening of an event
E is known as the probability of success and the
4
4 C3 1 20
= = = ; P ( E ) = .
probability [P ( E )] of the non-happening of the event C3 84 21
9
21
is the probability of failure.  dd against the three balls being red are
O
2. If P(E) = 1, the event is called a certain event and if 20 1
P(E) = 0 the event is called an impossible event. = P( E ) : P( E ) = : = 20 : 1.
21 21
3. Instead of saying that the chance of happening of an
event is m/n, we can also say that the odds in favour Example 4:When two dice are rolled together, find the
of the event are m to (n m) or the odds against the probability of getting at least one 4.
event are (n m) to m. Solution: Let E be the event that at least one die shows 4.
E be the event that no die shows 4. The number of favourable
Addition Theorem of Probability outcomes to E is 5 5 = 25. P ( E ) =
25
If A and B are two events, then 36
P(A B) = P(A) + P(B) P(A B). 25 11
This result follows from the corresponding result in set P(E) = 1 P ( E ) = 1 = .
36 36
theory. If n (X ) represents the number of elements in set X,
n (X Y ) = n (X ) + n (Y ) n (X Y). Example 5:When two dice are rolled together, find the
probability that the total score on the two dice will be 8 or 9?
Example: If a die is rolled, what is the probability that the
number that comes up is either even or prime? Solution: When two dice are rolled, the total number of
A = The event of getting an even number = {2, 4, 6} outcomes = 6 6 = 36.
B = The event of getting a prime = {2, 3, 5} Favourable outcomes for getting the sum 8 or 9 are
A B = {2, 3, 4, 5, 6} {(2, 6), (6, 2), (3, 5), (5, 3), (4, 4), (3, 6), (6, 3), (4, 5), (5, 4)}
A B = {2} i.e., the total number of favourable outcomes = 9.
9 1
3 3 5 The required probability = = .
P(A) = , P(B) = , P(A B) = and 36 4
6 6 6
1 Example 6: If two cards are drawn simultaneously from a
P(A B) = . We can verify that pack of cards, what is the probability that both will be jacks
6
or both are queens?
P(A B) = P(A) + P(B) P(A B).
Solution: Here, two events are mutually exclusive, P(J Q) =
Example 1: When a cubical die is rolled, find the probability
of getting an even integer.
4
C2
P(J ) + P(Q). Probability of drawing two jacks is P(J ) = .
52
C2
Solution: When a die is rolled, the number of possible 4
C2
outcomes is 6. The number of favourable outcomes of Probability of drawing two queens is P(Q) = .
getting an even integer is 3.
52
C2
P(J Q) = P(J ) + P(Q)
3 1
The required probability = = .
6 2
4
C2 4
C2 4
C2 2
= + = 2. = .
Example 2: If a card is drawn from a pack of cards, find the 52
C2 52
C2 52
C2 221
probability of getting a queen.
Example 7:When two cards are drawn from a pack of
Solution: When a card is drawn, the number of possible cards, find the probability that the two cards will be kings
outcomes is 52. The number of favourable outcomes of or blacks.
getting a queen card is 4. 4
C
4 1 Solution: The probability of drawing two kings = 52 2 .
The required probability = = . C2
52 13
26
C2
Example 3: A bag contains 5 green balls and 4 red balls. The probability of drawing two black cards is = .
If 3 balls are picked from it at random, then find the odds
52
C2
against the three balls being red. 2
C2
The probability of drawing two black kings is .
Solution: The total number of balls in the bag = 9. Three The required probability
52
C2
balls can be selected from 9 balls in 9C3 ways.
Three red balls can be selected from 4 red balls in 4C3
4
C2 26
C2 2
C2 55
= + = .
ways. 52
C2 52
C2 52
C2 221

Chapter 05.indd 113 8/28/2015 7:48:31 PM


2.114|Engineering Mathematics

Conditional Probability Independent Events


Let S be a finite sample space of a random experiment and A, In a random experiment, if A, B are events such that P(A) >
B be events, such that P(A) > 0, P(B) > 0. If it is known that the 0, P(B) > 0 and if P(A/B) = P(A) or P(B/A) = P(B) (condi-
event B has occurred, in light of this, we wish to complete tional probability equals to unconditional probability) then,
the probability of A, we mean conditional probability of A we say A, B are independent events.
given B. The occurrence of event B would reduce the sample If A, B are independent P(A B) = P(A) P(B).
space to B, and the favourable cases would now be A B. Example 9: Two coins are tossed one after the other and A
A B (new favourable set)
is the event of getting a tail on the second coin and B is the
A Sample
event of getting a head on the first coin, then find P(A/B).
space
Solution: Sample space = {HH, HT, TH, TT}, A = {HT,
TT} and B = {HH, HT}, (A B) = {HT}.
B 2 1
B (new sample space) P(A) = = and
4 2
P ( A B) 1 / 4 1
Notation: The conditional probability of A given B is P(A/B) = = = .
denoted by P(A/B). P ( B) 1/ 2 2
Thus, P(A/B) = P(A).
n( A B) n( A B) / n( S ) P ( A B )
P(A/B) = = = .  Logically too, we understand the occurrence or non-
n( B) n( B) / n( S ) P ( B) occurrence of tail in the 2nd coin.
Notes:
1. This definition is also valid for infinite sample spaces. Bayes Rule
2. The conditional probability of B given A is denoted Suppose A1, A2,.., An are n mutually exclusive and exhaus-
P ( A B) tive events, such that P(Ai) 0. Then for i = 1, 2, 3, .., n,
by P(B/A) and P(B/A) =
P ( A) A
P ( Ai ) P
A A
i
Multiplication Theorem P i =
A
A n
Let A and B be two events of certain random experiment P ( AK ) P
A
such that A occurs only when B has already occurred. Then, k =1 K
A where, A is an arbitrary event of S.
for the conditional event , the total possible outcomes are
B Example 10: Akshay speaks the truth in 45% of the cases.
the outcomes favourable to the event B and its favourable
In rainy season, on each day there is 75% chance of raining.
outcomes are the outcomes favourable to both A and B.
On a certain day in the rainy season, Akshay tells his mother
A n ( A B) that it is raining outside. What is the probability that it is
So, P =
B n ( B) actually raining?
Solution: Let E denote the event that it is raining and
n ( A B) n (S ) 1
= = P ( A B) A denote the event that Akshay tells his mother that it is
n (S ) n ( B) P ( B) raining outside.
A
i.e., P P ( B ) = P ( A B ).
B
3
Then, P(E) = , P E =
4
1
4
( )
This is called the multiplication theorem of probability. A 45 9 A 11
P = = and P = .
Example 8: A letter is selected at random from the set of E 100 20 E 20
English alphabets and it is found to be a vowel. What is the By Bayes Rule, we have
probability that it is e? A
P (E) P
Solution: Let A be the event that the letter selected is e and E E
P =
B be the event that the letter is a vowel. Then, A B = {e} A
and B = {a, e, i o, u }.
A
P (E) P + P E P
E ( )
A
E
1 3 9
A
So, P =
P ( A B ) = 26 = 1 .
4 20 27
B P ( B) = = .
5 5 3 9 1 11 38
+
26 4 20 4 20

Chapter 05.indd 114 8/28/2015 7:48:38 PM


Chapter 5 Probability and Statistics | 2.115

Advanced Probability Mathematical Expectation [E(X)]


Random Variable Mathematical expectation is the weighted mean of values
of a variable.
A random variable is a real valued function defined over the If X is a random variable which can assume any one of
sample space (discrete or continuous). A discrete random the values x1, x2, xn with the respective probabilities p1,
variable takes the values that are finite or countable. For p2, ..... pn, then the mathematical expectation of X is given
example, when we consider the experiment of tossing of 3 by E(X ) = p1x1 + p2x2 + + pnxn
coins, the number of heads can be appreciated as a discrete For a continuous random variable,
random variable (X). X would take 0, 1, 2 and 3 as the
+
possible values. E(X ) =
A continuous random variable takes values in the form x f ( x ) dx, where f(x) is the p.d.f. of X.

of intervals. Also, in the case of a continuous random vari-
able P(X = c) = 0, where c is a specified point. Heights and
Some Special Discrete Distributions
weights of people, area of land held by individuals, etc., are Discrete uniform distribution: A discrete random variable
examples of continuous random variables. defined for values of x from 1 to n is said to have a uniform
distribution if its probability mass function is given by
Probability Mass Function (p.m.f.) 1
; for x = 1, 2, 3..., n
If X is a discrete random variable, which can take the values F(x) = n
x1, x2,. and f (x) denotes the probability that X takes the 0, otherwise.
value xi, then p(x) is called the probability mass function
The cumulative distribution function F(x) of the discrete
of X. p(xi) = P(x = xi). The values that X can take and the
uniform random variable x is given by
corresponding probabilities determine the probability dis-
tribution of X. We also have 0, for x < 1
x
(i) p(x) 0; (ii) p(x) = 1. F(x) = ; for 1 x n
n
Probability Density Function (p.d.f.) 1; for x > 1.
If X is a continuous random variable, then a function f(x), n +1
x I (interval) is called the probability density function. Mean of x = m =
2
The probability statements are made as P(x I ) = f ( x ) dx.
n2 1
We also have, Variance of x = s2 =
12
(i) f(x) 0 (ii) f ( x ) dx = 1.
Binomial distribution: If an experiment which is made of n
The probability P(X x) is called the cumulative dis- independent trials, each of which results in either success
tribution function (c.d.f .) of X and is denoted by F(X). It with probability p or failure with probability q (q = 1 p),
is a point function. It is defined for discrete and continuous then the probability distribution for the random variable X,
random variables. when it represents the number of successes is called a
The following are the properties of probability distribu- binomial distribution. The probability mass function is
tion function F(x): p(x) = b(x; n, p) = nCx px qnx; x = 0, 1, 2, n.
1. F(x) 0. Example: Hitting a target in 5 trials.
2. F(x) is non-decreasing Here, the random variable (X) represents the number of
i.e., for x > y, F(x) F(y). trials made for hitting the target, i.e., x = 0 or 1 or 2 or 3 or
3. F(x) is right continuous. 4 or 5.
4. F( ) = 0 and F(+ ) = 1. We have a set of 5 trials n = 5.
Also, Each trial may hit the target termed to be success (p) or
5. P(a < x b) = F(b) F(a). not termed to be failure (q) which are independent.
For a continuous random variable, This is an example for binomial distribution.

6. Pr{x < X x + dx} = F(x + dx) F(x) = f(x) dx, where


Properties of binomial distribution
dx is very small.
d 1. E (X ) = np (mean).
7. f(x) = [F ( x )] where,
dx 2. V(X ) = E (X 2) (E(X ))2 = npq; (variance) (mean >
(a)f (x) 0 x R. variance).
(b) f ( x ) dx = 1. 3. S.D. (X) = npq.
R

Chapter 05.indd 115 8/28/2015 7:48:41 PM


2.116|Engineering Mathematics

4. Mode of a binomial distribution lies between ( b a) 2


Variance of x = s2 = .
(n + 1)p 1 x (n + 1)p. 12
5. If X1 b(n1, p) and X2 b (n2, p) and if X1 and X2 are
independent, then Normal distribution: A continuous random variable X is
X1 + X2 b (n1 + n2, p) where (n, p) is the p.m.f. of said to have a normal distribution with parameters m and
binomial distribution. s2 if its density function is given by the probability density
function
Poisson Distribution (x m ) 2 < x <
1 e 2s < m <

2

A random variable X is said to follow a Poisson distribu- f ( x ) = s 2p .


tion with a parameter l; l > 0 if it assumes only nonnega- s >0
tive values and its probability mass function is given by 0 otherwise
e 1l x : x = 0, 1, 2, .....
It is denoted as X N (m, s2).
p(x ) = p(x ; l ) = x ! l>0 The graphical representation of a normal distribution is
0 otherwise.
as given below.
In a binomial distribution, if n is large compared with p,
then np approaches a fixed constant, say l. Such a distribu-
tion is called Poisson distribution (limiting case of binomial
distribution). +

Properties of poisson distribution


e l l x
Properties of normal distribution
1. E(X ) = x = l. 1. The function is symmetrical about the value m.
x x!
2. It has a maximum at x = m.
2. V(X ) = E(X 2) (E(X ))2 = l 3. The area under the curve within the interval (m s) is
S.D. (X ) = l 68%
Mean = l = variance. i.e.P (m s x m + s) = 0.68.
3. Mode of a Poisson distribution lies between l 1 and l.
4. If X1 P (l1) and X2 P (l2), and X1, X2 are independent 4. A fairly large number of samples taken from a
then, X1 + X2 P (l1 + l2). Normal population will have average, median
and mode nearly the same, and within the limits of
average 2 S.D.; there will be 95% of the values.
Some Special Continuous +

5. E (X ) = x . f (x ) dx = m.
Distributions

Continuous Uniform Distribution (OR) 6. V (X ) = s2 ;S.D. (X ) = s.


Rectangular Distribution 7. For a normal distribution,
A continuous random variable x defined on [a, b] is said to Mean = median = mode.
have a uniform distribution, if its probability density func- 8. All odd order moments about mean vanish for a
tion is given by normal distribution

1 i.e. m2 n+1 = 0 = n = 0, 1, 2, .
; for x [a, b ]
F(x) = b a
0; otherwise. 9. If X1 N (m1, s12) and X2 N (m2, s22), X1, X2 is

independent, then
The cumulative distribution function of the continuous
X1 + X2 N (m1 + m2, s12 + s22).
uniform random variable x is given by
0; if x a Also, X1 X2 N (m1 m2, s12 + s22).
x a 10. If m = 0 and s2 = 1, we call it as standard normal

F(x) = ; if a < x < b distribution. The standardisation can be obtained by
b a
1; if x b . the transformation
xm X m
a+b z= . Also, N (0, 1).
Mean of x = m = . s s
2

Chapter 05.indd 116 8/28/2015 7:48:45 PM


Chapter 5 Probability and Statistics | 2.117

Exponential Distribution Example 14: A family consists of five children. If the random
A continuous random variable x is said to have an expo- variable (X ) represents the number of boys in that family, then
nential distribution, if its probability density function f (x) (a) find the expected value E(X ) of X
is given by (b) find the variance of X.
le lx ; for x > 0 Solution: This situation can be modelled as a binomial
F(x) =
0; otherwise. distribution.
1 1
Here, l is the parameter of the exponential distribution X b 5, ; E ( X ) = np = 5 = 2.5
and l > 0. 2 2
The cumulative distribution function F(x) of an expo- 1 1
V(X ) = npq = 5 = 1.25.
nential distribution with l as the parameter is 2 2
1 e lx ; if x > 0 Example 15: Ram and Shyam play a game in which their
F(x) = chances of winning are in the ratio 2 : 3. Find Shyams
0, otherwise.
chance of winning at least 3 games out of five games played.
1 1
Mean = m = , Variance = s2 = 2 . 3 2
l l Solution: P(Shyam wins) = ;P(Shyam loses) =
Example 11: An unbiased die is thrown at random. What is 5 5
the expectation of the number on it? Let x denote the number of games won by Shyam.
P(Shyam wins at least 3 games) = P(X 3)
Solution: Let X denote the number on the die, which can
x 5 x
take the values 1, 2, 3, 4, 5 or 6.
5
3 2 5
3 x 25 x
= 5C x = 5C x
1 5 5 55
The probability of each will be equal to x =3 x =3
6 3 3
= 5C 22 + 5C4 3 2 + 1 32 1
X 1 2 3 4 5 6 55 3
1 1 1 1 1 1 27 79
P(X = x) = = 0.68.
6 6 6 6 6 6 3125
Example 16: The p.d.f. of a random variable X is
E(X ) = x P ( X = x )
x (1 / 10) e ( x /10 ) ; x > 0
1 1 1 1 1 1 f(x) =
= 1 + 2 + 3 + 4 + 5 + 6 0 otherwise.
6 6 6 6 6 6
What is P(X 10) (given e1 = 0.3679)?
1 67 7
= (1 + 2 + 3 + 4 + 5 + 6) = = = 3.5. 1 10x
f ( x ) dx =
10 10
6 62 2 Solution: P(X 10) = e dx
0 0 10
Example 12: In a city, 5 accidents take place in a span of 10
x
25 days. Assuming that the number of accidents follow the 1 e 10
Poisson distribution, what is the probability that there will =
10 1
be 3 or more accidents in a day (given e0.2 = 0.8187)?
10 0
5
Solution: Average number of accidents per day = = 0.2. = 1 e1 = 0.6321.
l = 0.2. 25
Probability (3 or more accidents per day)
= 1 P (2 or less accidents) Joint Distribution of Random
= 1 [P(X = 0) + P(X = 1) + P (X = 2)] Variables
= 1 [e0.2 + 0.2e0.2 + 0.02e0.2]
= 1 e0.2[1.22] = 1 0.99814 = 0.001186. Joint Probability Mass Function
Let x and y be two discrete random variables on the same
Example 13: What is the area under the normal curve to
sample space S with the range space of x as Rx = {x1, x2, ..,
the left of Z = 1.54 (the given area between 0 and 1.54
xm} and the range space of y as
= 0.4382)?
Ry = {y1, y2, .., yn} and Px(x) and Py(y) as the prob-
Solution: Required area = 0.5 0.4382 = 0.0618. ability mass functions of x and y. Then, the joint probability
f(z)
mass function Pxy(x, y) of the two dimensional random vari-
able (x, y) on the range space Rx Ry is defined as

Pxy (xi, yj) = ( i j )


P x = x , y = y , for x , y R R
i j x (
y )
1.54 0 +1.54 z
0, otherwise.

Chapter 05.indd 117 8/28/2015 7:48:55 PM


2.118|Engineering Mathematics

This joint probability mass function can be represented in the form of a table as follows:

y
P (x , y )
n

x y1 y2 y3.. yn
xy i j
j =1

X1 Pxy(x1, y1) Pxy(x1, y2) Pxy(x1, y3) Pxy(x1, yn) Px(x1)


X2 Pxy(x2, y1) Pxy(x2, y2) Pxy(x3, y3) Pxy(x3, yn) Px(x2)
X3 Pxy(x3, y1) Pxy(x3, y2) Pxy(x3, y3).. Pxy(x3, yn) Px(x3)
. . . . . .
. . . . . .
. . . . . .
. . . . . .
Xm Pxy(xm, y1) Pxy(xm, y2) Pxy(xm, y3). Pxy(xm, yn) Px(xm)

(x , y )
m

P
i =1
xy i j
Py(y1) Py(y2) Py(y3).. Py(yn)

From the above table, it can be easily observed that the The cumulative joint distribution function FXY(x, y) of the
marginal probability mass functions of x and y namely, Px(x) two dimensional random variable (x, y) (where x and y are
and Py(y) can be obtained from the joint probability mass any two continuous random variables defined on the same
function Pxy (x, y) as sample space) is given by

( )
n
Px (xi) = Pxy xi , y j , for i = 1, 2, . m

j =1
FXY(x, y) = f

XY
( x , y )dx dy .

( )
m
and Py(yj) = Pxy xi , y j Pxy (xi, yj) for j = 1, 2, 3, ..n.
j =1 Conditional Probability Functions
Pxy (xi, yj) 0 i, j. of Random Variables
Let x and y be two discrete (continuous) random variables
( x y ) = 1.
m n

P xy i j defined on the same sample space with joint probability


i =1 j =1
mass (density) function fXY(x, y). Then,
The cumulative joint distribution function of the two 1. the conditional probability mass (density) function
dimensional random variable (x, y) is given by fX/Y(x/y)of x, given Y = y is defined as
Fxy(x, y) = P(X x, Y y).
f XY ( x, y )
FX/Y (x/y) = , where fY(y) 0 and
Joint Probability Density Function fY ( y )
Let X and Y be two continuous random variables on the
2. the conditional probability mass (density) function
same sample space S with fx(x) and fy(y) as the probabil-
fY/X(y/x) of y, given X = x is defined as fY/X (y/x) =
ity density functions, respectively. Then a function fxy(x, y)
is called the joint probability density function of the two f XY ( x, y )
where fX(x) 0.
dimensional random variable (X, Y) if, the probability that f X ( x)
the point (x, y) will lie in the infinitesimal rectangular region
of the area dxdy is fxy(x, y) dx dy Independent Random Variables
Two discrete (continuous) random variables X and Y defined
1 1 1 1 on the same sample space with joint probability mass (den-
i.e. P x dx X x + dx, y dy Y y + dy
2 2 2 2 sity) function PXY(x, y) are said to be independent, if and
= fXY (x, y) dx dy, only if
PXY(x, y) = PX(x) PY(y)
f XY
( x , y )dx dy =1.
where, PX(x) and PY(y) are the marginal probability mass
The marginal probability density functions fX(x) and fY(y) of (density) functions of the random variables X and Y,
the two continuous random variables x and y are given by respectively.

fx(x) =

f XY ( x, y )dy and fy(y) =

f XY ( x, y )dx. Note: If the random variables X and Y are independent, then
Pxy (a X b, c Y d) = Px (a X b) Py(c Y d).

Chapter 05.indd 118 8/28/2015 7:48:59 PM


Chapter 5 Probability and Statistics | 2.119

Statistics (c) Harmonic mean


(d) Median
Statistics is basically the study of numeric data. It includes
(e) Mode.
methods of collection, classification, presentation, analysis
and inference of data. Data as such is qualitative or quan- The first three are mathematical averages and the last two
titative in nature. If one speaks of honesty, beauty, colour, are averages of position.
etc., the data is qualitative while height, weight, distance,
2.Measures of dispersion:It is possible that two sets of
marks, etc. are quantitative.
data may have the same central value, yet they may differ in
The present course aims to systematically study statistics
spread. So, there is a need to study about the spread of data.
of quantitative data. The quantitative data can be divided
The measures we deal with are:
into three categories:
(a) Range
1. Individual series (b) Quartile deviation or semi inter-quartile range
2. Discrete series and (c) Mean deviation
3. Continuous series. (d) Standard deviation (including variance).
The formulae for each of the above mentioned measures
Individual Series are listed for each of the series in what follows.
Examples:
(a) Height of 8 students: Measures of Central Tendencies
5. 0, 4.9, 4.5, 5.1, 5.3, 4.8, 5.1, 5.3 (in feet).
1. Arithmetic mean (A.M. or x):
(b) Weight of 10 students:
(a) Individual series:
46, 48, 52, 53.4, 47, 56.8, 52, 59, 55, 52 (in kgs).
x + x2 + + xn xi
x = 1 =
Discrete Series n n
(b) Discrete series:
Example:
f x + f 2 x2 + + f n xn f i xi
x : Number of children in a family x= 1 1 =
f : Number of families f1 + f 2 + + f n fi
Total number of families = 50. where, x1, x2, . xn are n distinct values with
x
frequencies f1, f2, f3, ., fn, respectively.
0 1 2 3 4
f 8 10 19 8 5 (c) Continuous series:

Continuous Series x=
f1m1 + f 2 m2 + + f n mn
=
fm
i i

Example: Total number of students = 50.


f1 + f 2 + + f n f i

where, f1, f2, f3 ,. fn are the frequencies of the


Class Interval Frequency classes whose mid-values are m1, m2, mn,
(CI ) (f)
respectively.
0 10 8
10 20 12
20 30 13
Some important results based on A.M.
(i) The algebraic sum of deviations taken about the mean
30 40 10
is zero.
40 50 7
(ii) Its value is based on all items.
In order to analyse and get insight into the data, some (iii) The mean of first n natural numbers is (n + 1)/2.
mathematical constants are devised. These constants con- (iv) Arithmetic mean of two numbers a and b is (a + b)/2.
cisely describe any given series of data. Basically, we deal (v) If b is the A.M. of a and c then, a, b, c are in arithmetic
with two of these constants: progression.

1. Averages or measures of central tendencies Combined mean: If x1 and x2 are the arithmetic means
2. Measures of spread or dispersion. of two series with n1 and n2 observations respectively, the

1.Measures of central tendencies:These tell us about n1 x1 + n2 x 2


combined mean, x c =
how the data is clustered or concentrated. They give the n1 + n2
central idea about the data. The measures are:
2.Median: If for a value, the total frequency above (or
(a) Arithmetic mean or mean below) it is half of the overall total frequency, then the value
(b) Geometric mean is termed as Median. Median is the middle-most item.

Chapter 05.indd 119 8/28/2015 7:49:02 PM


2.120|Engineering Mathematics

Individual series: If x1, x2, . xn are arranged in the where, x1, x2 . xn are the n observations and A is the
ascending order of magnitude, then the median is the size mean or median or mode.
n + 1 th
of
2
item. Some results based on M.D.:
(i) Mean deviation depends on all items.
Some results based on median (ii) By default, mean deviation is to be computed about
(i) Median does not take into consideration all the items. the mean.
(ii) The sum of the absolute deviations taken about the (iii) Mean deviation about the median is the least.
median is least. ab
(iii) Median is the abscissa of the point of intersection of (iv) Mean deviation of two numbers a and b is .
2
the cumulative frequency curves.
(iv) Median is the best suited measure for open end classes. 4. Standard Deviation (S.D.): Standard deviation is
referred to as the root mean squared deviation about the
3.Mode:The most frequently found item is called the mean.
mode. Being so, it is easy and straight forward to find for
individual and discrete series. Individual series:
S.D. (s)
Empirical formula:
For a moderately symmetrical distribution,
( x1 x ) 2 + ( x2 x ) 2 + + ( xn x ) 2
mode = 3 median 2 mean. =
For a symmetrical distribution, n
mode = mean = median. This formula is to be applied
in the absence of sufficient data. Given any two, of the where, x1, x2, . xn are n observations with the mean as x.
mean, median or mode, the third can be found.
x xi
2
2

Measures of Dispersion Alternatively, = i


is a useful formula
n n
1. Range: The range of a distribution is the difference
for computational purpose.
between the greatest and the least values observed.

Some important results based on range Some results based on S.D.


(i) The square of standard deviation is termed as
(i) Range is a crude measure of dispersion as it is based
variance.
only on the value of extreme observations.
(ii) S.D. is the least mean square deviation.
(ii) It is also very easy to calculate.
(iii) If each item is increased by a fixed constant, the S.D.
(iii) It does not depend on the frequency of items.
does not alter or S.D. is independent of the change of
Q3 Q1 origin.
2. Quartile Deviation (Q.D.): Q.D. =
2 (iv) Standard deviation depends on each and every data
item.
Individual series: The numbers are first arranged in the
(v) For a discrete series in the form a, a + d, a + 2d,
ascending or descending order, then we find the quartiles
(A.P.), the standard deviation is given by S.D.
Q1 and Q3 as
Q1 size of (n + 1)/4th item n2 1
=d , where n is the number of terms in the series.
Q3 size of 3(n + 1)/4th item. 12
The first quartile (or the lower quartile) Q1 is that value
Co-efficient of variation (CV): Coefficient of variation
of the variable, which is such that one-quarter of the obser-
vations lies below it. The third quartile Q3 is that value of S .D .
(CV ) is defined as, CV = 100.
the variable, which is such that three-quarters of the obser- A.M
vations lie below it.
This is a relative measure, which helps in measuring the
3. Mean Deviation (M.D.): It is defined as the arithmetic consistency. Smaller the co-efficient of variation, greater is
mean of the deviation from the origin, which may be the consistency.
either mean or median or mode.
Example 17: For the individual series, compute the mean,
Individual series: median and mode 8, 11, 14, 17, 20, 23, 26, 29.

M.D. =
x1 A + x2 A + + xn A
Solution: Mean = x =
xi = 8 + 11 + ....... + 29 = 18.5.
n n 8

Chapter 05.indd 120 8/28/2015 7:49:05 PM


Chapter 5 Probability and Statistics | 2.121

Median: As the numbers are in the ascending order and Solution: The given set of observations is {2, 3, 5, 7, 11}
the numbers 17 and 20 being middle terms, x2/n = 208/5
x/n = 28/5
17 + 20 37
median = = = 18.5.
x x
2
2
2 2
S.D. =
Mode: As no term can be regarded as the most often n n
found, mode is not-defined. However, using empirical
2
formula, 208 28
= = 3.2.
mode = 3 median 2 mean 5 5
= 3(18.5) 2(18.5) = 18.5.
Example 20:In a series of observations, coefficient of
Example 18: If the arithmetic mean of 8, 14, x, 20 and 24 variation is 25 and mean is 50. Find the variance.
is 16, then find x. S .D .
Solution: Coefficient of variation: C.V. = 100
8 + 14 + x + 20 + 24 x
Solution: x = = 16 C .V
5 S.D. = x
100
x = 80 66 = 14. = 50 25/100 = 12.5
Example 19: Calculate standard deviation of the first five variance = (12.5)2 = 156.25.
prime numbers.

Exercises
Practice Problems 1 6
C 6
C 12
C5 12
C5
1 18 5 (B) 18 5 (C)
(A) (D) 1 .
Directions for questions 1 to 70: Select correct alternative from C5 C5 18
C5 18
C5
the given choices.
1. If eight unbiased coins are tossed together, then the probabil- 6. On a biased dice, any even number appears four times as fre-
ity that the number of heads exceeds the number of tails is quently as any odd number. If the dice is rolled thrice, what
is the probability that the sum of the scores on them is more
31 1 than 16?
(A) (B)
128 2 26 112 26 112
(A) (B) (C) (D) .
93 57 375 375 3375 3375
(C) (D) .
256 256
7. A five digit number is formed using the digits 0, 1, 2, 3, 4 and
2. If A and B are two mutually exclusive and exhaustive events 5 at random but without repetition. The probability that the
and the probability that the non-occurrence of A is 3/4, then number so formed is divisible by 5 is
the probability of occurrence of B is 1 2 4 9
(A) (B) (C) (D) .
1 1 5 5 25 25
(A) (B)
4 2 8. If ten students are to be seated in a row, then the probability
3 1 that two particular students never sit together is
(C) (D) . (A) 2/5 (B) 4/5
4 16
(C) 1/5 (D) 3/5.
3. If four fair dice are rolled together, then the probability that 9. If six people sit around a circular table, the probability that
the total score on the four dice is less than 22 is two specified persons always sit side by side is
26 3 427 83 (A) 14/15 (B) 11/15
(A) (B) (C) (D) .
27 432 432 108 (C) 2/5 (D) 4/15.

4. A bag contains five red balls, three black balls and a white 10. Eight letters are to be placed in eight addressed envelopes. If
ball. If three balls are drawn from the bag, the probability that the letters are placed at random into the envelopes, the prob-
the three balls are of different colours is ability that exactly one letter is placed in a wrong addressed
(A) 23/28 (B) 5/28 envelopes is
(C) 3/28 (D) none of these. 1 1
5. From a box containing 18 bulbs, of which exactly 1/3rd are (A) (B)
6 8!
defective, five bulbs are chosen at random to fit into the five
bulb holders in a room. The probability that the room gets 1
(C) (D) none of these.
lighted is 7!

Chapter 05.indd 121 8/28/2015 7:49:15 PM


2.122|Engineering Mathematics

11. A puzzle in logic was given to three students A, B and C 20. S = {1, 2, 3, 4, 5}. A, B, C are three subsets of S and each
whose chances of solving it are 1/2, 3/4 and 1/4, respectively. contains three elements from S. It is known that n(ABC) =
The probability that the problem being solved is 5. Find the probability that n(AB) > 3.
22 66 9 99
(A)
29/32 (B)
31/32 (C)
1/8 (D)
7/8. (A) (B) (C) (D)
23 100 10 100
12. If A and B are two events of an experiment such that P(A B)
3 7 21. There are two groups X and Y. Each group contains three dif-
= , P(A) = , then find P(B) given that ferent types of people. In group X, there are 10 Indians, 8
4 20
Americans and 7 Japanese. In group Y, there are 8 Indians,
(i) A and B are mutually exclusive 6 Japanese and 6 Americans. If two people are selected at
1 1 3 2 random from the two groups, then what is the probability that
(A) (B) (C) (D)
4 5 5 5 both selected are Indians from group X?
(ii) A and B are equally likely 56 57 57 57
(A) (B) (C) (D)
7 3 2 13 113 103 113 123
(A) (B) (C) (D)
20 4 5 20 22. An unbiased coin is tossed and a person gets H30 if the coin
(iii) A and B are independent events shows head, and he loses H15 if the coin shows tail. If three
7 8 6 2 coins are tossed, the probability that the person gets H45 is
(A) (B) (C) (D) . (A) 3/8 (B) 1/2 (C) 1/10 (D) 1/25.
13 13 13 5
23. A and B pick a card at random from a well-shuffled pack of
13. The probability that a square selected at random from a 8 8
cards, one after the other replacing it every time till one of
chessboard of size 3 3 is
them gets a spade. The person who picks a spade is declared
8 14 3 25 the winner. If A begins the game, then the probability that B
(A) (B) (C) (D) .
51 17 17 204 wins the game is
14. A dice has two of its sides painted pink, two blue and two 5 4 3 4
(A) (B) (C) (D) .
green. If the dice is rolled twice, the probability that the same 9 9 7 7
colour appears both the times is
24. If three unbiased dice are rolled simultaneously in a random
(A) 1/3 (B) 2/3 (C) 7/9 (D) 8/9.
experiment, the sum of the numbers showing up on them is
15. X and Y are independent events. The probability that both X 14. What is the probability of an event of showing up 6 on any
and Y occur is 1/8 and the probability that neither of these one of the dice?
occur is 3/8. The probability of occurrence of X can be 7 11 4 3
(A) 2/3 (B) 1/4 (C) 1/3 (D) 3/4. (A) (B) (C) (D)
15 15 5 5
16. A letter is taken randomly from the word SISTER, and
another letter is taken randomly from the word RESIST. 25. What is the probability of getting at least 6 heads when a coin
The probability that the two letters are same letters is is tossed 7 times if it is known that there are at least 5 heads?
(A) 2/3 (B) 3/5 (C) 7/36 (D) 2/9. 5 8
(A) (B)
17. A bag contains 12 cards. Five of these cards have the let- 29 29
ter M printed on them, 3 cards have the letter A printed 9
(C) (D) None of the above
on them and the remaining cards have the letter N printed 29
on them. If three cards are picked up one after the other at
26. At the Wimbledon, the probability that Federer qualifies
random, and placed on a table in that order, then what is the
for the final is 0.7, and the probability that Nadal qualifies
probability that the word formed will be MAN?
for the semifinal is 0.5. The probability that Federer qualifies for
5 1 3 3 the final or Nadal qualifies for the semi-final is 0.8. Given that
(A) (B) (C) (D)
44 22 22 44 Nadal qualifies for the semifinal, find the probability that
18. A number is randomly chosen from the numbers 10 to 99. It Federer qualifies for the final.
is observed that the sum of the digits of the number is ten. (A) 0.2 (B) 0.8 (C) 0.6 (D) 0.9
Find the probability that it is divisible by five.
1 1
1 1 1 2 27. If P(A B) = 2/3, P( A B ) A = and P( A B ) + P( A B ) =
(A) (B) (C) (D) 3 2
9 3 2 9 1
P( A B ) + P( A B ) = , then what is the value of P(A/B)?
19. I had to type a 6-character password. The probability that I 2
make a mistake in typing a character is 0.3. The password that 1 1 1 1
(A) (B) (C) (D)
I typed turned out to be wrong. Find the probability that only 3 2 6 5
the last character that I entered is wrong.
( 0.7 )5 ( 0.3 ) ( 0.7 )6 3 6 1 Ac
(A) (B) 2 8. If P(A) = , P(BC
) = and P(A B) = , then find P B c .
1 - ( 0.7 ) 6 1 - ( 0.7 )6 5 7 4
( 0.7 )5 ( 0.7 )( 0.3 )5 17 71 19 29
(C) (D) (A) (B) (C) (D)
1 - ( 0.7 )6 1 - ( 0.7 ) 6 60 120 60 60

Chapter 05.indd 122 8/28/2015 7:49:34 PM


Chapter 5 Probability and Statistics | 2.123

29. If two events A and B are such that P( A ) = 0.4, P(B) = 0.7 and 37. Values of E(x) and E(x2), respectively are

P(A B) = 0.2, then P
B 104 160 102 150
is
A B (A) ,
15 3
(B) ,
15 3
(A) 3/5 (B) 2/5 (C) 1/4 (D) 4/5. 21 160 104 151
(C) , (D) , .
30. A cinema historian noted that for a brief period, all movies 3 5 15 3
released were either directed by Nolan or starred Bale. Also, 38. The value of E[(3x + 2)2] is ________.
no movie directed by Nolan starred Bale. The probability that
(A) 675.2 (B) 560.2
a movie was directed by Nolan is 0.5, and the probability that a
movie starred Bale is 0.5. The probability that a movie is a hit (C) 134.56 (D) 567.2
if directed by Nolan is 0.6, while the probability that a movie 39. For a random variable x, the p.d.f.
is a hit given that Bale acted in it is 0.4. Given that a movie is
a hit, find the probability that it was directed by Nolan. kx 2 0 x1
f(x) = . Find E(x).
0 otherwise
(A) 0.4 (B)0.5 (C) 0.6 (D) 0.7
1 1 3 3
31. A random variable X has mean 2 and E(X ) = 6. Then the
2 (A) (B) (C) (D)
8 4 8 4
standard deviation of X is
40. In the random experiment of drawing a card from 15 cards
(A) 6 (B) 2 (C) 2 (D) 4.
numbered 1 to 15, if x is the random variable defined by the
32. The standard deviation of 3x + 2 is 4; then the variance of x is number appeared on the card, then the expectation of x is
(A) 12 (A) 8 (B) 7 (C) 6 (D) 5.
(B) 4/3
41. For a binominal distribution, the mean is 6 and the variance is
(C) 16/9
2. The number of Bernouli trials is
(D) cannot be determined.
(A) 8 (B) 9 (C) 10 (D) 11.
Linked answer for Questions 33 and 34: Probability mass 42. If X(n, p) follows a binominal distribution with n = 6, such
function of a variate x is as follows: that 9P [X = 4] = P[X = 2], then p =
1 1 1
x 0 1 2 3 4 (A) (B) (C) 1 (D)
3 2 4
P(X = x) k 2k 3k 4k 5k
43. The distribution of the number of male children in a family of
33. k = 5 children follows which of the following distributions?
2 1 3 4 (A) Normal (B) Poisson
(A) (B) (C) (D) . (C) Binomial (D) Negative binomial
15 15 15 15
Linked answer for Questions 44 and 45: Let ABC be a bulb
34. P(x 3) = manufacturing company. The probability that a bulb manufactured
1 4 3 5 by ABC is defective is 0.25.
(A) (B) (C) (D) .
3 15 5 7 44. What is the probability that in a sample of 8 bulbs at most,
one will be defective?
8
35. If X is a random variable taking values 0, 1, 2, 3, 4, 5 with 11 3 3
8

probabilities 3a, 4a, 5a, 6a, 7a, 8a respectively, then P(X 1) = (A) (B)
11
3 4 4
7
3 4 7 5 11 3 1
(A) (B) (C) (D) . (C) (D)
33 33 33 33 3 4 48

36. The expected number of trials required to open a door using a 45. Atleast 3 will be defective?
bunch of n keys of which only one is the correct key is 5 8
3 25 3 61
n n-1 n+1 (A) 1 (B) 1
(A) (B) (C) (D)
n. 4 48 4 9
2 2 2 8 6
3 24 3 1
(C) 1 (D) 1
4 48 4 48
Linked answer for Questions 37 and 38: A variate x has the
probability distribution as: 46. The variance of a Poisson variate is given to be 1. Then,
P(X = 3) is
x 4 8 12
1 1 1 1
(A) (B) (C) (D) .
e 2e 3e 6e
1 3 1 47. A random variable X follows a Poisson distribution such that
P(X = x)
3 5 15 P[X = 1] = P[X = 2]. Its mean and variance are, respectively,

Chapter 05.indd 123 8/28/2015 7:49:48 PM


2.124|Engineering Mathematics

(A) 1, 1 (B) 2, 2 Let X2 and Y2 be two discrete random variables with a joint
(C) 3, 2 (D) 2 , 2. probability mass function as given below:

48. The probability that a person hits a target is 0.003. What is the Y2
0 4 7 P(X2 = xi)
probability of hitting the target with 2 or more bullets if the X2
number of shots is 2000? 1 3 1 3
(A) 1 e6 (B) 1 e6 1
7 14 14 7
(C) 1 7e6 (D) 1 7e6
49. The expected value of a random variable with uniform distri- 4 2 2 4
3
bution over the interval (2, 5) is 21 7 21 7

1 1 1 1 1 1
(A) 2 (B) 2 (C)3 (D) 4 . P(Y2 = yi)
2 2 2 3 2 6
1
50. If X is a continuous random variable with p.d.f. f(x) = if 2
4
x 2 and f(x) = 0 elsewhere, the mean of X is _____ Which of the following statements is TRUE about the random
variables X1, X2, Y1 and Y2?
(A) 1 (B) 1.5 (C) 2 (D) 0.
(A) Only X1 and Y1 are independent.
51. If x is a uniformly distributed random variable in [1, 4] then (B) Only X2 and Y2 are independent.
P x > is
3 (C)  X1 and Y1 are independent as well as X2 and Y2 are
2 independent.
1 1 5 1 (D) Neither X1 and Y1 are independent nor X2 and Y2 are
(A) (B) (C) (D) . independent.
6 2 6 4
52. If x is a uniformly distributed random variable in [2, 5] then 57. If X and Y are two independent random variables with expec-
E(x2) is tations 3 and 4 respectively, then the expectation of X Y is
(A) 1 (B) 7 (C) 12 (D) 16.
(A) 2 (B) 8 (C) 13 (D) 15.
58. If X and Y are two independent random variables that are
53. If the life time of bulbs (in months) is exponential with a uniformly distributed over the same interval [2, 5], then
mean of 5 months, then the probability that the bulb lasts for
P X , Y is
atleast 7 months is 11 11
(A) 0.2466 (B) 0.7534 4 3
(C) 0.4932 (D) 0.5068. 1 2 1 4
(A) (B) (C) (D) .
54. The median of a normal variate X with a p.d.f. 9 9 3 7
1 2 2
f(x ) = e - ( x - m ) / 2s is, 59. Two fair dice are rolled simultaneously. Let X denote the
2p s number on the first die and Y denote the number on the sec-
m
(A) 0 (B) s (C) m (D) . ond die. Then the value of P(X + Y 7/Y 5) is
s
55. x is a normal variate with mean 35 and variance 25. The prob- 1 1 1 1
(A) (B) (C) (D) .
ability of 31 x < 45 is ( 0.8 z < 0 = 0.2881). 2 3 4 6
60. The mean of the cubes of first 10 natural numbers is
0.4772
(A) 305 (B) 300 (C) 302.5 (D) 310.
61. The mean of 25 observations was found to be 38. It was later
discovered that 23 and 38 were misread as 25 and 36. Then
0 2 the mean is
(A) 32 (B) 36
(A) 0.6735 (B) 0.7563 (C) 0.7653 (D) 0.5736 (C) 38 (D) none of these.
56. Let X1 and Y1 be two discrete random variables with a joint
62. If 3, 2 and 9 occur with frequencies 2, 5 and 3 respectively,
probability mass function as given below.
then their arithmetic mean is
Y (A) 4.3 (B) 5 (C) 6 (D) 4.8.
2 3 P(X1 = xi)
X
63. The median of the first ten prime numbers is
1 2 1 (A) 11 (B) 13 (C) 12 (D) 10.
1
15 15 5
64. If the mean of a set of 12 observations is 10 and another set
4 8 4 of 8 observations is 12, then the mean of the combined set is
4 (A) 12.6 (B) 10.8 (C) 12.8 (D) 10.6.
15 15 5
65. The mode of a distribution of 13 and its mean is 4, then its
1 2
P(Y1 = yi) median is
3 3 (A) 7 (B) 9 (C) 8 (D) 11.

Chapter 05.indd 124 8/28/2015 7:50:01 PM


Chapter 5 Probability and Statistics | 2.125

66. Consider the non-decreasing series of the numbers, (A) M (B) 100 + M
1, 8, 8, 13, 14, 14, x, y, 18, 20, 31, 34, 38 and 40. If the median (C) 100 M (D) M 100.
of the series is 15, then the mode of the series is 69. If the standard deviation of 10, 20, 30, 40 and 50 is S, then the
(A) 14 (B) 16 standard deviation of 20, 30, 40, 50 and 60 is
(C) 18 (D) cannot be determined. (A) S (B) S + 10 (C) S 10 (D) 10 S.
67. The standard deviation of 5, 5, 5, 5, 5, 5, 5, 13 is 70. The arithmetic mean of five observations is 6.4 and the vari-
(A)
2 2 (B) 6 (C) 5 (D) 7 . ance is 8.24. If three of the observations are 3, 4, 8, then find
the other two observations.
68. If the standard deviation of 1, 2, 3, 4, 5, 6, 7, 8, 9, 10, 11 is
(A) 6, 11 (B) 10, 7 (C) 8, 9 (D) 5, 12.
M, then the standard deviation of 101, 102, 103, 104, .. and
111 is

Practice Problems 2 11. The point (a, b) is chosen on the xy plane such that a2 +
Directions for questions 1 to 70: Select correct alternative from b2 < 4a. Find the probability that a2 + b2 < 4.
the given choices. 2p 3 3 4p 3 3
(A) (B)
1. A committee of three is to be chosen from a group of three 6p 2p
men and four women. The probability that the committee 4p 3 3 2p 3 3
contains only women is (C) (D)
6p 2p
(A) 4/35 (B) 3/35 (C) 31/35 (D) 34/35.
12. A sugar particle is floating in the air, a fly roams around it, it
2. There are 25 bulbs, out of which two are fused. If two bulbs
doesnt move more than 10cm from it. Find the probability
are drawn at random, the probability of drawing one good and
that it is at a distance d cm, from the sugar particle, where
one fused bulb is
6 d 10.
(A) 7/13 (B) 5/13 (C) 23/150 (D) 7/15.
98 57 97 75
3. A two-digit number is formed by using the digits 0, 2, 3, 5, 8 (A) (B) (C) (D)
125 81 125 127
without repetition. If a two digit number is selected at random
out of these numbers, then the probability that it is a multiple 13. A bag has 5 red balls and 4 blue balls. Three balls are drawn
of 2 is in succession without replacement.
(A) 5/8 (B) 3/8 (C) 7/8 (D) 1/8. Event A: getting a red ball the first time.
Event B: getting a blue ball the second time.
4. The probability of getting at least one 4 when two unbiased
Event C: getting a red ball the third time.
dice are thrown together is
Which of the following is true?
(A) 5/18 (B) 11/36 (C) 1/3 (D) 13/36.
(A) A and B are independent.
5. If two balls are drawn at random from a bag containing four (B) A and C are independent.
green balls and three red balls, the odds against the two balls (C) A, B, C are pairwise independent.
being green is (D) A, B, C are dependent events.
(A) 3 : 5 (B) 2 : 5 (C) 5 : 2 (D) 5 : 3.
14. Three mountaineers Akil, Nikil, and Sunil are climbing up
6. If a card is picked at random from a well-shuffled pack of a mountain with their respective probability of reaching the
cards, what is the probability that it is neither a king nor a 2 5 4
summit being , and , respectively. What is the probabil-
spade? 3 8 7
(A) 4/13 (B) 11/13 (C) 2/13 (D) 9/13 ity that none of them reaches the summit?
7. A person gets as many rupees as the number shows up when 1 3 5 3
an unbiased 6-faced dice is rolled. If two dice are rolled, the (A) (B) (C) (D)
14 56 56 14
probability that the person gets H9 is
(A) 5/36 (B) 1/9 (C) 1/6 (D) 1/12. 15. Three mountaineers Akil, Nikil, and Sunil are climbing up
a mountain with their respective probability of reaching the
8. If A and B are two independent events such that P(A) = 2/3
2 5 4
and P( B ) = 1/5, then find P( A B ). summit being , and respectively. What is the probabil-
3 8 7
(A)8/15 (B)4/15 (C) 7/15 (D) 11/15 ity that exactly two of them reaches the summit?
9. If A and B are two independent events such that 37 5 19 6
(A) (B) (C) (D)
P(A B) = 0.75 and P(B) = 0.25, then P(A) = 84 12 28 17
(A) 2/3 (B) 1/3 16. Three mountaineers Akil, Nikil, and Sunil are climbing up
(C) 1/2 (D) none of these. a mountain with their respective probability of reaching the
10. S = {1, 2, 3, 4, 5}. Three subsets of S, viz. A, B, C are con- 2 5 4
summit being , and respectively. What is the probabil-
structed, each containing three elements from S. It is known 3 8 7
ity that atleast two of them reaches the summit?
that n(A B) = 4. Find the probability that n(A B C) = 5.
1 3 5 3 5 3 37 19
(A) (B) (C) (D) (A) (B) (C) (D)
5 7 7 5 21 56 84 28

Chapter 05.indd 125 8/28/2015 7:50:12 PM


2.126|Engineering Mathematics

17. The probability of drawing a blue and an orange ball in suc- 28. Avinash picks a number from the numbers 1 to 25 and found
cession in that order from a bag containing 6 blue and 4 it to be an even number. What is the probability that the num-
orange balls, when the ball that is drawn first is not replaced is ber is 12?
11 4 4 3 1 12 1 1
(A) (B) (C) (D) . (A) (B) (C) (D)
15 15 9 5 13 25 12 6
18. The probability of drawing a blue and an orange ball in 29. Neelu picked up a letter from the English alphabet and found
succession in that order from a bag containing 6 blue and it to be a consonant. What is the probability that it is letter C?
4 orange balls, when the ball that is drawn first is replaced is 1 21 5
1
4 2 3 6 (A) (B) (C) (D)
(A) (B) (C) (D) . 21 26 26 7
25 25 25 25
30. If an unbiased coin is tossed five times, then the probability of
19. A bag contains 7 red and 3 blue balls and another bag contain getting exactly four heads is
6 blue and 4 red balls. If one of the bags is selected at random 5 1 1 3
and two balls are drawn at random from the bag thus selected, (A) (B) (C) (D) .
32 8 32 26
the probability that the two balls are of same colour is
8 1 7 31. Eight letters are to be placed in eight addressed envelopes. If
(A) 1 (B) (C) (D) . the letters are placed at random into the e nvelopes, the prob-
15 2 15
ability that at least seven letters are placed into the right enve-
20. The odds against an event are 4 to 5 and the odds in favour of lopes is
another independent event are 3 : 7. The probability that none 1 9
of them occur is (A) (B) (C) 1 (D) 0.
8! 8!
14 31 7 8
(A) (B) (C) (D) . 32. A fair die is rolled four times and the sum of four numbers is
45 45 15 15 20. Find the probability that the first throw was 6.
21. The odds against an event are 4 to 5 and the odds in favour 10 12 5 3
(A) (B) (C) (D)
of another independent event are 3 : 7. The probability that at 23 23 23 7
least one of them occurs is
33. If a number is selected randomly from the natural numbers
11 13 31 21
(A) (B) (C) (D) . 1 to 30, then the probability that the number is divisible by
32 61 45 45 either 4 or 7 is
22. In a biased coin, head occurs three times as frequently as tail 2 7 11 1
(A) (B) (C) (D) .
occurs. If the coin is tossed 3 times, what is the probability of 5 15 30 3
getting two heads?
34. For a discrete random variable X with cumulative distri-
3 3 9 27 1 5
(A) (B) (C) (D) bution function F(X ), such that F(1) = and F(3) = ,
32 64 64 64 P[1 < X 3] = 4 7
23. When four dice are rolled, the probability that the total score 1 5 13 11
on the four dice is maximum is (A) (B) (C) (D)
4 7 28 28
1 1 1 5 35. If x is a continuous random variable with p.d.f. f(x) then,
(A) (B) (C) (D) .
1296 216 432 1296

24. In the month of February of a non-leap year, the probability




x . f ( x ) dx =

that it will have 5 Saturdays is (A) 0 s (D)


(B) (C) m.
1 6
(A) (B) 1 (C) (D) 0. 36. If f(x) = Cx (1 x); 0 < x < 1 defines a probability density
2

7 7 function, then the constant C =


25. When two fair coins are tossed together, what is the probabil- 1 1
ity that they both show the same face?
(A) (B)
12 12
(A) 1/2 (B) 1/3 (C) 1/4 (D) 1
1

(C) (D) 12.
26. If a dice is rolled two times, what is the probability of the 4
product of the numbers obtained, being 12? 37. If x is a continuous random variable with probability density
(A) 1/9 (B) 8/9 (C) 5/36 (D) 1/6 function given by
27. A and B appear for an interview. The probability that A is f(x) = k.x2 (0 x < 2)
short listed in the interview is 3/7 and that of B is 1/7. The = 2k (2 x 4),
probability that both are short listed for the interview is then the value of k is _______.
(A) 24/49 (B) 4/7 (C) 1/49 (D) 3/49. (A) 3/20 (B) 3/8 (C) 1/4 (D) 5/24

Chapter 05.indd 126 8/28/2015 7:50:31 PM


Chapter 5 Probability and Statistics | 2.127

e 2 x x 0 48. For a uniformly distributed random variable x in [2, 3], the


38. f(x) = is a probability density function, then value of E(x3) is
0 x<0
(A) 0.125 (B) 3.25
P(1 x 3) is
(C) 6.50 (D) 4.75.
e -2 - e -6 e -2 e -6 49. Suppose the life expectancy x of a mobile battery (in years)
(A) (B) +
2 2 6 is exponentially distributed with a variance of 36, then the
probability that the life expectancy of the mobile battery will
e -2 e -6 lie between 7 years and 11 years is
e2 + e6 (D)
(C) -
2 6 (A) 0.1717 (B) 0.1514
39. A 12 faced fair die is rolled. A random variable x is defined (C) 0.8283 (D) 0.8486.
as The number appeared on the bottom of the die. Then the 50. If X is a normal variate with mean m and standard deviation s,
variance of x is X -m
then is a normal variate with variance
(A) 10.35 (B) 11.92 (C) 12.5 (D) 13.2. s
m
(A) m (B)
Linked answer for Questions 40 and 41: The mean and variance s
of a binomial distribution are 8 and 8/3 respectively. (C) 1 (D) 0.
40. The number of trails is ______. 51. If X is a normal variate with mean 6 and variance 49 and if
(A) 15 (B) 18 (C) 12 (D) 21 X -6
Y = , then the mean of Y is
7
41. P(x 1) = ______. 6
(A) 6 (B)
1 1 7
(A) 1 (B) 1
3 -12 312 (C) 1 (D) 0.
1 1
(C) 12 (D)
3 512 52. Let X1 and Y1 be two discrete random variables with a joint
probability mass function as given below:
42. For which of the following distributions, the mean and vari-
ance are equal?
X1
(A) Normal (B) Binomial 2 7 P(X1 = xi)
Y1
(C) Poisson (D) Negative Binomial
43. If X is a random variable which follows Poisson distribution 3 0.16 0.24 0.4
with mean 16, then the co-efficient of variation is given by 4 0.22 0.28 0.5
(A) 25 (B) 50 (C) 75 (D) 100.
6 0.02 0.08 0.1
44. The relation between the mean of the binomial vari-
ate and the Poisson variate (l) is (let n = number of trials, P(Y1 = yj) 0.4 0.6
p = probability of success)
(A) l = np (B) l = 2np
Let X2 and Y2 be two discrete random variables with a joint
np probability mass function as given below:
l=
(C) (D) 3l = np.
2
X2
Linked answer for Questions 45 and 46: A random variable has -1 0 +1 P(X2 = xi)
Y2
a Poisson distribution such that P(x = 2) = P(x = 3).
3 0.2 0.12 0.08 0.4
45. The value of mean is _______.
(A) 1 (B) 2 (C) 3 (D) 8 6 0.2 0.12 0.08 0.4

8 0.1 0.06 0.04 0.2


46. The value of P(x = 5) is ______.
P(Y2 = yj) 0.5 0.3 0.2
e -1 8 5 e -8
(A) (B)
5! 5! Consider the two statements,
25 e -2 35 e -3 P1: X1 and Y1 are independent random variables
(C) (D)
5! 5! P2: X2 and Y2 are independent random variables.
5 5 Which of these two statements is/are TRUE?
47. If X is a uniformly distributed random variable in , (A) P1 only (B) P2 only
3 3
then, P(X 1) is (C) Both P1 and P2 (D) Neither P1 nor P2
53. If X and Y are two independent random variables, then which
(A) 0.4 (B) 0.6 (C) 0.7 (D) 0.8. of the following is/are TRUE?

Chapter 05.indd 127 8/28/2015 7:50:37 PM


2.128|Engineering Mathematics

(I) Covariance of (X, Y ) = 0 62. The median of 5, 5, 5, 5, 12, 12, 12, 13, 13, 14 is
(II) Variance of (X + Y ) = variance of X + variance of Y. (A) 13 (B) 5 (C) 12 (D) 12.5.
(A) I only (B) II only
63. If the mean of 100 observations is 50 and their standard devi-
(C) Both I and II (D) Neither I nor II
ation is 4, then the sum of the squares of the observations is
54. If X and Y are two uniformly distributed random vari- (A) 216500 (B) 251600
ables in the interval [1, 7] that are independent, then (C) 256100 (D) 261500.
3 9 7 13 64. The average of a set of 120 observations is 20. If the average
P X , Y is
2 4 3 3 of 80 of these observations is also 20, then the average of the
1 1 1 1 remaining 40 observations is
(A) (B) (C) (D) .
6 12 24 48 (A) 20 (B) 40 (C) 10 (D) 30.

55. Two fair dice are rolled simultaneously. If X and Y are two 65. There are 100 employees in an organisation. The average
random variables defined as X = the number on the first die wage of 40 of these employees is H2000 per month and
and Y = the number on the second die, then the value of the average wage of the remaining employees is H3000 per
P(X 4/X + Y = 9) is month. Then the average wage of all the 100 employees per
month is
3 1 1 1
(A) (B) (C) (D) . H2600 (B)
(A) H2800 (C)
H3000 (D)
H2500.
4 2 3 4
66. a1, a2, , an are n distinct real numbers such that, a1 < a2
56. For a symmetric distribution, which of the following is true? < a3 < .. < an and the arithmetic mean of a1, a2, .., an is M.
(A) mode = 2 median 3 mean Let bi = max {a1, a2. ., ai}. Then the arithmetic mean of b1,
(B) mode = 3 median + 2 mean b2 , bn is
(C) mean = median = mode
M
(D) mean = mode + median (A) 2M (B) M (C) (D) M 2.
2
57. If the arithmetic mean of x1, x2, ., xn is A, then the arithmetic 67. Which of the following can be the range of the values of the
mean of x1 + a, x2 + a, .., xn + a is median for the series 14, 12, 23, x, 15, 29, 5?
A + a (B)
(A) A a (C)
a . A (D)
A. (A) [12, 14] (B) [14, 15]
58. The arithmetic mean of the first n natural numbers is8. Then (C) [15, 29] (D) [13, 18].
n= 68. The median of a set of 21 observations is 50. If 2 is subtracted
(A) 16 (B) 15 (C) 14 (D) 17. from each of the observations, then the median of the new set
59. The arithmetic mean of the squares of first n natural numbers of observations is
is 11, then n = (A) 25 (B) 52 (C) 50 (D) 48.
(A) 4 (B) 5 (C) 6 (D) 7. 69. If the standard deviation of the series x1 + 2, x2 + 2, .. xn + 2
60. If the difference between median and mean of a moderately is s, then the standard deviation of x1 2, x2 2, xn 2
symmetric distribution is 8, then the difference between mode is
s
and mean is (A) s (B) s 2 (C) s + 2 (D) .
2
(A) 8 (B) 24 (C) 4 (D) 16.
61. If the sum of the squares of eight observations is 160 and 70. The standard deviation of 7, 7, 7, 7, 7, 7, 7, 7, 7, 7 and 18 is
their mean is 4, then their standard deviation is (A) 10 (B) 2 2 (C) 10 (D) 2 5.
(A) 2 (B) 4 (C) 6 (D) 8.

Previous Years Questions


7 42
1. In the following table, X is a discrete random variable and (A) (B)
P(x) is the probability density. The standard deviation of 20 125
X is [2014] 25 5
(C) (D)
29 9
X 1 2 3 3. Consider an unbiased cubic die with the opposite faces col-
P(x) 0.3 0.6 0.1 oured identically and each face coloured red, blue or green
such that each colour appears only two times on the die.
(A) 0.18 (B) 0.36 If the die is thrown thrice, the probability of obtaining red
(C) 0.54 (D) 0.6. colour on the top face of the die at least twice is _____.
2. A box contains 25 parts of which 10 are defective. Two parts  [2014]
are being drawn simultaneously in a random manner from 4. A group consists of equal number of men and women. Of
the box. The probability of both the parts being good is these 20% of the men and 50% of the women are unem-
 [2014] ployed. If a person is selected at random from these, the

Chapter 05.indd 128 8/29/2015 10:39:15 AM


Chapter 5 Probability and Statistics | 2.129

probability of the selected person being employed is 13. If three coins are tossed simultaneously, the probability of
________. getting at least one head is [2009]
5. A machine produces 0, 1 or 2 defective pieces in a day with
an associated probability of 1/6, 2/3 and 1/6, respectively. (A) 1/8 (B) 3/8 (C) 1/2 (D) 7/8.
The mean value and the variance of the number of defective 14. The standard deviation of a uniformly distributed random
pieces produced by the machine in a day respectively, are variable between 0 and 1 is [2009]
 [2014]
(A) 1 and 1/3 (B) 1/3 and 1 1 1 5 7
(A) (B) (C) (D) .
(C) 1 and 4/3 (D) 1/3 and 4/3. 12 3 12 12
6. A nationalised bank has found that the daily balance avail- 15. A coin is tossed 4 times. What is the probability of getting
able in its savings accounts follows a normal distribution heads exactly 3 times? [2008]
with a mean of H500 and a standard deviation of H50. The
percentage of savings account holders, who maintain an 1 3 1 3
(A) (B) (C) (D)
average daily balance more than H500 is _____. [2014] 4 8 2 4
7. The number of accidents occurring in a plant in a month
16. Let X and Y be two independent random variables. Which
follows Poisson distribution with the mean as 5.2. The prob-
of the relations between expectation (E), variance (Var) and
ability of occurrence of less than 2 accidents in the plant
covariance (Cov) given below is FALSE? [2007]
during a randomly selected month is [2014]
(A) E(XY) = E(X )E(Y )
(A) 0.029 (B) 0.034
(B) Cov(X, Y) = 0
(C) 0.039 (D) 0.044.
(C) Var(X + Y ) = Var(X ) + Var(Y )
8. Let X be a normal random variable with mean 1 and vari- (D) E(X 2Y 2) = (E(X ))2 (E(Y ))2
ance 4. The probability P{X<0} is [2013]
17. A box contains 20 defective items and 80 non defective
(A) 0.5
items. If two items are selected at random without replace-
(B) greater than zero and less than 0.5
ment, what will be the probability that both the items are
(C) greater than 0.5 and less than 1.0
defective? [2006]
(D) 1.0.
1 1 20 19
9. The probability that a student knows the correct answer to a (A) (B) (C) (D)
2 5 25 99 495
multiple choice question is . If the student does not know
3 18. Consider a continuous random variable with a probability
the answer, then the student guesses the answer. The prob- density function f(t) = 1 + t for 1 t 0
1
ability of the guessed answer being correct is . Given that = 1 t for 0 t 1.
4
the student has answered the question correctly, the condi- The standard deviation of the random variable is [2006]
tional probability that the student knows the correct answer 1 1 1 1
is [2013] (A) (B) (C) (D) .
3 6 3 6
2 3 5 8
(A) (B) (C) (D) .
3 4 6 9 19. A lot has 10% defective items. Ten items are chosen ran-
10. A box contains 4 red balls and 6 black balls. Three balls are domly from this lot. Then the probability that exactly 2 of
selected randomly from the box one after the other, without the chosen items are defective is[2005]
replacement. The probability that the selected set contains (A) 0.0036 (B) 0.1937
one red ball and two black balls is [2012] (C) 0.2234 (D) 0.3874.
20. A single die is thrown twice. What is the probability that the
(A) 1/20 (B) 1/12 (C) 3/10 (D) 1/2. sum is neither 8 nor 9? [2005]
11. An unbiased coin is tossed five times. The outcome of each 1 5 1 3
toss is either a head or a tail. The probability of getting at (A) (B) (C) (D)
9 36 4 4
least one head is [2011]
1 13 16 31 21. From a pack of regular playing cards, two cards are drawn
(A) (B) (C) (D) .
32 32 32 32 at random. What is the probability that both the cards will
be Kings, if the first card is NOT replaced?
12. A box contains 2 washers, 3 nuts and 4 bolts. Items
[2004]
are drawn from the box at random one at a time without 1 1
(A) (B)
replacement. The probability of drawing 2 washers first fol- 26 52
lowed by 3 nuts and subsequently the 4 bolts is [2010]
1 1
(A) 2/315 (B) 1/630 (C) (D)
169 221
(C) 1/1260 (D) 1/2520.

Chapter 05.indd 129 8/28/2015 7:50:53 PM


2.130|Engineering Mathematics

Answer Keys
Exercises
Practice Problems 1
1.C 2.C 3.C 4.B 5.A 6.D 7.D 8.B 9.C 10.D
11.A 12.(i)D(ii)A(iii)B 13.C 14.A 15.B 16.D 17.B 18.A 19.A
20.A 21.C 22.A 23.C 24.C 25.B 26.B 27.B 28.B 29.B
30.C 31.B 32.C 33.B 34.C 35.C 36.C 37.A 38.D 39.D
40.A 41.B 42.D 43.C 44.A 45.B 46.D 47.B 48.D 49.C
50.D 51.C 52.C 53.A 54.C 55.C 56.C 57.C 58.A 59.C
60.C 61.C 62.A 63.C 64.B 65.A 66.D 67.D 68.A 69.A
70.A

Practice Problems 2
1.A 2.C 3.A 4.B 5.C 6.D 7.B 8.C 9.A 10.D
11.C 12.A 13.D 14.B 15.A 16.D 17.B 18.D 19.C 20.A
21.C 22.D 23.A 24.D 25.A 26.A 27.D 28.C 29.A 30.A
31.A 32.D 33.D 34.C 35.D 36.A 37.A 38.A 39.B 40.C
41.B 42.C 43.A 44.A 45.C 46.D 47.D 48.B 49.B 50.C
51.D 52.B 53.C 54.C 55.B 56.C 57.A 58.B 59.B 60.B
61.A 62.C 63.B 64.A 65.A 66.B 67.B 68.D 69.A 70.C

Previous Years Questions


1.D 2.A 3.0.25 to 0.27 4. 0.64 to 0.66 5.A 6. 49 to 51
7.B 8.B 9.D 10.D 11.D 12.C
13.D 14.A 15.A 16.D 17.D 18.B
19.B 20.D 21.D

Chapter 05.indd 130 8/28/2015 7:50:53 PM


Chapter 6
Numerical Methods
LEARNING OBJECTIVES

After reading this chapter, you will be able to understand:


Numerical Methods Solutions of Linear System of Equations
Methods for Finding the Real Roots Gauss Elimination Method
Bisection Method Gauss-Jordan Elimination Method
Regula Falsi Method Numerical Integration
The Secant Method Simpsons Three Eight Rule
Newton-Raphson Method

nuMerical Methods We start with an initial approximate value, say x0, and then find
the better approximations successively x1, x2, x3.., xn by repeating
We encounter problems in Engineering Mathematics for which
the same method.
analytical methods are not available to find solutions. Further, it
If the successive approximations at each step of a method approach
may be sufficient in engineering applications to find approximate
the root more and more closely, we say that the method converges.
solutions. The numerical methods offer us approximate solutions
1. Method for finding zeros an algebraic or transdental The intermediate value theorem
equations If a function f (x) is continuous between a and b and f (a) and
2. Solutions to system of linear equation f (b) are of opposite signs, then there exists at least one root say a
3. Numerical Integration between a and b of the equation
4. Numerical solutions of ordinary differential equation. f (x) = 0, such that f (a) = 0
Note 1: Root a of f (x) = 0, will be unique in (a, b) if f (x) has
Methods for Finding the Real Roots (Zeros) the same sign in (a, b)
of f(x) = 0 (i.e. f (x) > 0 or f (x) < 0 in a < x < b)
The equation of the form f(x) = 0 is called an Algebraic or
Transcendental if f(x) is purely a polynomial in x or contains some Relations between roots and coefficients
other functions such as exponential, logarithmic and trigonometric An nth order equation has n roots. Corresponding to every root,
functions, etc. there is a factor. If a is a root of f(x) = 0, then x - a is a factor of
That is, f(x). Sometimes (x - a)2 may also be a factor. In such a case, a is
1. x9 + 8x5 - 4x3 - 11x + 3 = 0 Algebraic equation said to be a double root. Similarly, equations can have triple roots,
2. 10x4 - log (x2 - 3) + e-xsin x + tan2 x = 0 Transcendental quadruple roots and roots of multiplicity m. If m is the greatest
equation value of k, for which (x - a)k is a factor of f(a), then a is said to
be a root of multiplicity m. If all the roots are counted by taking
In this chapter, we obtain the solution of an equation f (x) = 0, their multiplicity into account, the number of roots is equal to n,
i.e. we mean to find the zeros of f (x). the degree of the equation.
We shall now discuss a few methods to find the real roots of both
algebraic (with numerical coefficients) and transcendental equations. If a1, a2, .., an (not necessarily distinct) are the roots of
We first find an approximate value of the root of the given equa- f(x) = 0, then
tion, and then successively improve it to some desired degree of f(x) = an(x - a1) (x - a2) . (x - an)
accuracy. = an[xn - S1 xn-1 + S2xn-2..+ (-1)n Sn]

Chapter 06.indd 131 8/28/2015 7:45:57 PM


2.132|Engineering Mathematics

where S1 = the sum of the roots The second root results in a second sign change [x2 -
S2 = the sum of the products of the roots taken 2 at a time (a1 + a2)x + a1 a2 has 2 sign changes] and so on.
S3 = the sum of the product of the roots taken 3 at a time But every sign change need not correspond to a real pos-
and so on. itive root. (For example x2 - 2x + 4 has two sign changes but
Sn = the sum of the product of the roots taken n(or all) the corresponding equation x2 - 2x + 4 = 0 has no real roots.
at a time. Thus, Sn is a single term. The number of positive roots is at the most equal to the
number of sign changes. It could also be less than that by
Sn = a1 a2 ..an
2, 4 i.e., if there are k sign changes in f(x), the number of
Let us write down the polynomial f(x) in two forms: positive roots could be k, k-2, k-4,
The standard form This is called Descartes Rule of Signs. This rule can be
f(x) = an xn + an-1 xn-1 + an-2 xn-2 + a1x + a0 extended to negative roots as follows. The number of nega-
tive roots of f(x) = 0 is equal to the number of positive roots
In terms of the roots of the corresponding equation.
of g(x) = f(-x) = 0
f(x) = an [xn - S1xn-1 + S2xn-2 + .+ (-1)x-1 Sn-1x + (-1)nSn] For example, consider
These polynomials are identically equal, i.e., equal for f(x) = x5 - 3x3 + 6x2 - 28x + 24.There are 4 sign changes
all values of x. Therefore, the corresponding coefficients are in f(x)
equal. The sum of the roots S1 = - an-1/an The number of positive roots could be 4, 2 or 0,
The sum of the products of the roots, taken two at a time, Consider g(x) = f(-x)
S2 = an-2 /an (-x)5 - 3(-x)3 + 6(-x)2 - 28 (-x) + 24
The sum of the products of the roots, taken three at a
= -x5 + 3x3 + 6x2 + 28x + 24
time, S3 = -an-3 /an and so on.
The sum of the products of the roots taken m (m n) There is only one sign change in f(- x).
a The number of negative roots of f(x) = 0 is 1. (It cant
at a time Sm = a1 a2 a3.. am = (-1)m n m be -1, -3, )
an
a The following table shows the various possibilities for
Sn = a1 a2 a3..an = (-1)n 0 the roots.
an
Negative Positive Complex
For example, consider the polynomial equation
1 4 0
(x - 1)(x - 2)(x - 3) = x3 - 6x2 + 11x - 6 = 0 1 2 2
(We can see immediately that the roots are 1, 2, 3) 1 0 4
The sum of roots = (1 + 2 + 3) = - (-6)/1
The sum of the products of the roots, taken two at a time We have considered one specific equation and this specific
S2 = 1(2) + 1(3) + 2(3) = 11 = 11/1 equation has 5 specific roots. We can use more advanced
We can drop the word sum and products for the last techniques to find the actual roots. But even without that,
relation, because there is only one term (only one product). using only Descartes Rule, we expect exactly one of the 3
The product = 1(2)(3) = 6 = -(-6)/1. situations shown in the table above.

Roots of equations and descartes rule Solved Examples


If the coefficients are all real and the complex number z1, is
Example 1: Find the nature of roots of the equation, f(x) =
a root of f(x) = 0, then the conjugate of z1, viz, z 1 is also a x3 + x - 2 = 0.
root of f(x) = 0. Thus, for equations with real, coefficients,
Solution: There is only 1 change of sign in f(x). We know
complex roots occur in pairs. that when f(x) has r changes of sign then f(x) has r, r - 2,
A consequence of this is that any equation of an odd r - 4, .. positive roots.
degree must have at least one real root. f(x) = 0 has one positive root.
The number of roots is related to very simple properties Now, f(-x) -x3 - x - 2 = 0. q = 0
of the equation as illustrated below. As there is no change of sign in f(-x), f(x) has no negative
Let a1 be a positive root, ie x - a1, is a factor. roots. The number of complex roots is even.
Let a2 be another positive root, i.e., x2 - (a1 + a2)x + a1 The equation has one positive root, and two complex roots.
a2 is a factor. Hence, f(x) = 0 has 1 real root and two complex roots.
Let a3 be another positive root ie x3 - (a1 + a2 + a3)x2 +
Example 2: How many non-real roots does the equation
(a1 a2 + a2 a3 + a3 a1) x - a1 a2 a3 is a factor
x4 - 2x2 + 3x - 2 = 0 have?
We note that every positive root introduces a sign change
in the polynomial. For 1 root, there is 1 sign change (the Solution: Let f(x) = x4 - 2x2 + 3x - 2
coefficient of x is positive and -a1 is negative) f(x) has 3 sign changes

Chapter 06.indd 132 8/29/2015 11:03:25 AM


Chapter 6 Numerical Methods | 2.133

f(x) has 3 or 1 positive roots. 0.25 + 0.5 0.75


f(- x) = x4 - 2x2 - 3x - 2 The third approximation to the root is =
2 2
f(- x) has one sign change = 0.375.
f(x) has exactly one negative root. Now, f (0.375) = (0.375)3 - 2 (0.375)2 + 3 (0.375) - 1 =
As the sum of the co-efficient of f(x) is zero, -0.103 < 0 and f (0.5) > 0
x = 1 is a root f f(x) = 0 The root lies between 0.375 and 0.5.
0.375 + 0.5
f(x) = (x - 1) (x3 + x2 - x + 2) = (x - 1) f1(x). By trial, f1(-2) = 0 The fourth approximation to the root is =
f1(x) = (x + 2) (x2 - x + 1) 0.875 2
= 0.4375
We can see that x2 - x + 1 = 0 has two non real roots. 2
 f(x) has one positive, one negative and two non-real roots.
Example 3: If p - q, p, p + q are the roots of the equation Convergence of Bisection Method
x3 - 18x2 + 99x - 162 = 0, then find the values of p and q. If x1, x2, x3.., xn is the sequence of midpoints obtained by
ba
Solution: Given p - q, p, p + q are the roots of the equation. bisection method, then |c - xn| n , n = 1, 2, 3where
The sum of the roots is (p - q) + p + (p + q) = 18 2
c is between a and b
3p = 18 p = 6 Note: In bisection method, the convergence is very slow but
and the product of the roots is (p - q) p (p + q) = 162 definite. The order of convergence is linear or of the first order.
162
p2 - q2 = = 27 36 - q2 = 27 Regula Falsi Method or (The Method of
6
q = 3 p = 6 and q = 3 False Position)
In this method, to find the real root of the equation f(x) = 0,
Bisection Method (Bolzano Method) we consider a sufficiently small interval (a, b), a < b such that
f (a) and f(b) will have opposite signs. This implies a root
or (Halving Method) lies between a and b according to intermediate value theo-
Consider the equation f (x) = 0 (1) rem. Also, the curve y = f(x) will meet the x-axis at some
If f (x) is continuous between a and b and f(a) f(b) > 0, point between A [a, f(a)] and B [b, f(b)]. The equation of the
then there exists one root between a and b. Let f(a) be chord joining A [a, f(a)] and B [b, f (b)] is given by
negative and f(b) be positive. The bisection method isolates f ( b) f ( a)
the root in [a, b] by halving process, approximately dividing y - f (a) = (x - a)  (1)
ba
the given interval [a, b] into two, four, eight, etc. equal parts.
The point of intersection of the chord (1) with x-axis is
Thus, the first approximation to the root is given by
given by y = 0 in (1)
a+b
x0 = f ( b) f ( a) af (b) bf ( a)
2 -f (a) = (x - a), x =
ba f ( b) f ( a)
ab
af (b) bf ( a)
a+b The first approximation x1 =  (2)
f ( b) f ( a)
2
If f(x1) = 0, then x1 is the root. If f(x1) 0 and if f(x1)
If f (x0) = 0, then x0 is a root, otherwise the root lies either
and f(a) have opposite signs, the second approximation
between a and x0 or x0 and b depending on whether f (x0) is
af ( x1 ) x1 f ( a)
positive or negative. We again bisect the interval and repeat x2 =  (3)
the process until the root is obtained to desired accuracy. f ( x1 ) f ( a)
Proceeding in the same way, we get x3, x4 and so on.
Example 4: Find a real root of the equation f (x) = x3 - 2x2 Geometrically, the required root is shown in the figure below.
+ 3x - 1 on the interval (0, 1) using bisection method with
y A[a,f(a)]
four iterations.
Solution: We have f (0) = -1 < 0 and f (1) = 1 - 2 + 3 - 1 = 1 > 0
A root lies between 0 and 1
0 +1
The first approximation to the root is = 0.5. Now,
2
f (0.5) = (0.5) - 2 (0.5) + 3 (0.5) - 1 = 0.125 > 0 and f (0) < 0
3 2
x3 x2 x1
x
O a b
The root lies between 0 and 0.5. The second approximation
0 + 0.5 f
to the root is = 0.25 e
2
Now, f (0.25) = (0.25)3 - 2 (0.25)2 + 3 (0.25) - 1 = -0.359 d
< 0 and f (0.5) > 0 B[b,f(b)]
The root lies between 0.25 and 0.5. y = f ( x)

Chapter 06.indd 133 8/29/2015 11:14:59 AM


2.134|Engineering Mathematics

Note: This method is faster than the first order fixed point The best approximation to the root upto three decimal
iteration. places is 0.768

Convergence of Regula-Falsi Method Newton-raphson Method


The order of convergence of the method of false position is Let x0 be the approximate root of f (x) = 0 and let x1 = x0 + h
greater than 1. be the correct root. Then, f (x1) = 0
The Secant Method f (x0 + h) = 0 (1)
This method is quite similar to that of Regula-Falsi method Expanding (1) using Taylors theorem
except for the condition f (a) f (b) < 0. The interval at each we get f (x0) + hf1 (x0) + ..= 0
iteration may not contain the root. Let the initial limits of f ( x0 ) f ( x0 )
the interval be a and b. h= , x1 = x0 -
f ( x0 ) f ( x0 )
The formula for successive approximation general form is
Now, x1 is the better approximation than x0. Proceeding this
( x xn 1 ) f ( xn )
xn+1 = xn + n way, the successive approximations x2, x3xn+1 are given by
f ( xn 1 ) f ( xn )
f ( xn )
In case at any stage f (xn) = f (xn-1) the method fails. xn+1 = xn -
f ( xn )
Note: This is called Newton-Raphson formula.
1. This method does not converge always, but Regula-
Falsi method always converges. Geometrical Interpretation of
2. If it converges, it converges with order 1.62
Newton-Raphson Formula
approximately, which is more repidly than the Regula-
Falsi method. y
y = f(x)
Example 5: Find a root for 2exsin x = 3 using Regula-Falsi
method and correct to three decimal places with three iterations.
Solution: Let f (x) = 2exsin x - 3
f(x0)
f (0) = -3 < 0, f (1) = 2e1sin 1 - 3
= 1.5747 > 0
A root lies between 0 and 1.
Here, a = 0 and b = 1
The first approximation f(x1)
af (b) bf ( a) 0(1.5747) 1( 3)
x1 = = f(x2)
f ( b) f ( a) 1.5747 ( 3)
x
3 O x2 x1 x0
= = 0.6557
4.5747
The geometrical meaning of Newtons Raphson method is a
Now, f (0.6557) = 2e0.6557sin (0.6557) - 3
tangent is drawn at the point [x0, f(x0)] to the curve y = f(x).
= - 0.6507 < 0 and f (1) > 0
It cuts the x-axis at x1 which will be a better approxima-
The root lies between 0.6557 and 1. The second
tion of the root. Now, drawing another tangent at [x1, f(x1)]
approximation x2
which cuts the x-axis at x2 which is a still better approxi-
(0.6557)(1.5747) 1( 0.6507) mation and the process can be continued till the desired
=
1.5747 ( 0.6507) accuracy is achieved.
1.0325 + 0.6507 1.6832
= = = 0.7563 Convergence of Newton-Raphson Method
2.2254 2.2254
The order of convergence of Newton-Raphson method is 2
Now, f (0.7563) = -0.0761 < 0 and f (1) > 0 or the convergence is quadratic. It converges if |f (x). f (x)|
The root lies between 0.7563 and 1 < |f(x)|2. Also this method fails if f(x) = 0
The third approximation to the root x3
(0.7653)(1.5747) 1( 0.0761) Newtons iterative formula to find bth
=
1.5747 ( 0.0761) root of a positive real number a
1.1909 + 0.0761 1 a
= = 0.7675 The iterative formula is given by xn+1 = (b 1) xn + b 1
1.6508 b xn

Chapter 06.indd 134 8/28/2015 7:46:07 PM


Chapter 6 Numerical Methods | 2.135

Newtons iterative formula to find a Example 8:Find the reciprocal of 24 using Newton-
reciprocal of a number N Raphson method.
1
The iterative formula is given by Solution: The iterative formula to find is
N
xi+1 = xi (2 - xiN) xn+1 = xn (2 - xnN)
Let x0 = 0.041. Then, x1 = x0 (2 - x0 (24))
Example 6: Find a real root of the equation -4x + cos x + 2 = 0,
by Newton-Raphson method upto four decimal places x1 = (0.041) (2 - (24) (0.041))
assuming x0 = 0.5 = 0.04165
Solution: Let f (x) = -4x + cos x + 2 and x2 = (0.0416) (2 - (24) (0.04165)] = 0.04161, similarly
f (x) = -4 - sin x proceeding we get x3 = 0.041666
Also, f (0) = 1 + 2 = 3 > 0 and f (1) = -4 + cos 1 + 2 The reciprocal of 24 is 0.04166.
= -1.4596 < 0
So, a root lies between 0 and 1. Numerical Integration
Given x0 = 0.5 The numerical integration can be stated as follows:
The first approximation Given a set of (n + 1) data points (xi, yi), i = 0, 1, 2, 3, . n
f ( x0 ) of the funciton y = f (x), where f (x) is not known explicitly,
x1 = x0 - xn
f ( x0 ) it is required to find f ( x )dx.
x0
[ 4(0.5) + cos( 0.5) + 2
= 0.5 - Note: Numerical integration is also known as Numerical
4 sin(0.5) quadrature.
[ 2 + 2 + cos(0.5) Newton-Cotes Quadrature Formula
= (0.5) -
4 sin 0.5 [General Quadrature formula]
0.8775 b
= 0.5 - Consider the integral I = f ( x )dx
4.4794 a
= 0.5 + 0.1958 = 0.6958 Let the interval [a, b] be divided into n equal subintervals of
Example 7:Obtain the cube root of 14 using Newton- width h so that a = x0, x1 = x0 + h, x2 = x0 + 2h..b = x0 + nh
x0 + nh
Raphson method.
I =
x0
f ( x )dx
Solution: We know that, the iterative formula to find b a is
Put x = x0 + mh dx = h.dm as x x0, m 0 and x x0
1 a + nh, m n
xn+1 = (b 1) xn + b 1
b xn n

I = h f ( x0 + mh)dm
Here, b = 3 and a = 14 and let x0 = 2.5 0
Applying Newtons forward interpolation formula
1 14
x1 = 2 x0 + 2 n
m( m 1) 2
3 x0 = h ( y0 + my0 + y0 + ......)dm
0
2!
1 14
x1 = 2( 2.5) + Integrating term by term and applying the limits, we get
3 ( 2.5) 2
x 0 + nh
n n( 2n 3) 2 n( n 2) 2 3
1
= 5 +
14 1 f ( x )dx = nh y0 + y0 +
2 12
y0 +
24
y0 + ....

= {5 + 2.24} = 2.413 x
3 6.25 3
0

n n( 2n 3) 2 n( n 2) 2 3
1 + y0 +
y0 14 y0 + y0 + .... (Newton Cotes quadrature formula)
x2 = 2( 2.413) + 2 12 24
3 ( 2.413) 2
y
1 14 y = f(x)
= 4.826 +
3 5.822569
1
= {4.826 + 2.4044} = 2.410
3 x
The approximate cube root of 14 is 2.41. x0 x0 + h x0 + 2h x0 + nh

Chapter 06.indd 135 8/28/2015 7:46:14 PM


2.136|Engineering Mathematics

On substituting n = 1, 2, 3. in Newton Cotes quadrature x0 + 2 h


1
formula, we get various quadrature formula.
x0
f ( x )dx = 2h( y0 + y0 + 2 y0 )
6
Trapezoidal rule [two-point quadrature]
h
Substituting n = 1 in the Newton Cotes formula and taking = ( y0 + 4 y1 + y2 )
3
the curve y = f (x) through (x0, y0) and (x1, y1) as a straight
line so that differences of order higher than one becomes Similarly,
zero, we get
x1 x0 + h x0+4h
1 h
f ( x)dx = f ( x )dx = h y0 + y0 f ( x )dx = ( y + 4y 3 + y 4 )
3 2
x0 x0 2 x0+ 2h

1 h x 0 + nh
= h y0 + ( y1 y0 ) = y0 + y1 Similarly, h
2 2 f ( x )dx =
x 0 + ( n 2) h
( y + 4 y n 1 + y n )
3 n 2
x2 x0 + 2 h
1 h
f ( x)dx =
x1 x0 + h
f ( x )dx = h y1 + y1 = y1 + y2
2 2
Therefore, adding all these we get when n is even,
x 0 + nh
h ( y 0 + y n ) + 4( y 1 + y 3 + ....+ )
x3 x0 + 3 h
h f ( x )dx =
3 +2( y 2 + y 4 + ..... + y n 2 )

f ( x)dx =
x2 x0 + 2 h
f ( x )dx = ( y2 + y3 )
2
x0
h
=  [(sum of the first and last ordinates
( x0 + nh )
h 3
Proceeding,
x0 + ( n 1) h
f ( x )dx = y + yn
2 n 1
+ 4 (sum of the odd ordinates) + 2

xn
(sum of the even ordinates)]
h 1
Hence, f ( x )dx = ( y0 + yn ) + 2( y1 + y2 + ...... + yn 1) This is known as Simpsons rd rule.
x0
2 3
xn
h (sum of the first and last ordinates) Simpsons three-eighth rule
Thus, f ( x )dx =
x0
2 + 2(sum of remainning ordinates) Substituting n = 3 in the Newton Cotes quadrature formula
and taking curve through (x0, y0), (x1, y1), (x2, y2) and (x3, y3)
The above rule is known as Trapezoidal rule. so that the differences of order higher than three becomes
Geometrical interpretation of trapezoidal rule: zero, we get
x3
y 3 3 1
(x1, y1)
(x2, y2)
(xn, yn) f ( x)dx = 3h y
x0
0
+
2
y0 + 2 y0 + 3 y0
2 8
(x0, y0) (xn1, yn1)
3h
= ( y + 3 y1 + 3 y2 + y3 )
y0 y1 y2 8 0
yn1 yn
Similarly,
x6
3h
O x0 x1 x2 xn1 xn
x f ( x)dx =
x3
8 3
( y + 3 y4 + 3 y5 + y6 ) and so on.

Geometrically, the curve y = f (x) is replaced by n straight


line segments joining the points (x0, y0) and (x1, y1); (x1, y1) Adding all these integrals from x0 to xn where n is a
and (x2, y2);.;(xn-1, yn-1) and (xn, yn). The area bounded by multiple of 3, we get
the curve y = f (x) is then approximately equal to the sum of xn
the areas of n trapeziums as shown in the figure. 3h
f ( x)dx =
x0
[(y + y ) + 3 (y1 + y2 + y4 + y5 + .+ yn-2)
8 0 n
Simpsons one-third rule [three-point quadrature] + 2 (y3 + y6 + y9 + + yn-3)]
Substituting n = 2 in the Newton Cotes quadrature formula 3
taking the curve through (x0, y0), (x1, y1) and (x2, y2) as a The above rule is called Simpsons rule which is
8
parabola, so that the differences of order higher than 2 applicable only when n is a multiple of 3.
becomes zero, we get

Chapter 06.indd 136 8/28/2015 7:46:21 PM


Chapter 6 Numerical Methods | 2.137
2

Example 9: Evaluate 1 + x 2 dx taking h = 0.2 using Solution: Here, a = 0, b = 2, h = 0.2


0
ba 20
(i) Trapezoidal rule and So n = = = 10
h 0.2
1
(ii) Simpsons rd rule The values of x and y are tabulated as below.
3

x 0 0.2 0.4 0.6 0.8 1 1.2 1.4 1.6 1.8 2


1 1.0198 1.077 1.1661 1.2806 1.414 1.562 1.7204 1.8867 2.059 2.236
y = 1+ x 2 y0 y1 y2 y3 y4 y5 y6 y7 y8 y9 y10

(i) By Trapezoidal rule, By Simpons three-eighth rule,


2 p
h

2
1 + x 2 dx = ( y + y10 ) + 2( y1 + y2 + ..... + y9 ) 3h
0
2 0 e
0
cos x
dx = [(y + y ) + 3 (y1 + y2)]
8 0 3
0.2 3 p
=  [(1 + 2.236) + 2 (1.0198 + 1.077 = [(2.718 + 1) + 3 (2.3774 + 1.6487)]
2 8 6
+ 1.1661 + 1.2806 + 1.414 + 1.562 +
p
1.7204 + 1.8867 + 2.059)] = [(3.718) + (12.0783)] = 3.10159
16
= 0.1 [(3.236) + 2 (13. 1856)]
LU Decomposition Method of F actorisation
= 0.1 [29.6072] = 2.96072 (or) Method of Triangularisation
1 Consider the system of equations
(ii) By Simpsons rd rule,
3 a11x1 + a12x2 + a13x3 = b1
2 a21x1 + a22x2 + a23x3 = b2 (1)
h
0 1 + x dx = 3 ( y 0 + y 10 ) + 4( y 1 + y 3 + y 5 + y 7 + y 9 ) + 2a( y312x1++ya4 32+xy2 +6 +a33y x8 3)= b3
2

h In matrix notation, (1) can be written as


( y + y 10 ) + 4( y 1 + y 3 + y 5 + y 7 + y 9 ) + 2( y 2 + y 4 + y 6 + y 8 )
3 0 AX = B (2)
0.2 a11 a12 a13 x1
=  [(1 + 2.236) + 4 (1.0198 +
3
where A = a21 a22 a23 , X = x2 ,
1.1661 + 1.414 + 1.7204 + 2.059) a x
31 a32 a33 3
+ 2 (1.077 + 1.2806 + 1.562 +
1.8867)] b1

and B = b2
0.2 b
= [(3.236) + 29.5172 + 11.6126] 3
3
Step 1:Write A = LU, where L Lower triangular matrix
= 2.95772
of A and U Upper triangular matrix of A.
p
2 1 0 0 u11 u12 u13
Example 10:Evaluate e cos x dx by Simpsons three-eighth rule
0
i. e. L = l21 1 0 and U = 0 u22 u23
p l l32 1 0 0 u33
Solution: Taking h = , the range can be divided into three 31
6
equal, sub intervals with the division points. Step 2: Now, equation (2) becomes LUX = B (3)

The values of x and y are tabulated as below. y1



Step 3:Let UX = Y (4) where Y = y2
p p p y
x 0 3
6 3 2
Step 4: Combining (3) and (4), we get LY = B (5)
y = ecos x 2.718(y0) 2.3774(y1) 1.6487(y2) 1(y3)
On solving (5) we get y1, y2, y3

Chapter 06.indd 137 8/28/2015 7:46:29 PM


2.138|Engineering Mathematics

Step 5:Substituting Y in (4), we get UX = Y 3 1


R2 R2 - R and R3 R3 - R1,
On solving, we get X i.e. x1, x2, x3. 2 1 2
Example 11:Solve the system of equations x1 + x2 +
x3 = 1, 3x1 + x2 - 3x3 = 5 and x1 - 2x2 - 5x3 = 10 by LU 2 1 1 10
decomposition method.
1 3
we get [A, B] = 0 3
1 1 1 x1 1 2 2

Solution: AX = B 3 1 3 x2 = 5 0 7 17
11
1 2 5 x3 10 2 2
Now, R3 R3 - 7R2,
Step 1: LU = A
2 1 1 10
1 0 0 u11 u12 u13 1 1 1
1 3
l21 1 0 0 u22 u23 = 3 1 3 [A, B] ~ 0 3
2 2
l 1 0 u33 1 2 5 0
31 l32 0
0 2 10
Expanding and on solving we get, u11 = 1, u12 = 1, Now, by backward subtraction,
u13 = 1, u22 = -2, u23 = -6, u33 = 3, l21 = 3, l31 = 1, 1 3
-2z = -10 z = 5 and y + z = 3
3 2 2
l32 = y + 3z = 6
2
y = 6 - (3 5) = y = -9 and x = 7
Step 2: Now, LUX = B The solution set is (7, -9, 5)
Step 3: Let UX = Y
Step 4: LY = B

Numerical Solutions of Ordinary
1 0 0 y 1 Differential Equations
1 0 0 y 1 1
3 1 0 y 12 = 5 The following methods are discussed on the numerical solu-
3 1 0 y = 5
3 2 tion of ordinary differential equations.
1 3 1 yy 3 10
10
1 2 1 3
2 Single-step Methods
On solving, y1 = 1, y2 = 2 and y3 = 6 1. Taylors series Method
Step 5: UX = Y 2. Picards Method of successive approximation

1 1 1 x1 1 Multi-step Methods

0 2 6 x2 = 2

1. Eulers Method
0 0 3 x3 6 2. Modified Eulers Method
3. Runge-Kutta Method
On solving we get x1 = 6, x2 = -7 and x3 = 2 4. Predictor-Corrector Methods
The sol is (6, -7, 2) [Milnes and Adams]

Gauss Elimination Method Taylors Series Method


This method of solving a system of n linear equations in n Consider the differential equation
unknowns consists of eliminating the coefficients in such
a way that the system reduces to upper triangular system dy
= f (x, y) with initial condition y (x0) = y0  (1)
which may be solved by backward substitution. dx

Example 12: Solve the equations 2x + y + z = 10, 3x + 2y + Let y = f (x) be the solution of (1)
3z = 18 and x + 4y + 9z = 16 using Gauss elimination method. Writing the Taylor series expansion of f(x) at x0

Solution: The augmented matrix of the system is ( x x0 ) 2


f(x) = f(x0) + (x - x0) f (x0) + f (x0) +
2 1 1 10 2!
[A, B] = 3 2 3 18 ( x x0 ) 2
y = y0 + (x - x0) y0 + y0 +
1 4 9 16 2!
Put x = x1, we get
Now we reduce the matrix A to upper triangular matrix ( x1 x0 ) 2
y1 = y0 + (x1 - x0) y0 + y0 +..
doing some operations. 2!

Chapter 06.indd 138 8/28/2015 7:46:34 PM


Chapter 6 Numerical Methods | 2.139

If we take h = x1 - x0 Proceeding this way, in general


h 2
x
y1 = y0 + hy0 + y + ..
2! 0 yn = y0 + f ( x, yn 1 )dx
In general, x0

h 2
yn+1 = yn + hyn + y + will be the iterative formula. dy
2! n Example 14:Given = 1 + xy and y (0) = 1. Evaluate
dx
dy y (0.1) by Picards Method upto three approximations.
Example 13: Given = x - y2 with the initial condition
y (0) = 1 dx
Solution: f (x, y) = 1 + xy
x0 = 0, y0 = 1
Find y (0.1) using Taylor series method x

Solution: f (x, y) = x - y 2 The first approximation y1 = y0 + f ( x, y0 )dx


x0
x = 0.1, x0 = 0, y0 = 1, h = 0.1 x x

y = x - y2 y (0) = x0 - y02 = - 1; 1 + 1 + xy0 dx = 1 + 1 + xdx


x0 0
y = 1 - 2yy y (0) = 1 - 2y0y0
x2
= 1 - 2 (1) (-1) = 3 y1 = 1 + x +
2
y = -2yy - 2(y)2
(0.1) 2
y (0) = -2 (1) (3) - 2 (-1)2 At x = 0.1, y1 = 1 + (0.1) + = 1.105
2
= -6 - 2 = -8 The second approximation y2
By Taylors formula,
x
h2 h3 = y0 + f ( x, y1 )dx
y (0.1) = y1 = y0 + hy1 (0) + y11 (0) + y111 (0)
2! 3! x0

x
(0.1) 2
(0.1)3
y1 = 1 + (0.1) (-1) + (3) + ( 8 ) + .. y2 = 1 + 1 + xy1dx
2! 3! 0

= 1 - 0.1 + 0.015 - 0.0013 + .. x


x2
y2 = 1 + 1 + x 1 + x + dx
y1 = 0.9137 0
2
x
x3
Picards Method of Successive = 1 + 1 + x + x 2 + dx
2
Approximation 0

x 2 x3 x 4
dy =1+ x+ + +
Given the differential equation = f (x, y) (1) 2 3 8
dx
Integrate (1) from x0 to x, we get (0.1) 2 (0.1)3 (0.1) 4
At x = 0.1, y2 = 1 + (0.1) + + +
x x
2 3 8
y (0.1) = 1.10534
dy = f ( x, y)dx x

The third approximation y3 = y0 +


x0 x0
f ( x, y2 )dx
x x0
y (x) - y (x0) = f ( x, y )dx x

x0 y3 = 1 + 1 + xy2 dx
x 0

y (x) = y (x0) + f ( x, y )dx (2) x


x 2 x 3 x 4
x0 1 + 1 + x 1 + x + + + dx
0 2 3 8
Put y = y0, we get the first approximation,
x
x
y1 = y0 + f ( x, y0 )dx x3 x 4 x5
1 + 1 + x + x 2 + + + dx
x0
0
2 3 8
Similarly, put y = y1 in f(x, y) of (2), the second approximation
x 2 x3 x 4 x5 x6
x =1+x+ + + + +
y2 = y0 + f ( x, y1 )dx 2 3 8 15 48
x0 At x = 0.1,

Chapter 06.indd 139 8/28/2015 7:46:44 PM


2.140|Engineering Mathematics

(0.1) 2 (0.1)3 (0.1) 4 (0.1)5 (0.1)6Note: To find yn, we proceed to find the approximations
y3 = 1 + (0.1) +
+ + + +
48 yn , yn , yn ..until the two successive approximations
(0) (1) (2)
2 3 8 15
(0.1) 2 (0.1)3 (0.1) 4 (0.1)5 (0.1)6 are approximately equal.
+ + + +
2 3 8 15 48 Example 16:Find y for x = 0.1 using modified Eulers method
= 1 + 0.1 + 0.005 + 0.0003 + 0.0000125 + dy
0.0000006 + 0.00000002 for the differential equation = log (x + y) with y (0) = 1
dx
y3 = 1.105313
Solution: Given f (x, y) = log (x + y)
Multi-step Methods x0 = 0, y0 = 1, h = 0.1
Eulers Method To find y1, x1 = 0.1
dy y1 (0) = y0 + h f (x0, y0)
For the differential equation = f (x, y)
dx
with initial condition y (x0) = y0, the Eulers iteration for- = 1 + (0.1) log (0 + 1) = 1
mula is h
y1 (1) = y0 + [f (x0, y0) + f (x1, y1(0)]
yn = yn-1 + h f (xn-1, yn-1), n = 1, 2, 3 2

Note: The process is very slow and to obtain accuracy, h h


= y0 + [log (x0 + y0) + log (x1 + y1(0))
must be very small, i.e. we have to divide [x0, xn] into a more 2
number of subintervals of length h. 0.1
=1+ [log (0+1) + log (0.1 + 1)]
dy y x 2
Example 15:Solve = , y (0) = 1, find y (0.5) by
dx y + x 0.1
=1+ [log 1 + log 1.1] = 1.0047
Eulers method choosing h = 0.25 2
yx h
Solution: f (x, y) = y1(2) = y0 + [f (x0, y0) + f (x1, y1(1)]
y+x 2
x0 = 0, y0 = 1, h = 0.25 h
= y0 + [log(0 + 1) + log(x1 + y1(1)]
Eulers iteration formula, 2
yn = yn-1 + h f (xn-1, yn-1) 0.1
=1+ [log (0 + 1) + log (0.1 + 1.0047)]
Put n = 1, 2
x1 = 0.25 y1 = y (0.25) = y0 + h f (x0, y0) = 1.0049
y x0 h
= 1 + (0.25) 0 y1(3) = y0 + [f(x0, y0) + f(x1, y1(2))]
y0 + x0 2
1 0 0.1
= 1 + (0.25) = 1.25 =1+ [log(0 + 1) + log(0.1 + 1.0049)]
1+ 0 2
Put n = 2 = 1.0049
x2 = 0.5 y2 = y (0.5) = y1 + h f (x1, y1) y1 = 1.0049
y x
= (1.25) + (0.25) 1 1
y1 + x1 Runge-Kutta Methods
1.25 0.25 First Order Runge-Kutta Method
= 1.25 + (0.25)
1.25 + 0.25 y1 = y0 + hy01 [same as Eulers method]
= 1.25 + 0.166666 = 1.4166
y (0.5) = 1.4166 Second-Order Runge-Kutta Method
Modified Eulers Method 1
The formula is y1 = y0 + (k1 + k2)
2
dy
For the differential equation = f (x, y) with initial condi- where k1 = h f (x0, y0) and k2 = h f (x0 + h, y0 + k1)
dx
tion y (x0) = y0, the Modified Eulers iteration formula is Third Order Runge-Kutta Method
h 1
yr (n) = yr-1 + [f (xr-1, yr-1) + f (xr, yrn-1)] The formula is y1 = y0 + (k1 + 4k2 + k3)
2 6

Chapter 06.indd 140 8/28/2015 7:46:51 PM


Chapter 6 Numerical Methods | 2.141

where k1 = h f (x0, y0) h k


k3 = h f x0 + , y0 + 2
1 1 2 2
k2 = h f x 0 + h , y 0 + k 1 and
2 2 = (0.05) f (1 + 0.025, 1.2 + 0.066)
= (0.05) f (1.025, 1.266)
k3 = h f (x0 + h, y0 + k1) where k1 h f(x0 + h, y0 + k1) = (0.05) [(1.025)2 + (1.266)2] = 0.1326 and k4 = h
f (x0 + h, y0 + k3)
Fourth-Order Runge-Kutta Method = (0.05) f (1 + 0.05, 1.2 + 0.1326)
1 = (0.05) f (1.05, 1.3326)
The formula is y1 = y0 + (k + 2k2 + 2k3 + k4) = (0.05) [(1.05)2 + (1.3326)2] = 0.1439
6 1
1
where k1 = h f (x0, y0) y1 = y (1.05) = y0 + (k1 + 2k2 + 2k3 + k4)
6
1 1 1
k2 = h f x 0 + h , y 0 + k 1 = 1.2 + [0.122 + 2 (0.1320) + 2 (0.1326)
2 2 6
+ 0.1439]
1 1 1
k3 = h f x 0 + h , y 0 + k 2 = 1.2 + [0.7951] = 1.3325
2 2 6
and k4 = h f (x0 + h, y0 + k3)
Predictor-Corrector Methods
Example 17:Given
dy
= x2 + y2, y (1) = 1.2. Find y (1.05)
Milnes Predictor Formula
dx 4h
ynp+1= yn-3 + (2 fn-2 - fn-1 + 2 fn)
applying fourth order Runge-Kutta method 3
Solution: f (x, y) = x2 + y2, x0 = 1, y0 = 1.2, h = 0.05 Milnes Corrector Formula
k1 = h f (x0, y0) = (0.05) [x02 + y02] h
ync+1= yn-1 + [fn-1 + 4fn + fn+1p]
= (0.05) [12 + (1.2)2] = 0.122 3

h k Adams-Bashforth Predictor Formula


k2 = h f x0 + , y0 + 1 h
2 2 ynp+1= yn + [55fn - 59fn-1 + 37fn-2 - 9fn-3]
24
= (0.05) [f (x0 + 0.025, y0 + 0.061] Adams-Moulton Corrector Formula
= (0.05) [f (1.025, 1.261)] h
ync+1 = yn + [9 f p + 19fn - 5fn-1 + fn-2]
= (0.05) [(1.025)2 + (1.261)2] = 0.1320 24 n +1

Exercises
Practice Problems 1 5. If the equation 3x4 - 13x3 + 7x2 + 17x + a - 10 = 0 has exactly
three positive roots, then a can be
Directions for questions 1 to 45: Select the correct alternative
(A) 11 (B) 4 (C) 13 (D) 12
from the given choices.
1. Three of the roots of the equation x4 + x3 + mx2 + nx + 24 = 0 6. If two of the roots of the equation x3 + 3x2 - 10x - 24 = 0
are 3, 1 and -2. Which of the following could be the value of are such that one is twice the other, then the third root is
+ m - n? (A) -4 (B) -3 (C) -2 (D) 3
(A) 0 (B) 1 (C) 2 (D) 3 7. Which of the following is an algebraic equation?
2. If one of the roots of the equation x3 + 5x2 - 12x - 36 = 0 is (A) 4x3 - 3x2 + 10x - 1 = 0
thrice another root, then the third root is (B) sin x + xex = 0
(A) - 6 (B) 3 (C) -2 (D) - 89/13
(C) x4 + log x - 11 = 0
3. If the equation x6 + 5x5 + 11x4 + 34x2 + 20x + 24 = 0 has exactly
(D) x2e-x = 3
four non-real roots, then the number of negative roots is
(A) 1 (B) 0 (C) 3 (D) 2 8. The Transcendental equation of the following is
4. A student finds, by trial, two negative and one positive root(s) (A) x2 - 2x5 + 9 - x = 0
of the equation x5 + 5x4 + 2802x + 3024 = 103x3 + 329x2. How (B) 4x - sin2 x + log x = 0
many non-real roots does the equation have? (C) 2x3 - 3x2 + 12x = 0
(A) 0 (B) 1 (C) 2 (D) 4 (D) None of these

Chapter 06.indd 141 8/28/2015 7:46:57 PM


2.142|Engineering Mathematics

9. Let f(x) = x3 - x - 5 = 0. By bisection method first two approx- 21. Find the reciprocal of 22 using Newton-Raphson method.
imations x0 and x1 are 1.5 and 2.25, respectively, then x2 is (A) 0.0454545 (B) 0.4504504
(A) 1.625 (B) 1.875 (C) 0.54054 (D) None of these
(C) 1.999 (D) None of these
22. If the first approximation of the root of x3 - 3x - 5 = 0 is
10. The second approximation of the positive root of the equation (x0 =) 2, then find x1 by Newton-Raphson method.
2x - cos x = 0 between 0 and 0.5 using Bisection method is (A) 2.2806 (B) 2.2790
(A) 0.405 (B) 0.258 (C) 2.3333 (D) None of these
(C) 0.375 (D) None of these
23. Find the first approximation of the real root by Newton-
11. Find the fourth approximation of the root of the equation x3 + Raphson method for x4 + x3 - 7x2 - x + 5 = 0 by taking x0 = 2.
x - 11 = 0, between 2 and 3, using Bisection method. (A) 2.066 (B) 2.981
(A) 1.925 (B) 2.832 (C) 2.819 (D) None of these
(C) 2.5215 (D) 2.0625
24. Solve the following system by LU-decomposition method
12. The absolute error in Bisection method is
x + 2y + 3z = 16
1
(A) 2n (B) |b - a| 3x + 5y + 8z = 43
2n 4x + 9y + 10z = 57
1
(C) (D) |b - a| 2n
|b-a| (A) (2, -1, 4) (B) (2, 1, 4)
(C) (-2, 1, 4) (D) None of these
13. If the first two approximations x0 and x1 to a root of x - x - 4 3

= 0 are 1.666 and 1.780 respectively, then find x2 by Regula- 3


1
Falsi method. 25. Find the value of dx taking four intervals by trapezoi-
1 + x2
(A) 1.974 (B) 1.794 2

(C) 1.896 (D) None of these dal rule and also find the error when compared to its exact
value
14. Find the second approximation to the root of the equation 2x (A) 0.1759, 0.000004 (B) 0.1826, 0.04
- 5 = 3sinx between (2, 3) using the method of false position. (C) 0.1953, 0.004 (D) 0.1423, -0.0004
(A) 2.2523 (B) 2.012
(C) 2.8804 (D) None of these 1
x2
26. Find dx using Trapezoidal rule by taking 4 strips
0 1 + 8x
3
15. The third approximation to the root of x - 2logex - 10 = 0 by the
2

method of false position is (A) 0.0911 (B) 0.9011


(A) 3.5392 (B) 3.7856 (C) 0.1901 (D) None of these
(C) 3.9310 (D) None of these
27. The error obtained in the above problem is
16. The Newtons iterative formula to find the value of N is (A) 0.401 (B) 0.0004
(C) 0.0489 (D) None of these
1 N 1 N
xi+1 =
(A) xi + (B)xi+1 = xi 3 - 6
1
2 xi 2 xi 28. Find the value of x log x dx taking 4 strips by Simpsons rd
2
3
1 N 1 rule upto four decimals.
xi+1 =
(C) x - (D)xi+1 = (xi - N)
2 i xi 2 (A) 21.8901 (B) 22.8661
17. For N = 28 and x0 = 5.5, the first approximation to N by (C) 23.6581 (D) None of these
Newtons iteration formula is p /2
1
(A) 5.295 (B) 5.582 29. Evaluate sin x dx by Simpsons 3 rd rule using six intervals
(C) 5.396 (D) None of these 0

3
(A) 0.97768 (B) 0.98869
18. The Newtons iterative formula to find the value of N is (C) 0.99968 (D) None of these
NN
xi+1 =1 12x N 1 11 N N
(A) xi xi+i-+ 2 (B) xi+1 = 3xi +xxi i-+ 2 30. Find the maximum error in evaluating the above when
2 2 xxi xii 2 2 xi xxii compared to its exact value.
1 NN 1 NN (A) 0.000032 (B) 0.00032
(C) xi+1 = 1 12x x x+++ (D) xi+1 = 1 12x x x+-+ (C) 0.00000032 (D) 0.0032
32 2 i i i xxxi2 32 2 i i i xxxi2
i
i i i
3
1 3
19. Find the second approximation to the cube root of 24 correct
to three decimal places using Newtons iterative formula.
31. Evaluate 2+ x
0
2
dx by using Simpsons
8
th rule by taking
3 strips.
(A) 2.695 (B) 2.885
(C) 3.001 (D) None of these (A) 0.507 (B) 0.5007
1 (C) 0.7839 (D) None of these
20. The Newtons iterative formula to find the value of is
N 32. A solid revolution is formed by rotating about the x-axis, the
(A) xi+1 = xi (2 + xiN) (B) xi+1 = xi (2 - xiN) area between the x-axis and the lines x = 0 and x = 1 and a
(C) xi+1 = xi2 (2 + xiN) (D) None of these curve through the points with the following coordinates

Chapter 06.indd 142 8/28/2015 7:47:05 PM


Chapter 6 Numerical Methods | 2.143

x0 x1 x2 x3 x4 (A) 1.3902 (B) 1.2309


x 0.00 0.25 0.50 0.75 1.00 (C) 1.3092 (D) None of these
y 1.0000 0.9783 0.9412 0.9003 0.8536
dy
39. Find k1, by Runge-Kutta method of fourth order if
dx
= 2x + 3y2 and y (0.1) = 1.1165, h = 0.1.
Find the volume of the solid formed (A) 0.3993 (B) 0.9393
(A) 2.9814 (B) 2.7672 (C) 0.3939 (D) None of these
(C) 2.0012 (D) None of these
40. If y (0) = 1, h = 0.5 and k1 = 0.8, find k2 by Runge-Kutta
1 y 2 - 3x 2
33. Evaluate loge5 using Simpsons rd rule by taking h = 1. method of order 4, given yI = 2 .
3 y + 3x 2
(A) 1.62 (B) 2.17
(A) 0.2416 (B) 0.0052
(C) 3.18 (D) None of these
(C) 0.6184 (D) 0.4126
34. A river is 40 m wide. The depth d in meters at a distance of x
meters from a bank is given in the following table Linked data for Questions 41 and 42:
dy
x 0 10 20 30 40 41. Find y(0.8) by Milnes predictor formula, given = x - y2,
dx
d 0 4 16 6 0 y2 = 0.0795, y (0.6) = 0.1762, y0 = 0.0000, y11 = 0.1996, y21 =
0.3937, y31 = 0.5689, h = 0.2
Then, the area of cross section of the river, in square meters, is
(A) 300 (B) 240 (A) 0.9304 (B) 0.4930
(C) 270 (D) 200 (C) 0.3049 (D) None of these
42. For the above problem find y (0.8) using Milnes corrector
dy formula.
35. If = 1 - 3xy2, y (0) = 0, then by Taylors method y (0.1) =
dx (A) 0.3046 (B) 0.4036
(A) 0.02 (B) 0.001 (C) 0.436 (D) None of these
(C) 0.05 (D) 0.1
Linked data for Questions 43 and 44:
dy
36. If = 2x + y, y (0) = 1, the Picards approximate of y upto
dx 43. Find using the Adams-Bashforth corrector formula y(0.4), for
second degree terms is dy 1
the differential equation = xy , given y(0.1) = 1.01, y(0.2)
x2 dx 2
(A) 1 + x + x2 (B)1 + x + = 1.022, y(0.3) = 1.023, y0 = 0, y11 = 0.0505, y21 = 0.1022,
1
2
x2 y31 = 0.1535
(C) 1 - x - (D) None of these (A) 1.5418 (B) 1.0410
2
(C) 1.4100 (D) None of these
37. If y0 = 1, f (x0, y0) = 1.2, f (x1, y1(0)) = 1.9312, h = 0.3, by 44. For the above differential equation find y(0.5) using Adams-
modified Eulers formula y1 (1) = Bashforth predictor formula
(A) 1.4696 (B) 1.2015 (A) 1.00463 (B) 1.06463
(C) 1.325 (D) 1.525 (C) 1.00599 (D) None of these
38. Using Eulers modified method, find a solution of the equation 45. The Runge-Kutta methods has the error of order _____
dy
= x + y with y(0) = 1 at y (0.2). (A) 1 (B) 3 (C) 5 (D) 2
dx

Practice Problems 2 4. If f(x) = 0 is an eighth degree equation and it has four positive
Directions for questions 1 to 150: Select the correct alternative roots, then which of the following cannot be the number of
from the given choices. sign changes in f (x)?
(A) 4 (B) 3 (C) 6 (D) 8
1. a, b, c are in arithmetic progression and a, b, are the roots of
x3 + ax2 + bx + c = 0 Which of the following is true if g = -1? 5. If ax4 + bx3 + cx2 + dx + e = 0 has exactly two negative roots
(A) 3ab - a - b = 1 (B) ab + a + b = 1 and a > 0, b < 0, then how many of the following statements
(C) 2ab - a - b = -1 (D) ab - a - b = 1 cannot be true?
2. The sum of the squares of two positive numbers is 185 and I. c > 0, d > 0, e > 0
the sum of the larger and thrice the smaller is 35. Find the II. c > 0, d > 0, e < 0
larger of the two numbers. III. c > 0, d < 0, e > 0
(A) 11 (B) 8 (C) 13 (D) 14 IV. c < 0, d < 0, e < 0
(A) 0 (B) 1 (C) 2 (D) 3
3. Two of the roots of the equation x3 - 7x2 + 36 = 0 are such that
one is thrice the other. These roots have opposite signs. Find 6. A real root for e2x = 9x using bisection method is
the difference of the greatest two roots. (A) 0.0981 (B) 0.1875
(A) 2 (B) 3 (C) 5 (D) - 4 (C) 0.6789 (D) None of these

Chapter 06.indd 143 8/28/2015 7:47:09 PM


2.144|Engineering Mathematics

7. In case of bisection method, the convergence is 16. Let N = 52 and x0 = 7.5. The first approximation to N by
(A) quadratic (B) linear Newtons iteration formula is _____
(C) very slow (D) of order 3 (A) 7.263 (B) 7.216
8. Let a < b and f(a), f(b) have opposite signs. The first approxi- (C) 7.418 (D) None of these
mation of one of the roots of f(x) = 0, by Regula-Falsi method 17. For P = 30 and x0 = 3.5 the second approximation to cube root
is of P by Newton-Raphson iterative formula is _____
af ( a) + bf (b) af ( a) - bf (b) (A) 3.1496 (B) 3.1078
(A) x0 = (B) x0 =
f ( a) + f ( b) f ( a) - f ( b) (C) 3.1823 (D) None of these
1/ 6
af (b) - bf ( a) af (b) + bf ( a) 18. Evaluate , with x0 = 0.5 using Newton-Raphson upto 5
1
x0 =
(C) x0 =
(D)
f ( b) - f ( a) f ( a) - f ( b) 5
approximations
9. Compute a root for 0.32sin(0.3 + x) = x by the method of false (A) 0.8051 (B) 0.5081
position up to three approximations (C) 0.1058 (D) None of these
(A) 0.3411 (B) 0.4311 19. The order of convergence in secant method is
(C) 0.3144 (D) 0.1344 (A) 0.89 (B) -0.782
10. Find the third approximation of the root of the equation (C) 1.62 (D) None of these
5x - 2cos x - 1 = 0 using the method of false position,
20. If we put n = 2 in general quadrature formula, we get
between 0 and 1.
3
(A) 0.2454 (B) 0.5424 (A) Trapezoidal rule (B) Simpsons th rule
(C) 0.4524 (D) None of these 8
1
11. Newtons iterative formula to find the (p + 1)th approximation (C) Booles rule. (D) Simpsons rd rule
to a root of f(x) = 0 is 3
f (xp ) f (xp ) 21. If we put n = 1 in the general quadrature formula, we get
(A) xp+1 = xp + (B) xp+1 = xp - 1
f ( x p ) f ( x p ) (A) Simpsons rd rule (B) Trapezoidal rule
3
f ( x p ) f ( x p ) 3
xp+1 = xp +
(C) xp+1 = xp -
(D) (C) Simpsons th rule (D) None of these
f (xp ) f (xp ) 8
b 22. If we put n = 3, in the general quadrature formula, we get
12. The Newtons iterative formula for a is
(A) Trapezoidal rule (B) Weddles rule
1 a 1 3
xn+1 =
(A) (b + 1) xn - b -1 (C) Simpsons rd rule (D) Simpsons th rule
b xn 3 8
1
1 a 23. Simpsons rd rule is a _____
xn+1 =
(B) (b - 1) xn + b -1 3
b xn (A) Newton-cotes 4 point formula
(B) Newton-cotes 1 point formula
1 a
xn+1 =
(C) (b - 1) xn - b -1 (C) 2-point Newton-Cotes formula
b xn (D) 3-point Newton-Cotes formula
7
(D) None of these 5
24. By trapezoidal rule the value of the integral 3+ x
5
2
dx divid-
13. The Newton-Raphsons iteration formula for finding R 1/7
ing the range into eight equal parts is
where R is a positive real number is (A) 0.42971 (B) 0.4356
(C) 0.3285 (D) 0.2625
1 R 1 R
(A) 6 xn - 6 (B) 6 xn + 25. The inherent error in the trapezoidal rule is given by
7 xn 7 xn -16
h4 h3
(A) - fII () (B) - n.fII ()
1 R 12 8
(C) 6 xn + 6 (D) None of these
7 xn (b - a) 2 II
(C) - h .f () (D) None of these
12
14. What is the order of convergence in Newton-Raphson
method? 1
26. In Simpsons rd rule, the number of subintervals should be
(A) 1 (B) 3 (C) 2 (D) 0 ______ 3
15. Find the second successive approximation for the root of the (A) even (B) a multiple of 3
equation x4 - 2x3 + x2 - 3x - 1 = 0 which is nearer to three (C) odd (D) none of these
decimal places by Newton-Raphson method 27. The maximum inherent associated with composite Simpsons
(A) 2.248 (B) 2.901 1
rd rule is _____
(C) 2.482 (D) None of these 3

Chapter 06.indd 144 8/28/2015 7:47:18 PM


Chapter 6 Numerical Methods | 2.145

( b - a) 4 4 h5 n 3 dy
-
(A) -
h f () (B) f () 35. Find the approximation to the solution = xy + 1, y (0) = 1
180 180 2 upto bi-quadratic equation. dx
b-a x 2 x3 x 4 x 2 x3 x 4
(C) h4.f 4 () (D) None of these (A) 1 + x + + - (B) 1 - x - - -
180 2 3 12 2 3 12
6
x 2
x 3
x 4
x 2
x3 x 4
28. Find the error obtained in finding 2 exdx using Simpsons (C) 1 - x + - + (D) 1 + x + + +
2 3 6 2 3 8
1 0
rd rule taking 6 intervals dy
3 36. Eulers formula for (n + 1)th iteration, if = f (x, y) is
dx
(A) 4.512 (B) 3.987 (A) yn+1 = yn + f (xn, yn) (B) yn+1 = yn + hf (xn, yn)
(C) -3.989 (D) 2.978 (C) yn+1 = f (xn, yn) (D) yn+1 = hf (xn, yn)
3
29. Which among the following represents the Simpsons th 37. Using Eulers method, the value of y of the differential
rule when n = 3? 8 1
dy y - x
h equation = , y (0) = 1, and h = 0.005
(A) [y0 + 2 (y1 + y2 +..+ yn-1) +yn] dx y + x
2 (A) 1.502 (B) 1.005
3h (C) 1.002 (D) None of these
(B) [y0 + 2 (y3 + y6 ++ yn-3) + yn]
8 38. Which of the following iterative formula denote Modified
3h Eulers Method?
(C) [y0 + 3y1 + 3y2 + y3]
8 h
(A) y1k+1 = y0 + [ f (x0, y0) + f (x1, y1k-1)]
3h 2
(D) [y3 + 3y4 + 3y5 + y6] h
8 (B) y1k = y0 - [ f (x0, y0) + f (x1, y1k)]
3 2
30. The number of sub-intervals in Simpsons th rule must be h
8 (C) y1k+1 = y0 + [ f (x0, y0) + f (x1, y1k)]
______ 2
(A) Odd (B) A multiple of 3 (D) None of these
(C) Even (D) A multiple of 4 dy
39. = x + cos y, y (0) = 1. The value of y (0.2) with
4 dx
31. The value of 1 + x dx upto four decimal places by
3
h = 0.2 by Modified Eulers Method is _____
1
3 (A) 1.7711 (B) 1.3344
Simpsons th rule taking h = 0.5 is (C) 1.2266 (D) 1.1177
8
(A) 15.0123 (B) 12.8709 dy
40. For = 2xy, y (0) = 1. h = 0.22, find y (0.22) using modified
(C) 13.1254 (D) None of these dx
Eulers method.
32. Match list-I and list-II: (A) 1.0508 (B) 1.5080
(C) 1.8501 (D) None of these
List-I List-II
41. Using Runge-Kutta method of fourth order, to solve the dif-
A. Regula-Falsi a. Solution of system of linear dy 2 xy + e x
equations ferential equation, = 2 , y (1) = 0, h = 0.2, the
dx x + ex
B. Modified Eulers b. Integration value of k2 is
C. LU decomposition c. Finding the root (A) 0.0512 (B) 0.1502
(C) 0.2816 (D) None of these
D. Trapezoidal rule d. Solution of ordinary differen-
tial equation 42. What will be the value of y(0.3) from the differential equation
dy
= 3ex + 2y with y (0) = 0, h = 0.3 by Runge-Kutta Method
(A) A d, B c, C b, D a dx
(B) A c, B d, C a, D b of order 4?
(C) A a, B b, C c, D d (A) 1.414 (B) 1.844
(D) None of these (C) 1.444 (D) None of these
dy Linked data for Questions 43 and 44:
33. To solve the differential equation = -4y + 3ex using
dx 43. Using Runge-Kutta method, to solve the differential equation
Taylors series method, with y(0) = 0, y(iv) is dy
= x + y, h = 0.1 and y (0) = 1, the values of k1, k2, k3 and
(A) 3 (B) -21 (C) -153 (D) 145 dx
k4 respectively are
dy (A) 0.11, 0.121, 0.1, 0.005
34. To solve the differential equation = x2y - 1, y (0) = 1,
dx (B) 0.1, 0.11, 0.1105, 0.12105
h = 0.1, by Taylors series method the value of y is (C) 0.111, 0.11105, 0.121005, 0.121
(A) -1 (B) 0 (C) 2 (D) 6 (D) None of these

Chapter 06.indd 145 8/28/2015 7:47:27 PM


2.146|Engineering Mathematics

44. For the above problem, y (0.1) = _____ x


(A) 1.11034 (B) 1.33011
R. Eulers method c. yn = y0 + f(x, yn-1)dx,
(C) 1.43001 (D) None of these x0

45. The first order Runge-Kutta method is n = 1, 2, 3.


(A) Picards Method
S. Modified Eulers Method d. h h2
(B) Modified Eulers Method yn+1 = yn + y + y
1! n 2 ! n +..
| |

(C) Eulers Method


(D) Taylors Method h
Runge-Kutta Method of yn+1(k+1) = yn + [f (xn, yn)
46. The second order Runge-Kutta method is T. e. 2
order 4
(A) Taylor series Method + f (xn+1, yn+1)]
(B) Eulers Method
(C) Milnes Method (A) P - d, Q - c, R - a, S - e, T - b
(D) Modified Eulers Method (B) P - d, Q - c, R - b, S - e, T - a
(C) P - d, Q - c, R - e, S - b, T - a
47. The formula for the values of k1 and k2 respectively for second (D) P - d, Q - a, R - c, S - e, T - b
order Runge-Kutta method are
50. In which of the following methods successive approximations
(A) k1 = hf (x0, y0) and k2 = hf (x0 + h, y0 + k1)
used?
(B) k1 = f (x0, y0) and k2 = hf (x0, y0)
(A) Picards method
(C) k1 = hf(x0, y0) and k2 = hf(x0 + h/2, y0 + k1/2)
(B) Taylor series method
(D) None of these
(C) Adams-Bashforth method
48. Match list-I and list-II: (D) None of these
51. Which among the following is a step-by-step method?
List-I List-II (A) Picards
A. Regula-Falsi a. Solution of system of (B) Taylors series
linear equations (C) Adams-Bashforth
(D) None of these
B. Modified Eulers b. Integration
52. Which among the following is the self-starting method?
C. LU decomposition c. Finding the root (A) Adams-Bashforth
D. Trapezoidal rule d. Solution of ordinary (B) Milnes method
differential equation (C) Runge-Kutta method
(D) Predictor method
(A) A d, B c, C b, D a
53. Among the following, which is the best for solving initial
(B) A c, B d, C a, D b
value problems?
(C) A a, B b, C c, D d
(A) Modified Eulers method
(D) None of these
(B) Picards method
49. Match list-I and list-II: (C) Runge-Kutta mehod of fourth order
(D) Taylor series method
List-I (Solution of ordinary List-II (Formula) 3

differential equation) 54. The value of x dx


-3
4 approximately, when [- 3, 3] is divided

yn+1 = yn + hf (xn, yn), into 6 equal parts by (using the trapezoidal rule) is
P. Taylors series method a.
n = 0, 1, 2 (A) 115 (B) 110 (C) 120 (D) 100

1
55. Find the roots of the equation x4 - 2x3 - 13x2 + 14x
Q. Picards method b. y1 = y0 + (k1 + 2k2 + + 24 = 0.
6
2k3 + k4) (A) - 1, - 4, 1, 6 (B) 4, -3, - 2, - 1
(C) - 1, - 2, 2, 6 (D) - 3, - 1, 2, 4

Previous Years Questions


13
1. Using the trapezoidal rule, and dividing the interval of 3. The definite integral
1 x
dx is evaluated using Trapezodial
integration into three equal subintervals, the definite

+1 rule with a step size of 1. The correct answer is ________.
integral | x | dx is _____ [2014]
[2014]
-1 4
4. The real root of the equation 5x - 2cosx - 1 = 0 (up two
2. The value of ln(x)dx calculated using the Trapezoidal
2.5 decimal accuracy) is _______ [2014]
rule with five subintervals is _____ [2014]

Chapter 06.indd 146 8/28/2015 7:47:30 PM


Chapter 6 Numerical Methods | 2.147

dx
5. Consider an ordinary differential equation = 4t + 4. If Column I Column II
dt
x = x0 at t = 0, the increment in x calculated using Runge - P. Gauss - Seldel 1. Interpolation
method
Kutta fourth order multi-step method with a step size t = 0.2
is [2014] Q. Forward 2. Non - linear
(A) 0.22 (B) 0.44 Newton - Gauss differential
(C) 0.66 (D) 0.88 method equations

6. Match the CORRECT pairs. R. Runge - Kutta 3. Numerical


 [2013] Method Integration
S. Trapezoidal 4. Linear algebraic
Numerical Integration Order of Fitting Rule equation
Scheme Polynomial
P. Simpsons 3/8 rule 1. First (A) P - 1 Q - 4 R - 3 S - 2
Q. 2.
(B) P - 1 Q - 4 R - 2 S - 3
Trapezoidal rule Second
(C) P - 1 Q - 3 R - 2 S - 4
R. Simpsons 1/3 rule 3. Third (D) P - 4 Q - 1 R - 2 S - 3

(A) P2, Q1, R3 (B) P3, Q2, R1 10. Starting from x0 = 1, one step of Newton-Raphson method
(C) P1, Q2, R3 (D) P3, Q1, R2 in solving the equation x3 + 3x - 7 = 0 gives the next value
(x1) as [2005]
3
1 x1 = 0.5
(A) (B) x1 = 1.406
7. The integral x dx
1
when evaluated by using Simpsons
x1 = 1.5
(C) (D) x1 = 2
1/3 rule on two equal subintervals each of length l, equals. 11. The values of a function f(x) are tabulated below
[2011]
(A)1.000 (B)1.098 (C)1.111 (D)1.120 x f(x)
0 1
8. A calculator has accuracy up to 8 digits after decimal place.
2p 1 2
The value of sin x dx
0
when evaluated using this calcula-
2 1
tor by trapezoidal method with 8 equal intervals, to 5 sig- 3 10
nificant digits is[2007]
(A) 0.00000 (B) 1.0000 Using Newtons forward difference formula, the cubic poly-
(C) 0.00500 (D) 0.00025 nomial that can be fitted to the above data, is [2004]
(A) 2x3 + 7x2 - 6x + 2 (B) 2x3 - 7x2 + 6x - 2
9. Match the items in Columns I and II [2006] (C) x3 - 7x2 - 6x + 1 (D) 2x3 - 7x2 + 6x + 1

Answer Keys
Exercises
Practice Problems 1
1.D 2.B 3.D 4.A 5.B 6.B 7.A 8.B 9.B 10.C
11.D 12.B 13.B 14.C 15.A 16.A 17.A 18.C 19.B 20.B
21.A 22.C 23.A 24.B 25.D 26.A 27.B 28.B 29.C 30.B
31.C 32.B 33.A 34.B 35.D 36.A 37.A 38.B 39.C 40.D
41.C 42.A 43.B 44.B 45.C

Practice Problems 2
1.D 2.A 3.B 4.B 5.D 6.B 7.C 8.C 9.D 10.B
11.B 12.B 13.C 14.C 15.A 16.B 17.B 18.A 19.C 20.D
21.B 22.D 23.C 24.D 25.C 26.A 27.A 28.C 29.C 30.B
31.B 32.B 33.C 34.C 35.D 36.B 37.B 38.C 39.D 40.A
41.B 42.A 43.B 44.A 45.C 46.D 47.A 48.B 49.A 50.A
51.C 52.C 53.C 54.A 55.D

Previous Years Questions


1. 1.10 to 1.12 2. 1.74 to 1.76 3. 1.1 to 1.2 4.0.53 5.D 6.D 7.C 8.A 9.D
10.C 11.D

Chapter 06.indd 147 8/28/2015 7:47:31 PM


2.148 | Engineering Mathematics

Test
Engineering Mathematics
Directions for questions 1 to 30:Select the correct alternative y
from the given choices.
1. If l is an eigen value of an orthogonal matrix A, then which B (0,1)
of the following is always TRUE?
1
P: is also an eigen value of A
l C
Q: l is a non zero eigen value.
(A) P only (B) Q only
(C) Both P and Q (D) Neither P nor Q
x
0
2. A fair die is rolled independently four times. The probability A (1,0)
that a non-composite number turns up for atleast 3 times is
_______ 1 1
(A) (B)
16 11 3 3
(A) (B)
27 27 1 1
(C) (D)
1 5 6 6
(C) (D)
27 27
7 a
2
8. If the eigen values of a matrix A = are 10 and 2, then
3. The value of I = x 3 ln xdx is ________ 5 b
1
the values of a and b respectively are _______
(A) 4ln 16 - 15
a = 3, b = -5
(A) (B) a = - 3, b = -5
(B) 4ln 16 + 15 a = 3, b = 5
(C) (D) a = 3, b = - 5
15
(C) ln 16 + cos x 3p
16 9. The Taylors series expansion of at x = is given by
3p 2
15 x
(D) ln 16 - 2
16
2 4
y 3p 3p
4. If x = ln , then y has x x
2 2
x 1
(A) + ......
2! 4!
a local maximum at x = -1
(A)
a local minimum at x = -1
(B) 3p 3p
3 5

a local maximum at x = 1
(C) x x
3p 2 2
a local minimum at x = 1
(D) x
(B) + ......
2 3! 5!
5. The complete solution to the initial value problem
2 4
d2y 3p 3p
+ 4y = 0; y(0) = 4 and y(0) = 8 is ________ x x
dx 2 2 2
1
(C) + .....
3! 5!
(A) y = 4 (cos 2x + 2sin 2x)
(B) y = 4 cos 2x 3p
2
3p
4

(C) y = 4 sin 2x x x
2 2
(D) None of these 1 +
(D) + .....
3! 5!
1
1 + 2n n 10. If a scalar field f and a vector field V are related by f = Div V ,
6. The value of Lt is _______
n 0 1 + 3n
which of the following is TRUE?
(A)
e (B) e1 Here, S is a closed surface, enclosing a volume VS.
(C)
e
2
(D) 1

(A) curlV d s = fdv,
 F dr, where F = xy 2i x 2 y j over the path
S VS
7. The value of
c

shown in the figure is ________


d s = fdv,
(B) V
S VS

Part 2_Test.indd 148 12/31/2015 11:15:42 AM


Test |2.149

(A) 0.5862 (B) 0.6929


fdv = DivV d s
(C)
S VS (C) 0.5928 (D) 0.7234

18. The arithematic mean of the observations 12, 22, 32, . . . 102 is

(D) curlV d s = f dv,
S VS
(A) 40 (B) 38.5 (C) 50.5 (D) 11

19. If r is the standard deviation of a1, a2, a3 . an, then the


11. The curl of the vector field 2 xyi + xy 2 z j 6 xz 3 k at the point
standard deviation of ka1 + 1, ka2 + 1, ka3 + 1 .. kan + 1 is
(0, 2, 3) is ______
r r
2i + 5 j 6 k (B)
(A) 5i + 2 j (A) kr + 1
kr (B) (C) - 1 (D)
k k
162 j + 12k
(C) (D) None of these
20. If a = i + j - 2k and b = i - 2 j + k , then determinant of the
12. Let S denote the set of all possible arrangements of letters of
the word AUTHORISED without repetitions. If an element matrix a.a a.b , where a.b denotes the dot product of the
in S is selected at random, then what is the probability that, b.a b.b
the letter I appears at a later position, than all other vowels?
vector a and b is _________
1 1
(A) (B) (A) 9 (B) 27 (C) 18 (D) 54
5 5!
4! 4! 5! 21. If p = p, Q = q and R = r are the position vector of the verti-
(C) 2 (D)
5! 10! ces of a triangle PQR, then area of the triangle PQR is ______
13. In the process of finding a solution to the equation f(x) = 4x2 1 1
= 4x 15 = 0 by Newton Raphson method, with initial solu- (A) p q + q r + r p (B) p q + r q + r p
2 2
tion as xo = 1.6, the method converges to actual solution after
_______ interaction, when the calculator is fixed to four deci- 1 1
(C) q p + q r + r p (D) p q + q r + p r
mal places 2 2
(A) 2nd (B) 4th (C) 6th (D) 8th
22. If x and y are two random variables, a and b are any two con-
14. If w = f (z) = u (x,y) + I v(x,y) be an analytic function then stants and E(x) and var(x) denote the expectation and the vari-
which of the following need not be true? ance of the random variable x, then which of the following is
INCORRECT?
2u 2u 2v 2v
(A) 2 + 2 = 0; (B) + = 0; (A) var(ax + b) = a2 var(x)
x y x 2 y 2 (B) E(ax + b) = aE(x) + b

u + v u (C) E(x + y) = E(x) + E(y)
(C) + =0 (D) + =0
x y x y (D) E(x2) = var(x)

15. In the process of finding a root for x3 3x2 5x + 6 = 0 in 23. If the roots of the equation x3 + px2 + qx + r = 0 are 2, 3 and
between a = 3 and b = 4, by Regula Falsi method, the value of 6, then
the root in the first iteration is ____ (A) p = 7, q = 0 and r = 36
(A) 3.8182 (B) 3.5 (B) p = 7, q = 0 and r = 36
(C) 3.6235 (D) 3.3218 (C) p = 7, q = 0 and r = 36
(D) p = 7, q = 0 and r = 36
16. For the system of linear equations
24. Which of the following pairs of vectors are orthonormal?
x + 2y + 3z = 4
2x + 3y + (a 4) z = b 1 1

4x + 7y z = 5 4 2
1 -1
has infinite number of solutions, then the values of a and b 1 1
1 ; 2 (B)
(A) ;
are ____ 4 4
1 -1
(A) a = b = 3 (B) a = b = 3 1 -1
(C) a = b = 3 (D) a = b = 3
2 4
p /3
3
17. Evaluate f ( x ) dx by Simpsons 8 Rule using the following 1 1
0
table 2 3
1 3
2 ; 2 -1 1
p p p 2p 5p p (C)
(D) ;
x 0 3 1 2 3
18 9 6 9 18 3 -1

y = f(x) 0 0.1762 0.3638 0.5770 0.8385 1.1907 1.7299 0 3

Part 2_Test.indd 149 12/31/2015 11:17:24 AM


2.150 | Engineering Mathematics

2 + 3i dy
25. The value of is _____ 28. If x = y + y + y + ....... , then the value of at x = 1 is
3 + 4i dx
_______
18 1 18 1
(A) - i (B) +i (A) 0(B)1(C)4(D)undefined
25 25 25 25
18 24 18 24 3
(C) - i (D) +i 2 x - 3 ; for x 2
25 25 25 25 29. Let f(x) =
3 - 2 x ; for x < 3
26. An integrating factor of the non-exact differential equation 2
(x2 + 2xy 2y2)dx + (y2 + 2xy 2x2)dy = 0 is
Then, which of the following is true?
1 (A) 
f(x) is continuous and differentiable for all real values of x
x +y
(A) 3 3
(B)
x3 + y3 3
f(x) is not continuous at x =
(B) 
1 2
x3 y3
(C) (D) 3
x3 - y3 f(x) is continuous for real values of x, except x =
(C) 
2
dy
27. The solution of the differential equation x + y = 3x2, (D) 
f(x) is continuous for every x and differentiable for all
y(1) = 2 is dx
3
values of x, except x =
1 1 2
y = x + (B)
(A) y=x+ 2
x x 2s + 3
30. If L[f(t)] = , then the initial value of f(t) is _______
1 s 2 + 5s + 6
y = x3
(C) (D) y = x2 +
x (A) 1 (B) 2 (C) 3 (D) 6

Answer Keys
1.C 2.A 3.D 4.B 5.D 6.B 7.D 8.C 9.D 10.B
11.C 12.A 13.B 14.C 15.A 16.C 17.B 18.B 19.A 20.B
21.A 22.D 23.C 24.D 25.B 26.B 27.D 28.B 29.D 30.B

Part 2_Test.indd 150 12/31/2015 11:18:12 AM

You might also like